Download as pdf or txt
Download as pdf or txt
You are on page 1of 121

INCOME TAXATION WHOLE BOOK Individual taxpayers education and social protection among

CHEAT SHEET (Progressive Income tax) others.A system of government is


LAWS, PRINCIPLES, AND APPLICATIONS Corporate income taxpayers(corporate indispensable to every society. Without
By Rex B. Banggawan, CPA, MBA income tax) it, the people will not relish the benefits
Philippine Copyright, 2014 of a civilized and orderly society.
However, a government cannot exist
OVERVIEW OF CONTENTS Special individual tax rules: without a system of funding. The
Introductory concepts to taxation Taxable trust &Estates government’s necessity for funding is
The concept of tax, tax laws and tax Special corporate tax rules: the theory of taxation.
administration Gross Income tax The basis of taxation
The concept of gross income under MCIT The government provides benefits to
taxation IAET the people in the form of public
Taxation schemes, accounting period, Branch profit remittance tax services, and the people provide the
methods and income reporting funds that finance the government. This
Final income taxes CHAPTER 1 mutuality of support between the
Capital gains taxes INTRODUCTION TO TAXATION people and the government is referred
Exclusions and exempt income Chapter Overview and Objectives to as the basis of taxation.
Income subject to regular income tax This chapter discusses the fundamental This mutuality is illustrated as:
Compensation income principles of taxation. Public services-people-taxes-
Fringe benefits and the fringe benefitsAfter this chapter, readers must be able government-public services
tax to comprehend and demonstrate Receipt of benefits is conclusively
Dealings in properties subject to regular
mastery of the following: presumed
tax 1.Concept of taxation and its necessity Every citizen and resident of the state
Allowable deductions from gross for every government directly or indirectly benefits from the
income 2. Lifeblood doctrine and its implication public services rendered by the
Specific regular tax rules applicable to taxation government. These benefits can be in
individuals 3. Theories of government cost the form of daily free usage of public
Specific regular tax rules applicable to
allocation infrastructures, access to public health
corporations 4.Inherent power of the state or educational services, the protection
5.Scope of the taxation power and security of person and property, or
THE CONCEPT MAP OF INCOME 6.Limitations of the taxation power simply the comfort of living in a civilized
TAXATION 7.Stages of taxation and peaceful society which is
Introductory concepts 8.Concept of situs in taxation maintained by the government.
9.Fundamental principles surrounding While most public services are received
taxation indirectly, their realization by every
GROSS INCOME 10.Various escapes from taxation citizen and resident is undeniable. In
11. Concept of tax amnesty and taxation, the receipt of these benefits
condonation by the people is conclusively presumed.
What is taxation? Thus, taxpayers cannot avoid payment
Taxation may be defined as a state of taxes under the defense of absence
Final Income Regular power, a legislative process, and a of benefit received. The direct receipt
Income mode of government cost distribution. or actual availment of government
Taxation Capital Gains taxation 1.As a state power-Taxation is an services is not a precondition to
taxation inherent power of the state to enforce a taxation.
proportional contribution from its THEORIES OF COST ALLOCATION
subjects for public purpose. Taxation is a mode of allocating
2.As a process-Taxation is a process of government costs of burden to the
levying taxes by the legislature of the people. In distributing the costs or
state to enforce proportional burden, the government regards the
Fringe benefits Gains on contributions from its subjects for following general considerations in the
dealings in public purpose. exercise of its taxation power:
properties 3.As a mode of cost distribution 1.Benefit received theory
Taxation is a mode by which the state 2.Ability to pay theory.
Gross Income PXXX allocates its costs or burden to its Benefit received theory
Less: subjects who are benefited by its The benefit received theory
Deductions XXX spending. presupposes that the more benefit one
Personal exemptions XXX receives from the government, the
Taxable income PXXX The theory of taxation more taxes he should pay.
Every government provides a vast array Ability to pay theory.
of public services including defense, The ability to pay theory presupposes
public order and safety, health, that taxation should also consider the
taxpayer’s ability to pay. Taxpayers a.Income received in advance is taxable the general
should be required to contribute based upon receipt. govern welfare
on their relative capacity to sacrifice for b.Deduction for capital expenditures ment of the
the support of the government. and prepayments is not allowed as it people
In short, those who have more should effectively defers the collection of Perso Comm Commun Owner
be taxed more even if they benefit less income tax. ns unity ity if of the
from the government. Those who have c.A lower amount of deduction is affect or class of propert
less shall contribute less even if they preferred when a claimable expense is ed class individua y
receive more of the benefits from the subject to limit. of ls
government. d. A higher tax base is preferred when individ
Aspects of the ability to pay theory the tax object has multiple tax bases. uals
1.Vertical equity-vertical equity INHERENT POWERS OF THE STATE Amou Unlimi Limited(I No
proposes that the extent of one’s ability A government has its basic needs and nt of ted(Ta mpositio amount
to pay is directly proportional to the rights which co-exist with its creation. It Impo x n is impose
level of his tax base. has rights to sustenance, protection, sition based limited d. (The
For example, A has P200,000 income and properties. The government on to cover govern
while B has P400,000. In taxing income, sustains itself by the power of taxation, govern cost of ment
the government should tax B more than secures itself and the well-being of its ment regulatio pays
A because B has greater income; hence people by police power, and secures its needs. n) just
a greater capacity to contribute. own properties to carry out its public ) compe
2.Horizonatl equity-requires services by the power of eminent nsation
consideration of the particular domain. .)
circumstance of the taxpayer. For These rights, dubbed as “powers” are Impor Most Most Import
example, both A and B have P300,000 natural, inseparable, and inherent to tance import superior ant
income. A is single without child while B every government, No government can ant
is married with four children. In taxing sustain or effectively operate without Relati Inferio Superior Superio
income, the government should tax A these powers. Therefore, the exercise, onshi r to to r to the
more than B because A has a greater of these powers by the government is p the the”Non- “Non-
capacity to contribute since he has no presumed understood and with “Non- impairme impair
dependents to support. B has less acknowledged by the people from the the impair nt ment
capacity to contribute considering the very moment they establish their Cons ment Clause” Clause”
support requirements of his government. These powers are tituti Clause of the of the
dependents. naturally exercisable by the government on ” of Constitu Constit
Vertical equity is a gross concept while even in the absence of an express grant the tion ution.
horizontal equity is a net concept. of power in the Constitution. Constit
The lifeblood doctrine The Inherent Powers of the State ution
Taxes are essential and indispensable to 1.Taxation power-is the power of the Limit Constit Public Public
the continued subsistence of the State to enforce proportional ation utional interest purpos
government. Without taxes, the contribution from its subjects to sustain and and due e and
government would be paralyzed for lack itself. inhere process just
of motive power to activate or operate 2.Police power-is the general power of nt compe
it.(CIR vs. Algue) the State to enact laws to protect the limitati nsation
Taxes are the lifeblood of the well-being of the people. ons
government, and their prompt and 3.Eminent domain-is the power of the
certain availability are an imperious State to take private property for public Similarities of the three powers of the
need. Upon taxation depends the use after paying just compensation. State
government’s ability to serve the Comparison of the three powers of the They are all necessary attributed of
people for whose benefit taxes are State sovereignty.
collected.(Vera vs. Fernandez) Point Taxatio Police Eminen They are all inherent to the state.
Implication of the lifeblood doctrine in of n Power t They are all legislative in nature
taxation: Differ Domain They are all ways in which the State
1.Tax is imposed even in the absence of ence interferes with private rights and
a Constitutional grant. Exerci Gover Governm Govern properties.
2. Claims for tax exemption are sing nment ent ment They all exist independently of the
construed against taxpayers. autho and Constitution and are exercisable by the
3.The government reserves the right to rity private government even without
choose the objects of taxation. utilities Constitutional grant. However, the
4.The courts are not allowed to Purpo For the To For Constitution may impose conditions or
interfere with the collection of the se suppor protect public limits for their exercise.
taxes. t of the use
5. In income taxation:
They all presuppose and equivalent Thus,it can be imposed only within the from its properties and activities
form of compensation received by the territories of the state. There is no basis conducted for profit including income
persons affected by the exercise of the in taxing foreign subjects abroad since from government owned and controlled
power. they do not derive benefits from our corporations is subject to tax.
The exercise of these powers by the government. Furthermore, Non-delegation of the taxing power
local government units may be limited extraterritorial taxation will amount to The legislative power is vested
by the national legislature. encroachment of foreign sovereignty. exclusively in Congress and is non-
Scope of the taxation power Exception to the territoriality principle delegable pursuant to the doctrine of
The scope of taxation is widely regarded 1.In income taxation, resident citizens separation of the branches of the
as comprehensive, plenary, unlimited and domestic corporations are taxable government to ensure a system of
and supreme. income derived within and outside the checks and balances.
However, despite the seemingly Philippines. The power of lawmaking, including
unlimited nature of taxation, it is not 2. In transfer taxation, residents or taxation, is delegated by the people to
absolutely unlimited. Taxation has its citizens such as resident citizens, nor the legislature. So as not to spoil the
own inherent limitations and limitations resident citizens and resident aliens are purpose of delegation, it is held that
imposed by the Constitution. taxable on transfers of property located what has been delegated cannot be
THE LIMITATIONS OF THE TAXATION within or outside the Philippines. further delegated.
POWER International Comity Exceptions to the rule of non-delegation
Inherent limitations International comity pertains to mutual 1.Under the Constitution, local
1.Territoriality of taxation courtesy or reciprocity between states government units are allowed to
2.International comity is a basic principle of international law exercise the power to tax to enable
3.Public purpose that all states are equally sovereign. them to exercise their fiscal autonomy.
4.Exemption of the government Each states observes co-equal 2.Under the tariff and customs code,
5.Non-delegation of the taxing power sovereignty by not taxing the The president is empowered to fix the
B.Constitutional limitations properties, income, or effects of fellow amount of tariffs to be flexible to trade
1.Due process of law state. conditions.
2.Equal protection of the law Consistent with this, embassies or 3.Other cases that require expedient
3.Uniformity rule in taxation consular offices of foreign governments and effective administration and
4.Progressive system of taxation in the Philippines including implementation of assessment and
5.Non-improsonment for non-payment international organizations and their collection of taxes.
of debt or poll tax non-filipino staff are not subject to CONSTITUTIONAL LIMITATIONS OF
7.Free worship rule Philippine taxation. Under the National TAXATION
8.Exemption of religious or charitable Internal Revenue Code(NIRC), The Observance of due process of law
entities, non-profit cemeteries, income of foreign government and No one should be deprived of his life,
churches and mosque from property foreign government-owned and liberty, or property without due process
taxes corporations are not subject to income of law. Tax laws should neither be harsh
9.Non-appropriation of public funds or tax. nor oppressive.
property for the benefit of any church, When a state enters into treaties with Aspects of Due process
sect or system of religion other states, it is bound to honor the 1.Substantive due process
10.Exemption from taxes of the agreements as a matter of mutual Tax must be imposed only for public
revenues and assets of non-profit, non- courtesy, and in the case its treaty purpose, collected only under authority
stock educational institutions. agreements with other states conflict of a valid law and only by the taxing
11. Concurrence of a majority of all with its local tax law, its treaty power having jurisdiction. An
members of Congress for the passage of agreements are given primacy. assessment without a legal basis
a law granting tax exemption. Public purpose violates the requirement of due
12. Non-diversification of tax collections Tax is intended for the common good. process.
13. Non-delegation of the power of Taxation must be exercised absolutely 2.Procedural due process-There should
taxation for public purpose. It cannot be be no arbitrariness in assessment and
14. Non-impairment of the jurisdiction exercised to further any private interest. collection of taxes, and the government
of the Supreme Court to review tax Exemption of the government shall observe the taxpayer’s right to
cases The taxation power is broad. The notice and hearing. The law established
15. The requirement that government can exercise the power procedures which must be adhered to
appropriations, revenue or tariff bills upon anything including itself. However, in making assessments and in enforcing
shall originate exclusively in the House the government normally does not tax collections.
of Representatives. itself as this will not raise additional Under the NIRC, assessments shall be
16. The delegation of taxing power to funds but will only impute additional made within three years from due date
local government units. costs. of filing of the return of from the date
INHERENT LIMITATION OF TAXATION Under the NIRC, government properties of actual filing, whichever is later.
Territoriality of Taxation and income from essential public Collection shall be made within five
Public services are normally provided functions are not subject to taxation. years from the date of assessment. The
within the boundaries of the state. However, income of the government failure of the government to observe
these rules violates the requirements of non-improsonment for non-payment of exempt from real property tax. This,
due process debt does not extend to non-payment however, is not applied in the
Equal protection of the law of tax except poll tax. Philippines.
No person shall be denied the equal Poll, personal, community or residency Non-appropriation of public funds or
protection of the law. Taxpayers should tax property for the benefit of any church,
be treated equally both in terms of Poll tax has two components: sect, or system of religion
rights conferred and obligations Basic community tax This constitutional limitation is intended
imposed. Additional community tax to highlight the separation of religion
This rule applies where taxpayers are The constitutional guarantee of non- and the state. To support freedom of
under the same circumstances and imprisonment for non-payment of poll religion, the government should not
conditions. This requirement would tax applies only to the basic community favor any particular system of religion
mean Congress cannot exempt sellers tax. Non-payment of the additional by appropriating public funds or
“ballot” while subjecting sellers of community tax is an act of tax evasion property in support thereof.
“penoy” to tax since they are essentially punishable by imprisonment. It should be noted, however, that
the same goods. Non-impairment of obligation and compensation to priests, imams, or
Uniformity rule in taxation contract religious ministers working with the
The rule of taxation shall be uniform The state should set an example of military, penal institutions, orphanages,
and equitable. Taxpayers under good faith among its constituents. It or leprosarium is not considered
dissimilar circumstances should not be should not set aside obligations from religious appropriation.
taxed the same. Taxpayers should be contacts by the exercise of its taxation Exemption from taxes of the revenues
classified according to commonality in power. Tax exemptions granted under and assets of non-profit, non-stock,
attributes, and the tax classification to contract should be honored and should educational institutions including
be adopted should be based on not be cancelled by a unilateral grants, endowments, donations, and
substantial distinction. Each class is government action. contributions for educational purposes.
taxed differently, but taxpayers filing Free worship rule The Constitution recognizes the
under the same class are taxed the The Philippine adopts free exercise of necessity of education in state building
same. Hence, uniformity is relative religion and does not subject its by granting tax exemption on revenues
equality. exercise to taxation. Consequently, the and assets of non-profit educational
Progressive system of taxation properties and revenues of religious granting tax exemption on revenues
Congress shall evolve a progressive institutions such as tithes or offerings and assets of non-profit educational
system of taxation. Under the are not subject to tax. This exemption, institutions. This exemptions, however,
progressive system, tax rates increase however, does not extend to income applies only on revenues and assets
as the tax base increases. The from properties or activities of religious they are actually, directly, and
constitution favor progressive tax as it is institution that are propriety or exclusively devoted for educational
consistent with the taxpayer’s ability to commercial in nature. purposes.
pay. Moreover, the progressive system Exemption of religious, charitable or Consistent with this constitutional
aids in an equitable distribution of educational entities, non-profit recognition of education as a necessity,
wealth to society by taxing the rich cemeteries, churches and mosques, the NIRC also exempts government
more than the poor. lands, buildings, and improvements educational institutions from income
Non-imprisonment for non-payment of from property taxes tax subjects private educational
debt or poll tax The Constitutional exemption from institutions to a minimal 10% income
As a policy, no one shall be imprisoned property tax applies for properties tax.
because of his poverty, and no one shall actually, directly, and exclusively(i.e. Concurrence of a majority of all
be imprisoned for mere inability to pay primarily) used for charitable, religious members of Congress for the passage of
debt. and educational purposes. law granting tax exemptions.
However, this Constitutional guarantee In observing this Constitutional Tax exemption law counters against the
applies only when the debt is acquired limitation, the Philippines follows the lifeblood doctrine as it deprives the
by the debtor in good faith. Debt doctrine of use wherein only properties government of revenues. Hence, the
acquired in bad faith constitutes estafa, actually devoted for religious, grant of tax exemption must proceed on
a criminal offense punishable by charitable, or educational activities are upon a valid basis. As a safety net, the
imprisonment. exempt from real property tax. Constitution requires the vote of the
Is non-payment of tax equivalent to In observing this Constitutional majority of all members of Congress in
non-payment of debt? limitation, the Philippines follows the the grant of tax exemption.
Tax arises from law and is a demand of doctrine of use wherein only properties In the approval of an exemption law, an
sovereignity. It is distinguished from actually devoted for religious, absolute majority or the majority of a
debt which arises from private contacts. charitable, or educational activities are members of Congress, not a relative
Non-payment of tax compromises exempt from real property tax. Under majority or quorum majority, is
public interest while the non-payment the doctrine of ownership, the required.However, in the withdrawal of
of debt compromises private interest. properties of religious, charitable, or tax exemption, only a relative majority
The nonpayment of tax is similar to a educational entities whether or not is required.
crime. The Constitutional guarantee on used in their primary operations are Non-diversification of tax collections
Tax collections should be used for public 2.Assessment and collection Illustration
purpose. It should never be diversified Levy or imposition A foreign corporation leases a
or used for private purpose. This process involves the enactment of residential space to a non-resident
Non-delegation of the power of a tax law by Congress and is called Filipino citizen abroad. The rent income
taxation impact of taxation. It is also referred to will exempt from Philippines taxation as
The principle of checks and balances in as the legislative act in taxation. the leasing service is rendered abroad.
a republican state requires that taxation Congress is composed of two bodies: 3.Income tax situs on sale of goods-The
power as part of lawmaking be vested 1.The house of representatives,and gain on sale is subject to tax in the place
exclusively in Congress. 2.The senate of sale.
However, delegation may be made on As mandated by the Constitution, a tax Illustration
matters involving the expedient and bill must originate from the House of While in China, a non-resident OFW
effective administration and Representatives. Each may, however, citizen agreed with a Chinese friend to
implementation of assessment and have their own versions of a proposed sell his diamond necklace to the latter.
collection of taxes. Also, certain aspects law which is approved by both bodies, They stipulated that the delivery of the
of the taxing process that are non- but tax bill cannot originate exclusively item and the payment will be made a
legislative in character are delegated. from the senate. week later in the Philippines. The sale
Hence, implementing administrative Matters of legislative discretion in the consummated as agreed.
agencies such as the Department of exercise of taxation The contract of sale is consensual and is
Finance and the Bureau of Internal 1.Determining the object of taxation perfected by the meeting of the minds
Revenue (BIR) issues revenue 2.Setting the tax rate or amount to be of the contracting parties. The
regulations, ruling orders, or circulars to collected perfection of the contract of sale is in
interpret and clarify the application of 3.Determining the purpose for the levy China. The situs of taxation is in China.
the law. But even so, their functions are which must be public use The gain on sale of the necklace will be
merely intended to interpret or clarify 4.kind of tax to be imposed. taxable abroad and exempt in the
the proper application of the law. They 5.Apportionment of the tax between Philippines.
are not allowed to introduced new the national and local government 4.Property tax situs:Properties are
legislations within their quasi legislative 6.Situs of taxation taxable in their location
authority. 7. Method of collection Illustration-An overseas Filipino worker
Non-impairment of the jurisdiction of Assessment and Collection has a residential lot in the Philippines
the Supreme Court to review tax cases The tax law is implemented by the He will still pay real property tax despite
Notwithstanding the existence of the administrative branch of the his absence in the Philippines because
Court of Appeals, which is a special government. Implementation involves his property is located herein.
court, all cases involving taxes can be assessment of the determination of the 5.Personal tax situs:Persons are taxable
raised to and finally decided by the tax liabilities of taxpayers and in their place of residence
Supreme Court of the Philippines. collection. This stage is referred to as Illustrations:
Appropriations, revenue, or tariff bills incidence of taxation or the Ahmed Lofti is a Sudanese studying
shall originate exclusively in the House administrative act of taxation. medicine in the Philippines
of representatives, but the Senate may SITUS OF TAXATION Ahmed will pay personal tax in the
propose or concur with amendments. Situs is the place of taxation. It is the tax Philippines even if he is an alien
Laws that add income to the national jurisdiction that has the power to levy because he is residing in the
treasury and those that allows spending taxes upon the tax object. Situs rules Philippines.
therein must originate from the House serves as frames of reference in gauging Other fundamental doctrines in
of Representatives while Senate may whether the tax object is within or taxation
concur with amendments. The outside the tax jurisdiction of the taxing 1.Marshall doctrine-“The power to tax
origination of a bill by Congress does authority. involves the power to destroy.” Taxation
not necessarily mean that the House Examples of Situs rules: power can be used as an instrument of
bill must become the final law. It was 1.Business tax situs:Businesses are police power. It can be used to
held constitutional by the Supreme subject to tax in the place where the discourage or prohibit undesirable
court when Senate changed the entire business is conducted. activities or occupation. As such,
house version of a tax bill. Illustration taxation power carries with it power to
Each local government unit shall A taxpayer is involved in car dealership destroy.
exercise the power to create its own abroad and restaurant operation in the However, the taxation power does not
sources of revenue and shall have a just Philippines. include the power to destroy if it is used
share in the national taxes The restaurant business will be subject solely for the purpose pf raising
This is a constitutional recognition of tax in the Philippines since the business revenues.(Roxas vs. CTA)
the local autonomy of local is conducted herein, but the car dealing 2.Holme’s Doctrine-“Taxation power is
governments and an express delegation business is exempt because the not the power to destroy while the
of the taxing power. business is conducted abroad. court sits”. Taxation power may be used
STAGES OF THE EXERCISE OF TAXATION 2.Income tax situs on services-Service to build or encourage beneficial
POWER fees are subject to tax where they are activities or industries by the grant of
1.Levy or imposition rendered. tax incentives.
While the Marshall doctrine and the of its assessment. In the absence of an construed strictly against the tax payer
Holme’s doctrine appear to contradict assessment, tax prescribes if not in accordance with the lifeblood
each other both are actually employed collected by judicial action within 3 doctrine.
in practice. A good manifestation of the years from the date the return is The right of taxation is inherent to the
Marshall doctrine is the imposition of required to filed. However, taxes due state. It is a prerogative essential to the
excessive tax on cigarretes while from taxpayers who did not file a return perpetuity of the government. He who
application of the Holme’s doctrine or the who filed fraudulent returns do claims exemption from the common
include creation of Ecozones with not prescribe. burden must justify his claim by the
holidays and provision of incentives, 7.Doctrine of Estoppel clearest grant of organic or statue law.
such as the Omnibus Investment Under the doctrine of estoppel, any (Iloilo, et al. vs. Smart Communications,
Code(E.0 226) and the Barangay Micro- misrepresentation made by one party Inc, G.R. No. 167260, February 27,
Business Enterprise(BMBE) Law. toward another who relied therein in 2009)
3. Prospectivity of tax laws good faith will be held true and binding When exemption is claimed, it must be
tax laws are generally prospective in against that person who made the shown indubitably to exist. At the
operation. An ex post facto law or that misrepresentation. outset, every presumption is against it.
retroacts is prohibited by the The government is not subject to A well-founded doubt is fatal to the
Constitution. estoppel. The error of any government claim, it is only when the terms of the
Exceptionally, income tax laws may employee does not bind the concession are too explicit to admit
operate retrospectively if so intended government. It is held that the neglect fairly of any other construction that the
congress under certain justifiable or omission of government officials proposition can be supported. (Ibid)
conditions. For example, even after that entrusted with the collection of taxes Tax exemption cannot arise from vague
tax on income earned during periods of should not be allowed to bring harm or inference. Tax exemption must be clear
foreign occupation even after the war. detriment to the interest of the people. and unequivocal. A taxpayer claiming a
4.Non-compensation or set-off Also, erroneous applications of the law tax exemption must point to a specific
Taxes are not subject to automatic set- by public officers do not block the provision of law conferring on the
off or compensation. The taxpayer subsequent correct application of the taxpayer, in clear and plain terms,
cannot delay payment of tax to wait for same. exemption from a common burden. Any
the resolution of a lawsuit involving his 8.Judicial Non-interference doubt whether a tax exemption exists is
pending claim against the government. Generally, courts are not allowed to resolved against the taxpayer.(see
Tax is not a debt, hence, it is not subject issue injunction against the Digital Telecommunications, Inc. vs. City
to set-off. This rule is important to allow government’s pursuit to collect tax as Government of Batangas, et al)
the government sufficient period to this would unnecessarily defer tax DOUBLE TAXATION
evaluate the validity of the claim. (see collection. This rule is anchored on the Double taxation occurs when the
Philex Mining Corporation CIR G.R. Lifeblood doctrine. taxpayer is taxed twice by the same tax
125704) 9.Strict Construction of tax laws jurisdiction for the same thing.
Exceptions: When the law clearly provides for Elements of double taxation
a.Where the taxpayer’s claim has taxation, taxation is the general rule 1.Primary element:Same object
already become due and demandable unless there is a clear exemption.Hence 2.Secondary elements:
such as when the government already the maxim, “Taxation is the rule, a.same type of tax
recognized the same and appropriation exemption is the exception.” b.same purpose of tax
for refund was made When the language of the law is clear c.same taxing jurisdiction
b.Cases of obvious overpayment of and categorical, there is no room for d.same tax period.
taxes interpretation. There is only room for Types of Double taxation
c.Local taxes application. However, when taxation 1.Direct double taxation
5.Non-assignment of taxes laws are vague, the doctrine of strict This occurs when all the element of
Tax obligations cannot be assigned or legal construction is observed. double taxation exists for both
transferred to another entity contract. Vague tax laws impositions.
Contracts executed by the taxpayer to Vague tax laws are construed against Examples:
such effect shall not prejudice the right the government and in favor of the a.An income tax of 10% on monthly sale
of the government to collect. taxpayers. A vague tax laws means no and a 2% income tax on the annual
6.Imprescritibility in taxation tax laws. Obligation arising from law is sales(total of monthly sales)
Prescription is the lapsing of a right due not presumed. The Constitutional b.A 5% tax on bank reserve deficiency
to the passage of time.When one sleep requirement of due process requires and another 1% penalty per day as a
on his right over an unreasonable laws to be sufficiently clear and consequence of such reserve deficiency.
period of time, he is presumed to be expressed in their provision. 2.Indirect double taxation
waiving his right. The government’s Vague exemption laws This occurs when at least one of the
right to collect taxes does not prescribe Vague tax exemption laws are secondary elements of double taxation
unless the law itself provides such construed against the taxpayer and in is not common for both impositions.
prescription favor of the government. A vague tax Examples:
Under the NIRC, tax prescribes if not exemption law means no exemption a.The national government levies
collected within 5 years from the date law. The claim for exemption is business tax on the sales or gross
receipts of business while the local to a tax which others are subject to.Tax under certain justifiable grounds. This is
government levies business tax upon exemptions may be granted by the also referred to as tax remission.
the same sales or receipts. Constitution, law, or contract. Because they deprive the government
b.The national government collects All forms of tax exemptions can be of revenues, tax exemption, tax refund,
income tax from a taxpayer on his revoked by Congress except the granted tax amnesty, and tax condonation are
income while the local government by the Constitution and those granted construed against the taxpayer and in
collects community tax upon the same under contracts. favor of the government.
income. B.Those that do not result to loss of Tax amnesty vs. tax condonation
c.The Philippine government taxes government revenue Amnesty covers both civil and criminal
foreign incomes of domestic 1.Shifting-This is the process of liabilities, but condonation covers only
corporations and resident citizens while transferring tax burden to other civil liabilities of the taxpayer.
a foreign government also taxes the taxpayers. Amnesty operates retrospectively by
same income (international double Forms of shifting forgiving past violations. Condonation
taxation) a.Forward shifting-This is the shifting of applies prospectively to any unpaid
Nothing in our law expressly prohibits tax which follows the norm flow of balance of the tax; hence, portion
double taxation. In fact, indirect double distribution (i.e, from manufacturer to already paid by the taxpayer will not be
taxation is prevalent in practice. wholesalers, retailers, consumers). refunded.
However, direct double taxation is Forward shifting is common with Amnesty is also conditional upon the
discouraged because it is oppressive essential commodities and services taxpayer paying the government of a
and burdensome to taxpayers. It is also such as food and fuel. portion of the tax whereas condonation
believed to counter the rule of equal b.Backward shifting-This is the reverse requires no payment.
protection and uniformity in the of forward shifting, Backward shifting is CHAPTER 1:SELF-TEST EXERCISES
Constitution common with non-essential Discussion Questions
How can double taxation be minimized? commodities where buyer have 1.Define taxation
The impact of double taxation can be considerable market power and 2.Distinguish the theory and the basis
minimized by any one or a combination commodities with numerous substitute of taxation
of the following: products. 3.What are the theories of government
a.Provision of tax exemption c.Onward shifting-This refers to any tax cost allocation?Explain each
b.Allowing foreign tax credit (deduction shifting in the distribution channel that 4.Differentiate the Lifeblood doctrine
for taxes paid abroad) exhibits forward shifting or backward 5.Enumerate and explain the inherent
c.Allowing reciprocal tax treatment shifting. powers of the state
between the home country and a Shifting is common with business taxes 6.Distinguish the three powers of the
foreign country. where taxes imposed on business state and enumerate their similarities
d.Entering into treaties or bilateral revenue can be shifted or passed-on to 7.Differentiate vertical and horizontal
agreements. customers. equity
ESCAPE FROM TAXATION 2.Capitalization-This pertains to the 8.Describe the scope of the power of
Escapes from taxation are the means adjustment of the value of an asset taxation
available to the taxpayer to limit or caused by changes in tax rates. 9.Distinguish substantive due process
even avoid the impact of taxation. For instance, the value of a mining from procedural due process.
Categories of Escapes from taxation property will correspondingly decrease 10. Distinguish the concept of equality
A.Those that result to loss government mining output is subjected to higher from the concept of uniformity in
revenue taxes. This is a form of backward taxation
1.Tax evasion-also known as tax shifting of tax. 11.Distinguish non-payment of debt
dodging, refers to any act or trick that 3.Transformation-This pertains to the versus non-payment of tax in terms of
tends to illegally reduce or avoid the elimination of wastes or losses the consequences
payment of tax. In income taxation, this taxpayers to form tax savings to 12. What institutions are exempt from
can be perpetrated by undue compensate for the tax imposition on real property tax in the Constitution?
understatement of income, increase in taxes. 13.Which of the constitutional
overstatement of expenses or non- Tax amnesty limitations are also classified as
declaration of income. Amnesty is a general pardon granted by inherent limitations?
2.Tax avoidance-also known as tax the government for erring taxpayers to 14. Explain the stages of the exercise of
minimization, refers to any act or trick give them a chance to reform and taxation power.
that reduces or totally escapes taxes by enable them to have a fresh start to be 15. Explain the concept of situs.
any legally permissible means. This may part of a society with a clean slate. It is 16. Distinguish the Marshall Doctrine
be done by selecting tax options an absolute forgiveness or waiver by from the Holme’s Doctrine
allowed by law which minimizes tax the government on its right to collect 17.Discuss the doctrine of strict
liability or by careful tax planning to and is retrospective in application. construction of tax laws.
reduce tax exposure. Tax condonation 18.Explain double taxation, its
3.Tax exemption-also known as tax Tax condonation is forgiveness of the elements, and its types.
holiday, refers to the immunity, tax obligation of a certain taxpayer 19.What are the categories of escapes
privilege or freedom from being subject from taxation? Enumerate and explain
each means of escape under each 5.There should be direct receipt of b.The BIR has five deputy
category. benefit before one could be compelled commissioners
20.Distinguish tax amnesty from tax to pay taxes.F c.The government can still collect tax in
condonation. 6.The exercise of taxation power disregard of a constitutional limitation
Exercise drills requires Constitutional grant.F because taxes are the lifeblood of the
In the space provided for, indicate 7.Taxation is inherent in sovereignity.T government
whether the statement relates to a 8.Police power is the most superior d.The president of the Philippines can
Constitutional limitation©, or inherent power of the government. Its exercise change tariff or imposts without
limitation(I).If it is not a limitation to the needs to be sanctioned by the necessity of calling Congress to pass a
taxing power, indicate (N). Constitution.F law for that purpose.-
1.Non-assignment of taxes-N 9.All inherent powers presupposes an 4.A.The power to tax includes the
2.Territoriality of taxation-I equivalent form of compensation. power to exempt.
3.Taxes must be for public use-I 10.The reciprocal duty of support B.The power to license includes the
4.Exemption of the property of religious between the government and the power to tax
institutions from income tax-C people underscores the basis of Which is true?
5.Exemption of the revenues and assets taxation.T a.A only
of non-profit, non-stock educational True or false b.B only
institutions.-C 1.The constitutional exemption of C. A and B
6.Non-delegation of the taxing power-I religious, charitable, and non-profit d.Neither A nor B-
7.Non-appropriation for religious cemeteries, churches and mosques 5.International double taxation can be
purposes-C refers to income tax and real property mitigated by any of the following
8.The requirement of absolute majority tax.F except?
in the passage of a tax exemption law-N 2.Taxpayers under the same a.Providing allowance for tax credit
9.Non-improsonment for non-payment circumstance should be taxed b.Provision of reciprocity provisions in
of tax or debt-C differently.F tax laws
10.Taxpayers under the same 3.Taxation is subject to inherent and c.Provision of tax exemptions
circumstances should be treated equal Constitutional limitations.T d.Entering into treaties to form regional
both in terms of privileges and 4.International comity connotes trade blockage against the rest of the
obligations.C courtesy between nations.T world.-
11.Exemption from property taxes of 5.Collection of taxes in the absence of a 6.Which is not an object taxation?
religious, educational and charitable law is violative of the Constitutional a.Persons
entities.C requirement for due process.T b.Business
12.Government income and properties 6.The scope of taxation is regarded as c.Transactions
are not objects of taxation.I comprehensive, plenary, unlimited, and d.Public properties-
13.Each local government shall have the supreme.T 7.That courts cannot issue injunction
power to create its own sources of 7.No one shall be imprisoned for non- against the government’s effort to
revenue.C payment of tax.T collect tax is justified by
14.Imprescribility in taxation N 8.The lifeblood doctrine requires to a.Lifeblood doctrine-a
15.Non-impairment of obligation and override its obligations an contracts b.imprescribility of taxes
contracts.C when necessary.F c.the ability to pay theory
16.Gurantee of proportional system of 9.2/3 of all members Congress is d.the doctrine of estoppel
taxation.C required to pass a tax exemption law.F 8.The power to enforce proportional
17.International courtesy-I 10.The government should tax itself.F contribution from the people for the
18.Non-impairement of the jurisdiction Multiple Choice-Theory:Part 1 support of the government is
of the Supreme Court to review tax 1.The point at which tax is levied is also a.Taxation-
cases.C called b.Police power
19.The government is not subject to a.Impact of taxation- c.Eminent domain
estppel.N b.Situs of taxation d.Exploitation
20.Improsonment for non-payment of c.Incidence of taxation 9.This theory underscores that taxes are
poll taxN d.Assessment indispensable to the existence of the
TRUE OR FALSE 2.Which of the following state.
1.Eminent domain involves confiscation inappropriately describes the nature of a.Doctrine of equitable recoupment
of prohibited commodities to protect taxation? b.The lifeblood doctrine-
the well-being of the people.F a.Inherent in sovereignty c.The benefit received theory
2.Horizontal equity requires b.Essentially a legislative function d.The Holmes Doctrine
consideration of the circumstances of c.Subject to inherent and constitutional 10.A. Taxation is the rule, exeption is
the taxpayer.T limitation the exemption.
3.Taxes are the lifeblood of the d.Generally for public purpose- B.Vague taxation laws are interpreted
government.T 3.Which is correct? liberally in favor of the government.
4.Taxation is a mode of apportionment a.Tax condonation is a general pardon Which is false?
of government costs to the people.T granted by the government a.A only
b.B only a.Tax exemption based on contract 24.This refers to the privilege or
c.Both A and B- b.Tax exemption based on the immunity from a tax burden which
d.Neither A nor B Constitution others are subject to:
11.Select the incorrect statement c.Tax exemption based on law a. Exclusion
a.The power to tax includes the power d.Both A and B b.deduction
to exempt D c.Tax holiday-
b.Exemption is construed against the 20.Which statement is incorrect? d.Reciprocity
taxpayer and in favor of the a.Every person must contribute his 25.Statement 1: The benefit received
government share in government costs. theory presupposes that some
c.Tax statues are construed against the b.The existence of a government is taxpayers within the territorial
government in case of doubt expected to improve the lives of the jurisdiction of the Philippines will be
d.Taxes should be collected only for people. exempted from paying tax so long as
public improvements- c.The government provides protection they do not receive benefits from the
12. Which is not a public purpose? and other benefits while the people government.
a.Public education provide support Statement 2:The ability to pay theory
b.National defense d.Only those who are able to pay tax suggests that some taxpayers may be
c.Transportation can enjoy the privileges and protection exempted from tax provided they do
c.None of these.- of the government.- not have the ability to pay the same.
13.Which does not properly describe 21.Which is the most incorrect Which statement is true?
the scope of taxation? statement regarding taxes? a.Only statement 1
a.Comprehensive a.Taxes are necessary for the continued b.Only statement 2-
b.Supreme existence of the government. c.Both statements 1 and 2
c.Discretionary- b.The obligation to pay taxes does not d.Neither statement 1 and 2
d.Unlimted rest upon the privilege enjoyed by or 26.Which is not a legislative act?
14. the protection afforded to the citizen of a.Determiantion of the subject of the
All of these are secondary purposes of the government but upon the necessity tax
taxation except of money for the support of the state. b.Setting the amount of the tax
a.To reduce social inequality c.There should be personal benefit c.Assessment of the tax-
b.To protect local industries enjoyed from the government before d.Determining the purpose of the tax
c.To raise revenue for the support of the one is required to pay tax- 27.Statement 1:Taxation is the rule;
government- d.Taxes should be collected without exemption is the exception
d.To encourage growth of local unnecessary delay but its collection Statement 2:Taxation may be used to
industries should not be tainted with implement the police power of the
15.What is the theory of taxation? arbitrariness. state
a.Reciprocal duties of support and 22.Statement 1: In the selection of the a.1 is true
protection objects of taxation, the courts have no b.2 is true
b.Necessity- power to inquire into the wisdom, c.1 and 2 are true
c.Constitutionality objectivity, motive, expediency, or d.1 and 2 are not true
d.Public purpose necessity of a tax law. c
16.A. Taxes should not operate Statement 2:An imposition can be both 28.Which of the following powers of the
retrospectively a tax and a regulation. Taxes may be Commissioner of Internal Revenue
B.Tax is generally for public purpose levied to provide means for cannot be delegated?
Which is true? rehabilitation and stabilization of a.The examination of tax return and the
a.A only threatened industry. determination of tax due thereon
b.B only Which is incorrect? b.To refund or credit tax liabilities in
c.A and B a.Statement 1 only certain cases
d.Neither A nor B b.Statement 2 only c.The power to compromise or abate
17.Which provision of the Constitution c.Both statements any tax liability involving basic
is double taxation believed to violate? d.Neither statements- deficiency tax of P500,000 and minor
a.Equal protection guarantee 23. Which of the following acts in criminal violations
b.Progressive scheme of taxation taxation is administrative by nature? d.The power to reverse a ruling of the
c.Uniformity rule a.Determination of the amount to be Bureau of Internal Revenue
d.Either A or C imposed d
d b.Fixing the allocation of the amount to 29.When exemption from a tax
18.Which limitation of taxation is the be collected between the local imposition is silent or not clearly stated,
concept of “situs of taxation” based? government and the national which is true?
a.Territoriality- government. a.Taxation applies since exemptions are
b.Public purpose c.Levy or distraint of taxpayer’s construed against the government.
c.International comity property for tax delinquency-C b.Exemption still applies since this is an
d.Exemption of the government d.Determining the purpose of the tax to instance of exemption by omission.
19.Which tax exemption is irrevocable? impose.
c.Taxation applies since exemptions are a.Real property tax- d.None of these
construed against the taxpayer.- b.Income tax 8,The following are limitations of
d.Exemption applies since obligation c.Property tax and income tax taxation:
arising from law cannot be presumed d.Business tax a.Territoriality of taxation
and hence construed against the 2.The agreement among nations to b.Exemption of the government
government lessen tax burden of their respective c.Taxation is for public purpose
30.What is the basis of taxation? subjects is called d. non-impairment of contracts
a.Reciprocal duties of support and a.Reciprocity e.Non-delegation of the power to tax.
protection-a b.International comity- Which of these are classified as both
b.Constitutionality c.Territoriality constitutional and inherent limitations?
c.Public purpose d.Tax minimization a.A and B
d.Necessity 3.An educational institution operated b.B and C
31.When the provisions of tax laws are by a religious organization was being c.C and E
silent as to the taxability of an item, required by a local government to pay d. D and E
which is true? real property tax. Is the assessment b
a.Taxation applies since taxation is the valid? 9.The provisions in the Constitution
rule,exemption is the exception. a.Yes, with respect to all properties held regarding taxation are
b.Exception applies since vague tax laws by such educational institution a.Grants of the power to tax
are construed against government.- b.Yes, with respect to properties not b.Limitations to the power to tax-
c.Taxation applies due to the Lifeblood actually devoted to educational c.Grants and limitations to the power to
doctrine purposes.- tax
d.Exemption applies since obligation c.No, with respect to any properties d.Limitations against double taxation.
arising from law is presumed; ignorance held by such educational institution 10.The Constitutional exemption of
of the law is not an excuse. d.No, with respect to properties not non-stock, non-profit educational
32.Which of the following statements actually devoted to educational institutions refers to
does not support the principle that tax purposes. a.Real property tax
is not subject to compensation or set- 4.Which is not a Constitutional b.Income tax-
off? limitation? c.Property tax and income tax
a.The government and the taxpayer are a.No tax law shall be passed without d.Business tax
not creditors and debtors of each other. the concurrence of a majority of all 11.Which of the following is violative of
b.Tax is not in the nature of contract but members of Congress.-a the principle of non-delegation?
it grows out of a duty wherein b.Non-appropriation for religious a.Requiring that legislative enactment
taxpayers are bound to obey even purposes must exclusively pertain to Congress.
without the personal consent of the c.No law impairing government b.Authorizing the President to fix the
taxpayer. obligations on contracts shall be amount of impost on imported and
c.Taxes arises from law, not from passed. exported commodities
contracts. d.Non-impairment of religious freedom c.Authorizing certain private
d.Both tax and debt partake the nature 5.Which of the following is not an corporation to collect taxes
of an obligation.- inherent limitation of the power to tax? d.Allowing the secretary of finance and
33.Which is not legally tenable in a.Tax should be levied for public the BIR to issue regulation or rulings
refusing to pay tax? purpose which go beyond the scope of a tax
a.Absence of benefit from the b.Taxation is limited to its territorial law.-
government-a jurisdiction 12.Which of the following violates
b.Lack of jurisdiction of the taxing c.Tax laws shall be uniform and Constitutional provisions?
authority equitable- a.Payment of salaries to priests or
c.Prescription of the tax authority’s d.Exemption of government agencies religious ministers employed by the
right to collect and instrumentalities armed forces of the Philippines.
d.All of these 6.The following are inherent limitations b,Imposing tax on properties of
34. What is the primary purpose of to the power of taxation except religious institutions which are not
taxation? one.Choose the exception directly and exclusively used for
a.To enforce contribution from its a.Territorialy of taxes religious purposes.
subjects for public purpose b.Legislative in character c.Imposition of license for the sale of
b.To raise revenue- c.For public purpose religious literature.-
c.To achieve economic and social d.Non-appropriation for religious d.Authorizing the President of the
stability purposes.-d Philippines to fix the rates of tariffs or
d.To regulate the conduct of business or 7.That all taxable articles or properties imposts.
profession of the same class shall be taxed at the 13.In order to phase-out a huge deficit,
Multiple choice-Theory:Part 2 same rate underscores the President of the Philippines passed
1.The Constitutional exemption of a.Equality in taxation a law offering all taxpayers with
religious or charitable institutions refers b.Equity of taxation previous tax delinquency to pay a
only to c.Uniformity in taxation- minimum tax in exchange for relief from
tax assessment in the period of exemptions to rice farmers who c.University of Pangasinan, a private
delinquency. Is this a valid exercise of produce the staple food of the propriety educational institution
taxation power? Philippines. Is the new law valid? d.AM Property Holdings, a registered
a.Yes, because the measure adopted is a.Yes, since there is a valid classification property development company.-
grounded upon necessity of the taxpayers who would be 21.Tax exemption bills are approved by
b.Yes, because the President is merely exempted from tax.- a.Majority of all members of Congress
exercising his presidential discretion b.Yes, since sugar is more important b.Solely by the President of the
c.No, because the power of taxation is than rice. Republic
non-delegated.- c.No, since the grant of exemption is c.2/3 of all members of Congress
d.No, because only the Department of construed in favor of taxpayers. d.Majority of the representatives
Finance can issue such ruling. d.No, because there is no uniformity in constituting a quorum
14.Concerned with increasing the grant of tax exemption. -a
unemployment rates in the country, the 17.Congress passed a law subjecting 22.The Japanese government invested
president of the Philippines encouraged government-owned and controlled P100,000,000 in a Philippine local bank
the Philippine Senate to pass a law corporations(GOCCs) to income tax. Is and earned P10,000,000 interest.
granting special tax privileges to foreign the law valid? Which is correct?
investors who will establish businesses a.Yes, because all government agencies a.The income is exempt on grounds of
in the country. The senate accordingly and instrumentalities are subject to tax. territoriality.
drafted the bill and passed to Congress b.Yes, because GOCCs are not b.The income is exempt due to
for approval. government agencies and are international comity--
Is this valid exercise of taxation power? essentially commercial in nature.- c. The income is subject to tax on the
a.Yes, it is the discretion of the c.No, because government agencies are basis of sovereignty
President to adopt any measures he exempt. This would pose a violations of d. The income is subject to tax because
deemed necessary to alleviate poor the equality clause in the Constitution. the income is earned within the
conditions in the country. d.No, because GOCCs are Philippines.
b.Yes, any means beneficial to the constitutionally exempted from paying Multiple choice-Theory:Part 3
public interest should be given taxes. 1.When a legislative body taxes persons
optimum priority. 18.Which of the following is not a and property, rights and privileges
c.Yes, the president’s proposal will have constitutional limitation of the power to under the same taxable category at the
to be finally approved and passes by the tax? same rate, this is referred to as
legislature. The rule on non-delegation a.Non-impairment of obligations or compliance with the constitutional
of taxation would not be violated. contracts limitation of:
d.No.tax bills shall originate from the b.Due process and equal protection of a.Equity
House of Representatives.- the law b.Uniformity---
15.Ram is the only practicing lung c.Non-appropriation for religious c.Due process
transplant specialist in Baguio City. The purposes d.Equal protection clause
City Government of Baguio passed a d.Non-delegation of police power- 2.Which is not a legislative act?
local ordinance subjecting the practice 19.The Philippine enacted a law a.Assessment of the tax--
of lung transplant to 2% tax based on requiring foreign banks to withhold b.Setting the amount of the tax
receipts. Ram objected claiming that taxes earned by Filipino residents in c.Determination of the subject of the
other transplant specialists in other their country and to remit the same to tax
regions of the country are not subjected the Philippine Government. Is this a d.Determining the purpose of the tax
to tax. valid exercise of taxation power? 3.The inherent powers of the state are
Is Ram’s contention valid? a.Yes, because foreign banks are within similar in the following respect. except:
a.Yes, because the rule of taxation the territorial jurisdiction of the a.They are inherent to the existence of
should be uniform and equitably Philippines. the state.
enforced. b.Yes, The Philippines can enforce tax b.They are exercisable without the need
b.Yes, because Ram is the only one requirements to subjects of foreign for an express constitutional grant.
subject. Other practitioners who would sovereignty even if they are outside the c.All are not exercised by private
later practice would not be covered by country, entities.-
the ordninance. c.No, as this leads to encroachment of d.They are exercised primarily by the
c.No, because the ordinance would foreign sovereignty legislature.
cover all transplant specialist who c.No, this is prohibited by the c
would practice in Baguio City. The Constitution. 4.Which is mandatorily observed in
uniformity rule would not be violated. C implementing police power?
d.No, because subjecting the new 20.Which of the following normally pays a.Public interest
industry to taxation would hamper real property tax? b.Just compensation
economic growth. a.Bantay Bata, a non-profit institution c.Public use
16.With the country under incessant b.Jesus Crusade movement, a religious d.All of these—
shortage of sugar, the Philippine institution 5.Which is considered in the exercise of
Congress enacted a law providing tax eminent domain?
a.Public use d,neither statement 1 nor 2 purpose of raising revenue is an invalid
b.Just compensation 13.Which of the state affects the least tax law.
c.Both a and b number of people? Which is incorrect?
d.Nether a nor b a.police power a.Statement 1
--c b.eminent domain--d b.statement 2
6.The general power to enact laws to c.taxation c.Both statements
protect the well-being of the people is d.taxation and police power d.Neither statement
called 14.Select the correct statement. d
a.police power-a. a.The benefit received theory explains 19.Select the correct statement
b.eminent domain that the government is obliged to serve a.The provisions on taxation in the
c.taxation the people since it is benefiting from Philippines Constitution are grants of
d.all of these the tax collection from its subject. the power to tax.
7.Which of the following entities will b.The lifeblood theory underscores that b.The power to tax includes the power
least likely exercise the power of taxation is the most superior power of to destroy
eminent domain? the state. c.When taxation is used as a tool for
a.Electric cooperatives c.The police power of the state is general and economic welfare, this is
b.water cooperatives superior to the non-impairment clause called fiscal purpose.
c.telecommunication business of the constitution.c d.The sumptuary purpose of taxation is
d.transportation operators.d d.The power of taxation is superior to to raise funds for the government.
8.In exercising taxation, the government the non-impairment caluse of the B.
need not consider constitution. 20.Which of the following powers is
a.Inherent limitations 15.Which of the following is not inherent or co-existent with the
b.just compensation exercised by the government? creation of the government?
c.due pricess of law a.Taxation a.Police power
d.constitutional limitations—d b.Police power b.Eminent domain
9.Licensing of business or profession is c.eminent domain c.Taxation
an exercise of d.exploitation d.all of these
a.police power- d D
b.taxation 16.Select the incorrect statements 21.Which of the following is not an
c.eminent domain a.Since thre is compensation, eminent inherent limitation of the power to tax?
d.all of these domain raises money for the a.Tax should be levied for public
10.Select the correct statement government purpose
a.Eminent domain refers to the power b.Once a government is established, b.Taxation is limited to its territorial
to take public property for private use taxation is exercisable jurisdiction
after paying just compensation c.The most important of the power of c.Tax laws shall be uniform and
b.Police power being the most superior taxation equitable.
power of the state is not subject to any d.Police power is more superior than d.Government agencies and
limitation.- the non-impairment clause of the instrumentalities are exempt from tax
C.Taxation power shall be exercised by Constitution c
Congress even without an express a. 22.Select the incorrect statement
Constitutional grant.C 17.The following statements reflect the a.The power to tax includes the power
d.Taxes may be collected even in the differences among the inherent powers to exempt
absence of a laws since obligation except: b.Exemption is construed against the
arising from law is always presumed. a.The property taken under eminent taxpayer and in favor of the
11.Which is principally limited by the domain and taxation are preserved but government.
requirement of due process? that of police power is destroyed. c.Tax statutes are construed against the
a.Eminent domain b.Eminent domain and police power do government in case of doubt
b.police power-- not require Constitutional grant, but d.Taxes should be collected only for
c.taxation taxation, being a formidable power, public improvement.d
d.all of these requires constitutional grant.-b 23.Which of the following is not a
12.Statement 1: Congress can exercise c.Only eminent domain can be constitutional limitation of the power of
the power of taxation even without exercised by private entities. tax?
Constitutional delegation of the power d.Taxation, police power, and eminent a.Non-impairment of obligation or
to tax domain are ways which the government contracts
Statement 2:Only the legislature can interferes with private right and b.Due process and equal protection of
exercise the power of taxation, eminent property. the law
domain, and police power. 18.Statement 1:The taxation power can c.Non-appropriation for religious
Which statement is correct? be used to destroy if the law is valid. purposes.
a.Statement 1 Statement 2:A tax law which destroys d.Non-delegation of the taxing power.
b.statement 2 things, business, or enterprises for the d
c.statements 1 and 2
24.Which of powers of the state is the 1.Constitution issued by the BIR and other agencies or
most superior? Which is regarded as 2.Statues and Presidential decrees offices.
the most important? 3.Judicial decisions or case laws Revenue Bulletins(RB) refer to periodic
a.Taxation;Eminent domain 4.Executive orders and batas pambansa issuances, notices and official
b.Police power; taxation 5.Administrative issuances announcements of the Commissioner of
c.Eminent domain;police power 6.Local ordinances Internal Revenue that consolidate the
d.All the powers are equally superior 7.Tax treaties and conventions with Bureau of Internal Revenue’s position
and important foreign countries on certain specific issues of law or
b 8.Revenue Regulations administration in relation to the
CHAPTER 2 Types of Administrative Issuances provisions of the Tax code, relevant tax
TAXES, TAX LAWS, AND TAX 1.Revenue regulations laws, and other issuances for the
ADMINSTRATION 2.Revenue memorandum orders guidance of the public.
Chapter Overview and Objectives 3.Revenue memorandum rulings BIR Rulings are official positions of the
This chapter discusses tax laws, taxes, 4.Revenue memorandum circulars Bureau to queries raised by taxpayers
and their distinction from similar items, 5.Revenue bulletins and other stakeholders relative to
and the administration of the tax 6.BIR rulings clarification and interpretation of tax
system. Revenue regulations are issuances laws.
After this chapter, readers are expected signed by the secretary of finance upon Rulings are merely advisory or a sort of
to comprehend and demonstrate recommendations of the Commissioner information service to the taxpayer
knowledge on the following: of Internal Revenue(CIR) that specify, such that none of them is binding
a.The type of taxation laws prescribe, or define rules and except to the addressee and may be
b.Distinction among tax laws, revenue regulations for the effective reversed by the BIR at anytime.
regulations, and rulings enforcement of the provisions of the Types of rulings
c.tax, its elements, and classifications National Internal Revenue Code(NIRC) 1.Value added tax(VAT) rulings
d.Distinction of tax from similar items and related statutes. 2.International Tax affairs division (ITAD)
e.Tax system and its types Revenue regulations are formal rulings
f.The principles of a sound tax system pronouncements intended to clarify or 3.BIR rulings
g.How tax is administered explain the tax law and carry into effect 4,Delegated Authority (DA) rulings
h.The powers of the Bureau of Internal its general provisions by providing Generally Accepted Accounting
Revenue(BIR) and the Commissioner of details of administration and procedure. Principles (GAAP) vs. Tax laws
Internal Revenue(CIR) and the non- Revenue regulation has the force and Generally accepted accounting
delegated powers of the CIR. effect of a law, but it is not intended to principles or GAAP are not laws, but are
9.The criteria for selection of large expand or limit the application of the mere conventions of financial reporting.
taxpayers law; otherwise, it is void. They are benchmarks for the fair and
TAXATION LAW Revenue Memorandum relevant valuation and recognition of
Taxation law refers to any law that Orders(RMOs)are issuances that income, expenses, assets, liabilities, and
arises from the exercise of the taxation provide directives or equity of a valuation and recognition of
power of the state. instructions;prescribe guidelines; and income, expenses, assets, liabilities and
TYPES OF TAXATION LAWS outline processes, operations, activities, equity of a reporting entity for general
1.Tax laws refers to any law that arises workflows, methods, and procedures purpose financial reporting. GAAP
from the exercise of the taxation power necessary in the implementation of accounting reports are intended to
Types of taxation power stated policies, goals, objectives, and meet the common needs of a vast
1.Tax laws-These are laws that provide programs of the Bureau in all areas of number of users in the general public.
for the assessment and collection of operations except auditing. Tax laws including rules, regulations,
taxes. Revenue Memorandum and rulings prescribe the criteria for tax
Examples: Rulings(RMRs)are rulings, opinions and reporting, a special form of financial
a.The National Internal Revenue Code interpretations of the CIR with respect reporting which is intended to meet
(NIRC) to the provisions of the Tax code and specific needs of tax authorities.
b.The tariff and Customs Code other tax laws applied to a specific set Taxpayers normally follow GAAP in
c.The local tax code of facts, with or without established recording transactions in their books.
d.The real property tax code precedents, and which the CIR may However, in the preparation and filing
2.Tax exemption laws-These are laws issue from time to time for the purpose of tax returns, taxpayers are mandated
that grant immunity from taxation of providing taxpayers guidance on the to follow the tax law in cases of conflict
Examples: tax issued RMRs; otherwise, the Rulings with GAAP.
a.The minimum wage law are null and void ab initio. NATURE OF PHILIPPINE TAX LAW
b.The Omnibus Investment Code of Revenue Memorandum Philippine tax laws are civil and not
1987(E.O 226) Circulars(RMCs)are issuances that political in nature. They are effective
c.Barangay Micro-Business Enterprise publish pertinent and applicable even during periods of enemy
Law(BMBE) portions as well as amplifications of occupation. They are laws of the
d.Cooperative Development Act laws, rules, regulations, and precedents occupied territory and not by the
Sources of taxation laws occupying enemy. Tax payments made
during occupations of foreign enemies 1.Specific tax-a tax of a fixed amount e.Tax on banks and other financial
are valid. imposed on a per nit basis such as per institutions
Our internal revenue laws are not penal kilo, liter or meter, etc. DISTINCTION OF TAXES WITH SIMILAR
in nature because they do not define 2.Ad valorem-a tax of a fixed proportion ITEMS
crime. Their penalty provisions are imposed upon the value of the object. Tax vs. Revenue
merely intended to secure taxpayer’s E.As to rate- Tax refers to the amount imposed by
compliance. 1.Proportional tax-this is a flat or fixed the government for public purpose.
TAX rate tax. The use of proportional tax Revenue refers to all income collections
Tax is an enforced proportional emphasizes equality as it subjects all of the government which includes
contribution levied by the lawmaking taxpayers with the same rate without taxes, tariff, licenses, toll, penalties and
body of the State to raise revenue for regard to their ability to pay. others. The amount imposed is tax but
public purpose. 2.Progressive or graduated tax-This is a the amount collected is revenue.
Elements of a valid tax tax which imposes increasing rates as Tax vs. License fee
1.Tax must be levied by the taxing the tax base increases. The use of Tax is broader subject than license. Tax
power having jurisdiction over the progressive tax rates results in equitable emanates from taxation power and is
object of taxation taxation because it gets more tax to imposed upon any object such as
2.Tax must be uniform and equitable those who are more capable. It aids in persons, properties, or privileges to
3.Tax must not violate constitutional lessening the gap between the rich and raise revenue.
and inherent limitations. the poor. License fee emanates from police
4.Tax must be for public purpose. 3.Regressive tax-This tax imposes power and is imposed to regulate the
5.Tax must be proportional in character. decreasing tax rates as the tax base exercise of a privilege such as
6.Tax is generally payable in money. increase.This is total reverse of commencement of a business or a
Classification of taxes progressive tax. Regressive tax regarded profession.
A.As to purpose as anti-poor. It directly violates the Taxes are imposed after
1.Fiscal or revenue tax-a tax imposed Constitutional guarantee progressive commencement of a business or
for general purpose taxation. profession whereas license fee is
2.Regulatory- a tax imposed to regulate 4.Mixed tax-this tax manifest tax rates imposed before engagement in those
business, conduct, acts or transactions. which is a combination of any of the activities. In other words, tax is a post-
3.Sumptuary-a tax levied to achieve above types of tax. activity imposition whereas license is a
some social or economic objectives. F.As to imposing authority pre-activity imposition.
B.As to subject matter 1.National tax-tax imposed by the Tax vs. Toll
1.Personal, poll or capitation-a tax on national government Tax is a levy of government; hence, it is
persons who are residents of a Examples: a demand of sovereignty. Toll is a
particular territory. a.Income tax-tax on annual income, charge for the use of other’s property;
2.Property tax-a tax on properties, real gains or profits hence, it is a demand of ownership.
or personal b.Estate tax-tax on gratuitous transfer The amount of tax depends upon the
3.Excise or privilege tax-a tax imposed of properties by a decedent upon death needs of the government, but the
upon the performance of an act, c.Donor’s tax-tax on gratuitous tax amount of toll is dependent upon the
enjoyment of a privilege or engagement collected by VAT business taxpayers. value of the property leased.
in an occupation. e.Other percentage tax-consumption Both the government and private
C.as to incidence tax collected by non-VAT business entities impose toll, but private entities
1.Direct tax-When both the impact and taxpayers cannot impose taxes.
incidence of taxation rest upon the f.Excise tax-tax on sin products and non- Tax vs. Debt
same taxpayer, the tax is said to be essential commodities such as alcohol, Tax arises from law while debt arises
direct. The tax is collected from the cigarettes and metallic minerals. This from private contracts. Non-payment of
person who is intended to pay the should be differentiated with the tax leads to imprisonment, but non-
same. The statutory taxpayer is the privilege tax which is also called excise payment of debt does not lead to
economic taxpayer. tax. imprisonment. Debt can be subject to
2.Indirect tax-when the tax basis is paid g.Documentary stamp tax-a tax on set-off but tax is not. Debt can be paid
by any person other than the one who documents, instruments, loan in kind(dacion en pago) but tax is
is intended to pay the same, the tax is agreements and papers evidencing the generally payable in money.
said to be indirect. This occurs in the acceptance, assignment, sale or transfer Tax draws interest only when the
case of business taxes where the of an obligation, right, or property taxpayer is delinquent. Debt draws
statutory taxpayer is not the economic incident thereto. interest when it is so stipulated by the
taxpayer. 2.Local tax- tax imposed by the contracting parties or when the debtor
The statutory taxpayer is the person municipal or local government. incurs a legal delay.
named by law to pay the tax. An Examples: Tax vs. Special Assessment
economic taxpayer is the one who a.Real property tax Tax is an amount imposed upon
actually pays the tax. b.Professional tax persons, properties, or privileges.
D.As to amount c.Business taxes, fees, and charges. Special assessment is levied by the
d.Community tax government on lands adjacent to a
public improvement. It is imposed on 2.Regressive system due after such credit or claim refund or
land only and is intended to A regressive system is one that tax credit for excess tax withheld.
compensate the government for a part emphasizes indirect taxes. Indirect taxes 3.Assessment or enforcement system-
of the cost of the improvement. are shifted by businesses to consumers; Under this collection system, the
The basis of special assessment if the hence, the impact of taxation rests government identifies non-compliant
benefits in terms of the appreciation in upon the bottom end of the society. In taxpayers, assesses their tax dues and
land value caused by the public effect, e regressive tax system is anti- penalties, and enforces collections by
improvement. On the other hand, tax is poor. coercive means such as summary
levied without expectation of a direct It is widely believed that despite the proceeding or judicial proceedings
proximate benefit. Constitutional guarantee of a when necessary.
Unlike taxes, special assessment progressive taxation, the Philippine has PRINCIPLES OF A SOUND TAX SYSTEM
attaches to the land. It will not become a dominantly regressive tax system due According to Adam Smith, governments
a personal obligation of the land owner. to the prevalence of business taxes. should adhere to certain principles or
Therefore. the non-payment of special TAX COLLECTION SYSTEMS canons to evolve a sound tax system:
assessment will not result to 1.Withholding system-Under this a.Fiscal adequacy
imprisonment of the owner (unlike in collection system, the payor of the b.Theoretica justice
non-payment of taxes). income withholds and remits the same c.Administrative fesibility
Tax vs. Tariff to the government. The following are Fiscal adequacy
Tax is broader than tariff. Tax is an the withholding taxes collected under Fiscal adequacy requires that the
amount imposed upon persons, this system: sources of government funds must be
privileges, transactions, or properties. a.Withholding tax on compensation-a sufficient to cover government costs.
Tariff is the amount imposed on tax withheld by the employer from The government must not incur a
imported or exported commodities. payments of compensation income of deficit. A budget deficit paralyzes the
Tax vs. Penalty employees. government’s ability to deliver the
Tax is an amount imposed for the b.Expanded withholding tax-a essential public services to the people.
support of the government. Penalty is withholding tax prescribed on certain Hence, taxes should increase in
an amount imposed to discourage an income payments and is creditable response to increase in government
act. Penalty may be imposed by both against the income tax due of the payee spending.
the government and private individuals. for the taxable quarter or year in which Theoretical justice
It may arise both from law or contract the particular income was earned. Theoretical justice or equity suggests
whereas tax arises from law. c.Final withholding tax-a kind of that taxation should consider the
TAX SYSTEM withholding tax which is prescribed on taxpayer’s ability to pay. It also suggests
The tax system refers to the methods or certain income payments and is not that the exercise of taxation should not
schemes of imposing, assessing, and creditable against any income tax due be oppressive, unjust, or confiscatory.
collecting taxes. It includes all the tax of the payee for the taxable year. Administrative feasibility
laws and regulations, the means of their d.Withholding tax on government suggests that the tax laws should be
enforcement, and the government payments-the tax withheld by the capable of efficient and effective
offices, bureaus and withholding agents national government agencies and administration to encourage
which are part of the machineries of instrumentalities including government- compliance. Government should make
the government in tax collection. The owned and controlled corporations on it easy for the taxpayer to comply by
Philippine tax system is divided into their payments to taxpayers, suppliers, avoiding administrative bottlenecks and
two: the national tax system and the or payees. reducing compliance costs.
local tax system 2.Voluntary compliance system-Under The following are applications of the
Types of tax systems according to this collection system, the taxpayer principle of administrative feasibility:
imposition himself determines his income, reports 1.E-filing and e-payment of taxes
1.Progressive-employed in the taxation the same through income tax returns 2.Substituted filing system for
of income of individuals, and transfers and pays the tax to the government. employees
of properties by individuals. This system is also referred to as the 3.Final withholding tax on non-resident
2.Proportional-employed in taxation of “self-assessment method”.A portion of aliens or corporations
corporate income and business. the tax due payable herein may have 4.Accreditation of authorized agent
3.Regressive-not employed in the been withheld under the withholding banks in the filing and payment of
Philippines. system, such as: taxes.
Types of tax system according to impact a.withholding tax on compensation by TAX ADMINSTRATION
1.Progressive system compensation earners Tax administration refers to the
A progressive tax system is one that b.Expanded withholding tax by taxpayer management of the tax system. Tax
emphasizes direct taxes. A direct tax engaged in business or exercise of administration of the national tax
cannot be shifted. Hence, it encourages profession system in the Philippines is entrusted to
economic efficiency as it leaves no The taxes withheld are treated as tax the Bureau of Internal Revenue which is
other resort to taxpayers than to be credit( deduction) against the tax due of under the supervision and
efficient. This type of tax system the taxpayer in the income tax return. administration of the Department of
impacts more upon the rich. The taxpayer shall pay any balance still finance.
Chief Officials of the Bureau of Internal the person whose internal revenue tax b.The CIR believes that the books or
Revenue liability is subject to audit. other records of the taxpayer do not
Commissioner c.To summon the person liable for tax or correctly reflect the declaration in the
4 Deputy Commissioners, each to be required to file a return, his employees, return
designated to the following: or any person having possession and The presumptive gross sales or receipt
a.Operations group custody of his books of accounts and shall be derived from the performance
b.Legal Enforcement group accounting records to produce such of similar business under similar
c.Information systems group books, papers, records or other data circumstances adjusted for other
d.Resource management group and to give testimony relevant information.
POWERS OF THE BUREAU OF INTERNAL d.To take testimony of the person 8.To terminate tax period when the
REVENUE concerned, under oath, as may be taxpayer is:
1.Assessment and collection of taxes relevant or material to the inquiry a.Retiring from business
2.Enforcement of all forfeitures, e.To cause revenue officers and b.Intending to leave the Philippines
penalties, and fines, and judgements in employees to make canvass of any c.Intending to remove, hide, or conceal
all cases decided in its favor by courts. revenue district his property
3.Giving effect to, and administering the 4.To make assessment and prescribe d.Intending to perform any act tending
supervisory and police powers additional requirement for to obstruct the proceedings for the
conferred to it by the NIRC and other administration and enforcement collection of the tax or render the same
laws. 5.To examine tax returns and determine ineffective
4.Assignment of internal revenue tax due thereon The termination of the taxable period
officers and other employees to other The CIR or his duly authorized shall be communicated through a
duties. representatives may authorize the notice to the taxpayer together with a
5.Provision and distribution of forms, examination of any taxpayer and the request for immediate payment. Taxes
receipts, certificates, stamps,etc. to assessment of the correct amount of shall be due and payable immediately.
proper officials. tax. Failure to file a return shall not 9.To prescribe real property values
6.Issuance of receipts and clearances prevent the CIR from authorizing the The CIR is authorized to divide the
7.Submission of annual report, examination. Philippines into zones and prescribe
pertinent information to Congress and Tax or deficiency assessments are due real property values after consultation
reports to the Congressional Oversight upon notice and demand by the CIR or with competent appraisers. The values
Committee in matters of taxation his representatives. prescribed are referred to as zonal
POWERS OF THE COMMISSIONER OF Returns, statements or declarations value.
INTERNAL REVENUE shall not be withdrawn but may be For purposes of internal revenue taxes,
1.To interpret the provisions of the modified, changed amended by the fair value of real property shall mean
NIRC, subject to review by the Secretary taxpayer within 3 years from the date of whichever is higher of:
of Finance filing, except when a notice for audit or a.Zonal value prescribed by the
2.To decide tax cases, subject to the investigation has been actually served Commissioner
exclusive appellate jurisdiction of the upon the taxpayer. b.Assessed value per the Provincial and
Court of tax appeals, such as: When a return shall not be forthcoming City Assessor’s Office
a.Disputed assessments within the prescribed deadline or when For purposes of local taxes, fair value of
b.Refunds of internal revenue taxes, there is a reason to believe that the real property pertains to the assessed
fees, or other charges return is false, incomplete or value.
c.Penalties imposed erroneous, the CIR assess the proper 10.To compromise tax liabilities of
d.Other NIRC and special law matters tax on the basis of best evidence taxpayers.
administered by the BIR available. 11.To inquire into bank deposits, only
3.To obtain information and to In case a person fails to file a required under the following instances:
summon, examine, and take testimony return or other documents at the time a.Determination of the gross estate of
of persons to effect tax collection prescribed by law or willfully files a false the decedent
Purpose:For the CIR to ascertain: or fraudulent return or other b.To substantiate the taxpayer’s claim of
a.The correctness of any tax return or in documents, the CIR shall make or financial incapacity to pay tax in an
making a return when none has been amend the return from his own application for tax compromise.
made by the taxpayer knowledge and from such information In cases of financial incapacity, inquiry
b.The tax liability of any person for any obtained from testimony. The return can proceed only if the taxpayer waives
internal revenue tax or in correcting any shall be presumed prima facie correct his privilege under the Bank deposit
such liability and sufficient for all legal purposes. secrecy act.
c.Tax compliance of the taxpayer. 6.To conduct inventory taking or 12.To accredit and register tax agents
Authorized acts: surveillance. The denial by the CIR of application for
a.To examine any book, paper, record or 7.To prescribe presumptive gross sales accreditation is appealable to the
other data relevant to such inquiry and receipts for a taxpayer when: Department of Finance. The failure of
b.To obtain on a regular basis any a.the taxpayer failed to issue receipts; the secretary of finance to act on the
information from any person other than or appeal within 60 days is deemed an
approval.
13.To refund or credit internal revenue 2.The head of appropriate government Local Government Tax Collecting Units
taxes offices and his subordinates with Provinces, municipalities, cities and
14.To abate or cancel tax liabilities in respect to the collection of agency tax. barangays also imposed and collect
certain cases 3. Banks duly accredited by the various taxes to rationalize their fiscal
15.To prescribe additional procedures Commissioner with respect to receipts autonomy. These will be discussed
or documentary requirements of payments of Internal revenue taxes under business and transfer taxation by
16.To delegate his powers to any authorized to be made thru banks. the same author.
subordinate officer with a rank These are referred to as authorized TAXPAYER CLASSIFICATION FOR
equivalent to division chief of an office. government depository banks (AGDB). PURPOSES OF TAX ADMINISTRATION
Non-delegated power of the CIR OTHER AGENTS TASKED WITH TAX For purposes of effective and efficient
The following powers of the COLLECTIONS OR TAX INCENTIVES tax administration, taxpayers are
Commissioner shall not be delegated: RELATED FUNCTIONS classified into large and non-large. Large
1.The power to recommend the 1.Bureau of Customs taxpayers are under the supervision of
promulgation of rules and regulations 2.Board of Investments the Large taxpayers service(LTS) of the
to the secretary of finance 3.Philippines Economic Zone Authority BIR. Non-Large taxpayers are under the
2.The power to issue rulings of first 4.Local Government Tax Collecting Unit supervision of the respective Revenue
impression or to reverse, revoke or Bureau of Customs(BOC) District Officers (RDOs) where the
modify any existing rulings of the Aside from its regulatory functions, the business, trade, or profession of the
Bureau bureau of Customs is tasked to taxpayer is situated.
3.The power to compromise or abate administer collection of tariffs on The following are the criteria for
any tax liability imported articles and collection of the determining large taxpayers:
Exceptionally, the regional evaluation Value Added Tax on importation. A.As to payment
boards may compromise tax liabilities Together with the BIR, the BOC is under 1.Value added tax-at least P200,000 per
under the following: the supervision of the Department of quarter for the preceding year
a.assessments are issued by the Finance. 2.Excise tax-At least P1,000,000 tax paid
regional offices involving basic The Bureau of Customs is headed by for the preceding year
deficiency tax of P500,000 or less, and the Customs Commissioner and assisted 3.Income tax-At least P1,000,000
b.minor criminal vioaltions discovered by five Deputy Commissioners and 14 annual income tax paid for the
by regional and district officials. District Collectors. preceding year.
Composition of the Regional Evaluation Board of Investments (BOI) 4.Withholding tax-At least P1,000,000
Board The BOI is tasked to lead the promotion annual withholding tax payments or
a.Regional Director as chairman of investments in the Philippines by remittances from all types of
b.Assistant Regional Director assisting Filipino investors to venture withholding taxes
c.Heads of the Legal, Assessment and and prosper in desirable areas of 5.Percentage tax-At least P200,000
Collection Division economic activities. It supervises the percentage tax paid or payable per
4.The power to assign and resign grant of tax incentives under the quarter for the preceding year
internal revenue officers to Omnibus Investment Code. The BOI is 6.Documentary stamp tax-At least
establishments where articles subject an attached agency of the Department P1,000,000 aggregate amount per year
to excise tax are produced or kept. of Trade and Industry (DTI). As to financial conditions and results of
Rules in assignments of revenue officers The BOI is composed of five full-time operations
to other duties governors, excluding the DTI secretary 1.Gross receipts or sales-
1.Revenue officers assigned to an as its chairman. The president of the P1,000,000,000 total annual gross sales
establishment where excisable articles Philippines shall appoint a vice or receipts
are kept shall in no case stay there for chairman of the board who shall act as 2.Net worth-P300,000,000 total net
more than 2 years. the BOI’s managing head. worth at the close of each calendar or
2.Revenue officers assigned to perform Philippine Economic Zone Authority fiscal year
assessment and collection function shall (PEZA) 3.Gross purchases-P800,000,000 total
not remain in the same assignment for The PEZA is created to promote annual purchases for the preceding year
more than 3 years. investments in export-oriented 4.Top corporate taxpayer listed and
3.Assignment of internal revenue manufacturing industries in the published by the Securities and
officers and employees of the Bureau to Philippines and, among other myriads Exchange Commission
special duties shall not exceed 1 year. of functions, supervise the grant of Automatic classification of taxpayer as
Agents and Deputies for collection of both fiscal and non-fiscal incentives. large taxpayers
National Internal Revenue Taxes PEZA registered enterprises enjoy tax The following taxpayers shall be
The following are constituted agents for holidays for certain years, exempt from automatically classified as large
the collection of internal revenue taxes: import and export taxes including local taxpayers upon notice in writing by the
1.The commissioner of Customs and his taxes. The PEZA is also an attached CIR:
subordinates with respect to collection agency of the DTI. 1.All branches of taxpayers under the
of national internal revenue taxes on The PEZA is headed by a director Large Taxpayer’s Service
imported goods. general and is assisted by three deputy
directors.
2.Subsidiaries, affiliates, and entities of 4.Tax collected upon persons who are 3.Income tax
conglomerates or group of companies not the stator taxpayers-indirect tax 1,000,000 per year
of a large taxpayer 5.Tax that is imposed based on the 4.Withholding tax
3.Surviving company in case of merger value of the tax object-ad valorem 1,000,000 per year
of consolidation of a large taxpayer 6. Tax for general purpose- 5.Percentage tax
4.A corporation that absorbs the fiscal/revenue tax 200,000 per quarter
operation or business in case of spin-off 7.Tax imposed by the national 6.Documentary stamp tax
of any large taxpayer government-national tax 1,000,000 per year
5.Corporation with an authorizes 8.A tax on sin products or non-essential As to condition and operations
capitalization of at least P300,000,000 commodities-excise tax 1.Gross receipts or sales
registered with the SEC 9.Imposed on the gratuitous transfer of 1,000,000 in a year
6.Multinational enterprises with an property upon death-estate tax 2.Net worth
authorized capitalization or assigned 10.Tax on residents of a country- 300,000 at year end
capital of at least P300,000,000 personal, poll, or capitation 3.Gross purchases
7.Publicly listed Corporations 11.Tax that remains at flat rate 800,000 in the preceding year
8.Universal, commercial, and foreign regardless of the value of the tax Multiple Choice-Theory:Part 1
banks(the regular business unit and object-proportional tax 1.When tax is collected upon someone
foreign currency deposit unit shall be 12.Tax which is collected on a per unit who is effectively reimbursed by
considered one taxpayer for purposes basis-specific tax another, the tax is regarded as
of classifying them as large taxpayer) 13.Tax is collected upon the statutory a.direct
9.Corporate taxpayers with at least taxpayer-direct tax b.indirect=
P100,000,000 authorizes capital in 14.Tax imposed to regulate business or c.personal
banking, insurance, telecommunication, profession-regulatory d.illegal
utilities, petroleum, tobacco, and 15.Tax upon performance of an act or 2.All are ad valorem taxes, except one,
alcohol industries enjoyment of a privilege- Select the exception
10.Corporate taxpayers engaged in the excise/privilege tax a.poll tax==
production of metallic minerals. Exercise Drill No.2 b.estate tax
CHAPTER 2:SELF-TEST EXERCISES Identify which item is described by the c.Real property tax
Discussion Questions following: d.Capital gains tax on real property
1.Distinguish tax law from tax 1.It refers to all income collections of capital asset.
exemption law. the government-Revenue 3.Taxation power can be used to destroy
2.Enumerate the sources of tax laws 2.It is an imposition for the support of a.as a revenue measure
3.Discuss the nature of Philippines tax the government-Tax b.even is the tax is invalid
law. 3.It is imposed upon land adjacent to c.as an implement of police power==
4.Distinguish tax laws, revenue public improvements-special d.when the state is in dire need of funds
regulations, and rulings assessment 4.Which is not a characteristic of tax?
5.Define tax and identify its elements 4.It is imposed on imported and a.It is an enforced contribution
6.What are the classifications of taxes? exported commodities-custom’s duties b.It is generally payable in money
Enumerate and provide examples for 5.It is a charge imposed prior to the c.It is subject to assignment==
each classification commencement of business or exercise d.It is levied by the law-making body of
7.Compare tax with revenue, license, of a profession-license fees the state having jurisdiction
toll, debt, special assessment, tariff and 6.It is a post-activity rather than a pre- 5.Which of the following is a local tax?
penalty activity imposition-Tax a.Value added tax
8.What is a tax system?What are its 7.It is subject to compensation or set- b.Real property tax==
types? off-Tax debt c.Documentary tax
9.Enumerate the principles of a sound 8.It is a charge for the use of others’ d.Other percentage tax
tax system. Explain each property-toll 6.Which is not a source of tax law?
10.Enumerate the powers of the BIR 9.It is an imposition intended to a.CHED regulations==
11.Enumerate the non-delegated discourage an act.-penalty b.BIR rulings
powers of the CIR 10.It arises from contracts rather than c.Judicial decisions
Exercise Drill No.1 from law-debt d.Other percentage taxes
Identify the type of tax that is describe Exercise Drill No. 3 7.Tax as to purpose is classified as
by the following: Indicate the criteria for the selection of a.Fiscal or regulatory=
1.A consumption tax collected by non- large taxpayer for each of the following: b.Direct or indirect tax
VAT businesses-other percentage tax As to payment c.National or local tax
2.Tax on gratuitous transfer of property Criteria d.Specific or ad valorem tax
by a living donor-donor’s tax 1.Value added tax 8.Tax as to incidence is classified as
3.Tax that decreases in rates as the 200,000 per year a.Fiscal or regulatory
amount or value of the tax object 2.Excise tax b.Direct or indirect tax
increases-regressive tax 1,000,000 per year c.National or local tax=
d.Specific or ad valorem tax
9.Tax as to source is classified as 19.Tax classifications as to object do not 27.Mr. A has a tax obligation to the
a.Fiscal or regulatory include government amounting to P80,000.
b.Direct or indirect tax a.Poll tax Since he is leaving the country, he
c.National or local tax= b.Property tax entered into a contract with Mr. B
d.Specific or ad vaorem tax c.Regulatory tax= wherein Mr. B shall pay the P80,000 tax
10.Which is not a measure of tax? d.Excise tax in his behalf. On due date, Mr. B failed
a.Enforced proportional contribution 20.A. Tax must be not violate to pay the tax. The BIR sent a letter of
b.Enforced within the territorial constitutional and inherent limitation demand to Mr. A which he refused to
jurisdiction of the taxing authority B.Tax must be uniform and equitable pay.
c.Levied by the lawmaking body C.Tax must be for public purpose Which of the following statements is
d.Generally payable in kind= D.Tax must be levied by the lawmaking correct?
11.Taxes that cannot be shifted by the body a.The government cannot enforce
statutory taxpayer are referred to as E.Tax must be proportionate in collection charges against Mr. A since he
a.direct tax= character has validly transferred his obligations to
b.insirect tax F.Tax is generally payable in money B under the contract.
c.business tax Which of the above is/are not an b.The government can no longer run
d.personal taxes essential characteristic of a valid tax? after Mr. A because he is already
12.Which is a local tax? a.All of the above outside the Philippine territory.
a.Donor’s tax b.All except F c.The government should wait until Mr.
b.Professional tax= c.None, except F B becomes solvent again.
c.Documentary stamp tax d.None of the above= d.The government should force Mr. A to
d.Excise tax 21.To limit the production of an pay because taxes are non-assignable.=
13.As to subject matter, taxes do not environmentally harmful commodity, 28.Philippine tax laws are by nature,
include Congress passed a law subjecting the a.political
a.Property tax sales of an environmentally unfriendly b.civil=
b.Regulatory tax= commodity to a P10/kilo tax but a 5% c.political and civil
c.Poll tax tax is imposed on sales exceeding d.penal and civil
d.Excise tax P100,000. 29.Motor vehicles tax is an example of
14.A tax that is imposed upon the Which is incorrect? a.Property tax
performance of an act, the enjoyment a.The tax is a combination of an ad b.Privilege tax=
of a privilege or the engagement in a valorem tax and specific tax c.Income tax
profession is known as b.This is an example of a regulatory tax d.Indirect tax
a.income tax c.This is a national tax 30.Which of the following statements is
b.license d.This is a local tax correct?
c.excise tax= 22.Which is not an excise tax? a.The Marchall Doctrine is not used in
d.transfer tax a.Income tax practice since it is unconstitutional.
15.Which is a national tax? b.Community tax= b.An ex post facto tax law violates the
a.Real property tax c.Estate tax constitution
b.Community tax d.Occupation tax c.A tax bill personally drafted by the
c.Income tax= 23.Which is an indirect tax? president shall become a law after
d.Professional tax a.Value added tax= approval by congress.=
16.Which of the following distinguishes b.Donor’s tax d.It is in the public interest that errors
license from tax? c.Income tax of public officials should bind the
a.Unlimited in imposition d.Donor’s tax government to limit government abuse.
b.Imposed for revenue 24.Which is not an ad valorem tax? 31.Tax rulings are issued by the
c.Does not renders business illegal a.Real property tax a.Secretary of finance
d.Pre-activity in application= b.Excise tax on cigar= b.Supreme court
17.Which is correct? c.Income tax c.Court of appeals
a.Taxes may be subject to compensation d.Donor’s tax d.Commissioner of Internal Revenue=
b.Toll, being a demand of ownership, is 25.A tax that is imposed based on a per 32.Which of the following is limited in
exercised only by private entities. unit or per head basis known as application?
c.Dacion en pago and cession in a.Proportional tax a.Tax laws
payment are applicable to taxation. b.Specific tax= b.Revenue regulations
d.Special assessment applies only when c.Ad valorem tax c.Tax treaties
public improvement is made= d.Progressive tax d.BIR ruling=
18.Tax as to determination of amount is 26.Tax as to rates excludes 33.Which is not a source of tax law?
classified as a.Specific tax= a.Judicial decisions
a.Fiscal or regulatory b.Progressive tax b.Revenue regulations
b.Direct or indirect tax c.Mixed tax c.Opinions of tax experts=
c.National or local tax d.Proportional tax d.Tax treaties and ordinances
d.Specific or ad valorem tax= 34.Which issues revenue regulations?
a.Department of finance= 45.Which of the following levy is fiscal b.License
b.Congress or revenue by nature? c.Police power
c.Commissioner of Internal Revenue a.Tax law geared to phase out a deficit d.Toll
d.Commissioner of Customs balance of the government 9.Toll exhibits all of the following
35.Which is not an element of tax? b.Tax law intended to prohibit gambling characteristic, except one. Which is the
a.It must be for public purpose in the Philippines exception?
b.It must not violate Constitutional or c.Tax law intended to protect local a.Demand of ownership
inherent limitation industries= b.Compensation for the use of
c.It must be progressive by nature d.Tax law supporting the development another’s property
d.It must be uniform and equitable= of a particular industry c.Maby impsed by private individuals
36.Tax as to purpose does not include Multiple choice-Theory:Part 2 d.Levied for the support of the
a.Revenue 1.Which is not an excise tax? government.=
b.Sumptuary= a.Income tax= 10.Which of the following is incorrect?
c.Regulatory b.Business tax a.The collected tax is referred to as
d.Poll c.Personal tax revenue
37.When the impact and incidence of d.Transfer tax b.Tax is the sole source of government
taxation are merged into the statutory 2.Which of the following do not relate revenue=
taxpayer, the tax is known as to tax? c.License is imposed before
a.Personal tax a.Does not render business illegal when commencement of a business or
b.Direct tax not paid profession
c.Indirect tax b.Arises from law rather than from d.Debt can be subject to compensation
d.National tax= contracts or set-off.
38.Tax as to object includes c.Intended to cover cost of regulations= 11.Debt as compared to tax
a. Personal tax d.Intended for public purpose a.It is a demand of ownership
b.Property tax= 3.A levy from a property which derives b.It is not assignable
c.Excise tax some special benefit from public c.It will not cause imprisonment when
d.All of these improvement is not paid.=
39.Which is not an indirect tax? a.special assessment = d.It is generally payable in money.
a.duties b.eminent domain 12.Select the incorrect statement.
b.impost c.taxation a.Tax may be unlimited in amount
c.excise tax d.toll b.Non-payment of license renders the
d.personal tax= 4.A.Government revenue may come business illegal.
40.A tax that cannot be avoided is from tax, license, toll and penalties. c.Special assessment is not a liability of
a.direct tax= B.Penalty may arise either from law or the person owning the property.
b.indirect tax contracts. d.Special assessment can be imposed
c.specific tax Which is false? on building and other real right
d. personal tax a.A only attaching or pertaining to land.=
41.Statement 1:Taxes are voluntary b.B only 13.Tax as to subject matter does not
contributions to the government c.A and B include
Statement 2:Taxes are mandatory d.Neither A nor B= a. Real property tax
contributions to the government 5.What distinguishes tax from license? b.Personal tax
Which is correct? a.Tax is a regulatory measure c.Excise tax
a.Only statement 2 is correct b.Tax is a demand of ownership d.Regulatory tax=
b.Only statement 2 is correct= c.Tax arises from contract 14.What distinguishes debt from tax?
c.Both statements are correct d.Tax is a post-activity imposition= a.Arises from contract=
d.Neither statements are correst 6.Which of the following distinguishes b.Never draws interest
42.Which is an indirect tax? license from tax? c.Non-payment will lead to
a.Other percentage tax= a.It is imposed under taxation power. imprisonment
b.Income tax b.It is a charge for other’s property d.Generally payable in money
c.Donor’s tax c.Non-compliance to it will render Multiple Choice-Theory:Part 3
d.Estate tax business illegal= 1.The Commissioner of Internal
43.Income tax is not a/an d.It is generally payable in money Revenue is not authorized to
a.Ad valorem tax 7.The amount imposed is based on the a.interpret the provisions of the
b.direct tax value of the property National Internal Revenue Code
c.revenue tax a.Eminent domain b.promulgate Revenue Regulations=
d.Property tax= b.License c.terminate an accounting period
44.A transfer tax is not a/an c.Toll= d.prescribe presumptive gross receipts
a.Regressive tax= d.Special assessment 2.Which is not a power of the
b.Ad valorem tax 8.Which is intended to regulate Commissioner of Internal Revenue?
c.National tax conduct? a.To change tax periods of taxpayers.
d.Excise tax a.Penalty= b.To refund internal revenue taxes
c.To prescribe assessed value of real d.Economic consistency c.Interpretation of the provisions of the
properties= NIRC=
d.To inquire into bank deposits only 11.Which among the following powers d.Giving effect to and administering the
under certain cases of the Commissioner of Internal supervisory and police powers
3.The principles of a sound tax system Revenue can be delegated? conferred by the NIRC and other laws.
exclude a.The power to conduct inventory 18.The Commissioner of Internal
a.Economic efficiency-= surveillance Revenue can delegate the power to
b.Fiscal adequacy b.The power to recommend a.refund or credit internal revenue tax=
c.Theoretical justice promulgation of revenue regulations b.recommend rules and regulations to
d.Administrative feasibility c.The power to issue rulings of first the secretary of finance
4.Which of the following best describes impression c.assign and re-assign revenue officer to
the effect of tax condonation? d.The power to reverse a ruling, amend, establishments of excisable articles.
a.It only covers the unpaid balance of a or modify an existing ruling= d.compromise or abate tax liability
tax liability= 12.The Commissioner of Internal 19.The BIR is under the supervision of
b.It is conditional on the taxpayer Revenue is not empowered to a.The Bureau of Customs
paying some portion of the unpaid tax a.Make or amend a tax return for an din b.The president
c.It generally applies to all taxpayers behalf of the taxpayer. c.The department of finance=
d.All of these b.Obtain information and to summon, d.Congress
5.Which is not an application of a examine, and take testimony of persons 20.Who is not a legal taxpayer?
principle of a sound tax system? to effect tax collections.= a.Mining companies
a.Taxes should adjust based on c.Compromise tax liabilities of taxpayers b.Listed companies
government needs d.Grant amnesty for erring taxpayers c.Banks with P120M authorized capital
b.Taxation should be progressive= 13.Which is true with tax amnesty? d.None of these=
c.Taxation should encourage convenient a.It is unconditional 21.In terms of financial measure, which
compliance b.It covers both criminal and civil of the following threshold for
d.None of these liability of the taxpayer= qualification as large taxpayers is
6.By which principle of a sound tax c.It applies for past and future non- incorrect?
system is the elasticity in tax rates is compliance a.Gross receipts exceeding P1B.
justified? d.All of these b.Net worth exceeding P300M.
a.Theoretical justice 14.Which of the following may tax c.Gross purchases exceeding P800M.
b.Fiscal adequacy exemption come from? d.Gross sales exceeding P1.5B.
c.Adminstrative feasibility a.Contract 22.As to tax payments measures, which
d.All of these= b.Constitution of the following threshold for the
7.Violation of this principles will make a c.Law qualification as large taxpayers is
tax law invalid d.All of these incorrect?
a.Fiscal adequacy 15.Exemption based upon which of the a.Annual income tax payments of P1M.
b.Theoretical justice following is repealable? b.Annual value added tax payments of
c.Administrative feasibility a.Contract P1M.=
d.Economic consistency b.Constitution c.Quarterly percentage tax payments of
8.Which of the following is not an c.Law= P200,000
application of the lifeblood doctrine? d.None of these d.Annual documentary stamp tax of
a.The government has the right to 16.Select the incorrect statement P1M.
select the object of taxation regarding tax amnesty and CHAPTER 3
b.Taxation is the rule, exemption is the condonation. INTRODUCTION TO INCOME TAXATION
exception a.In tax amnesty, violators are required Chapter Overview and Objectives
c.Claim for exemption is strictly to pay a portion of the tax assessed. This chapter discusses the concept of
construed against the taxpayer.= b.When the remaining unpaid portion tax income, the situs of income, and the
d.None of these of the tax is condoned, the taxpayer types of taxpayers.
9.Which one of the following is the BIR cannot ask for refund for the balance After this chapter, readers are expected
not empowered to do? already paid. to comprehend and demonstrate
a.Assess national taxes= c.Tax amnesty operates as a general knowledge on the following:
b.Collect income, business and transfer pardon and is rarely available 1.The concept of gross income
taxes d.Tax condonation operates on the 2.The types of income taxpayers
c.Assess and collect local taxes whole balance of the assessed tax; 3.The general rules in income taxation
d.Enforce forfeitures, penalties and hence, the taxpayer can ask for refund 4.The income tax situs rules
fines for the paid portion of the tax.= THE CONCEPT OF INCOME
10.Which principle demands that ax 17.Which of the following is a power of Why is income subject to tax?
should be just, reasonable, and fair? the Commissioner of Internal Revenue? Income is regarded as the best measure
a.Theoretical justice= a.Assessment and collection of taxes of taxpayer’s ability to pay tax. It is an
b.Fiscal adequacy b.Enforcement of all forfeitures, excellent object of taxation in the
c.Administractive feasibility penalties, and fines allocation of government costs.
What is income for taxation purposes? 3.Human reputation The recovery of lost profits through
The tax concept of income is simply Life insurance, indemnity contracts, or legal
referred to as “gross income” under the The value of life is immeasurable by suits constitutes a taxable return on
NIRC. A taxable item of income is money. Under Sec. 32 of the NIRC, the capital.
referred to as an “item of gross income” proceeds of life insurance policies paid The following are taxable recoveries of
or “inclusion in gross income”/ to the heirs or beneficiaries upon death lost profits:
Gross income simply means taxable of the insured, whether in a single sum a.Proceeds of crop or livestock
income in layman’s term. Under the or otherwise, are exempt from income insurance
NIRC however, the term “taxable tax. b.Guarantee payments
income” refers to certain item of gross The proceed of a life insurance contract c.Indemnity received from patent
income less deductions and personal collected by an employer as a infringement suit
exemptions allowable by law. beneficiary from the life insurance of an Illustration 1
Technically, gross income is broader to officer or any person directly interested Mang Tomas insured his strawberry
pertain to any income that can be with his trade are likewise exempt. crop in a P200,000 crop insurance
subjected to income tax. These proceeds are viewed as advanced coverage against calamities. The crop
Gross Income is broadly defined as any recovery of future loss. was eventually destroyed by an unusual
inflow of wealth to the taxpayer from However, the following are taxable frost. Mang Tomas was paid the
whatever source, legal or illegal, that return on capital from insurance P200,000 insurance proceeds.
increases net worth. It includes income policies: The P200,000 proceeds which is a
from employment, trade, business or a.Any excess amount received over reimbursements for the lost value of the
exercise of profession, income from premiums paid by the insured upon future harvest, is an item of gross
properties, and other sources such as surrender or maturity of the policy (i.e, income. The value of the lost crops is, in
dealings in properties and other regular the insured outlives the policy.) effect realized not through actual
or casual transactions. b. Gain realized by the insured from the harvest but through the insurance
ELEMENTS OF GROSS INCOME assignment or sale of his insurance contract.
1.It is a return on capital that increases policy. Illustration 2
net worth. c.Interest income from the unpaid Mr. Santiago purchased a franchise. The
2.It is a realized benefit. balance of the proceeds of the policy franchisor guaranteed an annual
3.It is not exempted by law,contract, or d.Any excess of the proceeds received franchise income of P100,000 to Mr.
treaty. over the acquisition costs and premium Santiago. In the first year of operation,
RETURN ON CAPITAL payments by an assignee of a life Mr. Santiago’s outlet only earned
Capital means any wealth or property. insurance policy. P60,000. The franchisor paid the
Gross Income is a return on wealth or Health P40,000 difference to Mr. Santiago.
property that increases the taxpayer’s Any compensation received in The P40,000 guarantee payment is not
net worth. consideration for the loss of health such a gratuity but a recovery of lost profit
Illustration as compensation for personal injuries or for Mr. Santiago; hence subject to
ABC purchased goods for P300 and sold tortuous act is deemed a return of income tax. Mr. Santiago shall report
them for P500. The P500 consideration capital. P100,000 as franchise income.
can be analyzed as follows: Human Reputation Illustration 3
Selling price (total consideration The value of one’s reputation cannot be Mindoro Inc. experienced an unusual
received)P500 Total return measured financially. Any indemnity decline in its income after a competitor
Cost (value of inventory forgone) received as compensation for its copied its patented invention. Mindoro
300 Return of capital impairment is deemed a return of sued the competitor for patent
Mark-up (Gross income) capital exempt from income tax. infringement and was awarded an
P200 Return on capital Examples include moral damages indemnity of P3,000,000.
The return on capital that increases net received from: The P3,000,000 indemnity is a
worth is income subject to income a.Oral defamation or slander compensation for the income not
tax.Return of capital merely maintains b.Alienation of affection realized by Mindoro due to the patent
net worth; hence; it is not taxable.An c.Breach of promise to marry infringement. The same is an item of
improvement in net worth indicates an Recovery of lost capital vs. Recovery of gross income.
ability to pay tax. lost profits The recovery of lost income or profits is
Capital items deemed with infinite The loss of capital results in decrease in not intended to compensate for the loss
value net worth while the loss of profits does of capital. It is as good as realization of
There are capital items that have not decrease net worth. The recovery of income;hence; it is an item of gross
infinite value and are incapable of lost capital merely maintains net worth income.
pecuniary valuation. Anything received while the recovery of lost profits REALIZED BENEFIT
as compensation for their loss is increases net worth. Therefore, the What is meant by realized benefit?
deemed a return of capital. recovery of lost profits is a return on The “benefit” concept
Examples: capital. The term “benefit” means any form of
1.Life Taxable recovery of lost profits advantage derived by the taxpayer.
2.Health There is benefit when there is an
increase in the net worth of the These are referred to as “gratuitous proprietor should not be taxes since
taxpayer. An increase in net worth transactions” proprietorship businesses are taxable
occurs when one derives income, Under current usage, unilateral upon the same owner. Note that a
donation, or inheritance. transfers are simply referred to as proprietorship business is not a judicial
The following are not benefits, hence, “transfers” while bilateral transfers are entity.
not taxable: called “exchanges”. Benefits derived Benefits in the absence of transfers
a.Receipt of a loan-properties increases from onerous transactions are “earned The increase in wealth of the taxpayer
but obligations also increase resulting in or realized”; hence, they are subject to in the form of appreciation or increase
an offsetting effect in networth. income tax, Benefits derived from in the value of his properties or
b.Discovery of lost properties-under the gratuitous transactions are not realized decrease in the value of his obligations
law, the finder has an obligation to because of the absence of an earning in the absence of a sale or barter
return the same to the owner. process. Benefits derived from transaction is not taxable.
c.Receipt of money or property to be gratuitous transactions are subject to These are referred to as unrealized
held in trust for, or to be remitted to, transfer tax,not income tax. gains or holding gains because they
another person 3.Complex transactions have not yet materialized in an
If the taxpayer is entitled to keep for his Complex transactions are partly exchange transaction.
account portion of a receipt, only that gratuitous and partly onerous. These Examples of unrealized gains or holding
portion is a benefit are commonly referred to as “transfers gains:
Illustration for less than full and adequate a.Increase in value of investments in
1.An employee was granted P20,000 consideration”.The gratuitous portion of equity or debt securities.
transportation advance. He liquidated the transaction is subject to transfer tax b.Increase in value of real properties
P18,000 transportation expenses and while the benefit from the onerous held(revaluation increment)
was allowed by his employer to keep portion is subject to income tax. c.Increase in value of foreign currencies
the P2,000. Only the P2,000 retained by Illustration held or receivable
the employee is considered income A taxpayer sold his car which was d. Decrease in value of foreign currency
since this was the extent he was previously purchased for P100,000 and denominated debt by virtue of
benefited.(RR2-98) with a current fair value of P180,000 for favorable fluctuation in exchange rates.
2.A security agency receives P120,000 only P130,000. e.Birth of animal offsprings, accruals of
from clients, P100,000 of which is for The transaction will be analyzed as fruits in an orchard or growth of farm
the salaries of security guards. Under follows: vegetables.
RMC 39-2007, only the P20,000 Fair value P180,000 f.Increase in value of land due to the
attributable to the agency is considered Selling price 130,000 P50,000- discovery of mineral reserves.
income of the agency since it is the Subject to transfer tax Benefits in the absence of transfers
extent it is benefited. The P100,000 Cost 100,000 P30,000- The increase in wealth of taxpayers in
pertaining to salaries of security guards Subject to income tax the form of appreciation or increase in
is recognized by the agency as a liability The excess of fair value over selling the value of his properties or decrease
upon receipt. price is a gratuity whereas the excess of in the value of his obligations in the
The “realized” concept the selling price over the cost is an item absence of a sale or barter transaction
The term realized means earned. It of gross income. is not taxable.
requires that there is a degree of What is meant by another entity? These are referred to as unrealized
undertaking or sacrifice from the Every person, natural, or juridical, is an gains or holding gains because they
taxpayer to be entitled of the benefit. entity. Natural persons are living have not materialized in an exchange
Requisites of a realized benefit: persons while juridical persons are transaction.
1.There must be an exchange those created by law such as Examples of unrealized gains or holding
transaction partnerships and corporations.An entity gains:
2.The transaction involves another may be taxable entity or an exempt a.Increase in value of investments in
entity entity. A taxable item of gross income equity or debt equities
3.It increases the net worth of the arises from transactions which involve b.Increase invalue of real properties
recipient another natural or juridical entity. held (revaluation increment)
Types of Transfers Gains or income derived between c.increase in value of foreign currencies
1.Bilateral transfers or exchanges, such relatives, corporations, and between a held or receivable
as: partner and the partnership are taxable d.Decrease in value of foreign currency
a.Sale since it is made between a holding or dominated debt by virtue of favorable
b.Barter parent company and its subsidiaries and fluctuation in exchange rates.
These are referred to as “onerous between sister companies regardless of e.Birth of animal offspring, accrual of
transactions”. the underlying economic relationship. fruits in an orchard or growth of farm
2.Unilateral transfers, such as: However, the sales of a home office to vegetables.
a.Succession-transfer of property upon its branch office are not taxable f.Increase in value of land due to the
death because they pertain to one and the discovery of mineral resources.
b.Donation same taxable entity. Furthermore, the Rebdering of services
income between businesses of a
The rendering of services for a Mode of Receipt/ Realization Benefits 8.Income of international missions and
consideration is an exchange but does Taxable items of income may be organizations with income tax immunity
not cause a lost of capital.Hence, the realized by the taxpayer in two ways: Items of gross income tax are exempted
entire consideration received from 1.Actual receipt from taxation are discussed extensively
rendering services such as Actual receipt involves physical taking of under Exclusions in Gross Income in
compensation income or fees is an item the income in the form of cash or Chapter 8.
of gross income. property. TYPES OF INCOME TAXPAYERS
Illustration 2.Constructive receipt A.Individuals
Mr. Saladin lists the following possible Constructive receipt involves no actual 1.Citizen
items of gross income: physical taking of the income but the a.Resident citizen
Compensation income P200,000 taxpayer is effectively benefited. b.non-resident citizen
Winnings from gambling 100,000 Examples: 2.Alien
Increase in value of investments 50,000 a.Offset of debt of the taxpayer in a.Resident alien
Appreciation in the value of land owned consideration for the sale of goods or b.Non-resident alien
300,000 service a.engaged in trade or business
Debt of Saladin cancelled by creditors in b.Deposit of the income to the b.not engage in trade or business
consideration for services he rendered taxpayer’s checking account 3.Taxable estates and trusts
to them 150,000 c.Matured detachable interest coupons B.Corporations
Debt of Saladin cancelled by his creditor on coupon bonds not yet encashed by 1.Domestic corporation
out of affection 250,000 the taxpayer. 2.Foreign corporations
Loan received from a bank 400,000 d.Increase in the capital of a partner a.Resident foreign corporations
The items of gross income are: from the profit of the partnership. b.Non-resident foreign corporation
Compensation income P200,000 Inflow of wealth without increase in net INDIVIDUAL INCOME TAXPAYERS
Winnings from gambling 100,000 worth Citizens
Debt of Saladin forgiven in The inflow of wealth to a person that Under the Constitution, Citizens are:
consideration for service rendered to does not increase his net worth is not a.Those who are citizens of the
his creditors 150,000 income due to the total absence of Philippines at the time of adoption of
Note: benefit. the Constitution on February 2, 1987
1.Gains from gambling and the Examples: b.Those whose fathers or mothers are
forgiveness of debt in consideration of a.Receipt of property in trust citizens of the Philippines
services or properties received are b.Borrowing of money under an c.Those born before January 17,1973 of
realized gains from exchanges. obligation to return Filipino mothers who elected Filipino
2. The forgiveness of debt out of In law, the proceeds of embezzlement citizenship upon reaching the age of the
affection or mere generosity of the or swindling where money is taken majority.
creditor is a gratuitous transfer subject without an original intention to return d.Those who are naturalized in
to transfer tax. are considered as income because of accordance with the law
3.The loan received from a bank the increase in net worth of the Classification of citizens:
constitutes a transfer but is not a swindler. A.Resident Citizen-A Filipino citizen
benefit. NOT EXEMPTED BY LAW, CONTRACT, OR residing in the Philippines
Basis of exemption of unrealized TREATY B.Non-residebt citizen-includes:
income An item of gross income is not 1.A citizen of the Philippines who
Normally, taxpayers will have the ability exempted by the Constitution, law, establishes to the satisfaction of the
to pay tax when their income contract, treaties from taxation. Commissioner the fact of his physical
materializes in an exchange transaction The following items of income are presence abroad with a definite
since tax is generally payable in money. exempted by law from taxation; hence, intention to reside therein;
This does not mean, however, that only they are not considered items of gross 2.A citizen of the Philippines who leaves
income realized in cash is subject to tax. income: the Philippines during the taxable year
Income realized in non-cash a.Income of qualified employee trust to reside abroad, either as an immigrant
properties are, in effect, received in fund of for an employment on a permanent
cash but the taxpayer used the same to b.Revenues of non-profit non-stock basis;
acquire the non-cash property. Income educational institutions 3.A citizen of the Philippines who works
received in non-cash properties are, in 3.PCSO or lotto winnings and derives from abroad and whose
effect, received in cash but the taxpayer 4.SSS,GSIS,Pag-ibig or PhilHealth employment thereat requires him to be
used the same to acquire the non-cash benefits physically present abroad most of the
property. Income received in non-cash 5.Salaries and wages or minimum wage time during the taxable year;
considerations is taxable at the fair earners and qualified senior citizen 4.A citizen who has been previously
value of the property received. 6.Regular income of Barangay Micro- considered as non-resident citizen and
Moreover, exempting income realized in business Enterprises (BMBEs) who arrives in the Philippines at
non-cash considerations would open 7.Income of foreign governments and anytime during the taxable year to
considerations would open a wide foreign government-owned and reside permanently in the Philippines
avenue for tax evasion. controlled corporations shall likewise be treated as a non-
resident citizen for the taxable year in b.Aliens who stayed in the Philippines extrajudicial settlement is taxable to the
which he arrives in the Philippines with for more than 1 year as of the end of heirs.
respect to his income derived from the taxable year are considered resident 2.Trusts
sources abroad until the date of his c.Aliens who are staying in the A trust is an arrangement whereby one
arrival in the Philippines. Philippines for not more than 1 year but person (grantor or trustor) transfers (i.e
Filipinos working in the Philippines more than 180 days are deemed non- donates) property to another person
embassies or Philippine consulate resident engaged in business (beneficiary), which will be held under
offices are not considered non-resident d.Aliens who stayed in the Philippines the management of a third party
citizens. for not more than 180 days are (trustee or fiduciary).
Alien considered non-resident aliens not A trust that is irrevocable designated by
a.Resident alien-an individual who is engaged in trade or business. the grantor is treated in taxation as if it
residing in the Philippines but is not a Illustration 1 is an individual taxpayer. The income of
citizen thereof such as: Luiz Mario Aresmedi, a Mexican actor, the property held in trust is taxable to
1.An alien who lives in the Philippines was contracted by the Philippine the trust. Trusts that are not designated
without definite intention as to his television company to do a project in as revocable by the grantor are not
stay;or the Philippines. He arrived in the properties held under revocable trusts
2.One who comes to the Philippines for country on February 29, 2015 and is taxable to the grantor not to the
a definite purpose which in its nature returned to Mexico three weeks later trust.When the trust agreement is silent
would require an extended stay and to upon completion of the project. as to revocability of the trust, the trust
that end makes his home temporarily in Luiz Mario Aresmedi shall be classified us presumed to be revocable.
the Philippines, although it may be his as an NRA-NETB in 2015. His stay is for CORPORATE INCOME TAXPAYERS
intention at all times to return to his a definite purpose which in its nature The term “corporation” shall include
domicile abroad. will be accomplished immediately. partnerships, no matter how created or
An alien who has acquired residence in Illustration 2 organized, joint-stock companies, joint
the Philippines retains his status as such Mamoud Jibril, a Libyan national arrived accruals, associations, or insurance
until he abandons the same or actually in the country on November 4, 2015. companies, except general professional
departs from the Philippines. Mr. Jibril stayed in the Philippines since partnerships and joint venture or
B.Non-resident alien-an individual who then without any working visa or work consortium formed for the purpose of
is not residing in the Philippines and permit. undertaking construction projects or
who is not a citizen thereof For the year 2015, Mr. Jibril would be engaging in petroleum, coal,
1.Non-resident aliens engaged in considered an NRA-NETB because he geothermal, and other energy
business(NRA-ETB)-aliens who stayed in stayed in the Philippines for less than operations pursuant to an operating
the Philippines for an aggregate period 180 days as of December 31,2015. If he consortium agreement under a service
of more than 180 days during the year. still within the Philippine until contract with the government.
2.Non-resident aliens not engaged in December 31,2015, he will qualify as a Hence, the term corporation includes
business (NRA-NETB)-include: resident alien for 2016. profit-oriented and non-profit
a.Aliens who come to the Philippines Illustration 3 institutions such as charitable
for a definite purpose which in its Without any definite intention as to the institutions, cooperatives, government
nature may be promptly accomplished; nature of his stay, Juan Masipag, a agencies, and instrumentalities,
b.Aliens who shall come to the Filipino citizen, left the Philippines and associations, leagues, civic or religious
Philippines and stay therein for an stayed abroad from March 15,2014 to and other organizations.
aggregate period of not more than 180 April 1,2015 before returning to the Domestic Corporations
days during the year. Philippines. Domestic Corporations is a corporation
THE GENERAL CLASSIFICATION RULE For the year 2014, Juan is still a non- that is organized in accordance with
FOR INDIVIDUALS resident citizen because he is absent for Philippine law.
1.Intention more than 183 days but he will be Foreign Corporation
The intention of the taxpayer regarding classified as resident citizen for the year A foreign corporation is one organized
the nature of his stay within or outside 2015 because he is absent for less than under foreign law.
the Philippines shall determine his 183 days in 2015. Types of foreign corporations:
appropriate residency classification. The Taxable estate and trusts 1.Resident Foreign Corporation(RFC)-a
taxpayer shall submit to the CIR of the 1.Estate foreign corporation which operates and
BIR documentary proofs such as visas, Estate refers to the properties, rights conducts business in the Philippines
work contracts and other documents and obligations of a deceased person through a permanent establishment (i.e
indicating such intention. not extinguished by his death a branch)
2.Lenght of stay Estates under juridical settlement are 2.Non-resident foreign
In default of such documentary proof, treated as individual taxpayers. The corporation(NRFC)-a foreign
the length of stay of the taxpayer is estate is taxable on the income of the corporation which does not operate or
considered: properties left by the decedent.Estates conduct business in the Philippines
a.Citizens staying abroad for a period of under extrajudicial settlement are Note:
at least 183 days are considered non- exempt entities. The income of the 1.A corporation that incorporates in the
resident properties of the estate under Phillippins is a domestic corporation
under the incorporation test even if the All other joint ventures are taxable as Resident citizens and domestic
same is controlled by foreigners. corporations corporations derive most of the
2.A foreign corporation that transacts 3.Co-ownership benefits from the Philippines
business with residents through a A co-ownership is a joint venture government compared to all other
resident branch is taxable on such ownership of a property formed for the classes of taxpayers by virtue of their
transactions as a resident foreign purpose of preserving the same and/ or proximity to the Philippines
corporation through its branch. dividing its income. government citizens and domestic
However, is it transacts directly to A co-ownership that is limited to corporations enjoy preferential privilege
residents outside the branch, it is property preservation or income over aliens. Also, between resident and
taxable as a non-resident foreign collection is not a taxable entity and is non-resident citizens, resident citizens
corporation on the direct transactions. exempt but the co-owners are taxable unarguable enjoy many of the public
Special Corporations on their share on the income of the co- services and privileges because they are
Special Corporations are domestic or owned property. in the country. The taxation of foreign
foreign corporations which are subject However, a co-ownership that reinvests income of resident citizens and
ot special tax rules or preferential tax the income of the co-owned property domestic corporations properly reflects
rates. to other income-producing properties this difference in benefits consistent
OTHER CORPORATE TAXPAYERS or ventures will be considered as with the Benefit received theory.
1.Partnerships unregistered partnership taxable as The extra-territorial tax treatment of
A partnership is a business organization corporation resident citizens and domestic
owned by two or more persons who THE GENERAL RULES IN INCOME corporations is also intended as a safety
contribute their industry o resources to TAXATION net to the potential loss of tax revenues
a common fund for the purpose of Individual taxpayers brought by situs relocation of the
dividing profits from the venture. Taxable on income earned practice of executing or structuring
Types of partnership transactions such that income will be
a.)General professional Within Without realized abroad to avoid Philippines
partnership(GPP) Resident citizen income taxes.
A GPP is a partnership formed for the / / The issue of international double
exercise of a common profession. All Non-resident citizen taxation
partners must belong to the same / The rule on extraterritorial taxation on
profession. Resident alien resident citizens and domestic
A GPP is not treated as a corporation / corporations exposes taxpayers to
and is not a taxable entity. It is exempt Non-resident alien double taxation. However, the NIRC
from income tax, but the partners are / allows a tax credit for taxes paid in
taxable in their individual capacity with Corporate taxpayers foreign countries. In fact, resident
respect to their share in the income of Domestic corporation citizens and domestic corporations pay
the partnership. / / minimal taxes in the Philippines on their
b.)Business partnership Resident foreign corporation foreign income because of the tax
A business partnership is one formed / credit.
for profit. It is taxable as a corporation. Non-resident foreign corporation SITUS OF INCOME
2.Joint Venture / The situs of income is the place of
A joint venture is a business Notes: taxation of income. It is the jurisdiction
undertaking for a particular purpose. It 1.Consistent with the territoriality rule, that has the authority to impose tax
may be organized as a partnership of a all taxpayers except resident citizens upon the income.
corporation. and domestic corporations are taxable Situs of income vs. sources of income
Types of joint venture only on income earned within the Situs of income should be differentiated
a.Exempt joint ventures Philippines. from the source of income. The latter
Exempt joint ventures are those formed 2.The NIRC uses the term “without the pertains to the activity or property that
for the purpose of undertaking Philippines” to mean outside the produces the income.
construction projects or engaging in Philippines. Situs is important in determining
petroleum, coal, geothermal, and other The residency and Citizenship Rule whether or not an income is taxable in
energy operations pursuant to an Taxpayers who are resident and citizens the Philippines. Situs is particularly
operating consortium agreement under of the Philippines such as resident important to taxpayers taxable only on
a service contract with the citizen and domestic corporations are income within, However, it is also
Government. taxable on all income from sources important to taxpayers taxable on
Similar to a GPP, this type of joint within and without the Philippines. A global income for purposes of the
venture is not treated as a corporation corporation is a citizen of the country of computation of the foreign tax credit.
and is tax-exempt on its regular income, incorporation. Thus, a domestic INCOME SITUS RULES
but their venturers are taxable to their corporation is a citizen of the Types of income Place of
share in the net income of the joint Philippines. taxation(situs)
venture. Basis of the extraterritorial taxation 1.Interest income Debtor’s
b.Taxable joint venture residence
2.Royalties Where Dividends on domestic stocks 1.Domestic Corporation-the entire
the tangible is employed 150,000 P400,000 is earned within
3.Rent income The following table summarizes the 2.Non-resident foreign corporation-the
Location of the property situs of the foregoing income: entire P400,000 is earned abroad.
4.Service income Place 3.Resident foreign corporation-The
where the service is rendered Within Without P400,000 dividend shall be split.
Illustration Gain on sale of domestic stocks Gross Income
A taxpayer had the following income: P200,000 Ratio=P600,000/P1,000,000=60%
Interest income from deposits in a Gain on sale of foreign bonds Earned within the Philippines
foreign bank P300,000 P100,000 (60%XP400,000) P240,000
Interest from domestic bonds Gain on sale of commercial lot Earned without the Philippines
50,000 500,000 (40%XP400,000) 160,000
Royalties from books published in the Gain on sale of car in Canada Total dividends
Philippines 100,000 200,000 P400,000
Rent income from properties abroad Gain on sale of machineries Supposing that the ratio is 40%.The
(the lease contracts were executed in 250,000 entire P400,000 will be deemed earned
the Philippines) 150,000 Interest on foreign bonds outside the Philippines.
Professional fees for services rendered 50,000 C.Merchandising Income-earned where
in the Philippine to non-resident clients Dividends on domestic stocks the property is sold.
(paid in US dollars) 400,000 150,000 Illustration
Applying the situs rules the following Total Source of gross income
are the situs of the aforementioned P1,100,000 Amount
income: P350,000 Goods purchased and sold within
B.Dividend income from: P200,000
Within Without 1.Domestic corporation-presumed Goods purchased within and sold
Interest on foreign deposits P- earned within abroad 100,000
P300,000 2.Foreign corporation Goods purchased abroad and sold
Interest from domestic bonds a.)Resident foreign corporation- within 150,000
50,000 depends on the pre-dominance test Goods purchased and sold abroad
Royalties from books in the Phils. The pre-dominance test 350,000
100,000 If the ratio of the Philippines gross The income within and without shall be:
Rent income of foreign properties income over the world gross income of
150,000 the resident foreign corporation in the Within Without
Professional fees three-year period preceding the year of Purchased and sold within
400,000 dividend declaration is: P200,000
Total -At least 50% the portion of the Purchased within and sold abroad
P550,000 P450,000 dividend corresponding to the P100,000
Other Income situs rules Philippine gross income ratio is earned Purchased abroad and sold within
A.Gain on sale of properties within 150,000
1.Personal properties -Less than 50%, the entire dividends Purchased abroad and sold abroad
-Domestic securities-presumed earned received is earned abroad 350,000
within the Philippines b.)Non-resident foreign corporation- Total
-Other personal properties-earned in earned abroad. P350,000 P450,000
the place where the property is sold Illustration D.Manufacturing income-Earned where
2.Real Property-Earned where the In 2014, Sarah received a P400,000 the goods are manufactured and sold
property is located. dividend income from ABC Corporation. Operations
Illustration ABC Corporation had the following Remark
A taxpayer had the following income: gross income in 2011 through 2013: Production Distribution
Gain on sale of domestic stocks 2011 Within Within
P200,000 2012 2013 Total Total income from production and
Gain on sale of foreign bonds Philippines P100,000 distribution is earned within the
100,000 P200,000 P300,000 Philippines
Gain on sale of a commercial lot in P600,000 Without Without
Baguio City 500,000 Abroad 200,000 Total income from production and
Gain on sale of car in Ontario, Canada 100,000 100,000 distribution is earned without the
200,000 400,000 Philippines.
Gain on sale of machineries in Mexico. Total P 300,000 Within Without
Pampanga 250,000 P300,000 P400,000 Production income is earned within,
Interest income on foreign bonds P1,000,000 Distribution income is earned without
50,000 If ABC Corporation is a :
Without Within taxation shall be the place of sale g.Dividend income from domestic
Distribution income is earned within, without regard to the seller or the corporation
production income is earned without supplier. h.Dividend income from resident
Illustration 1 The following are the situs of income foreign corporation
Butual Inc. manufactures goods and for the parent corporation: i.Merchandising income
sells them through its branch, Butuan Scenario Parent Subsidiary Exercise Drill No. 1:Return of capital and
bills its branch at established market Within Without return on capital
prices. Butuan reported the following No.1 Philippines Philippines Indicate the amount representing
gross income: P1,600,000 P- return of capital or return on capital:
Hom No.2 Abroad Abroad Consideration For the loss of
e office Branch Total - 1,600,000 Return of capital Return on
Sales No. 3 Philippines Abroad capital
P4,000,000 P2,000,000 1,600,000 - 1.P1,000,000 Health
P6,000,000 No. 4 Abroad Philippines 1,000,000 0
Cost of goods sold 2,400,000 - 1,600,000 2.P500,000 P400,000 car
1,200,000 3,600,000 The following are the situs of income 400,000 100,000
Gross Income for the subsidiary corporation 3.P300,000 P350,000
P1,600,000 P800,000 Scenario Parent Subsidiary building 300,000
P2,400,000 Within Without 0
The following shows the situs of the No.1 Philippines 4.P600,000 Income
gross income of Butuan under each of Philippines P800,000 P- 0 600,000
the following scenario: No.2 Abroad Abroad 5.P1,200,000 Life
Scenario Home office - 800,000 1,200,000 0
Branch Within Without No.3 Philippines Abroad Exercise Drill no.2 Income tax and
No.1 Philippines - 800,000 transfer tax
Philippines P2,400,000 P0 No.4 Abroad Philippines Transaction Income tax
No.2 Abroad 800,000 - Transfer tax
Abroad 0 CHAPTER 3:SELF-TEST EXERCISES 1.Barter of properties /
2,400,000 Discussion Questions 2.Sale of goods /
No.3 Philippines 1.Enumerate the characteristics of gross 3.Indemnity of moral damages
Abroad 1,600,000 income. /
800,000 2.What are capital items considered 4.Harvested fruits from an orchard /
No.4 Abroad with infinite value?Enumerate 5.Compensation income /
Philippines 800,000 3.When is income considered realized? 6.Interest income /
1,600,000 4.Distinguish exchange from transfer 7.Amount received by the insured in
Note:Both production and distribution 5.What is a complex transaction?How is excess of insurance premiums paid /
are conducted by the same taxable it taxed?
entity, Butuan Inc. The branch is not a 6.What is a holding gain?Why is it
separate taxable entity but is an integral exempt from taxation?
part of Butuan Inc.;hence, its income is 7.Compare actual receipt with
taxable to Butuan Inc. constructive receipt
Illustration 2 8.Enumerate and explain the
Assuming production is conducted by a classifications of individual taxpayers
parent corporation and the distribution 9.What constitute a taxable estate and
is conducted by its subsidiary trust?
corporation: 10.Enumerate and explain the
Parent classifications of corporate taxpayers.
Subsidiary Total 11.Discuss the taxability of each of class
Sales of taxpayers.
P4,000,000 P2,000,000 12.Explain situs.Differentiate situs from 8.Proceeds of life insurance received by
P6,000,000 source of income the heirs of the insured
Cost of goods sold 13.What is the situs of the following /
2,400,000 1,200,000 income: 9.Gain on sale of goods by the home
3,600,000 a.Interest income office to its branch
Gross income b.Service income /
P1,600,000 P800,000 c.Royalty income 10.Gain on sale of goods and services
P2,400,000 d.Rental income between relatives /
The gross income recognized by each e.Gain on sale of movable property 11.Gain on sale of goods by the parent
corporation is taxable to each f.Dividend income from domestic corporation to a subsidiary
corporation because each corporation is corporation corporation /
a separate taxpayer. The situs of
12.Appreciation in the value of land 7.A Japanese who married a beautiful 9.Taxable trusts established by a Filipino
/ Filipina and has been residing in the citizen in the Philippines /
13.Birth of animal offspring Philippines for 2 years RA 10.Taxable estate of a non-resident
/ 8.A 2nd year Korean college student citizen judicially administered abroad
14.Income of a registered Barangay studying in the Philippines /
Micro-Business Enterprise RA Exercise Drill No. 6:Location and situs of
/ 9.A corporation incorporated under income
15.Cancellation of debt out of gratuity Philippine law Compute how much is earned within
of the creditor DC and earned outside the Philippine from
/ 10.A foreign corporation doing business each of the following independent
16.Cancellation of debt by the creditor in the Philippines cases:
in exchange of services rendered by the RFC Income description
debtor / 11.Trust designated by the donor as Within Without
17.Matured interest from coupon irrevocable 1.Rey earned P100,000 interest income;
bonds / RC 40% of these were from non-resident
18.Receipt of bank loan 12.Trust designated by the donor as debtors P60,000 P40,000
/ revocable 2.A finance company earned
19.Salaries of a minimum wage earner NT P1,000,000
/ 13.A business partnership royalties from a franchise; 40% of these
20.PCSO or lotto winnings DC were derived abroad P600,000
/ 14.A joint venture organized under a P400,000
21.Benefits from GSIS, SSS, Pag-Ibig or foreign law and is not operating in the 3.Raymond earned P100,000 rent from
Philhealth Philippines NRFC OFWs from his apartment in U.S.He
/ 15.An estate of a Filipino citizen also earned P40,000 rent from his
22.Discovery of hidden treasure judicially administered in Japan Philippine condominium unit P40,000
/ NRC P100,000
Exercise drill no.4:Income taxpayer 16.An estate of a Filipino citizen extra- 4.Chester, a resident citizen,works
classifications judicially administered in the home online and submits his output to
Indicate the appropriate classification Philippines NT clients
for each of the following taxpayers: 17.A taxable joint venture organized in He collected P100,000 service fee from
DC-Domestic Corporation the Philippines foreign clients and P20,000 from
RFC-Resident foreign corporation DC resident clients P120,000 P0
NRFC-Non-resident foreign corporation 18.A non-profit corporation organized 5.Mark rendered audit services to client
NRA-ETB- Non-resident engaged in in the Philippines in Afghanistan for P500,000. The
trade or business DC services were paid in Afghanistan 0
NRA-NETB-Non-resident alien not Exercise Drill No 5:General Income tax P500,000
engaged in trade or business rule 6.Jun has a store in a tourist park in
NT-Not a taxpayer Check the box properly corresponds to Baguio City, Philippines. He earned a
RC-Resident citizen the taxability of the following tax total of P40,000 gain from selling
NRC-Non-resident citizen payers: souvenir items 40% were from foreign
RA-Resident alien Taxpayer tourists P40,000 P0
Person or Entity World Income Philippine 7.Don Mariano sold at a gain of
Classification Income P2,000,000 to a client abroad a
1.A fat Mexican tourist 1.Non-resident citizen commercial building located in Quezon
NRA-NETB / City, Philippines P2,000,000
2.A hardworking overseas Filipino 2.Resident alien 8.John sold his stocks in a domestic
worker / corporation to a foreign investor at a
NRC 3.Non-resident alien engaged in trade gain of P50,000. P50,000 P0
3.An expatriate employee or business 9.Manso received P20,000 dividends
NRC / from a domestic corporation and
4.A Filipino who is privately employed 4.Resident foreign corporation P30,000 dividend income from a non-
in the Philippines / resident foreign corporationP20,000
RC 5.Resident citizen P30,000
5.An unemployed Filipino residing in / 10.Andrew received P40,000 dividends
the Philippines 6.Non-resident alien not engaged in from a resident foreign corporation;60%
RC business of its
6.A Chinses Businessman who has his / historical income is from the Philippines
domicile in the Philippines for 6 months 7.Non-resident foreign corporation P24,000 P16,000
NRA-NETB / 11.CDO,Inc. manufactures in the
8.Domestic corporation Philippines and sells to unaffiliated
/ export clients. A total of P100,000 gross
income was earned during the period d.Both a and b 15.Transfers for insufficient
P100,000 P0 8.Why is income subject to taxation? consideration are subject to
12.ABC manufactures abroad and sells a.Income is the most prevalent source a.Income tax
to its branch at market prices. of a taxpayer’s wealth b.Transfer tax
Production cost abroad were b.Income is the best measure of c.Either a or b
P200,000.Billings to the branch totaled taxpayer’s ability to pay tax= d.Both a and b=
P300,000 while branch sales totaled c.Rich people tend to have more 16.Which is specifically exempted from
P450,000 P150,000 P100,000 income than the poor. income taxation by virtue of legal
13.James received P100,000 dividends d.Any of these consideration
from resident foreign corporation which 9.Which is not an item of gross income a.Minimum wage
realized 40% of its income in the because of the absence of an b.Gain on sale of prohibited drugs
Philippines. P0 P100,000 undertaking from the taxpayer? c.Unrealized gain
14.Ellis received P20,000 dividends a.Proceeds of a life insurance policy d.all of the above=
from a non-resident foreign corporation b.Forgiveness of indebtedness as an act Multiple choice-Theory:Part 2
P0 P20,000 of gratuity= 1.A resident alien naturalized in
15.Davao plant manufactures tables c.Revaluation surplus on properties accordance with Philippine law is a
and sells to resident clients. A total of d.Service fees a.Resident citizen=
P400,000 gross income was realized 10.Which is subject to income tax? b.Resident alien
during the period P400,000 P0 a.Proceeds of life insurance policy c.Non-resident alien engaged in trade
Multiple Choice-Theory:Part 1 received by the family of the insured or business
1.Which is not a requisite of gross b.Excess of proceeds over the d.Non-resident alien not engaged in
income? premiums paid received by the trade or business
a.Return on capital taxpayer= 2.Who is not a resident alien
b.Realized profit c.Life insurance proceeds by the a.An alien who stayed in the Philippines
c.Exempted by law= corporation from the insurance of a for more than two years
d.Not exempted by law deceased officer b.An alien who married and stayed in
2.Which is taxable item of income? d.None of these the Philippines for one year=
a.Increase in numbers of a herd of 11.Which of the following is exempted c.An alien who stayed in the Philippines
animals from income taxation because of the for more than one year
b.Compensation for personal injuries absence of ability to pay? d.An alien who established his intention
c.Moral damages a.Damages received from patent before the CIR to stay in the Philippines
d.Interest on moral damages= infringement suit for an extended period of time
3.Which is not a subject to income tax? b.Unrealized income from investment= 3.Which taxpayer is not a natural
a.Donation= c.Gain on sale of goods person?
b.Sale of service d.inheritance a.Resident citizen
c.Sales of goods 12.Income tax may be imposed for the b.Taxable estate=
d.Barter of goods following purposes,except c.Non-resident alien engaged in trade
4.The total consideration received from a.To provide large amounts of or business
the sale of services constitute revenues.= d.Non-resident alien not engaged in
a.Return on capital= b.To limit corruption trade or business
b.Return of capital c.To offset regressive sales and 4.A Filipino who has been abroad for
c.Either a or b consumption taxes. more than 183 days is classified as a
d.Both a and b d.To mitigate the evils from the a.Resident alien
5.When paid for, which of the following inequalities in the distributions of the b.Non-resident alien
items may involve a return on capital? income and wealth c.Non-resident citizen=
a.House and lot= 13.Which of the following constitute d.Non-resident citizen not engaged in
b.Life taxable income? trade or business
c.Dignity a.Return of premium on life insurance 5.Which of the following is not an
d.Health received by the insured. income taxpayer classification?
6.The total consideration received from b.Moral damages from slander a.Resident citizen
the sale of goods at a gain represents? c.Proceeds of crop insurance= b.Non-resident alien
a.Return on capital d.Compensation for personal injury c.Resident foreign corporation
b.Return of capital 14.Which of the following is not a d.General professional partnership=
c.Either a or c constructive receipt of income? 6.An American who showed proof to
d.Both a and b= a.Foregiveness of indebtedness in the satisfaction of the Commissioner of
7.The total consideration received from consideration of service Internal Revenue of his intention to stay
the sale of goods and services at a loss b.Matured detachable interest coupons in the Philippines as an immigrant is
represents c.Deposit of income to taxpayer’s bank classified as a
a.Return on capital accounts a.Resident citizen
b.Return of capital= d.Cash salary of an employee= b.Resident alien=
c.Either a or b
c.Non-resident alien engaged in trade c.Joint venture engaged in energy a.wherever sold.
or business operation pursuant to a service contract b.If sold abroad only
d.Non-resident alien not engaged in with the government= c.without the Philippines only
trade or business d.Irrevocable trusts= d.within the Philippines only=
7.A Japanese who is staying in the 16.Which of the following taxpayers is 6.Yvonne, a resident alien, bought a car
Philippines for 183 days is a taxable only on income earned from the manufactured in the Philippines and
a.Resident citizen Philippines? exported the same at a gain to Carla, a
b.Resident alien= a.Resident Corporation= non-resident citizen. Which is correct?
c.Non-resident alien engaged in trade b.Domestic Corporation a.The gain is subject to tax in the
or business c.Resident Corporation Philippines since the commodity
d.Non-resident alien not engaged in d.All of these involved is manufactured in the
trade or business 17.All of the following are taxable only Philippines
8.A Canadian who is staying in the on income earned from sources within b.The gain is subject to tax in the
Philippines for more than one year is a the Philippines,except Philippines since the buyer is a citizen of
a.Resident alien= a.Resident alien the Philippines
b.Non-resident alien b.Non-resident alien c.The gain is both subject to tax in the
c.Non-resident alien engaged in trade c.Non-resident alien Philippines and abroad since the
or business d.Domestic Corporation= commodity involved is manufactured in
d.Non-resident alien not engaged in 18.Which is taxable on world income? the Philippines.=
trade or business a.Resident corporation d.The gain is taxable abroad because it
9.An alien who stayed less than one b.Non-resident corporation is sold abroad.
year in the Philippines is classified as a c.Resident citizen= 7.Juan, a resident alien, and Pedro, a
non-resident alien not engaged in trade d.Resident alien non-resident alien, executed a contract
or business if he stayed herein for less Multiple Choice-Theory:Part 3 of sale in Japan whereby Pedro shall
than 1.The place of taxation is purchase the lot owned by Juan in the
a.180 days= a.Situs rule= Philippines Juan gains P1,000,000 in the
b.1 year b.Situs exchange.
c.183 days c.Territoriality Which is true?
d. 2 years d.Gross Income a.The gain is exempt since the gain is
10.A corporation incorporated 2.Which is an incorrect statement derived outside the Philippines
according to Philippine laws is a regarding situs of income? b.The gain is not subject to Philippines
a.Domestic Corporation= a.Service Income is earned in the tax since Juan is a resident alien
b.Resident Corporation domicile of the taxpayer= c.The gain is subject to Philippine tax
c.Non-resident Corporation b.Interest income is earned in the because Juan is a resident alien
d.De jure Corporation residence of the debtor d.The gain is subject to Philippine tax
11.A foreign corporation which c.Royalty is earned where the intangible because the property is in the
operates a branch in the Philippines is a is employed. Philippines.
a.Domestic Corporation d.Rent is earned in the location of the Multiple Choice-Problems
b.Resident Corporation property Beth negotiated a P1,000,000 non-
c.Non-resident Corporation= 3.Which statement is correct regarding interest bearing promissory note to
d.De jure Corporation situs of income? Candy. Candy paid Beth P950,000.On
12.A foreign corporation which a.The gain on the sale of real property is due date, Beth paid Candy
operates a branch in the Philippines is a earned in the location of the property= P1,000,000.Which is true?
a.Resident corporation b.The gain of sale of any property is a.Beth earned P50,000 return on capital
b.Resident Corporation= earned in the place of sale. b.Candy earned P50,000 return on
c.Non-resident Corporation c.Merchandising income is earned in capital=
d.De jure corporation the residence of the proprietor. c.Candy received P50,000 donation
13.A partnership which dominantly d.Manufacturing income is earned in d.Candy received P1,000,000 return of
operates business abroad is a the place of sale. capital
a.Domestic corporation 4.Pedro, a non-resident citizen, lent Problem 3-2
b.Resident corporation money to Shino, a resident Chinese. The Andrew received a total sum of P42,000
c.Non-resident corporation= indebtedness was collateralized by a from his employer consisting of the
d.De jure corporation property located in Japan. The interest following
14.Which is required to pay income tax? income is earned in -P5,000 reimbursements for employer’s
a.Revocable trusts a. the Philippines expenses paid by Andrew
b.Estates under extrajudicial settlement b.China -P15,000 payment of Andrew’s
d.Business partnership= c.Japan computer set purchased by the
15.Which is not an income taxpayer? d.Japan, China and the Philippines= employer
a.Non-resident foreign corporation 5.Gains on the sale of goods -P22,000 monthly salary
b.Non-resident alien not engaged in manufactured and sold by the taxpayer Andrew’s computer set cost him
trade or business within the Philippines is subject to tax P12,000. Compute the total return on
capital which can be subjected to c.P60,000= c.P0
income tax d.P0 d.P200,000=
a.P42,000 Problem 3-7 Problem 3-11
b.P37,000 Alexander Company insured the life of Henson was one of the passengers of a
c.P25,000= its president for P2,000,000. A total of van that fell off a ravine. Henson sued
d.P22,000 P500,000 in premiums was paid before the bus company and was awarded an
Problem 3-3 the president died. The company indemnity of P800,000 for the
Betty paid P20,000 annual premium on collected the total proceeds. following:
a life insurance contract which would Compute the return on capital a.P500,000 for the impairment of his
pay her P1,000,000 in case of her death, a.P0= health resulting to the amputation of
After paying for 4 years. Betty assigned b.P1,500,000 his legs
the policy to Carlos for P120,000. c.P500,000 b.P200,000 for his loss of salaries during
Compute the return on capital d.P2,000,000 hospitalization
a.P42,000 Problem 3-8 c.P100,000 for his attorney’s fees
b.P37,000 Onyoc insured his newly constructed Copmute Henson’s return on capital
c.P25,000= building costing P1,000,000. Within a a.P800,000
d.P22,000 few days, the building was totally b.P300,000
Problem 3-3 destroyed by a fire. The insurance c.P200,000=
Betty paid P20,000 annual premium on company reimbursed Onyoc d.P0
a life insurance contract which would P1,500,000,which represents the fair Problem 3-12
pay her P1,000,000 in case of her value of the building. Jake sued an unscrupulous person for
death. After paying for 4 years, Betty Which statement is false? derogatory remarks which he
assigned the policy to Carlos for a.P1,000,000 of the proceeds is a return considered to have besmirched his
P120,000. Compute the return on of capital reputation. The court awarded him an
capital b.P500,000 of the proceeds is a return indemnity of P1,000,000 inclusive of
a.P120,000 on capital P200,000 reimbursement for Attorney’s
b.P80,000 c.P1,500,000 is a return of capital= fees and P100,000 exemplary damages.
c.P40,000= d.Only A and B Compute Henson’s total return on
d.P0 Problem 3-9 capital
Problem 3-4 Guilbert is worried that his entire a.P1,000,000
Becky purchased a P1, 500,000 life potato plantation which is expected to b.P800,000
insurance policy for P100,000. During yield P400,000 income will be totally c.P700,000
the year, Becky died and heir heirs devastated by bad weather conditions. d.P0=
collected the entire proceeds. How He obtained a P300,000 crop insurance Problem 3-13
much of the proceeds is exempt from cover for P30,000. Just before harvest, a Kendrick received the following items
income tax? rare frost totally destroyed Guilbert’s during the year:
a.P1,500,000= plantation. The insurance company paid -P200,000 donation from a girlfriend
b.P1,400,000 the policy proceeds. -P100,000 service fee from professional
c.P100,000 Compute the total recovery of loss services
d.P0 profits to be recognized by Guilbert as -P300,000 inheritance from his
Problem 3-5 income deceased father
Dan purchased the P1,000,000 life a.P0 -P100,000 income from illegal gambling
insurance policy of Ben for P120,000. b.P100,000 -P50,000 gain on sale of his personal car
Dan paid the P20,000 annual premium c.P300,000= -P250,000 profits from his bar
on the policy for 4 years after which ben d.P370,000 restaurant
died.Compute the total return on Problem 3-10 Compute the total income subject to
capital for Dan. Felix sells hot-chilli flavored using income tax
a.P1,000,000 pancakes using a secret formula he a.P1,050,000
b.P880,000 patented. He sued a competing pancake b.P750,000
c.P800,000= house for alleged patent infringement c.P550,000
d.P0 and claimed a total indemnity f d.P500,000=
Problem 3-6 P1,200,000. Problem 3-14
Carlos paid P20,000 annual premium *P1,000,000 for loss of profits from loss Pines Corporation has a branch in
for a P1,000,000 life insurance policy. of sales Manila and a 70% owned subsidiary,
After 7 years, Carlos surrendered the *P200,000 as Attorney’s fee Choco Hills, Inc. in Davao. The following
policy and was paid by the insurance reimbursement data shows Pines Corporation’s sales
company P200,000 which represents If Felix wins the case and is awarded the transactions during the year:
the cash surrender value of the policy. total indemnity, compute his total -Pines Corporation billed the Manila
Compute the return on capital return of capital branch P1,500,000 for merchandise
a.P1,000,000 a.P1,000,000 shipped to the latter at a mark-up of
b.P860,000 b.P800,000 50% above acquisition costs. The
branch stored the merchandise and did a.P1,800,000 b.P200,000=
not operate during the year. b.P1,550,000 c.P300,000
-Sold merchandise to unrelated parties c.P200,000 d.P500,000
at a gain of P800,000. d.P250,000= Problem 3-22
-Sold merchandise to Darrel Asuncion, Problem 3-17 A non-resident citizen is an
Pines Corporation’s controlling A condominium home owner’s international financier who earned
stockholder at a gain of P100,000 association collects dues from unit P400,000 interest income from resident
-Sold various merchandise to Choco holders and remits the same to service debtors and P300,000 from foreign
Hills, Inc. at a gain of P200,000 providers on their behalf. Such dues debtors. How much is subject to
Compute the total income of Pines include electricity, water, security, and Philippine income tax?
Corporation subject to income tax? maintenance. The association unit a.None
a.P1,700,000 holders an additional 2% of their utility b.P300,000
b.P1,200,000 bills as service charge. c.P400,000=
c.P1,100,000= During the year, the association d.P700,000
d.P900,000 processed utility bills for unit holders Problem 3-23
Problem 3-15 totaling P5,000,000 Sarah has the following items of
Denver is a supervisory employee of How much taxable income is realized by income:
Atlantis Corporation. He had the the association? P
following items of gross income during a.P5,000,000 hilippines Abroad
the year: b.P4,900,000 Business income
-Denver was paid P800,000 salaries c.P100,000= P200,000
-Denver’s P100,000 personal loan was d.P0 P100,000
paid by Atlantis Corporation as reward Problem 3-18 Professional fees
for his excellent performance Kenny used to bet in PCSO lotto. On 100,000
-Denver’s P50,000 advances to the June 3,2014, he won the P20,000,000 50,000
company was paid by Atlantis’ chief jackpot prize from the 6/45 lotto. One Compensation Income
executive officer as a gift P20-ticket out of 10 bets took the prize. 400,000
-Denver is entitled to excess How much is Kenny’s total income -
representation and transportation subject to tax? Rent Income
allowances. Denver received P200,000 a.P20,000,000 300,000
total allowance out of which P120,000 b.P19,999,990 200,000
was disbursed by him. c.P19,999,900 Interest Income
Compute Denver’s total income subject d.P0= 30,000
to income tax Problem 3-19 40,000
a.P980,000= An American citizen has been staying in 1.Assuming Sarah is a resident citizen,
b.P900,000 the Philippines since August compute the total income subject to
c.P880,000 15,2013.What would be his taxpayer Philippine income tax.
d.P800,000 classification for the year 2013 and a.P390,000
Problem 3-16 2014, respectively? b.P1,180,000
Jen is engaged in business. The a.Non-resident alien engaged in trade c.P1,030,000
following pertains to her transactions or business; resident alien d.P1,420,000=
during 2014: b.Non-resident alien not engaged on 2.Assuming Sarah is a resident alien,
-Sold his personal car which was trade or business; resident citizen compute the total income subject to
purchased at P200,000 to a friend who c.Non-resident alien engaged in trade Philippines income tax
paid only half of the car’s P500,000 or business; resident citizen a.P1,420,000
current fair value. d.Non-resident alien not engaged in b.P1,180,000
-Sales of merchandise was P800,000 trade or business; resident alien= c.P1,030,000=
and the cost of goods sold was Problem 3-20 d.P390,000
P600,000 A citizen who left the Philippines on 3.Assume Sarah is a resident
March 1,2013 would be classified as corporation, compute the total income
-Jen acquired several stocks from the a.Non-resident for the year 2013.= subject to Philippine income tax.
Philippine Stock Exchange for b.Resident citizen for the year 2013. a. P1,420,000
speculation.These stocks have an c.Non-resident for the year 2014 b.P1,030,000=
aggregate purchase price of P400,000 d.Resident citizen for the year 2014. c.P1,180,000
but with P700,000 fair value by Problem 3-21 d.P390,000
December 31,2014. An alien received P200,000 4.Assuming Sarah is a domestic
-Jen’s house and lot which he acquired compensation income in the Philippines corporation, compute the total income
for P1,500,000 in 2010 now have a and P300,000 rental income from subject to Philippines income tax
current fair value of P2,500,000. abroad. How much will be subject to a. P390,000
Compute Jen’s total income subject to Philippine income tax? b.P1,030,000
income tax a.None c.P1,180,000
d.P1,420,000= Compute the total income An item of gross income is taxable in
Case Problems earned from sources any of these taxa schemes:
Case Problem 1 a.Within the Philippines-750,000 Items of gross income-Taxable to any
Jaypee has the following income in b.Without the Philippines- one of:Final income taxation, capital
2017: 340,000 gains taxation and regular income
 P10,000 interest income from a Case Problem 3: taxation
non-resident Japanese friend TC Company manufactures Mutually exclusive coverage
 P40,000 interest income from wooden furniture for the local The tax schemes are mutually exclusive.
Philippine residents and export market. It has a An item of gross income that is subject
 P500,000 rent income from a distribution outlet abroad which to tax in one scheme will not be taxed
commercial complex located in handles foreign sales. It bills all by the other schemes. Similarly, items
the USA which is leased to customers, including the foreign of income that are exempted in one
resident Filipinos outlet, 70% above scheme are not taxable by the other
 P200,000 rent income from a manufacturing costs. The schemes.
boarding house in Baguio City, foreign outlet bills its customers CLASSIFICATION OF ITEMS OF GROSS
Philippines 100% above TC Company’s INCOME
 P200,000 professional fees billing price. TC Company Because of the different tax schemes,
rendered to Chinese clients in reports P3,400,000 in total sales, items of gross income can be classified
Hongkong exclusive of sales to the foreign as follows:
 P300,000 salary from a resident outlet. The foreign outlet 1.Gross income subject to final tax
employer reports P2,720,000 total sales to 2.Gross income subject to capital gains
 P100,000 gain from sale of customers tax
merchandise imported and sold Compute the manufacturing 3. Gross income subject to regular tax
to Filipino residents income,respectively earned *Readers are advised to master the
 P50,000 gain from sale of within and earned without the coverage of both final income tax and
merchandise imported and sold Philippines capital gains tax. A thorough
during business travel in a. P1,960,000; P1,360,000 understanding of these exceptional tax
Hongkong b. P1,400,000;P1,360,000 treatments is very essential to our
 P400,000 gain on sale of the c. P840,000;P1,920,000 masters of Income Taxation
boarding house located in d. d.P840,000;P1,360,000 FINAL INCOME TAXATION
Baguio City to a non-resident Final income taxation is characterized
buyer by final taxes where taxes are withheld
Required: CHAPTER 4 INCOME TAX or deducted at sources. The taxpayer
Compute the total income SCHEMES, ACCOUNTING receives income net of tax. The payor of
earned from sources PERIODS, ACCOUNTING income remits the tax to the
1.Within the Philippines- METHODS, AND REPORTING government. Final taxation is applicable
P1,040,000 Chapter Overview and only to certain passive income. Not all
2.Outside the Philippines- Objectives passive income is subject to final tax.
P760,000 This chapter provides and Passive Income vs. Active Income
Case Problem 2 overview of the income tax Passive income are earned with very
Joy earns franchise fees from his schemes under the NIRC. minimal or even without active
Hot burger franchise. He also After this chapter, readers are involvement of the taxpayer in the
deals in various properties. expected to gain familiarization earning process.
Johnny realized the following and demonstrate mastery of the Examples of passive income
gains in 2017: following: a.Interest income from banks
 P500,000 royalty fees from local a. Types of taxation schemes b. Dividends from domestic
Hot Burger outlets and their scope corporations
 P200,000 royalty fees from b. Concept of accounting c.Royalties
foreign Hot Burger outlets period and its types Active or regular income arises from
 P100,000 gain from sales of c. Concept of accounting transactions requiring a considerable
equipment to foreign methods and their degree of effort or undertaking from
franchisees accounting procedures the taxpayer. It is the direct opposite of
 P200,000 gain from sales of d. Types of tax returns, their passive income.
equipment to local franchisees deadline and place of filing Examples of active income
 P50,000 gains from sale of INCOME TAXATION SCHEMES 1.Compensation income
investment in domestic stocks to There are three income taxation 2.Business income
foreign investors schemes under the NIRC: 3.Professional income
 P40,000 gains from sale of a.Final income taxation Final income taxation will be discussed
investments in foreign stocks to b.Capital gains taxation in Chapter 5
Filipino investors c.Regular income taxation CAPITAL GAINS TAXATION
Required:
A capital gains tax is imposed on the This accounting period is available to Tawi-Tawi should file its last income tax
capital gain on the sale, exchange and both corporate taxpayers and individual return covering April 1 to August 15,
other disposition of certain capital taxpayers. 2014.
assets. Also, not all capital gains are Under the NIRC, the calendar year shall Under the old NIRC, dissolving
subject to capital gains tax. Most of be used when the: corporations shall file their return
them are subject to regular income tax. 1.taxpayer’s annual accounting period is within 30 days from the cessation of
Capital assets vs ordinary assets other than fiscal year activities or 30 days from the approval
Capital assets include all other assets 2.taxpayer has no annual accounting of merger by the Securities and
than ordinary assets. Ordinary assets period Exchange Commission in the case of
are assets directly used in the business, 3.taxpayer does not keep books merger. (BPI VS. CIR, GR 144653, August
trade or profession of the taxpayer such 4.taxpayer is an individual 28,2011).Hence, the return shall be
as inventory, supplies and items of Fiscal year filed on or before September 15, 2014.
property, plant and equipment. A fiscal accounting period is any 12- For individuals, the return shall be due
Capital gains vs. ordinary gains month period that ends on any day on or before April 15, 2015. There is no
Capital gains arise from the sale, other than December 31. the fiscal requirement for early filing under the
exchange and other disposition of accounting period is available only to NIRC.
capital assets. Ordinary gains arise from corporate income taxpayers and is not 3.Change of accounting period by
the sale, exchange and other allowed to individual income taxpayers. corporate taxpayers-The accounting
disposition of ordinary assets. Deadline of Filing of the Income tax period covers the start of the previous
The NIRC identifies capital gains tax as a return accounting period up to the designated
final tax but they are not actually final Under the NIRC, the return is due for year-end of the new accounting period.
tax similar to those imposed under final filing on the fifteenth day of fourth Note the BIR approval is required in
income taxation. The taxpayer still files month following the close of the changing an accounting period. It is not
a capital gains tax return to report the taxable year of the taxpayer. The regular automatic.
gain to the government and pay the tax due is payable upon filing of the Illustration 1
corresponding tax. Capital gains income tax return. Effective February, 2014, Sulu
taxation applies only to two types of Illustration:Due date of the annual Corporation changed its calendar
capital assets: domestic stocks and real income return accounting period to a fiscal year
property. 1.Taxpayers under the calendar year ending every June 30.
Capital gains taxation will be discussed must file their annual income tax return Sulu Corporation shall file an
in detail in Chapter 6. for the current period not later than adjustment return covering the income
REGULAR INCOME TAXATION April 15 of the following year. from January 1 to June 30, 2014 on or
The regular income taxation is the 2.A corporate taxpayer with fiscal year before October 15, 2014.
general rule in income taxation and ending June 30,2014 must file its Illustration 2
covers all other income such as: annual income tax return not later than Effective August 2014, Zamboanga
1.Active income October 15, 2014. Company changed its fiscal year
2.Gains from dealings in properties INSTANCES OF SHORT ACCOUNTING accounting period ending every June 30
a.Dealings in ordinary assets PERIOD to the calendar year.
b.Dealings in other asset not subject to 1.Newly commenced business-The Zamboanga Company should file an
capital gains tax accounting period covers the date of adjustment return covering July 1 to
3.Other income, active or passive, not the start of the business until the December 31,2014 or before April 15,
subject to final tax designated year-end of the business. 2015.
Items of gross income from these Illustration 4.Death of the taxpayer-The accounting
sources are measured using an Palawan Inc. started business operation period covers the start of the calendar
accounting method, accumulated over on June 30,2014 and opted to use the year until the death of the taxpayer
an accounting period, and reported calendar year accounting period. Illustration
through an income tax return. Palawan should file its first income tax Mr. Jacob died on November 2, 2014.
ACCOUNTING PERIOD return covering June 30 to December The heirs of Mr. Jacob or his estate
Accounting period is the length of time 31, 2014 for the year 2014. The return administrators or executors shall file his
over which income is measured and must be filed on or before April 15, last income tax return covering his
reported. 2015. income from January 1 to November 2,
TYPES OF ACCOUNTING PERIODS 2.Dissolution of business-The 2014. Hence, the income tax return
1.Regular accounting period-12 months accounting period covers the start of shall be filed early filing in case of death
in length the current year to date of dissolution of taxpayers. Hence, the income tax
a.Calendar of the business. return shall be filed on or before the
b.Fiscal Illustration usual deadline, April 15, 2015.
2.Short accounting period-less than 12 Tawi-Tawi Inc. is on the fiscal year of 5.Termination of the accounting period
months accounting period ending every March of the taxpayer by the Commissioner of
Calendar year 31. It ceased business operation on Internal Revenue-The accounting period
The calendar accounting period starts August 14, 2014. covers the start of the current year until
from January 1 and ends December 31.
the date of the termination of the the future period they expire or are Payment of accrued expenses of 2015
accounting period. used in the business, trade, or - 100,000
Illustration profession of the taxpayer. Payment of expenses of the following
The accounting period of a taxpayer Normally, the expensing of year 200,000 300,000
under the calendar year basis was prepayments does not properly reflect
terminated by the CIR on August 2, the income of the taxpayer. It also Tax Accrual Basis
2014. contradicts the Lifeblood Doctrine as it
The taxpayer must file an income tax effectively defers the recognition of 2015 2016
return covering January 1 to August 2, income. Cash Income
2014. The income tax return and the tax 3.Special tax accounting requirement P500,000 P800,000
shall be due and payable immediately. must be followed Accrued Income
ACCOUNTING METHODS There are cases where the tax law itself 500,000 400,000
Accounting methods are accounting provides for a specific accounting Collection for future services-advances
techniques used to measure income. treatment of an income or expense. The 300,000 200,000
Types of Accounting Methods specified method must be observed Total gross income
1.The general methods even it departs from the basis regularly P1,300,000 P1,400,000
a.Accrual basis employed by the taxpayer in keeping his Less:Deductions
b.Cash basis books. Cash expenses
2.Installment and deferred payment The accrual basis income is determined P400,000 P600,000
method as follows: Accrued expenses
3.Percentage of completion method Cash income Pxx 100,000 150,000
4.Outright and spread-out method Accrued (uncollected) income xx Amortization of 2015 prepaid expense
5.Crop year basis Advanced income xx - 200,000
General Methods for Income from sale Gross income Pxx Total deductions
of goods or services The tax accrual basis expense is P500,000
1.Accrual basis determined as follows: P950,000
Under the accrual basis of accounting, Cash expenses Pxx Net Income
income is recognized when earned Accrued(unpaid expense) xx P800,000
regardless of when received. Expense is Amortization of prepayments and P450,000
recognized when incurred regardless of depreciation of capital expenditures xx Points to consider in converting GAAP
when paid. Deductions Pxx Accrual Basis to Tax Accrual Basis
Income is said to have accrued when The tax cash basis income is determined 1.In accounting accrual basis income is
the right to receive is established or as follows: recognized when earned even if not yet
when an enforceable right to secure Cash income PXX received. Advanced income is
payment is created against the Advanced income XX inherently not included in net income.
counterparty. Gross Income PXX For purposes of taxation, advanced
2.Cash basis The tax cash basis expense is income is taxable. Hence, it must be
Under the cash basis of accounting, determined as follows: added to accrual basis gross income.
income is recognized when received Cash expenses Pxx 2.In accounting, expense is recognized
and expense is recognized when paid. Amortization of prepayments and when accrued even if not yet paid.
Tax and accounting concepts of accrual depreciation of capital expenditures Prepaid expenses are inherently not
basis and cash distinguished xx deducted. Hence, no adjustment for
The financial accounting concept of Deductions prepayments is necessary under accrual
accrual basis and cash basis are similar Pxx basis.
to their tax counterparts, except only Illustrations
for the following tax rules: A taxpayer providing services reported Tax Cash Basis
1.Advanced income is taxable upon the following in 2015 and 2016:
receipt. 20 2015 2016
Income received in advance is taxable 15 2016 Collection from services rendered
upon receipt in pursuant to the Collections from services rendered P500,000 P1,270,000
Lifeblood doctrine and the Ability to pay P500,000 P800,000 Collection for future services-advances
theory. The subsequent taxation of Accrued Income from services rendered 300,000 200,000
advanced income in the period earned 500,000 400,000 Total gross income
will expose the government to risk of Collection from accrued income of 2015 P1,300,000 P1,400,000
non-collection - 470,000 Less:deductions
2.Prepaid expense is non-deductible Collection for services not yet rendered Cash expenses
Prepaid expenses are advanced 300,000 200,000 P400,000 P700,000
payment for expense of future taxable Payment of expenses of current period Amortization of 2015 prepayments
periods. These are not deductible 400,000 600,000 - 200,000
against gross income in the year paid. Accrued expenses Total deductions
They are deducted against income in 100,000 150,000 P400,000 P900,000
Net Income The gross income as determined by Canlubang received a P200,000 cash
P400,000 P570,000 cash basis in the service business and downpayment and a P1,800,000
Note:P800,000+P470,000=P1,270,000;P the gross income as determined by the promissory note for the balance
600,000+P100,000=P700,000 accrual basis in the trading business are payable in six installment of P300,000
Points to consider in converting GAAP simply combined. There is no every July 3 and January 3 thereafter.
cash basis to Tax cash basis requirement to measure the income of The selling price and gross profit on the
1.Under the accounting cash basis, different businesses under a single sale is computed as follows:
income is recognized when received not accounting period. Cash downpayment P200,000
when it is earned .Advanced income is Sales of goods with extended payment Notes receivables 1,800,000
inherently recognized as income. terms Selling price P2,000,000
Hence, no adjustment is necessary on The sale of goods with extended Less:Tax basis of machine sold
income. payment terms may be reported using 1,200,000
2.Under accounting cash basis, expense the accrual basis, installment method, Gross profit P800,000
is deducted when paid including or deferred payment method. Accrual basis
prepaid expenses, Hence, the deducted Installment method Under the accrual basis, the entire
prepaid expenses must be reversed for Under the installment method, gross P800,000 gross profit shall be reported
purposes of taxation. income is recognized and reported is as gross income in 2016, the year of
Sellers of goods proportion to the collection from the sale.
The gross income of taxpayers selling installment sales. Installment basis
goods is determined as follows: Installment method is available to the Canlubang cannot readily use the
following taxpayers: installment method because it is a
Sales 1.Dealers of personal property of the dealer of cars rather than a dealer of
Pxx sale of properties they regularly sell machineries. The sale of properties of
Less:Cost of goods sold 2.Dealers of real properties, only if their which the seller is not a dealer is
xx initial payment does not exceed 25% of referred to as “casual sale”.Hence, the
Gross income the selling price. ratio of initial payment shall be tested
Pxx 3.Casual sale of non-dealers in property, first.
The cost of sales is computed using the real or personal, when their selling The initial payment of Canlubang can be
inventory method: price exceeds P1,000 and their initial computed as follows:
Beginning inventory payment does not exceed 25% of the Cash downpayment (January 3,2016)
Pxx selling price. P200,000
Add:Purchases Initial payment First installment (July 3, 2016)
xx Initial payment means total payments 300,000
Total goods available for sale by the buyer, in cash or property, in the Initial payment
Pxx taxable year the sale was made. The P500,000
Less:Ending inventory xx term “initial payment” is broader than Ratio of initial payment
Cost of goods sold downpayment. It also includes the (P500,000/P2,000,000) 25%
Pxx installment payments in the year of Canlubang can use the installment
The expensing of the purchase cost of sale. method. The contract price or the
goods does not properly and fairly Selling price amount due shall be determined next.
reflect the income of the taxpayer Selling price means the entire amount Since there is no mortgage assumed by
particularly when there are significant for which the buyer is obligated to the the buyer, the selling price is the
fluctuations in inventory levels between seller.It is computed as follows: contract price.
accounting periods. This could expose Cash received and/or receivable Pxx The gross profit will be reported in
the taxpayer to risk of BIR assessment. Fair market value of property received gross income throughout the
The use of the accrual method is or receivable xx installment period by the formula:
suggested but of course subject to Mortgage or any indebtedness assumed (Collection/Contract price)xGross profit
practical and cost considerations. by the buyer xx Canlubang shall recognize the following
Hybrid basis Selling price gross income:
The Hybrid basis is any combination of Pxx At the date of sale:
accrual basis, cash basis, and or other Contract price (P200k/P2MXP800,000)P80,000
methods of accounting. It is used when The contract price is the amount Upon every installment:
the taxpayer has several businesses receivable in cash or other property (P300k/P2MXP800,000) P120,000
which employ different accounting from the buyer. It is usually the selling If Canlubang is a dealer in machinery, it
methods. price in the absence of an agreement can avail of the installment method
Illustration whereby the debtor assumes even if the ratio of its initial payment
Mr.Roxas has two proprietorship indebtedness on the property. over selling price exceeds 25% so long
business: a service business which used Comprehensive illustration as the selling price of the installment
cash basis and a trading business which Canlubang Company, a car dealer, sold a sale exceeds P1,000.
accrual basis. machine with tax basis of P1,200,000 With indebtedness assumed by the
on installment on January 3, 2016. buyer
The application of the installment Upon every installment: Initial payment
method will slightly vary when the (P200,000/P1,000,000XP600,000) P500,000
buyer assumes indebtedness on the P120,000 Ratio of initial payment
property sold. Indebtedness assumed exceeds tax (P500,000/P2,000,000) 25%
In this case, the selling price is no basis of property sold The contract price shall be computed
longer the contract price. The contract When the indebtedness assumed by as:
price is the residual amount after the buyer exceeds the tax basis of the Selling price
deducting the mortgage from the property sold, the excess is an indirect P2,000,000
selling price. Thus, receipt realized by the seller. This is an Less:Mortgage assumed by the buyer
Selling price Pxx indirect downpayment which must be 1,500,000
Less:Motgage assumed by buyer xx added as part of the contract price and Cash collectible
Contract price Pxx the initial payment. Note also that P500,000
Illustration under this condition, all collection from Excess mortgage (P1,500,000-
On January 3, 2016, Tagaytay, Inc. a real the contract including the excess P1,300,000) 200,000
property dealer, sold a lot costing mortgage is a collection of income. Contract price
P1,400,000 for P2,000,000. The lot was The contract price shall be computed as P700,000
encumbered by a P1,000,000 mortgage follows:
which was assumed by the buyer. The Selling price Pxx Note that the gross profit on the sale is
buyer paid P20,000 downpayment. The Less:Mortgage assumed by buyer xx the same as the contract price. Hence,
balance is due over four installments of Cash Collectible Pxx any collection from the contract
P200,000 every July 3 and January 3 Add:Excess indebtedness-constructive including the excess mortgage shall be
thereafter. receipt xx recognized as gross income upon
The gross profit can be computed as Contract price collection.
follows: Pxx Canlubang shall recognize the following
Selling price P2,000,000 The initial payment shall be computed gross income:
less:tax basis of lot sold 1,400,000 as follows: At the date of sale
Gross profit P600,000 Downpayment (P200kdown+P100kexcess) P300,000
Note that dealers of real properties are Pxx Upon receipt of first installment-
subject to limitation on the use of Installment in the year of sale 12/31/2016 200,000
installment method. The ratio of initial xx Upon receipt of second installment-
payment shall be determined first. Excess of mortgage over tax basis 7/1/2017 200,000
January 3, 2016 cash downpayment xx Total gross profit on the contract
P200,000 Initial payment P700,000
June3, 2016 installment Pxx Deferred payment method
200,000 Illustration The deferred payment method is a
Initial payment On July 1,2016, a taxpayer made a variant of the accrual basis and is used
P400,000 casual sale of property with a tax basis in reporting income when a non-
Ratio of initial payment of P1,300,000 for P2,000,000. The interest bearing note is received as
(P400,000/P2,000,000) 20% property was subject to a P1,500,000 consideration in a sale.
Tagaytay is qualified to use the mortgage which was agreed to be Under the deferred payment method,
installment method. The contract price assumed by the buyer. The buyer paid a the gross income is computed based on
should be determined next. P100,000 down payment with the the present value(discounted value) of
Selling price balance due in two installments of a note receivable from the contract. The
P2,000,000 P200,000 on December 31,2016 and discount interest on the note is
Less:Motgage assumed by the buyer July 1,2017. amortized (i.e., spread) as interest
1,000,000 The gross profit on the sale is income over the installment term.
Contract price determined as follows: Illustration
P1,000,000 Selling price On December 31, 2015, a taxpayer sold
Alternatively, the contract price can be P2,000,000 an office building costing P1,400,000 for
computed directly as follows: Less:Tax basis of property sold P2,000,000. The buyer made
Cash downpayment 1,300,000 P1,000,000 downpayment and the
P200,000 Gross profit balance, evidence by a note is due in 2
Collectible balance(P200,000x4 P700,000 annual installments of P500,000 every
installments) 800,000 The initial payment shall be determined December 31 starting December
Contract price first: 31,2016.
P1,000,000 Downpayment Note that the installment method
Tagaytay shall recognize the following P100,000 cannot be allowed since the ratio of
gross income: December 31, 2016 installment initial payment is already 50%
At the date of sale: 200,000 (P1,000,000/P2,000,000)
(P200,000/P1,000,000XP600,000) Excess mortgage (P1,500,000- Assume the note is non-interest bearing
P120,000 P1,300,000) 200,000 but can be discounted at a local bank
for P900,000.Under the deferred income upon completion of the
payment method, the reportable gross 2015 2016 improvement in 2016.This is not income
income for each year shall be: Contract price in its totality, but this is the amount
P5,000,000 P5,000,000 referred to by the regulation.
2015 2016 2017 Multiply by:% of completion Spread-out method
Cash downpayment 70% 100% The depreciated value of the property
P1,000,000 Construction revenue at the termination of the lease is the
Present value of the note P3,500,000 P5,000,000 value of the years of usage of the lessor.
900,000 Less:Construction revenue in prior year This can be computed by splitting the
Selling price - 3,500,000 value of the improvement as follows:
P1,900,000 Construction revenue this year User Years of usage
Less:Tax basis of the property P3,500,000 P1,500,000 Allocation Cost
1,400,000 Less:Expense during the year -Lessee 20 20/30
Gross income 3,000,000 1,200,000 XP4,500,000 P3,000,000
P500,000 Construction gross income -Lessor 10 10/30
Interest income (P1,000,000-P900,000) P500,000 P300,000 XP4,500,000 1,500,000
P50,000 P50,000 Income from Leasehold Improvement Total 30
Note: Leasehold improvements are tangible P4,500,000
1.The difference between the face value improvements made by the lessee to Under the spread-out method,
and the present value of the note, the property of the Anderson shall spread the P1,500,000
known as discount, will not be lessor.Improvements will benefit the income over 20 periods or recognize an
recognized in gross income at the date lessor when their useful life extends annual income of P75,000 from the
of sale but will be deferred and beyond the lease term. This benefit is leasehold improvement from year 2016
recognized as interest income referred to as income from leasehold through year 2035.
2.The discount is amortized as interest improvement. Note to the readers
income upon every collection on the Under Revenue Regulations No. 2, the It should be pointed out that this rule
balance of the note as follows income from leasehold improvement only in regulation and is absent in the
:P500,000 installment/P1,000,000 total can be reported using either of the NIRC.Some taxpayers are questioning its
note balance XP100,000 discount following method at the option of the validity pointing out lack of legal basis.
In the case of interest-bearing notes, taxpayer: However, it is fairly proper to consider
the use of the deferred payment a.Outright method the depreciated value of the
method bear the same results as the The lessor may report as income the improvement that remains to the lessor
accrual basis of accounting fair market value of such buildings or upon termination of the lease as
The percentage of Completion method improvements subject to the lease at income because it is an actual benefit
for Construction Contracts the time when such buildings or to the lessor. These are, in effect,
Under the percentage of completion improvements are completed. additional rental consideration in kind.
method, the estimated gross income b.Spreadout method However, the treatment specified by
from construction is projected based on The lessor may spread over the life of the outright method is perceived as
the percentage of completion of the lessee the estimated depreciated unjust and abusive, and is improper
construction project. value of such buildings or introduction of legislation.
There are several methods of improvements at the termination of the The depreciated value of the
estimating project completion in lease and report as income for each improvement at the termination of the
practice, but the output method based year of the lease an aliquot part lease should be proper value to be
on engineering survey is prescribed by thereof. recognized as gross income under the
the NIRC. The depreciated value of the leasehold outright method. This view is supported
Illustration improvement is computed as: by the fact that the spread-out method
In 2015, Cagayan Construction Cost of improvementxExcess useful life could not have been an option if the
Company accepted a P5,000,000 fixed- over lease term/Useful life of the outright method intended to tax the
price construction contract. The improvement entire fair value of the improvement
following shows the details of its Illustration considering the huge disproportion in
construction activities: On January 1, 2016, Anderson leased a the reportable gross income in the two
vacant lot to Greg under a 20-year lease options.
2015 2016 contract. Greg immediately constructed The outright method as mandated by
Construction expenses a building on the lot at a total cost of the regulation will best apply in cases
P3,000,000 P1,200,000 P4,500,000. The building has a useful where lessees pay the lessor rentals in
Engineer’s estimate of completion life of 30 years. the form of leasehold improvements or
70% 100% Outright method when leasehold improvements made by
The reportable gross income on Under the plain wordings of section 49 lessees are treated as reductions to
construction will simply be computed as of Revenue Regulations No.2, Anderson cash rentals.
follows: shall recognize the entire P4,500,000 In such cases, the fair value of the
fair value of the improvement as gross leasehold improvements upon
completion is unquestionable income to Under the NIRC, the change in returns are also subject to penalties,
the lessor for taxation purposes. accounting methods by any taxpayer fines and or imprisonment.
Farming income and the change in accounting period by MODE OF FILING INCOME TAX RETURNS
Farming income is commonly corporate taxpayers require prior BIR 1.Manual Filing System
recognized using the cash basis or notice. The traditional manual system of filing
accrual basis. However, long-term crops INCOME TAX REPORTING income tax return is by paper
or those that take more than one year THE SELF-ASSESSMENT METHOD documents where taxpayers fill up BIR
to harvest may be accounted for under In preparing their tax return, taxpayers forms to report income, expenses, or
the crop year basis. declare their income and expenses, and any declaration required to be filed with
Crop year basis personally determine the tax due the BIR.
Under the crop year basis, farming thereon. The government relies on the Under the NIRC, the income tax return
income is recognized as the difference good faith of taxpayers in the shall be filed to the following, in
between the proceeds of harvest and preparation of their tax returns but descending order of priority, within the
expenses of the particular crop employs detective techniques to revenue district office where the
harvested. The expenses of each crop ascertain non-compliance or under- taxpayer is registered or required to
are accumulated and deducted upon compliance register:
the harvest of the crop. Types of income tax-related returns 1.An authorized agent bank
Illustration filed to the government 2.Revenue Collection Officer
Juan de la Cruz, a farmer, plants a 1.Income tax returns. 3.Duly authorized city or municipal
certain crop that takes more than a year 2.Withholding tax returns treasurer
to harvest. Juan had the following data 3.Information returns 2.e-BIR Forms
on his farming operations: Types of income tax return The BIR introduced the e-BIR forms
1.Capital gains tax return which come with an offline and online
2015 2016 2017 2.Regular income tax return versions. Taxpayers fill up their income
Proceeds of harvest P- THE WITHHOLDING SYSTEM tax returns in electronic spreadsheets
P750,000 P1,000,000 Aside from the income tax return, the without the need of writing on paper
st
1 cropping expenses government also requires taxpayers to returns. The system ensures
400,0000 200,000 withhold (i.e., deduct) taxes on their completeness of data on the return and
- income payments (i.e., expenses). is capable of online submission. If there
2nd cropping expenses These withheld taxes are called are no penalties that require BIR
- 500,0000 300,000 “withholding tax”.The taxpayer must assessments, taxpayers would have to
The reportable farming income using deduct the withholding tax on his print a hard copy of the filled tax
crop year method would be: income payments, file a withholding tax returns and proceed directly to the
return, and remit the withheld tax to bank for payment.
2015 2016 2017 the government. 3.Electronic Filing and Payment System
Proceeds of harvest Non-compliance to the withholding tax (Efps)
- P750,000 rules shall expose the taxpayers The Efps is a paperless tax filing system
P1,000,000 penalties and fines aside from the developed and maintained by the
Less:Cropping expenses disallowance of the expense as BIR.Taxpayers file tax returns including
Incurred last year deduction against income. attachments in electronic format and
400,000 500,000 Types of withholding tax pay the tax through the internet.
Incurred this year a.Final withholding tax Taxpayers mandated to use the Efps
200,000 300,000 b.Creditable withholding tax 1.Large taxpayers duly notified by the
Farming gross income Final withholding taxes are the full taxes BIR
P150,000 P200,000 upon the income of the recipient. The 2.Top 20,000 private corporations duly
Crop year basis is an accounting recipient will not pay additional taxes notified by the BIR
method and is not an accounting are advance taxes. The recipient of the 3.Top 5,000 individual taxpayers duly
period. income payments must report the notified by the BIR
Use of different accounting methods income in his income tax returns claim 4.Taxpayers who wish to enter into
Taxpayers with more than one type of the creditable withholding tax withheld contracts with government offices
business using different accounting as tax credit against his tax due. 5.Corporations with paid-up capital of
methods can consolidate the income INFORMATION RETURNS P10,000,000
reported using the different methods. Certain taxpayers are also required to 6.PEZA-registered entities and those
There is no need to restate the income file information returns to the located within Special Economic Zones
to a common accounting method. government. These information returns 7.Government offices, in so far as
However, the methods applied to each do not involve any payment or remittance of withheld VAT and
business should be applied consistently withholding of tax but essential to the business tax are concerned.
from period to period. government in its tax mapping efforts 8.Taxpayers included in the Taxpayer
and in its evaluation of tax compliance. Account Management program (TAMP)
Change in Accounting Methods and Non-filing of required information 9.Accredited importers, including
Accounting Periods prospective importers required to
secure the Importers Clearance b.Wholesale trade and commission PENALTIES FOR LATE FILING OR
Certificate (ICC) and Custom brokers trade PAYMENT OF TAX
Clearance Certificate(BCC) c.Sale, maintenance, repair of motor The late filing and payment of taxes is
In case of unavailability of the Efps vehicle, and sale of automotive fuel subject to the following additional
during maintenance or instances of d.Collection, purification, and charges:
technical errors, Efps enrolled taxpayers distribution of water 1.Surcharge
may file manually. e.Computer and related activityes a.25% of the basic tax for failure to file
GROUPING OF TAXPAYERS UNDER EFPS f.Real estate activities or pay deficiency tax on time
1.Group A 4.Group D b.50% for willful neglect to file and pay
a.Banking Institutions a.Air transport taxes
b.Insurance and pension funding b.Electrcity, gas, stream, and hot water The non-filing is considered “willful
c.Non-bank financial intermediation supply neglect” if the BIR discovered the non-
d.Activities auxiliary to financial c.Postal and telecommunications filing first. This is the case when the
intermediation d.Publishing, printing, and reproduction taxpayer received a notice from the BIR
e.Construction of recorded media to file return. If the taxpayer filed a
f.Water transport e.Recreational, cultural, and sporting return before the receipt of such notice,
g.Hotels and restaurants activities the same is considered simple neglect
h.Land transport f.Recycling subject to the 25% surcharge.
2.Group B g.Renting out of goods and services 2.Interest-20% per annum considering
a.Manufacture and repair of furniture h.Supporting and auxiliary transport the following period factor:
b.Manufacture of basic metals activities. Delay
c.Manufacture of chemicals, and 5.Group E Period factor
chemical products a.Activities of membership For everyday of delay
d.Manufacture of coke, refined organizations Inc. Number of days/365 days
petroleum, and fuel products b.Health and social work For every month of delay
e.Manufacture of electrical machinery, c.Private eeducational services Number of months/12
and apparatus NEC d.Puclic administration and defense For everyyear of delay 1
f.Manufacture of fabricated metal compulsory social security Note:
products e.Public educational services a.The period factor shall be multiplied
g.Manufacture of foods, products, and f.Research and development by 20% interest rate to get interest
beverages g.Agriculture, hunting, and forestry factor which will be multiplied to the
h.Manufacture of machineries, h.Farming of animals tax due to compute the interest penalty
precision, and optical instruments i.Fishing b.A 30-day period in a month is
i.Manufacture of medical, precision, j.Other service activities considered 1 month (i.e. March 1 to
and optical instruments k.Miscellaneous business activities March 31), but the 1-day excess on 31-
j.Manufacture of motor vehicles, l.Unclassified activities day month is ignored.Hence, March 15
trailers, and semi-trailers. PAYMENT ON INCOME TAXES to April 15 is still 1 month on interest if
k.Manufacture of office, accounting, The capital gains tax and regular income March has 31 days.
and computing machineries tax are paid as the taxpayer files his These rules were deducted after careful
l.Manufacture of other non-metallic return. Installment payment of income examination of the illustrative
mineral products tax is allowed on certain conditions. guidelines on interest penalty
m.Manufacture of other transport Taxpayers under the EFPS system shall calculation under RR12-99
equipment e-pay their tax online through the Illustration 1
n.Manufacture of other wearing internet banking service. The account of The tax return of the taxpayer was due
apparel the taxpayer will be auto-debited for on April 15,2017 but was filed on June
o.Manufacture of papers, and paper the amount of taxes to be paid. 30, 2017. The tax return of the taxpayer
products BASIC COMPARISON OF FILING AND amounts to P100,000.
p.Manufacture of radio, TV, and PAYMENT SYSTEMS April 15,2017 to June 15, 2017 is 2
communication equipment, and months;June 15 to June 30 is 30 days.
apparatus Manual e-BIR Forms Hence, the period factor shall be
q.Manufacture of rubber and plastic Efps (2/12+15/365) or 0.2077626. The
products Data entry interest factor shall be 0.2077626 x 20%
r.Manufacture of textiles Manual Electronic or 0.041555252. The interest penalty
s.Manufacture of tobacco products Electronic shall be computed as
t.Manufacture of wood and wood Filing/Submission P100,000x0.04155252=P4,155.25.
products Manual Electronic This may be computed directly as .
u.Manufacturing N.E.C. Electronic 2077626x20%XP100,000.
v.Metallic ore Mining Tax payment Illustration 2
w.Non-metallic mining and quarrying Manual Manual A taxpayer with a tax due of P100,000
3.Group C Electronic late filed on July 31, 2017. The deadline
a.Retain sale of the return was on April 15, 2016.
April 15,2015 to April 15,2017 is 1 year. of income tax in Appendix 4 for your income is reported is referred to as an
April 15,2017 to July 15, 2017 is 3 reference. accounting period.True
months.July 15 to July 31 is 16 days. PENALTIES FOR NON-FILING OR LATE OF 10.Regular accounting periods are
Hence, the period factor shall be INFORMATION RETURN calendar and fiscal-True
(1+3/12+16/365) or 1.2938356 For each failure to file a separate 11.Individuals file their income tax
XP100,000.The interest penalty shall be information return, statement of lists, return on or before April 15 of the
P25,876. or keep any record, or supply any following calendar year.True
3.Compromise Penalty information required by the Code or by 12.All taxpayers can change their
Compromise penalty is an amount paid the Commissioner on the date prescribe accounting period when there is a
in lieu of criminal prosecution over a tax therefor, unless it is shown that such change in the nature of their business,
violation. failure is due to reasonable cause not to but the BIR must be notified in all
The schedules of compromise penalty willful neglect, shall be subject to a cases.False
related to income taxes are included in penalty of P1,000 for each failure. 13.The first accounting period of a
Appendix 4 for your reference. Provided that the amount imposed for starting business will more likely to be
INTEGRATIVE ILLUSTRATION all failure during a calendar year shall less than 12 months.True
An individual taxpayer filed his 2014 not exceed P25,000.00 14.The accounting period of a deceased
income tax return with a computed tax CHAPTER 4:SELF-TEST EXERCISES taxpayer shall be terminated on
due of P100,000 on July 15, 2015. A Discussion Questions December 31 in the year of death.False
total of P20,000 withholding taxes was 1.What are the three income taxation 15.Accrual basis and cash basis are the
deducted by various income payors schemes? Briefly discuss the scope of most common accounting methods
from his gross income. each. used in practice.True.
The total amount to be paid by the 2.Discuss the nature of final income TRUE OR FALSE 2
taxpayer including penalties shall be: taxation 1.Advanced income is an item of gross
Tax due P100,000 3.What are ordinary assets? Compare income for accrual basis taxpayers.True
Less:Tax credits(withholding taxes) them with capital assets. 2.Generally, prepayments are non-
20,000 4.What capital gains are subject to deductible in the current accounting
Net tax due P80,000 capital gains taxation? period.True
Net tax due P80,000 5.What are the special features of 3.Prepayments are deductible but in
Plus:Penalties regular income taxation? the future period they expire or are
Surcharge(P80,000x25%) 20,000 6.Enumerate the instances wherein consumed in the business or trade of
Interest (P80,000x20%x3/12) 4,000 short accounting period will arise the taxpayer.True
Compromise penalty 15,000 7.What are the tax accounting methods 4.The use of different methods for
Total tax due P119,000 in reporting income? Briefly explain different businesses of the same
Note: each. taxpayer is permitted by law.True
1.The deadline of the 2014 income tax 8.What are the two types of income tax 5.Initial payment includes
return is April 15, 2015. April 15, 2015 return? downpayment and installments in the
to July 15, 2015 is a 3-month delay. TRUE OF FALSE 1 year of sale.True
2.Interest is computed from the net 1.Both active income and passive 6.Contract price is a synonymous with
amount of tax due before the 25% income do not require direct selling price.False
surcharge participation of the taxpayer in earning 7.The crop year method is an
3.The compromise penalty is taken from the income-F accounting method.True.
the table of compromise penalties for 2.There are three types of gross income 8.Under the percentage of completion
failure to file and or pay internal for taxation purposes.T method, gross income is reported
revenue tax at the time or times 3.The three tax schemes are mutually based on the cash collections from the
required by law, as follows: inclusive in coverage.False contract price.True
If the amount of tax unpaid 4.Regular income tax generally covers 9.The depreciated value of the property
Exceeds But not exceeds active income and capital gains-True upon termination of the lease
Compromise is 5.Final tax generally covers passive constitutes income to the lessee.False
… … income-True 10.The withheld taxes on the income
… 6.Capital gains arise from the sale, payment made by the taxpayers are tax
10,000 20,000 exchange, and other disposition of any credit against their income tax due.
5,000 assets.-False False.
20,000 50,000 7.There are only two types of assets for 11.There are three types of income tax
10,000 purposes of taxation-True return for each income tax
50,000 100,000 8.An ordinary asset is defined to include scheme.False.
15,000 all other assets other than capital asset- 12.All taxpayers, small or large, are
100,000 500,000 False encouraged to file their income tax
20,000 9.The technique used to measure return through EFPS system of the
You may check the schedule of income is referred to as an accounting BIR.False
compromise penalty for late payment method. The length of time over which
13.Large taxpayers are under the 6.Which is not an item of passive d.Dealers of properties can use the
supervision of the BIR Large Taxpayer income? installment method only if initial
Service.True a.Royalties payment does not exceed 25% of the
14.Non-filing and or non-payment of b.Prizes selling price.
tax is subject to penalties such as c.Deposit interest income 15.Leasehold income is recognized over
surcharges, interest, compromise and d.Professional income= the lease term in
imprisonment.False 7.Which is the following accounting a.Outright method
15.The interest on unpaid taxes is methods is most consistent with the b.Cash basis
computed on the basic tax only lifeblood doctrine? c.Spread-out method=
excluding the surcharge.True. a.Crop year method d.Percentage of completion method
16.Only large taxpayers shall file under b.Cash basis 16.Statement 1:The excess of mortgage
Efps-false c.Installment method assumed by the buyer over the basis of
17.Both manual filing and filing through d.Accrual method= the properties sold is the amount of the
e-BIR forms makes use of manual 8.These are accounting techniques or gain on the sale.
payment.True conventions used to measure income Statement 2:Any collection from an
18.Efps is fully electronic tax a.Accounting methods= installment contract where the
compliance-True b.Accounting periods mortgage exceeds the tax basis of the
19.e-BIR forms makes use of electronic c.Accrual basis properties received constitutes
data entry and filing –True d.Cash basis collection of income.
20.Efps filers may file manually when 9.These are distinct and equal time Which statement is true?
there is a BIR system downtime.True. periods over which income is measured a.Statement 1 only
Multiple Choice-Theory:General a.Accounting methods b.Statement 2 only=
Concepts b.Accounting periods= c.Both statements are true
1.Which is not a scheme in taxing c.Crop year basis d.Neither statement is true
income? d.Cash basis 17.Which is correct regarding the crop
a.Ordinary gain taxation= 10.Income is recognized when received year method?
b.Regular income taxation rather than when earned a.Crop year method is an accounting
c.Capital gains taxation a.Cash basis= period.
d.Final income taxation b.Installment basis b.Crop year method recognizes farming
2.Which is a correct statement c.Accrual basis income when the next planting season
regarding income taxes? d.Deferred payment basis commenced.
a.An item of income subjected to final 11.Income is recognized when earned c.Crop year method matches cropping
tax can still be subject to regular tax. regardless of when received expenses with the income upon
b.An item of income exempted from a.Cash harvest.=
final tax is nevertheless taxable to b.Accrual basis= d.Crop year method recognizes
regular tax. c.Installment method cropping expenses when incurred and
c.An item of income subjected to capital d.Percentage of completion harvest income when realized.
gains tax may be subject to regular 12.Initial payment means 18.Which is an incorrect statement
income tax. a.Downpayment regarding the use of accounting period?
d.An item of income exempted from tax b.Total collection within the year the a.Individuals can either choose the
is likewise exempt from capital gains tax installment sale was made.= calendar year of or fiscal year
and regular income tax.= c.Installment payments, exclusive of accounting period.=
3.Which is not a feature of final tax? downpayment, within the year b.The regular accounting period for any
a.Covers certain passive income installment sale was made. taxpayer is 12 months
b.Covers all capital gains= d.Total collections within one year from c.Individual taxpayers are not allowed
c.Withholding at source the date the installment sale was made to report income using the fiscal
d.None of these 13.Income is reported by reference to accounting period.
4.Which of the following properties the extent of project completion in d.Corporations may opt to use either
when classified as capital asset is a.Deferred payment method calendar or fiscal accounting period.
subject to capital gains tax? b.Installment method 19.Under which of the following will
a.Domestic stocks sold directly to buyer c.Percentage of completion method= short accounting period not arise?
b.Real property d.Completed contract method. a.Change of accounting period by a
c.Both a or b= 14.Which statement is correct? corporate taxpayer.
d.None of these a.Dealers of real properties can use the b.Change of accounting period by an
5.Capital asset means installment method without limitation. individual taxpayer=
a.real properties used in the business b.Dealers of personal properties can c.Death of a taxpayer
b.personal properties used in business use the installment method without d.Dissolution and liquidation of a
c.real properties not used in business limitation.= business.
d.any property, real or personal, not c.Non-dealers of properties can use the 20.Which is not a special feature of
used in business= installment method if initial payment regular income taxation?
exceeds 25% of the selling price. a.Use of accounting methods
b.Use of accounting periods c.August 15, 2013 to August 15, 2016 3.Berlin Corp. reported the following
c.Annual payment of income tax= d.August 15 to December 31,2016 data for 2015:
d.Final withholding tax at source 7.Effective March 1, 2017, Jonah, Inc. *Total net sales made to customers
Multiple Choice-Problem 4- changed its calendar year to fiscal year were P2,000,000 in cash
1:Accounting period ending every June 30. An adjustment *Goods purchased for sale totaled
1.An individual income taxpayer shall return shall be filed covering the period P1,200,000 in cash. Beginning inventory
file his or her income tax return on or a.March 1 to June 30, 2017 and ending inventory of goods for sale,
before the b.January 1 to June 30, 2017= respectively were P200,000 and
a.15th day of the fourth month of the c.January 1 to March 1, 2017 P300,000.
same calendar year. d.January 1 to December 31,2017 *1-year advanced rent of P30,000 to
b.15th day of the fourth month of the 8.During 2017, Klein Corporation apply for 2016 was received from
following calendar year.= changed its accounting period to the sublease contract
c.15th day of the fourth month of the calendar year. The adjustment return P40,000 total interest income from
same fiscal year. shall be filed on or before customers promissory note earned
d.15th day of the fourth month of the a.April 15, 2017 P30,000 of this was collected.
following fiscal year. b.July 15, 2017 *Unrealized foreign exchange gains
2.Anderson died on March 31,2017, c.April 15, 2018= from foreign currency receivable
Which is a correct statement? d.July 15, 2018 totaled P60,000
a.Anderson’s 2017 income tax return 9.Effective July 2, 2016, Rofelson Using cash basis, compute the total
shall cover January 1, 2016 to Company changed its fiscal year ending income subject to income tax
December 31. 2017. every March 31 to another fiscal year a.P990,000
b.Anderson’s 2017 income tax return ending every August 31. An adjustment d.P960,000=
shall cover January 1, 2017 to return shall be filed covering the period c.P750,000
December 31,2017. a.April 1 to July 2, 2016 d.P690,000
c.Anderson’s 2017 income tax return b.April 1 to July 1, 2016 2,000,000
shall cover January 1,2017 to March 31, c.April 1 to August 31, 2016= -1,200,000
2017= d.July 3 to August 31,2016 800,000
d.Anderson’s 2017 income tax return 10.Effective May 15, 2017, Tabuk, Inc. +30,000
shall cover April 1, 2017 to December changed its fiscal year ending every 40,000
31, 2017. April 30 to the calendar year. An 30,000
3.Mrs. Julian started business on July adjustment return shall be filed 900,000
15, 2016. What should be the coverage covering the period +60,000
of her 2016 income tax return? a.May 1 to December 31,2017= 960,000
a.January 1 to July 15, 2016 b.January 15 to December 31, 2017 4.Using accrual basis, compute the total
b.July 15, 2016 to December 31, 2016= c.January 1 to April 30, 2017 income subject to income tax.
c.July 16, 2016 to July 15, 2017 d.April 30 to December 31, 2017 a.P1,060,000
d.January 1 to December 31, 2016 Multiple Choice-Problems 4-2:Tax b.P1,000,000
4.A corporation reporting on a fiscal schemed and accounting methods c.P960,000
year ending every March 31 shall file its 1.Astercor, Inc. reported the following d.P970,000=
2017 income tax return not later than during the year: P2,000,000
a.April 15, 2017 -P400,000 proceeds of life insurance of -1,200,000
b.June 15,2017 an officer where Antercor is the 800,000
c.July 15,2017= beneficiary(P600,000 still uncollected.) 30,000
d.April 15, 2018 -P800,000 interest income, exclusive of 40,000
5.Metersbonwe Corporation started P200,000 uncollected and accrued 900,000
business on April 5, 2016 and opted to interest. 30,000
report income tax on a fiscal year -P300,000 increase in value of 40,000
ending every October 31. investment in stocks. 970,000
Metersbonwe’s first income tax return Compute the gross income under cash 5.Santiago, Inc. has the following
shall cover basis receipts during 2018:
a.April 6,2016 to October 31, 2016 a.P800,000= From service billings to clients P400,000
b.April 5,2016 to October 31, 2016= b.P1,000,000 Advances from clients 100,000
c.April 5,2016 to December 31,2016 c.P1,200,000 Total cash collection P500,000
d.April 5, 2016 to April 5, 2017 d.P1,400,000 The P100,000 advances refer to services
6.Beneco Corporation reports on a 2.Compute the gross income under which will be rendered next year. Total
calendar year. On August 15, 2016, it accrual basis uncollected billing increased from
stopped business due to persistent a.P800,000 P100,000 on December 31, 2017 to
losses. Beneco Corporation’s last d.P1,000,000= P150,000 on December 31, 2018
income tax return shall cover c.P1,200,000 Compute the gross income using cash
a.January 1 to December 31, 2016 d.P1,400,000 basis
b.January 1 to August 15,2016= a.P400,000
b.P450,000 Installment method-taxpayer, personal- How much gross income is to be
c.P500,000= dealer of household property- reported in 2016?
d.P550,000 appliances) a.P400,000
6. Compute the gross income using 10.using the accrual basis of b.P250,000
accrual basis accounting, compute Lancelot’s gross c.P116,667=
a.P400,000 income subject to income tax in 2016 d.P108,219
b.P450,000 a.P360,000 13.In the immediately preceding
c.P500,000 b.P320,000 problem, assuming Mr. Marangley is not
d.P550,000= c.P275,000 a dealer of car, how much gross income
7.In 2016, Bright Inc, sold its parking lot d.P250,000 is to be reported in 2016?
for P2,400,000 payable on installment. Cash sale(buo ang recognize na income) How much gross income is to be
The lot was previously acquired for 11.Merville is a dealer in real reported in 2016?
P1,500,000. The buyer has an properties. Merville requires 20% a.P400,000=
outstanding unpaid balance of downpayment, and the balance is b.P250,000
P1,800,000 as of December 31,2016. payable over 36 monthly installments c.P116,667
Bright’s year-end. starting on the last day of the month d.P108,219
Compute Exquisite’s gross income in following the month of sale. Merville 13.Luccio accepted a P1,000,000
2016 using the installment method. sold properties in 2015 and 2016 with construction contract in early 2016. As
a.P900,000 terms as follows: of December 31, Luccio incurred total
b.P675,000 .House and Lot No.1 was sold for construction costs of P600,000 and
c.P225,000= P1,350,000 on November 11, 2015 estimates additional P200,000 to bring
d.P112,500 .House and Lot No. 2 was sold for the project to completion. Compute
8.Equisite Corporation sold its old P1,800,000 on July 5, 2016 Luccio’s 2016 construction income using
warehouse with carrying amount (tax Both properties were sold at a gross the percentage of completion method.
basis) of P600,000 for P1,000,000. A profit rate of 40% based on the selling a.P40,000
downpayment of 15% was collected on price. b.P200,000=
July 1, 2016. Additional P150,000 Compute Merville’s gross income c.P160,000
installment payments were received as subject to income tax in 2015 and 2016, d.P400,000
of December 31, 2016. respectively. Solution:
Compute the gross profit to be reported a.P120,000;P720,000 Contract price P1,000,000
for the year 2016. b.P120,000;P864,000= Multiply by:% of completion 80%
a.P60,000 c.P540,000;P720,000 Construction revenue P800,000
b.P120,000 d.P540,000;P864,000 Less:Expense during the year P600,000
c.P400,000=(P1,000,000- DP P270,000 Construction gross income P200,000
P600,000=P400,000) Subsequent installment 30,000 15.Crevar constructs residential
d.P450,000 Total 300,000 additional personal rule properties for clients and reports
9.Lancelot is a dealer of household casual application of installment: income by the percentage of
applicanes. He reported the following in a.)SP>P1,000 completion method. In 2016, Crevar
2015 and 2016: S.)1P>25% of cash sale started a P2,000,000 construction
Sale 100% 1,350,000 contract. Details of his 2016 and 2017
2015 2016 COS 60% construction follow:
Installment sales GP rate 40%
P500,000 P800,000 300,000/1,350,000=0.22 Cash 2016 2017
Cost of installment sales sale=more than 65% Annual construction costs
250,000 440,000 200,000 payment for July/August to P200,000 P800,000
Collections December Estimated cost to finish
300,000 600,000- +360,000 800,000 250,000
100,000=P500,000 560,000 Initial payment Extent of completion
Lance’s 2016 collection is inclusive of 12.In 2016, Mr. Marangley, a dealer of 20% 80%
P100,000 accounts from 2015. car disposed a brand new sports utility Compute the construction income in
Using the installment method, compute vehicle (SUV) which costs P800,000 for 2016 and 2017.
Lancelot’s gross income subject to P1,200,000, inclusive of under the a.P200,000;P560,000
income tax in 2016. following terms: b.P200,000;P400,000=
a.P360,000 July 1, 2016-as down payment c.P400,000;P400,000
b.P320,000 P100,000 d.P400,000;P560,000
c.P250,000 Monthly installment thereafter 2016
d.P275,000= 50,000 2017
Collection/Contract price XGP= Mr. Marangley will choose whichever Construction revenue P2,000,000
225,000+50,000=275,000 favorable permissible income reporting P2,000,000
P500,000x45%=225,000 method for him. Extent of completion 20%
P100,000x50%=50,000 80%
Construction revenue P 400,000 lease contract. Just after the second Farming gross income P500,000
P1,600,000 year of the lease,Tomas renovated the Add:revenue from tree branches for
Less:Expenses 200,000 premises and made improvements at a firewood P50,000
400,000 Construction revenue in prior cost of P1,200,000. These Total income subject to tax P550,000
year improvements are expected to last for Multiple choice-Problem 4-3:Tax
Construction Gross income P200,000 12 years. Compute Rafi’s annual income compliance
P 800,000 expenses from the leasehold improvement using 1.A taxpayer was 6 months and 12 days
the spread-out method. in filing his income tax return. If he has
P 400,000 Gross income a.P50,000 P40,000 net tax due, compute the
16.Oliver entered into a 40-year lease b.P37,500 penalties in the form of interest
contract with Berna. Per agreement, c.P30,000 a.P4,263=
Oliver will construct a building on d.P20,000= b.P4,211
Berna’s lot and operate the same for 40 User Years of usage c.P4,267
years. Ownership of the building will Allocation Cost d.P5,329
transfer to Berna upon the termination -Lessee 10 10/12x1,200,000 Interest(P40,000x20%x(6/12+12/365))=
of the lease. The lease will not 1,000,000 4,263
commence until the building is -Lessor 2 2/12 x1,200,000 2.What is the total surcharge penalty?
completed. Oliver completed the 200,000/10=P20,000 a.P0
building a total cost of P40,000,000 on Total 12 1,200,000 1,200,000 b.P8,000
January 1, 2017. The building is Under the spread-out method, c.P10,000=
expected to be useful over 50 years. Anderson shall spread the P200,000 d.P20,000
Compute Berna’s income from the income over 10 periods or recognize an Surcharge(P40,000x25%)=P10,000
leasehold improvement to be reported annual income of P20,000 from the 3.A taxpayer received a notice from the
in 2017 using the spread-out method. leasehold improvement BIR to file his income tax return after
a.P200,000= 19.Hasan started raising swine for sale incurring 9 months of delay. The tax due
b.P4,000,000 by purchasing 5 gifts and a boar at a per his return is P100,000.What is the
c.P8,000,000 total purchase price of P50,000 on total surcharge penalty?
d.P32,000,000 January 2016. As of December 31, a.P0
User Years of usage 2016, Hassan’ herd grew to 15 guilts, 2 b.P20,000
Allocation Cost boars and 20 piglets. The total herd has c.P25,000
-Lessee 40 40/50 a fair value of P196,000 when sold as is. d.P50,000=
P32,000,000 During the year, Hassan earned Surcharge(50%XP100,000)=P50,000
-Lessor 10 10/50 P180,000 from selling piglets. How The non-filing is considered “willful
8,000,000/40=P200,000 much should hassan report as farming neglect” if the BIR discovered the non-
Total P40,000,000 income in 2016? filing first. This is the case when the
50 a.P326,000 taxpayer received a notice from the BIR
Under the spread-out method, b.P180,000= to file return. If the taxpayer filed a
Anderson shall spread the P8,000,000 c.P146,000 return before the receipt of such notice,
income over 40 periods or recognize an d.P130,000 the same is considered simple neglect
annual income of P200,000 from the subject to the 25% surcharge.
leasehold improvement from 2017 4.What is the total interest penalty?
through year 2056. 20.Pedro,a farmer, uses the crop-year a.P12,500
17.In the immediately preceding method in reporting his income from b.P14,900
problem, assume that the building was long-term crops. The following data are c.P15,000=
completed on July 1, 2017, what is the relevant to his farming operations in d.P25,000
income using outright method 2017: Interest (P100,000x20%x9/12)=15,000
a.P4,000,000 -Sales of crops harvested, P900,000 5.Compute the compromise penalty
b.P8,000,000 -Expenses on harvested crops, P400,000 a.P10,000
c.P32,000,000 -Expenses on maturing crops, P200,000 b.P15,000=
d.P40,000,000= -Expenses on newly crops, P100,000 c.P20,000
Outright method -Sales of tree branches for firewood, d.P30,000
Under the plain wordings of Section 49 P50,000 If the amount of tax unpaid
of Revenue Regulations No. 2, Anderson Pedro uses the crop year method in Exceeds But not exceeds Compromise
shall recognize the entire P40,000,000 reporting income. Compute Pedro’s is
fair value of the improvement as gross total income subject to tax. … … …
income upon completion of the a.P240,000 10,000 20,000 5,000
improvement in 2017. This is not b.P340,000 20,000 50,000 10,000
income in its totality, but this is the c.P540,000 50,000 100,000 15,000
amount referred to by the regulation d.P550,000= 100,000 500,000 20,000
18.Tomas leases an office space from Proceeds of harvest P900,000 6.Mani Pakyaw failed to pay file his
Rafi, Inc. in a non-renewable 10-year Less: Cropping expenses 400,000 income tax return for the year 2916
which should have been filed on or tax rates is extremely crucial to your Items of passive income are earned
before April 15, 2017. The BIR sent him mastery of income taxation. with very minimal involvement from
a notice to file and pay his taxes on or After finishing this chapter, readers are the taxpayer and are generally irregular
before July 18, 2017. Mani Pakyaw filed expected to demonstrate: in timing and amount. Unlike items of
return showing a basic tax due of a.Undestanding and appreciation of the active income, they are not usually
P1,000,000. Compute the total interest features and scope of final tax specifically monitored by taxpayers.
penalty. b.Mastery of those certain passive When not recorded by the taxpayers,
a.P0 income subject to final tax and their their existence can be difficult to
b.P51,644= corresponding final tax rates. predict while their actual amount may
c.P64,555 c.Mastery of the general final tax rates be difficult to determine. Thus, the final
d.P59,863 on certain non-residents and their withholding at source is the most
exxceptions. favored scheme in taxing items of
Interest penalty d.Knowledge of the other applications passive income.
(20%XP1,000,000X(3/12+3/365))=51,64 of the final income tax scheme Non-resident persons not engaged in
4 business in the Philippines
7.Mr. Pakyaw must pay a surcharge of FEATURES OF FINAL INCOME TAXATION Non-resident persons not engaged in
a.P0 1.Final tax trade of business in the Philippines,
b.P250,000 2.Tax withholding at source such as non-resident aliens not
c.P500,000= 3.Territorial imposition engaged in trade or business (NRA-
d.P1,000,000 4.Imposed on certain passive income NETBS) and non-resident foreign
Surcharge(50%x1,000,000)=500,000 and persons not engaged in business in corporations (NRFCs), have high risk of
8.Rich Goma Corporation failed to file the Philippines non-compliance. These taxpayers
its income tax return for the fiscal year THE FINAL WITHHOLDING SYSTEM usually do not have principal offices or
ending August 31, 2016. On June 6, The final withholding system imposes fixed places of business in the
2017, it filed an income tax return with upon the persons making income Philippines. Their distance and absence
a basic tax still due and payable for the payments the responsibility to withhold from the Philippines make it very
fiscal year amounting to P500,000 the tax. The taxpayer receives the unlikely for tax compliance to be
Compute the interest penalty to be income net of tax and there would be expected.
imposed by the BIR no need for him to file an income tax Thus, the law subjects them to final
a.P47,694= return to report the same. income tax. Philippine resident shall
b.P52,845 The final withholding system is withhold the following final tax from
c.P64,275 inherently territorial. It applies only to their gross income, active or passive,
d.P35,873 certain passive income earned from from all sources within the Philippines:
sources within the Philippines. The Non-resident General final tax
Interest (P500,000 Philippine government cannot compel person not rate
x20%x10/12x6/365)= non-resident subjects to foreign engaged in trade
9.Compute the total assessment to be countries to withhold the tax as this or business
paid, excluding the compromise would amount to infringement of Non-resident 25%
penalty. foreign sovereignty. Hence, all items of alien not
a.P672,694= income earned from sources abroad, engaged in trade
b.P677,845 passive or active, are subject to tax or business
c.P689,275 under the general scope of the regular Non-resident 30%
d.P660,873 income tax. foreign
Tax due P500,000 Rationale of Final Income taxation corporation
Less: penalties The final withholding tax is built upon
Surcharge (25%x500,000) 125,000 the taxpayer and government PASSIVE INCOME SUBJECT TO FINAL TAX
Interest 47,694 convenience. It relieves the taxpayer of 1.Interest or yield from bank deposits
Total tax due P672,694=a the obligation to file an income tax or deposit substitute
CHAPTER 5 return. This is very convenient for 2.Domestice dividends, in general
FINAL INCOME TAXATION taxpayers who are limited by distance, 3.Dividend income from a Real Estate
Chapter Overview and Objectives time and cost to comply. For the Investment Trust
This chapter discusses the features of government, the final withholding 4. Share in the net income of business
final income taxation, the items of gross system is the most convenient and partnership, taxable associations, joint
income, and the class of taxpayers effective system in collecting taxes on ventures, joint accounts, or co-
subject to final income tax income where there is high risk on non- ownership
Final tax is one of the exceptions to the compliance or tax evasion. 5.Royalties, in general
scope of regular income tax. An Under the NIRC, final income tax is 6.Prizes exceeding P10,000
excellent understanding of the items of imposed on certain passive income and 7. Winnings
passive income and those taxpayers upon non-resident persons not engaged 8. Informer’s tax reward
subject to final tax including their final in business in the Philippines.
Passive Income
9.Interest income on tax-free corporate 2-year time deposit 12,000 previously untaxed or exempted
covenant bonds x20% 2,400 interest income will be subjected to the
FINAL TAX ON INDIVIDUALS AND 5-year time deposit 40,000x0% 0 following final taxes upon pre-
CORPORATIONS Final withholding tax termination:
Unless otherwise indicated, the final tax P4,000 Holding period Final tax
rates to be discussed in the following Corporate taxpayers P60,000 x 20% Less than 3 years 20%
sections apply to all taxpayers P12,000 3 years to less 12%
(individuals and corporation) other The exemption of individuals on interest than 4 years
than: income on long-term deposits is 4 years to less 5%
a.Non-resident alien not engaged in anchored on the fact that long-term than 5 years
trade or business (NRA-NETB), and deposits are usually channeled to the 5 years or more 0%
b.Non-resident foreign financing of long-term projects such as Illustration 1
corporation(NRFC) infrastructures, property developments, On January 1, 2011, Alice invested
INTEREST INCOME OR YIELD and other construction projects which P1,000,000 in Baguio Bank’s 5-year time
Interest income or yield from local are deemed essential to the deposit. The deposit pays 10% interest
currency bank deposits or deposit development of the country. Note that annually. Alice pre-terminated the
substitutes are subject to final tax as exemption is limited only to individuals deposit on July 1, 2014.
follows: to the exclusion of corporations. The final tax on pre-termination will be
Recipient Illustration 2 computed as follows:
Source of Individ Corporati A resident taxpayer received a P16,000 2011 interest income (P1,000,000x10%)
interest uals ons interest income from a bank. Determine P100,000
income the final tax withheld at source. 2012 interest income (P1,000,000x10%)
Short term 20% 20% Solution: 100,000
deposits Gross Interest income (P16,000/80%) 2013 interest income (P1,000,000x10%)
Long-term Exempt 20% P20,000 100,000
deposits/invest * Multiply by:final tax rates 20% 2014 accrued interest income
ment Final tax withheld P4,000 (P1,000,000x10%x6/12 months) 50,000
certificates Illustration 3 Total interest income
Note:*Exemptions does not include Banko Negro incurs the following P350,000
NRA-NETB interest in its savings and time deposits Final tax rate applicable to less than 4-
Short-term deposits are those for a accounts from the following depositors: year pre-termination 12%
period of less than five years. Long-term Depositors Amount Final tax P42,000
deposits or investment certificates refer Resident individuals P600,000 The net proceeds of the deposit and
to certificate of time deposit or Resident and domestic corporations accrued interest to be released to the
investment in the form of savings, 800,000 depositor upon pre-termination shall
common or individual trust funds, Non-resident aliens not engaged in be:
deposit substitutes, investment business 200,000 Principal balance P1,000,000
management accounts, and other Non-resident corporation 100,000 Accrued interest for 2014 50,000
investments with a maturity of not less Total accrued interest expense Final tax to be withheld (42,000)
than five years,the form of which shall P1,700,000 Net proceeds to be released to the
be prescribe by the BSP and issued by Required:Compute the total final depositor P1,008,000
banks only (not by non-bank financial income tax to be withheld by Banko Savings or time deposits with
intermediaries or finance companies) to Negro. cooperatives are not subject to final tax
individuals in denominations of P10,000 Solution: The final tax is limited to banks and
and other denominations as may be Depositors Amount Rate Final tax shall not be applied with time and
prescribed by the BSP. (RMC 18-2011) Resident individuals P600,000 x20% savings account deposit maintained by
Illustration 1 P120,000 members with cooperatives and by
A taxpayer earned the following income Resident and domestic corporations primary cooperatives with their
from various time deposits: 800,000 x 20% federations.(Dumaguete Cathedral
6-month time deposit P8,000 P160,000 Credit Cooperatives vs. CIR, G.R.
2-year time deposit 12,000 NRA-NETB 200,000 X25% 182772)
5-year time deposit 40,000 50,000 Other applications of the final tax on
Total interest income P60,000 NRFCs 100,000 x30% 30,000 interest
Required: Compute the final tax if the Total accrued interest expense 1.Deposit substitute
taxpayer is an individual and if a P1,700,000 2.Government securities
corporation. P360,000 3.Money market placements
Solution: Tax on pre-termination of long-term 4.Trusts funds
Individual taxpayers deposits of individuals 5.Other investments evidenced by
6-month time deposit P8,000x20% If the deposit or investment placement certificates prescribed by the Bangko
P1,600 of individual taxpayers is pre- Sentral ng Pilipinas (BSP)
terminated before 5 years, any
Deposit substitute means an alternative Is the bank account is jointly in the 4.Stocks dividends-paid in the stocks of
form of obtaining funds from at least 20 name of a non-resident and a resident the corporations
persons at any one time other than taxpayer. 50% of the interest shall be 5. Liquidating dividends-distribution of
deposits through the issuance, exempt while the other 50% shall be corporate net asset
endorsements, or acceptance of debt subject to the 7.5% final tax .(RR10-98) As a rule, dividends are income subject
instruments for the borrowers own Illustration to tax. However, the following income
account, for the purpose of relending or Mr Siman is an Overseas Filipino for taxation purposes:
purchasing of receivables and other Worker, He deposits all his savings in a 1.Stock dividends
obligations, or financing their own savings account under the foreign Stock dividends representing transfer of
needs or the needs of their agent or currency deposit unit (FCDU) of a surplus to capital account shall be
dealer. domestic bank. During the month, the subject to tax. Stock dividends are in
Government debt instruments and savings deposit account earned $1,000 the form of increase in corporate value
securities including Treasury bonds, interest equivalent to P41,500. (i.e., capital gain) which should be
Treasury bills, and treasury notes shall Scenario 1: Mr. Siman deposited his properly taxable when realized through
be considered as deposit substitute savings through the account of his disposal or sale of the stocks
irrespective of the number of lender at resident wife. investment
origination if such debt The final tax shall be P3,112.50 The distribution of stocks of another
instrumentalities and securities are to computed as follows: corporation as dividends is a taxable
be traded or exchanged in the Interest income P41,500 property dividend and not a stock
secondary market. Final tax rate 7.5% dividend
Foreign currency deposit with foreign Final tax P3,112.50 2.Liquidating dividends
currency depositary banks Scenario 2:Mr. Siman deposited his Under the NIRC, the receipt of
The interest income from foreign savings through a joint account with his liquidating dividends in not viewed as
currency deposit under the foreign resident wife. income but as exchange of properties.
currency deposit system or expanded The final tax shall be P1,556.25 When the liquidating dividends exceeds
foreign currency deposit system by computed as follows: the costs of the investments, the excess
residents is subject to a final tax of Interest income P41,500 is a taxable capital gain, subject to
7.5%. Portion taxable 50% regular income tax. Any loss is
Taxpayer Individual Corporations Taxable interest income P20,750 deductible only to the extent of capital
s Multiply by: final tax rate 7.5% gain.
Resident 15% 15% Final tax P1,556.25 TAXABILITY OF STOCK DIVIDENDS
s Scenario 2:Mr. Siman deposited his Normally, stock dividends are exempt
Non- Exempt Exempt savings account through his own from income tax. Exceptionally, stock
residents account dividends are subject to tax at the fair
In this case, the interest income shall be value of the stocks received under the
Note: exempt from final tax. following conditions:
1.Resident taxpayers include resident Interest income subject to regular tax a.Subsequent cancellation and
citizens, resident aliens, domestic Interest income from the following redemption
corporations and resident foreign sources is subject to regular income tax, If a corporation cancels or redeems
corporations not to final tax: stock issued as a dividend at such time
2.Non-residents taxpayers is include 1.Lending activities, whether or not in and in such manner as to make the
non-resident citizens, non-resident the course of business distribution and cancellation
aliens and non-resident foreign 2.Investments in bonds redemption, in whole or in part,
corporations 3.Promissory notes equivalent to the distribution of a
3.It should be emphasized that NRA- 4.Foreign sources, whether bank or taxable dividend, the amount so
NETBs and NRFCs are also exempt. non-bank distributed shall be taxable to the
4.There is no long-term or short-term 5.Penalty for legal delay or default extent it represent a distribution of
classification of foreign currency DIVIDENDS earnings or profit.
deposits. “Dividends” means any distributions For instance, a corporation declared
The reduced final tax rates on interest made by a corporation to its stock dividends and immediately called
income on foreign currency deposit and shareholders out of its earnings or stock dividends for redemption and
the exemption of non-resident profits and payable to its shareholders, cancellation. This act is equivalent to
depositors are intended to encourage whether in money or in other property. declaration of cash dividends.
the deposit of foreign currencies in our (Sec 73, NIRC) b.If it leads to substantial alteration in
banks which will be used in the Types of dividends: ownership in the corporation
financing of our international trades. 1.Cash dividends-paid in cash Substantial alteration in ownership in a
Our Philippine Peso is not a globally 2.Property dividends-paid in no-cash corporation may occur when stock
accepted currency. Our foreign trade properties including stocks or securities dividends are given in lieu of cash
will be limited without adequate foreign of another corporation dividends or when the corporation
currency reserves in our banking sector. 3.Scrip dividends-those paid in notes or declared an optional stock or cash
Joint accounts on forex deposits. evidence of indebtedness corporation dividend
Stock dividend or stock split Total dividends P750,000 domestic corporations since business
Stock dividend is a capitalization of The total final tax to be withheld by partnerships are considered
earnings while stock split results in Aborian Company shall be: corporations under the NIRC. However,
reduction in the par value of stock and Shareholders Dividends Rate the exemption does not extend to
an increase in the number of shares of Final tax dividends received by general
shareholders. Assuming a 2-for-1 split, a Resident aliens and citizens professional partnership, exempt joint
shareholder holding one P50-par value P500,000x10% P50,000 ventures and exempt co-ownership
stock will be given two P25-par value NRAs engaged in trade or business because they are not considered
stocks. While stock dividend may be 100,000 x20% 20,000 corporations under the NIRC.
taxable under certain conditions, stock NRAs-NETBs 50,000 x25% 12,500 On the other hand, the exemption of
split will never be subject to income tax. NRFCs 100,000 x30% 30,000 inter-corporate dividend does not apply
DIVIDEND TAX RULES Total P750,000 P112,500 to the share of a corporation from the
Recipient Historical dividend tax rates net income of a business partnership
of The imposable final tax rates vary due to absence of express legal
dividends depending on the source of the exemption. Exemption is restricted to
Source of Individua Corporatio dividends declared: dividend declaration only.
dividends ls ns Source Final tax Dividends from Cooperatives
Domestic 10% final Exempt Earnings before January 1,1998 Under RA 9520, the distribution by an
Corporatio tax Exempt exempt cooperative to its members
ns Earnings from 1998 6% either representing interest on capital
Foreign Regular Regular tax Earnings from 1999 8% or as patronage refunds shall not be
Corporatio tax Earnings from 2000 and thereafter 10% subject to tax.
ns Any distribution made to shareholders ENTITIES TAXABLE AS CORPORATIONS
or members of a corporation shall be ARE SUBJECT TO 10% FINAL TAX
Note: deemed to have been made from the The 10% final withholding tax also
1.A NRA-ETB is subject to a 20% final most recently accumulated profits or applies to dividends or share in the net
tax on dividend, not to the usual 10%, surplus and shall constitute a part of income of entities considered
but an NRA-NETB is subject to a 25% the annual income of the distributive corporations under the NIRC and
final tax. for the year in which received.(Sec. special laws, such as:
2. A NRFC is not exempt but is subject 73( C), NIRC) 1.Real Estate Investment Trusts
to the 30% general final tax rate. Exempt Dividends 2.Business partnerships
However, the imposable dividend tax 1.Inter-corporate dividends 3.Taxable associations
rate shall be 15% when the tax sparing 2.Dividends from cooperatives 4.Taxable joint ventures, joint accounts
rule applies. This will be discussed later. Inter-corporate dividends or consortia
Illustrative 1 Inter-corporate dividends received by a 5.Taxable co-ownerships
Calbayog Company declared a total of domestic corporation are exempted Real Estate Investment Trust or REIT
P2,000,000 dividends. P800,000 is due under the NIRC to minimize double A REIT is a publicly listed corporation
to corporate shareholders while taxation principally for the purpose of owning
P1,200,000 is due to individual Illustration income-generating real estate assets.
shareholders. B, Inc. owns 100% of A Corp. During the The following recipients of REIT
The final tax to be withheld by Calbayog year, A Corp. declared P100,000 dividends are exempt from the final tax:
Company shall be: dividends to B, Inc., in turn, declared a.Non-resident alien indivudals or non-
Shareholders Amount Rate the same dividends to its shareholders. resident foreign corporations entitled to
Amount The following table illustrates the claim preferential tax rate pursuant to
Individual shareholders P1,200,000 double taxation: applicable tax treaty.
x10% P120,000 A Corp. B Inc. b.Domestic corporations or resident
Corporate shareholders Dividends declared P100,000 foreign corporations
800,000x0% P0 P90,000 c.Overseas Filipino investors-exempt
Final tax P120,000 Less:10% dividends tax 10,000 9,000 from REIT dividend tax until August
Illustration 2 Aborian Company Net Dividends P90,000 12,2018 (7 years from the effectivity of
declared a total of P1,000,000 dividends P81,000 RR13-2011 which took effect on August
in March 2014. An analysis of the This is a form of direct duplicate 12, 2011)
recipient shareholders is as follows: taxation. To eliminate the impact of Business partnership,taxable
Shareholders Amount double taxation, inter-corporate associations, joint ventures, joint
Resident aliens and citizens P500,000 dividends such as those declared by A accounts or co-ownerships
NRAs engaged in trade or business Corp. to B Inc. is exempted from final Under Sec. 73 of the NIRC, the net
100,000 tax. When the dividend finally falls to an income of these entities is deemed
NRAs not engaged in trade or business individual shareholder, the 10% final tax constructively received by the partners,
50,000 applies. members, or ventures, respectively, in
Non-resident corporations 100,000 This exemption extends to dividends the same year the net income is
received by business partnerships from reported. Hence, the 10% final tax
applies at the point of determination of Books, literary works, and musical The taxation of prizes varies. Prizes may
the income, not at the point of actual compositions Individuals be exempt from income tax or subject
distribution. Corporations to either final tax or regular income tax.
Share in business partnership net 10% final Exempt prizes
income tax 20% final tax 1.Prizes received by a recipient without
The “share in net income” includes the Other sources 20% final tax any effort on his part to join a contest.
share in the residual profit and 20% final tax Examples include prizes from such
provisions for salary, interest and bonus Note: awards as Nobel prize, Most
to a partner. However, if the provisions 1.Under the regulations, the 10% Outstanding Citizen, Most Benevolent
for salaries, interests and bonuses are preferential royalty final tax on books Citizen of the year, and similar awards.
expensed as such in the books of the and literary works pertain to printed 2.Prizes from sports competitions that
partnership, they are subject to regular literatures. Royalties on books sold on are sanctioned by their respective
tax to the receiving partner, not to the e-copies or CDs such as e-books are national sport organizations
final tax. In this case, only the share in subject to the 20% final tax. Requisite of exemption
the residual income after such 2.Royalties on cinematographic films 1.The recipient was selected without
provisions is subject to final tax. and similar works paid to NRA-ETBs, any action on his part to enter the
Illustration NRA-NETBs or NRFCs is subject to a final contest.
The partnership profit distribution of tax of 25%. 2.The recipient is not required to render
partners Andy and Mar based on their Passive vs. Active royalties substantial future services as a
agreed profit distribution scheme is as Royalties of a passive nature such as condition to receiving the price or
follows: royalties of claim owners or land reward.
Andy Mar owners of mining properties, royalties Taxable prizes
Salaries to industrial partner P40,000 of inventors from companies that For individual income taxpayers, taxable
P0 manufacture and sell their invention, prizes are subject to either final tax or
Interest to capitalist partner - and royalty from licensing agreements regular tax depending on the amount of
P12,000 that transfers the use of trademark or the price. However, there is no final tax
Bonus to industrial partner 25,000 - technology are subject to 20% final tax. imposition on corporate prizes under
Residual profit sharing 8,000 When royalties accrues from an the NIRC. Hence, the same must be
24,000 undertaking where the taxpayer has subject to regular income tax.
Profit sharing P73,000 active involvement, it is an active Amount of taxable prize Recipient
P36,000 income subject to the regular income Prizes exceeding P10,000 Individuals
Assuming the salaries, interest and tax. Corporations 20% final tax
bonus are not expense in the book, the Illustration Regular tax
10% final tax shall be: E-Soft Inc. develops application Prizes not exceeding P10,000 Regular
Profit sharing P73,000 P36,000 programs for establishments. These tax Regular tax
Multiply by: Final tax rate 10% programs were individually tailored to Recall also that final taxation does not
10% meet specific requirements of the apply to foreign passive income; hence,
Final tax P7,300 establishments and required upgrades, prizes from foreign sources are subject
P3,600 occasional troubleshooting, and to the regular income tax.
Note:A PARTNER, MEMBER, OR adjustments for problems. The WINNINGS
VENTURE WHO IS AN NRA-ETB, NRA- developer receives 1% of the sales of For individual income taxpayers,
NETB, OR NRFC SHALL BE SUBJECT the establishments as royalty. winnings received from sources within
RESPECTIVELY TO 20%, 25% AND 30% E-soft also developed a utility program the Philippines are generally subject to
FINAL TAX RATE. and assigned it to an e-marketer which 20% final tax with the exception of the
The Improperly Accumulated Earnings sells the utility program through the following exempt winnings:
Tax Internet. E-Soft receives 30% royalty on 1.Philippin Charity Sweepstakes Office
Domestic corporations cannot avoid the each copy of the program sold. (PCSO) winnings
dividends tax by simply not declaring The royalties from application programs 2.Lotto winnings
dividends. Corporations which are active income subject to regular Similar to prizes, there is no final tax
accumulate earnings beyond the income tax. The royalty from the utility imposed on corporate winnings under
reasonable needs of business will be programs is passive income subject to the NIRC. Winnings that are not
imposed the 10% Improperly final withholding tax, but if the e- subjected to final tax by the payor
Accumulated Earnings Tax, a penalty marketer is not a resident in the should be reported as part of the
tax. This topic will be discussed in detail Philippines, the passive income from regular income. Also, winnings from
in Chapter 15-B. abroad shall be subject to regular tax. foreign sources are subject to regular
ROYALTIES Royalties, active of passive, earned from income tax.
Passive Income received from sources sources abroad are subject to regular Recipient
within the Philippines is subject to the income tax. Types of Winnings Individuals
following final tax rates: PRIZES Corporations
Source of passive royalties Recipient PCSO or Lotto winnings Exempt
Exempt
Other winnings, in general 20% final tax any tax on the interest shall be subject Special aliens are NRA-NETBs employed
Regular tax to a final withholding tax of 30%. by regional or area headquarters and
Note:The exemption on PCSO or lotto Bond investor regional operating headquarters of
winnings also extends to NRA-NETBs Individuals multinational companies, offshore
and NRFCs. Corporations banking units, or petroleum service
TAX INFORMER’S AWARDS Tax on interest income on tax-free contractors or subcontractors. Special
A cash reward may be given to any corporate covenant bonds 30% final aliens are subject to a final tax of 15%
person instrumental in the discovery of tax Regular income tax on gross income from their employees.
violations of the National Internal Note: Illustration:NRA-NETBs
Revenue Code or discovery and seizure 1.The final tax applies to all individuals, In 2014, Mr. Tih Wong, an NRA-NETB, is
of smuggled goods. The tax informer’s regardless of classifications a consultant to Raha Humabon
reward is subject to 10% final tax. 2.There is no similar final tax provision Manufacturing Company (RHMC), a
Requisite of Tax Informer’s Reward: for corporate recipients of “tax-free” domestic manufacturer. RHMC also
1.Definite sworn information which is interest; hence, the regular income tax pays MR. Wong royalty for using his
not yet in the possession of the BIR shall apply invention. During the year, Mr. Wong
2.The information furnished lead to the EXCEPTIONS TO THE GENERAL FINAL purchased shares of RHMC and sold
discovery of fraud upon internal TAX ON NON-RESIDENT PERSONS NOT them directly to a buyer.
revenue laws or provisions thereof. ENGAGED IN TRADE OR BUSINESS IN Royalties from invention P300,000
3.Enforcements results in recovery of THE PHILIPPINES Professional fees 1,000,000
revenues, surcharges, and fees and/or NRA-NETB Gain on sale of domestic stocks directly
conviction of the guilty party or NRFC to a buyer 40,000
imposition of any fine or penalty. General Final Tax Rate 25% 30% Required:
4.The informer must not be a: Exceptions: Compute the total final tax withheld at
a.BIR official or employee 1.Capital gain on sale of domestic stocks source
b.Other public official or employee directly to buyer 5%-10% Solution:
c.relative within the 6th degree of capital gains tax 5%-10% Royalties from invention P300,000
consanguinity of those officials or capital gains tax Professional fees 1,000,000
employee in a, and b. 2.Rentals on cinematographic films and Total gross income P1,300,000
Amount of Cash Reward-whichever is similar works 25% of rentals 25% Multiply by:Final tax on NRA-NETB 25%
the lower of the following per case: of rentals Total final withholding tax
1.10% of revenues, surcharges, or fees 3.Rentals of vessels 25% of rentals P325,000
recovered and or fine or penalty 4.5% of rentals Note
imposed and collected or. 4.Rentals of aircrafts, machineries, and 1.The final tax applies on gross income,
2. P1,000,000 other equipments 25% of rentals whether active or passive. The same
The amount of cash reward is subject to 7.5% of rentals. rule applies with NRFC except that the
10% final withholding tax which shall be 5.Special aliens(special employees) final tax rate is 30%
withheld by the government. 15% of gross income from employer 2.Mr. Wong shall file a capital gains tax
Illustration N/A returns for the gain on the sale of
Ms. Kirsten provided information to the 6.Lotto and PCSO Winnings Exempt domestic stocks,
BIR leading to the recovery of Exempt The tax sparing rule
P12,0000,000 unpaid taxes. The cash 7.Interest income under the foreign NRFCs shall be subject to a 15% final tax
shall be computed as follows: currency deposit system Exempt on dividend income instead of the 30%
10% cash reward (P12,000,000x10%) Exempt general final tax if the country of
P1,200,000 8.Interest on foreign loans N/A 20% domicile of the NRFC credits against the
Cash reward limit P1,000,000 9.Dividend income 25% 15% tax due of such NRFC taxes presumed to
Cash reward (whichever is lower) if tax sparing rule is applicable have been paid by such as NRFC from
P1,000,000 10.Tax on corporate bonds 30% 30% the Philippines equivalent to 15% of the
Less:10% final withholding tax CAPITAL GAINS TAX dividends.
100,000 As a rule, NRA-ETBs and NRFCs do not In applying the tax sparing rule, the
Net amount to be released to the tax file tax returns. Exceptionally,NRA- Supreme Court ruled that the NIRC
informer P900,000 NETBs and NRFCs are required to file does not require that the foreign law of
TAX-FREE CORPORATE COVENANT income tax returns to report their gain the non-resident corporation must give
BONDS from dealings in domestic stocks a deemed paid tax credit for dividend
Interest income of non-resident aliens, directly to buyers. Ownership of the equivalent to the percentage points
citizens of the Philippines on bonds, stocks shall not be transferred to the waived by the Philippines pointing that
mortgages, deeds of trusts, or other assignee without the required return the NIRC merely require the country of
similar obligations of domestic or and tax clearance (Certificate the NRFC to a deemed paid tax
resident foreign corporations with tax- Authorizing Registration or CAR) from equivalent to that waived by the
free or tax-deduction provision where the BIR that the tax on the transfer has Philippines.(CIR vs. Procter and Gamble
the obligor shoulders in whole or in part been paid. Philippines Manufacturing Corporation
Special Aliens and the CTA (G.R.66836))
Thus, the requirement of the tax tax on income payments such as the month following the month in
sparing rule is deemed satisfied if the interest income on loans from offshore which withholding was made with:
country to which the NRFC is domiciled banking units (OBUs) and expanded a.The authorized agent bank of the
imposes no tax on dividends from foreign currency deposit units( FCDUs). revenue district office having
foreign sources.(BIR Ruling 104-2012, The final taxation of FCDUs, EFCDUs and jurisdiction over the withholding
March 22, 2012) OBUs will be discussed in Chapter 15-A. agent’s place of business
Illustration:The tax sparing rule with INCOME PAYMENTS TO SUB- b.In places where there are no
NRFCs CONTRACTORS OF PETROLEUM SERVICE authorized agent banks, to the revenue
An NRFC is due to receive a dividend of CONTRACTORS collection officer
P1,000,000 from a domestic Under PD 1354, every subcontractors, c.The authorized city or municipality
corporation. The final tax to be imposed whether domestic or foreign, entering agent’s place of business is located.
by the Philippines which shall be into a contract with a service contractor Deadline for Efps FILING
withheld by the domestic corporation engaged in petroleum operations in the In accordance with the schedule set
shall be 15%, not 30%, if the country of Philippines shall be liable to a final forth in RR No. 26-2002, the deadline
domicile of the NRFC also reduces its income tax equivalent to eight percent for filing of return is as follows:
income tax upon the P1,000,000 (8%) of its gross income derived from Group A-Fifteen (15) days following the
dividend by at least 15%, the dividend such contract, such tax to be in lieu of end of the month
tax percentage waived by the any and all taxes, whether national or Group B-Fourteen (14) days following
Philippines from the 30% general final local. the end of the month
tax rate. If the country of the NRFC does Provided, however, that any income Group C-(Thirteen (13) days following
not reduce its tax on the dividend by received from all other sources within the end of the month
atleast 15% the Philippines shall impose and without the Philippines in the case Group D-Twelve (12) days following the
the 30% final tax. of domestic subcontractors and within end of the month
OTHER APPLICATIONS OF FINAL the Philippines in the case of foreign Group E-Eleven (11) days following the
INCOME TAX subcontractors shall be subject to the end of the month
1.Compensation income of filipinos regular income tax under the NIRC. Note:Please check the groupings of
qualified as special aliens The term “gross income” means all taxpayers under Efps in Chapter 4.
2.Fringe benefits of managerial or income earned or received as a result of Penalties for Late Filing or Remittance
supervisory employees the contract entered into by the of Final Income taxes withheld
3.Income payments of residents other subcontractor with a service contractor The same penalties for late payment of
than depository banks under the engaged in petroleum operations in the income taxes as discussed in Chapter 4
expanded foreign currency deposit Philippines under Presidential Decree apply for non-withholding or non-
system (EFCDS) to offshore banking No. 87. remittance of final taxes.
units (OBUs) and expanded foreign Note that the 8% final tax applies to ENTITIES EXEMPT FROM FINAL INCOME
currency deposit units (EFCDUs) subcontractors, whether individuals or TAX
4.Income payments to oil explorations corporations, resident or non-resident. 1.Foreign governments and foreign
service contractors or sub-contractors Petroleum service contractors are government-owned and controlled
FILIPINO EMPLOYEES QUALIFIED AS subject to the regular income tax. corporations
SPECIAL ALIENS Persons or entities contracted by a 2.International missions or
Filipino employees occupying the same petroleum service contractor to locally organizations with tax immunity
position as those held by special aliens supply goods and materials that are 3.General Professional partnership
subject to 15% final tax may opt to be required by and in, or that are 4.Qualified employee trust fund
taxed at the same 15% final tax. This inherently necessary or incidental to, its The first two categories are exempt on
will be discussed in detail in Chapter 10. exploration and development of grounds of international comity.
FRINGE BENEFIT TAX petroleum mineral resources and are General professional partnerships and
Fringe Benefits include all entitled to the preferential 8% final tax qualified employee trust funds are
remunerations under an employer- on their gross income derived from expressly exempt from any income tax
employee relationship that do not form such contracts. imposed under the NIRC.
part of compensation income. The (BIR Ruling No. 024-2001, June 13, These entities are exempt not only to
fringe benefits of managerial and 2001) final tax but also to capital gains tax and
supervisory employees are subject to a FINAL WITHHOLDING TAX RETURN regular income tax.
final fringe benefits tax. The final withholding tax return (BIR A comprehensive summary of final tax
This will be discussed in detail in Form 1601-F), Monthly Remittance rates is presented in Appendix 1.
Chapter 11. Return of Final Income taxes CHAPTER 5:SELF-TEST EXERCISES
INTEREST AND OTHER INCOME withheld,shall be filed in triplicate by Discussion Questions
PAYMENTS TO DEPOSITORY BANKS every withholding agent or payor who is 1.What are the features of income
UNDER THE EXPANDED FOREIGN either an individual or corporation taxation?
CURRENCY DEPOSIT SYSTEM DEADLINE AND PLACE FOR MANUAL 2.Explain withholding at source
Residents other than depository banks FILING 3.Enumerate the passive income
under expanded foreign currency The return shall be filed and the tax subject to final tax and their
deposit system,shall withhold 10% final shall be paid or before the 10th day of corresponding tax rates.
4.What are the two types of taxpayers 3. Individuals, except corporations, are b.Final tax applies only on certain
who are subject to final tax rates on all subject to final tax on winnings passive income earned within the
income from within the Philippines? abroad.False Philippines-
What are the general tax rates 4. Winnings which are not more than c. Items of income exempt under final
applicable to each? P10,000 in amount are subject to the tax are subject to regular tax.
5.Enumerate the requisites of the tax regular income tax.-False d.Final income tax applies to all passive
informer’s reward. 5. PCSO or lotto winnings are exempt income from Philippines sources
6.Discuss the procedures and deadline from all types of income tax-True 3. Interest income from which of the
in remitting the final withholding taxes. 6.Special aliens do not file a regular following sources is subject to final
True or false income tax return on their income tax?
1.Generally, interest income from non- compensation income.-True a.Lending
bank sources is subject to regular 7. The final tax on interest on tax-free b.Mortgage loans
income tax. T corporate covenant bonds applies only c.Bond Investment
2. Foreign income is subjected to final to resident individual or corporate d.Money Market placements-
tax if the taxpayer is taxable on global taxpayers.-False 4.The final tax does not apply to interest
income.False 8. NRA-NETB and NRFC are subject to on (select the best answer)
3.Items of passive income from abroad final tax only to passive income from a.Long-term deposit
are subject to final tax-False sources in the Philippines. False b. Trust funds
4.Interest income on government 9. NRA-NETB and NRFCs are not c.Deposit substitute
securities are subject to final tax.T required to file the regular income tax d. Promissory notes
5.All items of passive income on return.-True 4. The final tax does not apply to
government securities are subject to 10. Residents shall withhold 10% final interest on(select the best answer)
final tax-False tax upon interest on foreign currency a.Long-term deposit
6. Final tax is collected at source; hence, loans of FCDU banks and OBUs under b. Trust funds
there is noe need to file an income tax the expanded foreign currency deposit c.Deposit substitute
return-False system. d. Promissory notes-
7. Corporations are tax exempt on inter- 11. A qualified employee trust fund is 5. The exemption on final tax on
corporate dividends from any not subject to final tax.-True interest from long-term deposit is not
corporation. –False 12. The income of FCDU banks from applicable to
8. Individuals and corporations are tax fellow depository banks under the a.citizens indivduals
exempt on interest income on long- expanded foreign currency deposit b.Corporations-
term deposits.-False system is subject to 10% final tax.-False c.Resident corporation or individuals
9. Dividends from resident corporations 13.Petroleum service subcontractors d.All of these
are subject to regular tax.-False are subject to 8% final tax on their 6.Which interest income is not subject
10. Dividends from real estate income from petroleum service to any income tax?
Investment Trusts are exempt from final contractors-True a.Interest income from discount notes
tax-False 14. The final withholding tax return b.Interest income from bonds issued by
11. Stock dividends are always exempt shall be filed by the withholding agent a bank
from final tax. False on or before the 15th day of the month c. Imputed interest-
12. Corporations are subject to final tax the final taxes were withheld.-False d. Interest income from deposit
on prizes-False 15. Foreign governments and foreign substitute
13. The share in the net income of a government-owned and controlled 7.The final tax rates on pre-termination
business partnership is subject to a corporations are exempt from final tax.- of long-term deposits is not relevant to
creditable withholding tax, not to final True a.any corporations-
tax.-False Multiple choice-Theory 1 b.resident citizens
14. General professional partnerships 1.Which is correct with regard to the c.resident aliens
are subject to final tax but not to final income taxation? d.non-resident citizens
regular tax-False a.Taxpayers need not to file an income 8.Which of the following is subject to
15. All non-residents are exempt from tax return.- 7.5% final tax on interest from foreign
final tax but not to regular tax-False b.It covers all items of passive income currency deposits made within
16. Royalties, active or passive, are from whatever sources. Philippine banks?
subject to regular income tax.-False c.It applies to all items of gross income a. Non-resident corporation
17. The tax sparing rule is applicable to of any non-resident earned from b.Non-resident alien engaged in trade
resident and non-resident foreign sources within the Philippines. or business
corporations-False d. It applies to passive income earned c. Non-resident alien engaged in trade
TRUE OR FALSE 2 abroad or business-c
1.A government employee may claim 2. Which statement is correct regarding d. None of these
the tax informer’s reward.False final income tax? 10. The interest income from long-term
2. The final tax on winnings applies to a.Items of income subjected to final tax peso deposit made with foreign banks is
corporations-False can still be subjected to regular tax. a.subject to 20% final tax
b.Exempt from any tax
c.subject to regular tax-c a.Royalties from literary works c.subject to either final tax or regular
d. None of these b.Dividends from a domestic tax
11. What is the final tax on interest corporation d. always subject to regular tax
income on 6-year deposit pre- c.Share in the net income of a business 12. The winnings of a resident alien
terminated less than 2 years before partnership from abroad is
maturity? d. Royalties from a business trademark- a.subject to regular tax
a.20% 4. Which of the following is taxable? b. subject to final tax
b.12% a.Prize on sports competitions c.subject to either final or regular tax.
c.5%- sanctioned by the national sports d. not subject to Philippine income tax-
d.0% organizations 13. Royalty income of a domestic
12. Which of the following recipients is b.Prize from dance competition abroad- corporation from abroad is
exempt from final tax on dividends? c.Nobel Prize a.subject to regular tax.-
a. Resident citizens d.Any prize received without effort b. subject to final tax
b.Non-resident alien 5. Which winning is taxable to a final c. subject to either final tax or regular
c.Resident corporation- tax? tax
d.Resident alien a. Winnings exceeding P10,000 d.not subject to Philippine income tax
13.A dividend declaration in 2015 is b.Winnings not exceeding P10,000 14.Winnigns from PCSO are
subject to a final tax of c.Winnings from illegal gambling a. exempt from any tax-
a.6% d.All of these- b. subject to regular tax
b.8% 6.Prizes from sources abroad is subject c.subject to 20% final tax
c.10%- to final tax d,subject to both final and regular tax.
d.20% a. if the amount of prizes exceeds 15.The final tax rates on passive income
14. Dividends declared by a resident P10,000 are generally similar to the following
corporation is b. if the amount of prizes does not taxpayers. except one.
a.exempt from any tax exceed P10,000 Select the exception.
b.subject to 10% final tax c. if it is given to the taxpayer without a.Domestic corporation
c.subject to regular income tax- effort b.Resident alien
d.subject to 20% final tax d.under no circumstance- c.Non-resident citizen
15. Between the date of declaration 7.The following winnings are exempt d.Non-resident corporation-
and the date of record, BCD, Inc. sold an from tax, except one, Which is the 16.The tax informer’s reward may be
investment representing stock of ABC, exception? granted to
Inc., a domestic corporation. Which a.Jueteng winnings- a.Officials of the BIR
statement is correct? b. Lotto winings b. Public officials or employees
a.ABC, Inc., shall withhold 10% dividend c.PCSO winnings c.Close relatives of public government
tax if the buyer of the sale is an d.All of these employees
individual. 8. Which of the following is exempt d.Relatives of government employees
b. ABC, Inc., shall withhold 10% from final tax? beyond the 6th degree of consanguinity-
dividend tax if the buyer of the sale is a a. Interest income earned by a bank 17.Which is not subject to final tax?
corporation. from deposits in another bank a.Compensation to non-resident aliens
c. ABC, Inc. shall withhold 10% dividend b.Royalties from books not engaged in trade or business
tax if the seller of the sale is an c.Interest income earned by a bank b.Fringe benefits of supervisory and
individual. from lending activities- managerial employees
d.ABC, Inc. shall withhold 10% dividend d.All of these c.Income of non-resident foreign
tax if the seller of the sale is a 9. Dividends income from domestic corporations
corporation. corporations is subject to final tax d.Fringe benefits of rank and file
Multiple Choices-Theory 2 effective employees-
1. Which is covered by final tax? a.2000 18. Which of the following is exempt
a.Interest income from foreign b.1999 from final tax?
banks c.1998- a.Winnings in competitions by a local
b.Share in the net income of a joint d.1997 trading company
venture- 10. Prizes arising from sources within b.Prizes in a body building competition
c. P10,000 taxable prizes from the the Philippines may be c.Dividends from a foreign corporation-
Philippines a.subject to final tax d. Winnings from a casino
d. Share in the net income of a b. subject to regular tax Multiple Choice-Problem 1
general professional partnership c.exempt from tax 1.Winnie, a resident citizen, deposited
2. Which is not subject to 20% final tax? d. exempt or subject to final tax or P2,000,000 in the 180-day time deposit
a.Interest income from trusts funds regular tax- of Banco de Oro. The deposit pays 8%
b.Royalties from musical compositions- 11. Any royalties earned abroad by a interest. Compute the final tax on
c.P11,000 worth of prizes non-resident corporation is maturity of the deposit.
d.Winnings a.not subject to Philippine income tax a.P6,000
3. Which is not subject to 10% final tax? b. always subject to final tax b.P8,000
c.P20,000 7.Jake Corporation, a domestic deposit pays 8% annual interest.
d.P16,000- corporation. deposited P1,000,000 in Compute the final tax to be withheld in
2,000,000x0.08=16,000 the five-year time deposit of Banco dollars in the first year.
16,000x6/2=80,000 Pilipino which pays 10% annual interest. a.$0=
80,000x.20=16,000 Compute Jake Corporation’s net interest b.$12,000
2.Assume that Winnie is a non-resident income in the first year of the deposit c.$32,000
alien not engaged in trade or business; a.P100,000 d.$42,000
compute the final tax on maturity of the b.P90,000 13. Vernon is a non-resident citizen
deposit c.P80,000 working abroad. He invested his
a.P6,000 d.P70,000 $1,000,000 personal saving in a FCDU
b.P8,000 1,000,000x0.10-100,000 bank under a joint account with his
c.P20,000- 8. Assume Jake Corporation is a non- resident wife. The bank pays 8% annual
d.P16,000 resident corporation; compute the final interest. Compute the final tax to be
2,000,000x0.08=160,000 tax on interest income withheld in one year.
160,000x180/360=80,000 a.P0 a.$0=
80,000x.25=20,000 b.P10,000 b.$3,000-
3.Renz received P42,000 interest from c.P20,000 c.$6,000
short-term deposits. Compute the final d.P30,000- d.$8,000
tax withheld on the interest 1,000,000x.30=300,000 1,000,000x0.08=40,000x15%=6k
a.P10,500- 300,000x0.10=30,000 14. Yvonne, a non-resident alien not
b.P9,000 9.On January 2, 2011, Quezon, Inc. engaged in trade or business, made a
c.P5,000 placed P1,000,000 in the 5-year time $5,000,000 deposit to an FCDU Bank.
d.P4,500 deposit of Bank of Laguna. The The deposit pays 8% annual interest.
42,000/0.8=52,500 placement earns 10% annual interest. Compute Yvonne’s final tax in the first
52,500x20%=10,500 Quezon, Inc. preterminated the deposit year.
4.On January 1, 2014, Mr. Albuera on June 30,2014. Compute the final tax a.$0
invested P2,000,000 in the six year time to be withheld on pre-termination. b.$30,000
deposit of Sulu Unibank which pays a.P0 c.$80,000
10% annual interest. Compute the final b.P10,000- d.$100,000
tax to be withheld by Sulu Unibank on c.P17,500 15. Gerry, a resident alien, deposited
December 31, 2014. d.P42,000 $1,000,000 in a depository bank under
a.P0- paid by the bank on a yearly basis at the expanded foreign currency deposit
b.P100,000 the end of 1,000,000x10%x6/2=50,000 system. The five-year savings deposit
c. P200,000 each yearx20%=10,000 pays 10% annual interest. Assuming a
d.P250,000 P45:$1 exchange rate, how much final
5. Assume Mr. Albuera pre-terminated 10. Francis, a resident citizen, invested tax on interest would be withheld in the
the time deposit on July 1, 2018; P1,000,000 in a debt first year?
compute the final tax to be withheld on instruments( deposit substitute) issued a.P0
pre- termination. by Unibank. The debt instruments pay b.P7,500
a.P5,000 10% annual interest payable every c.P20,000
b. P12,000 quarter. How much tax should Unibank d.P337,500=
c.P45,000 withhold in the first quarter of the Multiple Choice-Problems 2
d.P108,000 issue? 1.Kendrick Inc., a domestic corporation,
2,000,000x10%=200,000 a.P0 was credited $46, 250 interest income
200,000x45=900,000 b.P5,000- depositary account in an FCDU Bank.
900,000x.05=45,000 c.P6,250 How much final tax was withheld?
6.Boy, a resident citizen, received the d.P20,000 a.$0
following interest income: 11. Cyndrell, a resident citizen, b.$3,750-
-P80,000 from short-term peso deposit deposited P5,000,000 in a non-resident c.$3,468.75
-P16,000 from a five-year bonds issued foreign depository bank which pays d.$11,562.50
by a commercial bank 12% annual interest. How much final 2.Assuming Kendrick Inc, is a resident
-P24,000 from 5/6 lending to market tax shall be withheld in the first year? foreign corporation, how much final tax
vendors a.P0= withheld?
How much final tax must have been b.P45,000 a. $11,562.50
withheld from Boy’s interest income? c.P120,000 b.$3,468.75
a.P0 d.P150,000 c.$3,750-
b.P16,000 12. Rodrigo, a non-resident citizen, d.$0
c.P20,000- deposited his $2,000,000 savings in a 3. Tuzan Corporation declared
d.P24,000 two-year time deposit in Blue bank, a P1,000,000 dividends in 2014. 40% of
80,000/80%=`100,000 depository bank under the expanded its outstanding shares is held by its only
100,000x20%=20,000 foreign currency deposit system. The corporate investor, Abdul, Inc. Compute
the amount of dividends tax to be c.P10,000 14. Mr. Tuao won an P800,000 prize in a
withheld on declaration. d.P0- national dancing competition. The
a.P0 9. Teddy is a venture sharing 50% in the organizer of the event declared that the
b.P40,000- profits of Wise Ventures, an P800,000 grand prize was “tax free”.
c.P60,000 unicorporated joint venture, engaged in How much final tax must be remitted by
d.P100,000 the trading of home appliances. Wise the organizer to the BIR?
1m x.60=600,000x10%=60,000 subject Ventures reported P200,000 profits for a.P200,000=
to the individual 2014. How much final tax will be b.P160,000
4. Mr. Neis, a resident alien, owns 40% withheld by Wise ventures? c. P88,889
of the shares of Coco Corporation, a a.P0 d.P0
resident corporation. Coco declared b.P10,000 800,000/80=1m
P200,000 dividends in 2014. How much c.P20,000- 1mx.20=200,000
final tax should be withheld from Mr. d.P40,000 15. Mr. Crocodile owns a mining
Neis’ dividend income? 10.Zamboanga Corporation, a resident property in Prosperidad. He allowed a
a.P40,000 corporation, is a venture in Southern mining company to operate under a
b.P20,000 Philippine Ventures (SPV), an condition that he will be paid 10%
c.P8,00 incorporated joint venture formed royalty on the gold production. The
d. P0- under Philippine laws. Zamboanga owns mining company produced P8,000,000
E-RFC 30% interest on SPV. During 2014, SPV worth of gold during the year. How
Coco income out reported P3,000,000 profit and much net royalties will be paid to Mr.
NEIC RC=In out declared P1,000,000 in dividends. How Crocodile?
5. Carrots, Inc. owns 20% of the stocks much final tax will be withheld from the a.P800,000
of cabbage corporation. Carrots and dividend distribution to Zamboanga b.P640,000-
Cabbage are both domestic Corporation? c.P720,000
corporations. Cabbage declared a.P100,000 d.P160,000
P120,000 dividends in 2014. b.P30,000 8,000,000x.10=800,000x.20=160,000
How much dividend income after any c.P90,000 Multiple Choice-Problem 3
applicable tax will Carrots receive? d.P0- 1.Mr. Aguak earned the following prizes
a.P24,000- 11.Pedrodrill Ventures is a joint venture from two different competitions:
B.p20,000 which operates under a service contract Date Event Gross
C.P19,000 with the government. Petrodrill is 30% Prize
d.P18,000 owned by individual investors. The rest January 4, 2017 Singing competition
6. Mr. Siegfried, a resident citizen, is owned by corporate oil exploration P12,0000
received P100,000 dividend income companies. Petdrodrill reported February 14, 2017 Painting competition
from Vodka, Inc., a domestic P2,000,000 profit which was to be 10,100
Corporation. How much final tax must distributed. Compute the total final tax that should
have been withheld by Vodka, Inc.? How much final tax will Petrodtrill have been withheld from the above
a.P0 withhold? prizes.
b.P11,111- a.P200,000 a.P5,500
c.P20,000 b.P60,000 b.P4,420-
d.P25,000 c.P140,000 c.P3,000
7. Ms. Gina owns 40% of the capital and d.P0- d.P2,400
profits of Cebu Ventures, a business 12. Mr. Bayugan published his book 2.Mr. Rems Orayt participated in two
partnership engaged in the trade of abroad for the international market. different events on January 4, 2017:
tourist souvenirs. Cebu Ventures During the first quarter, he earned a Date Organizer Competition Gross
reported P200,000 net income in 2014. gross royalty of P1,000,000. How much Prize
Compute the total final tax to be final tax will be withheld by the foreign January 4, 2017 Ang mahiwagang Boses
withheld be the partnership on Gina’s publisher? Singing P10,000
profit sharing. a.P250,000 January 4, 2017 Samahang Sexy Body
a. P0 b.P200,000 Dancing 10,000
b.P8,000- c.P100,000 Compute the total final tax that should
c.P10,000 d.P0-RIT have been withheld from the above
d.P20,000 13.Sison. Inc. is the only corporate prizes.
8.Raymond owns 20% of the capital and shareholder of Surigao Inc. declared a a.P0-
profits of a general professional P1,000,000 dividend. Compute the final b.P4,420
partnership. The partnership reported income tax to be withheld by Surigao. c.P3,000
P500,000 profits in 2014. How much a.P100,000 d.P2,400
final tax will be withheld from b.P60,000 3.Mr. Conrado won the P50,000,000
Raymond? c.P40,000- 6/49 Superlotto Jackpot of the
a.P100,000 d.P0 Philippine Charity Sweepstakes Office.
b.P50,000 How much final tax will be withheld?
a. P500,000 withheld from the annual interest b.Real property held by the taxpayer
b. P200,000 payment to Roy? primarily for sale to customers in the
c.P100,000 a.P36,000 ordinary course of his trade or business.
d.P0- b.P16,800 c. Real property used in trade or
4.Ms. Matti won a gold medal and c.P37,500- business of a character which is subject
P50,000 cash prize for chess d.P0 to the allowance for depreciation.
competition in the Palarong Pambansa. 10. A taxpayer wants to manually file d.Real property used in trade or
How much final tax should be withheld his Monthly Remittance Return of Final business of the taxpayer.
from the prize? Income Tax Withheld for the month of Basically, ordinary assets are
a.P20,000 February 2017. What is the deadline for a. Assets held for sale-such as
b.P10,000 the return? inventory
c.P5,000 a.February 14, 2017 b. Assets held for use-such as
d.P0- b. February 29, 2017 supplies and items of property,
5.Tiong Sang Department Store c.March 10, 2017- plant and equipment like
conducted a sales promotion where d. April 15, 2017 buildings, property
customers purchases exceeding P1,000 11. In reviewing its tax compliance, improvement, and equipment
in one transaction shall be entitled to a Rang-ay Banco that it failed to remit the 2.Capital assets-any assets other than
ticket for a chance to win P500,000. Mr. P90,000 final taxes it withheld for the ordinary assets
Shing won the prize. month of February 2017. Compute the Basically, capital assets are:
How much shall Tiong Sang withhold total amount of taxes due including the 1. Personal(non-business) assets of
from the grand prize? penalties of Rang-ay Banco settles the individual taxpayer
a.P200,000 obligation on March 20, 2017. 2. Business assets of any taxpayer
b.P100,000- a.P90,000 which are:
c.P50,000 b.P112,500 a. Financial assets-such as
d.P0 c.P113,000 cash, receivables,
6. Tacurong, a resident citizen, won d.P128,000- investments
$1,000,000 from the U.S lottery. The 12. Bangko Illustrado failed to withhold b. Intangible assets-such as
lottery winning is the final tax on the P1,200,000 interest such as patents, copyrights,
a. exempt from tax expense which was credited to various leasehold rights; franchise
b. subject to 20% final tax accounts of individual depositors in the rights.
c. subject to 10% final tax month of April 2015. It wants to settle ANAYSIS OF PROPERTIES HELD
d. subject to regular tax- the unpaid final withholding tax on June BY TAXPAYERS
7. Mr. Curandang submitted a sworn 10, 2015. Compute the surcharge and INDIVIDUAL TAXPAYERS
statement regarding the alleged tax interest penalty due.
evasion practices of Chitie Corporation. a.P450,000
This led the BIR to recover P20,000,000 b.P320,000
unpaid taxes. How much net tax c.P64,000- Personal Assets Business asset
informer’s reward shall be paid to Mr. d.P58,000
(All capital assets)
Curandang? 13. The Ibanag Bank of Gattaran failed
a. P1, 800,000 to withhold final taxes on interest on its
b.P1,600,000 time deposits resulting in P8,000
c.P1,000,000 interest penalty for six months of
d.P900,000- delinquency. The delinquency was OrdinaryCapital asset-
asset-for
8. A concerned government employee subject to a compromise penalty of financial
sale or for use
furnished the BIR a statement detailing P15,000. Compute the total tax due intangible
the improper tax practices of a inclusive of penalties.
corporate taxpayer. The information a.P123,000-
was instrumental to the collection of b.P108,000 Corporate taxpayers
P500,000 tax. How much informer’s tax c.P95,000
reward after ta shall be released to the d.P88,000
informant? CHAPTER 6
a.P0- CAPITAL GAINS TAXATION
b.P40,000 CLASSIFICATION OF TAXPAYER’S Ordinary assets Capital asset
c.P45,000 PROPERTIES
d.P50,000 1.Ordinary assets-assets used in
9. Roy invested P1,000,000 in the “tax- business, such as:
free” corporate bonds of RBC a. Stock in trade of a taxpayer or other Asset Classification is relative
Corporation. The bond pays 8.75% real property of a kind which would The classification of assets or properties
interest annually net of taxes. How properly be included in the inventory of as ordinary asset or capital asset does
much final tax must RBC Corporation the taxpayer if on hand at the close of not depend upon the nature of the
the taxable year. property but upon the nature of the
taxpayer’s business and its usage by the taxable transaction involving not been use for more than two
business. such property. years.
Example: E. A depreciable assets is an Case 2: Mr. Alfonso is engaged in
1.A domestic stock is an ordinary asset ordinary asset even if it is fully real estate business
to a dealer in securities but is a capital depreciated, or there is a failure The old office shall continue to be
asset to a non-security dealer. to take depreciation during the an ordinary asset despite the
A ‘dealer in securities” is a merchant of period of ownership abandonment or idling of the
stocks of stocks or securities with a F. Real properties used by an property from active use.
registered place of business, regularly exempt corporation in its Illustration 2-Property acquired to
engaged in the purchase of securities exempt operations are be used in business
and their re-sale to customers. considered capital assets In June 1, 2014, Mr. Alfonso
3. A vacant and unused lot is an G. The classification of property purchased a building to be used as a
ordinary asset to a taxpayer transferred by sale, barter, or branch sale office. The building
engaged in the real estate exchange, inheritance, donation, remained idle as of December 31,
business such as realty dealer, declaration of property 2017 due to an ongoing civil war.
realty developer, or lessor but is dividends shall depend on Case1: Mr. Alfonso is not engaged in
a capital asset to those not whether or not the acquirer real estate business
engaged in the real estate used it in business. The property shall remain to be an
business. H. For real properties subject of ordinary asset. The tow-year rule
Interestingly, the revenue involuntary transfer such as applies only to properties which are
regulations classifies real and other exprorpriation and foreclosure classified as ordinary asset for being
properties acquired(ROPA) by banks sale, the involuntariness of such used in business. A property
as ordinary assets even if banks are sale shall have no effect on the purchased for future use in
not actually engaged in the realty classification of such real business, even though this is later
business. This is an apparent property. thwarted by circumstances beyond
recognition of the fact that ROPA I. Change in business from real the taxpayers control, does not lose
are normally acquire and sold by estate to non-real estate its character as an ordinary asset.
banks in their normal course of business shall not change the Case 2: Mr. Alfonso is engaged in
business. However, ROPA in the classification of ordinary assets real estate business
form of domestic stocks held by previously held/ The property shall remain to be
banks are capital asset” means all Taxpayers engaged in real estate considered as an ordinary asset.
stocks and securities held by business includes real estate dealer, Properties acquired by taxpayers
taxpayers other than dealers in real estate developer, real estate engaged in real estate business
securities. lessor and taxpayers habitually shall remain to be ordinary asset
Asset Classification Rules engaged in real estate business. even if discontinued from active use
A. A property purchased for future Taxpayers habitually engaged in and even if they discontinue their
use in business is an ordinary real estate business include those real estate business.
asset even though this purpose registered with the HLURB or Illustration 3-Disposal of Property
is later thwarted by BUDCC as dealer or developer or Juan, a realty dealer, donated one of
circumstances beyond the those with at least 6 taxable real his house and lot inventory to his
taxpayer’s control. estate sales transactions in the son as dowry for his upcoming
B. B. Discontinuance of the active preceding year. marriage. His son shall use the same
use of the property does not Illustration 1- Property previously as his family residence. He also
change its character previously used in business donated another house and lot as
established as a business Mr. Ginebra has a building which initial capital of his daughter who
property. was previously used as an office and will commence a realty leasing
C. C.Real properties used, being is subject to periodic allowance for business.
used, or have been previously depreciation. In July 1, 2015, Mr. The house and lot shall be
used, in trade of the taxpayer Ginebra implemented a strategic considered capital asset to the son
shall be considered ordinary shist in his business operations because he will not use it in
assets resulting to the relocation of its business. The house and lot donated
D. D.Properties classified as administrative office in another city to the daughter shall be considered
ordinary assets for being used in and the resultant abandonment of ordinary asset to the daughter
business by a taxpayer not its office. because she will use it in business.
engaged in the real estate Case 1: Mr. Alfonso is a not engaged Illustration 4- Exempt non-business
business are automatically in real estate business operation
converted to capital assets upon Effective July 3, 2015(i.e., more than Bantay Bata Foundation owns
showing of proof that the same two years from discontinuance of Building A-1. which its uses for its
have not been used in business use, the old office building shall be non-profit operations. It also owns
for more than 2 years prior to reclassified as capital asset upon another building A-2. which it
the consummation of the showing of proof that the same has
leased out to various commercial SALE, EXCHANGE AND OTHER share premium is an additional
lessees. DISPOSITION OF DOMESTIC STOCKS capital and is not income.
Building A-1 is a capital asset since DIRECTLY TO BUYER Under the US tax rules, treasury
it is employed in a non-taxable Domestic Stocks shares can be considered as
operations. Building 2 is an ordinary Domestic Stocks are evidence of investments if the corporation
asset since it is employed in taxable ownership or rights to ownership in trades on its shares as it would in
operations. a domestic corporation regardless the shares of other corporations. As
TYPES OF GAINS ON DEALINGS IN of its features, such as: such, the treasury share premium is
PROPERTIES 1.Preferred stocks (participative, viewed as a capital gain.
1.Ordinary gains-arises from the cumulative, etc.) Under the NIRC, and RR6-2008,
sale, exchange, and other 2.Common stocks however, there is no express
disposition including pacto de retro 3.Stock rights provision taxing treasury share
sales and other conditional sales of 4. Stock options premium. Hence, treasury share
ordinary assets 5. stock warrants premium should not be subjected to
2.capital gain-arises from the sale, 6. Units of participation in any capital gains tax.
exchange, and other disposition association, recreation, or Exchange of stocks for services
including pacto de retro sales and amusement club (golf,polo, or Similarly, the exchange or issue of
other conditional sales of capital similar clubs) stocks for services cannot be
assets The capital gains tax covers only considered as exchange for
Taxation of Gains on Dealings in sales of domestic stocks for cash but property. No gain or loss can be
Properties also exchange of domestic stocks in imputed as it involves payment of
Types of gain Applicable taxation kind and other dispositions such as: expense in kind.
scheme 1.Foreclosure of property in Redemption of shares in a mutual
Ordinary gains Regular income settlement of debt fund
tax 2. Pacto de retro sales- sale with Gains from redemption of shares in
Capital gains General rule: Regular buy back agreement a mutual fund are exempted by the
income tax 3. Conditional sales-sales which will NIRC from income taxation.
Exception rule: Capital be perfected upon completion of Worthlessness of stocks
gains tax certain specified conditions The value of stocks becoming
-Sale of:domestic stocks sold 4. Voluntary buy back of shares by worthless is considered a capital
directly to buyer(15%), real the issuing corporation-redemption loss subject to the rules of regular
properties not used in business(6%) of shares which may be re-issued income tax. This will be discussed in
CAPITAL GAINS CUBJECT TO CAPITAL and non intended for cancellation Chapter 12.
GAINS TAX The term other disposition does not Redemption of stocks by the issuing
There are only two types of capital include: corporation
gains tax subject to capital gains tax: 1.Issuance of stocks by a Under RR6-2008, any gain or loss on
1. Capital gains on the sale of corporation the mandatory redemption of
domestic stocks sold directly to 2. Exchange of stocks for services stocks by the issuing corporation for
buyer. 3. Redemption of shares in a mutual the purpose of stock cancellation
2. Capital gains on the sale of real fund shall be subject to the rules of
properties not used in business 4. Worthlessness of stocks regular income tax discussed under
SCOPE OF CAPITAL GAINS TAXATION 5. Redemption of stocks for Dealings in Properties in Chapter 12.
Gains on dealings in capital assets cancellation by the issuing It should be noted, therefore, that
Tax rates corporation the gain by the investor on
Gain on the sale, exchange, and 6. Gratuitous transfer of stocks redemption of redeemable
other disposition of domestic stocks Issue of stocks including treasury preferred shares shall be subject to
directly to buyer Train law 15% stocks regular income tax.
Sale, exchange, and other The issue of stocks to stockholders Note, however, that this does not
disposition of real property in the by a corporation is a financing include the voluntary buy-back of
Philippines 6% capital transaction rather than a sale the shares by the issuing
gains tax transaction. The excess of fair value corporation to be held in treasury
Gains from other capital assets received over the par value of which may later on be re-issued.
Regular income tax shares issued is an additional capital The gain or loss realized by the
to the corporation. investor on voluntary buy-back of
The tax treatment of gains on Stocks acquired by the corporation shares by the issuing corporation is
dealings in other properties other from its shareholders, treasury taxable under capital gains taxation.
than those subject to capital gains shares, cannot be considered assets Gratuitous transfer of stocks
tax will be discussed in detail in or investments in accounting sense. The gratuitous transfer of stocks
Chapter 12. The excess of the consideration either by way of donation inter-
received in the re-issuance of vivos or donation mortis causa is
treasury stocks is called treasury
subject to transfer tax, not to P100,000 loss is also an ordinary maintained by the seller
income tax. loss reportable as item of deduction where transaction of every
MODES OF DISPOSING DOMESTIC against gross income under the particular stock is recorded.
STOCKS regular income tax. First-in, First out method, if
Shares of stocks may be sold, TAX ON SALE, EXCHANGE, AND the stocks cannot be
exchanged, or disposed: OTHER DISPOSITIONS OF DOMESTIC specifically identified.
1.Through the Philippine Stock STOCK DIRECTLY TO BUYER.  If acquired by devise,
Exchange (PSE) or The net gain on the sale, exchange, bequest, or inheritance, the
2. Directly to buyer and other disposition of domestic tax basis is the fair value at
Stock transaction tax SPX1%X1/2 or stocks directly to a buyer, is subject the time of death of the
SPx0.5% to a two-tiered capital gains tax: decedent
TAX ON SALE OF DOMESTIC STOCKS Tax rate  If acquired by gif-the tax
THROUGH THE PSE Net gain up to P100,000.00 5% basis is the lower of the fair
The sale of domestic stocks Excess net gain above P100,000 10% market value at the time of
classified as capital assets through The net gain is determined as gift and the basis in the
the PSE is not subject to capital follows: hands of the donor or the
gains tax. It is subject to a stock Selling price last preceding owner by
transaction tax of ½ of 1% of the Pxxx.xxx whom it was not acquired by
selling price. Less: gift.
Illustration 1:Non-dealer in stocks Basis of stocks disposed Pxxx.xx  If acquired for inadequate
Mr. San Juan, not a dealer in stocks, Selling expenses xxx.xx consideration-the tax basis is
sold the following stock investments Documentary stamp tax on the the amount paid by the
through the Philippine stock sale xxx.xx transferee for the property.
Exchange: xxx.xx  If acquired under tax-free
Date Stock Code Selling Price Net capital gain(loss) Pxxx.xx exchanges, the tax basis is
Cost Gain (Loss) Selling price shall mean: the substituted basis of the
4/5/2015 AC P4,000,000 *In case of cash sale, the total stocks.
P3,700,000 P300,000 consideration received per deed of Illustration 1:Cost of acquisition
4/5/2014 SMB sale Mrs. La Carlota purchased 1,000
3,000,000 3,200,000 (200,000) *If total consideration is paid partly shares of Bacolod Corporation
P7,000,000 in money and partly in property, the for P100,000 and paid the
P6,900,000 P100,000 same sum of money and fair values broker’s commission of P1,000.
of consideration received. The stocks were subject to a
The stock transaction tax shall be Illustration chattel mortgage of P10,000
computed as follows: Mr. Cool sold his stocks receiving in which Mrs. La Carlota assumed.
Total selling prices of stocks through exchange a building with a tax basis The cost basis of the stocks shall
the PSE P7,000,000 of P2,000,000 but with a fair value be:
Multiply by: Transaction tax rate of P2,500,000 goods worth Cash paid P100,000
0.5% P100,000, and P400,000 cash Obligations assumed on the
Transaction tax The selling price shall be computed property purchased 10,000
P35,000 as follows: Direct acquisition costs-broker’s
Note: Fair value of buildings received commission 1,000
1. The stock transaction tax applies P2,500,000 Total cost(tax basis)
on the selling price regardless of Fair value of goods received P111,000
the existence of a gain or loss on 100,000 Illustration 2:Costing procedures
the sale transactions Cash Mr. Alcantara had the following
2. 2. The P65,000 net capital gains, 400,000 purchases and sales of shares of
after deduction of the P35,000 Total consideration or selling price the stocks of El Dorado
transaction tax, is exempt from P3,000,000 Corporation:
income tax. It will no longer be What is the tax basis of stocks? Date Transaction Shares Price
subject to capital gains tax or to  If acquired by purchase, tax Cost
regular income tax. basis is the cost of the January 1 Purchase 10,000
Illustration 2:Dealer in stocks property which shall be P10.00 P100,000
Assume the same data in the determined by the following March 1 Purchase 5,000
previous illustration except that Mr. methods in descending order 11.03 55,150
San Juan is a dealer in stocks. of priority: March 23 Purchase 20,000
Mr. San Juan shall not be subject to -Specific identification, if the 12.00 240,000
the stock transaction tax but the shares can be specifically April 4 Sale 25,000
P300,000 gain is an ordinary gain identified. 15.00
reportable as item of gross income -moving average method, if The cost of the shares sold shall
subject to regular income tax. The books of accounts are be determined as follows:
1.Assuming Mr. Alcantara presumed from the cost of the (Dona Karena) who did not
identified that the shares sold first 25,000 shares bought: acquire the property by gift, and
were those bought on March 1 Date Transaction Shares Unit -P2,500,000, the fair value at the
and March 23, the applicable cost Cost date of donation, hence,
method is specific identification January 1 Purchase 10,000 P400,000
method. P10.00 P100,000 2.Inheritance:Assuming the
Under specific identification, the March 1 Purchase 5,000 shares were inherited by Mrs.
actual cost of the shares sold 11.03 55,150 Lipa when Don Bosco died in
and remaining stocks shall be: March 23 Purchase 20,000 March 2013, the basis of the
March 1 purchase 5,000 shares 12.00 240,000 shares to Mrs. Lipa shall be
P55,150 35,000 P2,500,000, the fair value at the
March 23 purchase 20,000 P395,150 date of death of Don Bosco.
shares 240,000 Less: Cost of goods sold: 3.Purchase for an inadequate
Cost of the remaining 10,000 January 1 10,000 consideration:Assuming the
stocks P100,000 P10.00 P100,000 shares were bought by Mrs. Lipa
2.Assuming Mr. Alcantara March 1 5,000 from Don Bosco for only
cannot identify the shares 11.03 55,150 P1,200,000 the basis of the
actually sold but retains detailed March 23 10,000 shares to Mrs. Lipa shall be
records of purchase and sale in 12.00 120,000 P1,200,000, the actual price
the stocks and sale in the stocks (P275,150) paid for the,.
of El Dorado, the applicable 25,000 The computation of adjusted
method is the moving average Quantity and cost of ending basis on tax-free exchanges will
method. shares 10,000 12.00 be discussed under tax-free
Under the moving average P120,000 exchanges in this chapter.
method, the cost of the shares Note: Stocks sold below their fair
sold and the remaining shares 1.The 10,000 and 5,000 shares value
shall be computed as follows: from January 1 and March 1 The excess of the fair value over
Date Transaction Shares respectively are deemed first the selling price is subject to
Unit cost Cost sold. The other 10,000 shares donor’s tax.
January 1 Purchase 10,000 sold are deemed coming from Illustration
P10.00 P100,000 the last purchase on March23. Mr. Navotas sold his investment
March 1 Purchase 2. The cost of the 10,000 shares in domestic stocks directly to a
5,000 11.03 55,150 in the last purchase is computed buyer for P500,000. The shares
March 23 Purchase 20,000 as have a fair value of P650,000
12.00 240,000 10,000/20,000Xp240,000=P120, and P300,000 tax basis and
35,000 P11.29 P395,150 000 expenses on the date of sale.
Less:Cost of shares sold 25,000 Illustration 3:Acquisition by Fair value-P650,000 -150,000
P11.29 gratuitous title 0.5% gratuity subject to transfer
(P282,250) In March 2013, Mrs. Lipa tax
Quantity and cost of ending received by gratuitous Selling price 500,000
shares 10,000 P11.29 acquisition shares of stocks of Less:Cost and expenses 300,000
P112,900 Taal Corporation from her father P200,000 gain subject to capital
Don Bosco. Don Bosco acquired gains tax
Note: the same shares by donation in The rules on the determination
1.Average unit cost June 1999 from his mother. of fair value of stocks and
=P395,000/35,000=P11.29 Dona Karena, who bought the transfer taxation will be
2.Under the moving average shares for P400,000 in April discussed in the book, Business
method, the average unit cost of 1996. The shares had a fair value and Transfer Taxation, by the
the stocks is determined after of P700,000 in June 1999 and same author.
every purchased P2,500,000 in March 2013. SCOPE OF THE TWO-TIERED
3. The cost of ending shares can Assuming the shares were CAPITAL GAINS TAX
be computed as 10,000 acquired by Mrs. Lipa from her The two-tiered capital gains tax
Xp11.29/ father by way of: applies for all classes of
3.Assuming Mr. Alcantara 1.Donation:Assuming the shares taxpayers, individuals and
cannot identify the stocks and were donated by Don Bosco to corporations, regardless of the
does not maintain detailed Mrs. Lipa in March 2013- the place of sale, the identity of the
records of transactions in the basis of the shares to Mrs. Lipa buyer,and the length of time the
shares of El Dorado, the shall be whichever is lower of: domestic stocks were held by
applicable method is the First-in, -P400,000, the basis in the hand the taxpayer. Even NRA-NETBs
first-out method . The cost of of the last preceding owner and NRFCs are required to file
the shares sold shall be the capital gains tax return. Due
to this, it is regarded as the most Selling price P240,000 The final capital gains tax payable
universal rule in income Less: Cost and expenses 102,500 (refundable) shall be:
taxation. Capital gain P137,500 Annual net capital gain P170,000
The transfer by a non-resident x15%=20,625 Less: First P100,000 net gain
alien or a foreign corporation to First P100,000 gain at 5% 100,000x5%=P5,000
anyone of share of stocks issued P100,000 Excess net capital gain P70,000x10%
by a domestic corporation shall Excess gain at 10% 7,000
not be effected in its books 37,500 Annual capital gains tax due
unled the transferor has filed P5,000 P12,000
with the Commissioner a bond 3,750 Less: Total transactional capital gains
conditioned upon the future Capital gains tax due taxes paid
payment by him of any income P8,750 10,500
tax that may be due on the gains Capital gains tax payable (refundable)
derived from such transfer, or Deadline of the transactional capital P1,500
the Commissioner has certified gains tax return
that the taxes, if any, due on the The capital gains tax return (BIR Form Annual capital net gain
gain realized from such sale or 1707) shall be filed within 30 days after 170,000x15%=25,000
transfer have been paid. each sale, exchange, and other Annual capital gains tax due less
It shall be the duty of the disposition of stocks. If the tax is transactions (30,000)
transferor and the corporation qualified for payment under the Tax refundable 4,500
the shares of which are sold or installment method, the tax is due Deadline of annual capital gains tax
transferred, to advise the within 30 days after each installment. return
transferee of his requirement. The installment payment of capital The annual capital gains tax return, BIR
CAPITAL GAINS TAX gains tax will be discussed later in this Form 1707-A, shall be filed on or before
COMPLIANCE chapter. the 15th day of the fourth month
1.Transactional capital gains tax ANNUALIZED CAPITAL GAINS TAX following the close of taxable year of
2. Annual capital gains tax The tax on capital gains on the sale, the taxpayer.
TRANSACTIONAL CAPITAL GAINS exchange, and other disposition of Illustration
TAX domestic stocks directly to a buyer is In the immediately preceding
The capital gains or losses are based on the annual net capital gains. illustration, the P1,500 net capital gains
required to be reported after The annual net capital gain or loss is tax payable is due on or before October
th
each sale, exchange, and other computed as transactional capital gains 15, 2016, the 15 day of the fourth
dispositions through capital less transactional capital losses. month following the fiscal-year-end,
gains tax return, BIR form 1707. The transactional capital gains taxes June 30. 2016.
Illustration:Computation of paid during the year are deducted as For individual taxpayers, the P1,500
Capital Gains tax tax credit against the annual capital capital gains tax shall be payable on or
Cherry disposed her investments gains tax due. The excess of the annual before April 15, 2016 since individuals
in domestic stocks costing capital against ax due over the sum of are allowed only the calendar year
P100,000 directly to a buyer. She the transactional capital gains taxes accounting period.
paid on the sale of P2,000 and paid during the year is a capital gains INSTALLMENT PAYMENT OF THE 5%-
P500 respectively, for broker’s tax payable. The excess of the sum of 10%, CAPITAL GAINS TAX
commissions and documentary transactional capital gains tax over the When domestic stock is sold in
stamp tax expense. annual capital gains tax due is capital installments, the capital gains tax may
Compute the capital gains tax gains tax refundable. also be paid in installments if the:
under each of the following Illustration a.Selling price exceeds P1,000; and
conditions: Allisson, Inc., disposed several equity b.Initial payment does not exceed 25%
1.Selling price=P180,000 directly to a buyer during its fiscal year of the selling price
Selling price ending June 30,2016: Illustrative Case:Basic
P180,000 Date Equity Securities Selling price Cost On November 1, 2016, Mr. Batanes
Less: Cost and expenses and Expenses Capital gains(losses) made a sale of domestic stocks costing
Purchase cost P100,000 Capital gains tax Train Law P700,000 directly to a buyer for
Commission expense 2,000 1/12 Preferred stock P210,000 P1,000,000. The buyer agreed to pay in
Documentary stamp tax P100,000 P110,000 P6,000 16,500 P100,000 monthly installments starting
expense 500 102,500 3/18 Common stocks 80,000 90,000 November 30.
Capital gain (10,000)- The capital gains tax shall be:
P77,500x15%=11,625 5/14 Stock rights 160,000 Selling price P1,000,000
First P100,000 of gain P77,500 70,000 90,000 4,500 13,500=30,000 Less: Cost of shares sold 700,000
Multiply by applicable rate: 5% 6/17 Stock options 80,000 100,000 Net capital gain
Capital gains tax due P3,875 (20,000) - P300,000x15%=45,000-----subject to
2.Selling price=P240,000 P170,000 P10,500 capital gains tax because, SP 1,000,000
(exceeds 1,000) installments
100,000(did not exceed 25% of 1m Illustration 3:With excess mortgage 30
x25%)=250,000 over cost days
Less:First P100,000 x5% P5,000 Assume instead that the stock was
Excess gain P200,000x10% 20,000 subject to P750,000 mortgage which Securities for purposes of the 61-day
Net capital gains tax due P25,000 the buyer assumed. The P250,000 rule include stocks and bonds. The wash
Illustration 1:No mortgage on the balance is payable in monthly sales rule has significance on the
shares sold installments of P50,000 starting recognition of reportable capital losses
Initial payment: November 30,2016. on domestic stocks sold directly to
First installment(November30) The gain and the capital gains tax shall buyer.
P100,000 be the same as P300,000 and P25,000 For the purposes of this rule,
Second installment (December31) respectively. The excess of mortgage substantially identical means that stocks
100,000 over the basis of the stocks is an or bonds of the same class with the
100,000/1mx45,000=4,500 indirect downpayment, a form of same features. A common stocks is not
constructive receipt. substantially identical to a preferred
Total initial payment Selling price P1,000,000 stock. Participating and non-
P200,000 Less:Mortgage assumed 750,000 participating stocks are not substantially
Ratio of initial payment Cash collectible P250,000 identical.
(P200,000/P1,000,000) 20% Constructive receipt (P750k mortgage- Illustration 1:Acquisition of identical
The taxpayer is qualified to pay capital P700k basis) 50,000 shares before a losing sale
gains by installment. Contract price P300,000 In 2016, Mr. Toledo had the following
Under the installment method, the tax The initial payment shall be computed transactions in the shares of Talisay,
shall be paid based on the pattern of as follows: (1 year) Inc., a domestic corporation:
collection of the contract price. The Indirect downpayment (constructive Date Transaction Shares Price Cost
contract price is the total sum of money receipt) P50,000 January 5 Purchase 10,000 P4.00
collectible from the contract. It is First installment (November 30) P40,000
normally the selling price in the 50,000 March 1 Purchase 10,000 4.10
absence of any indebtedness on the Second installment(December 31) 41,000
shares sold. 50,000 March 18 Sale* 10,000 3.80
Under the installment method, the Total initial payment 38,000
capital gains tax payable every P150,000 *Those purchased January 5, 2016
installment shall be computed Ratio of initial payment The capital gain or loss on March 18,
as:Collection/Contract pricexCapital (P150,000/P1,000,000)=15% 2016 shall be computed as follows:
gains tax The taxpayer is qualified to pay capital Selling price P38,000
The capital gains tax payable for every gains tax in installments. Less:Cost of goods sold 40,000
installment shall be P2,500 computed The capital gains tax shall be as follows: Capital Loss P2,000
as P100,000/P1,000,000 XP25,000 For the sale :P50,000/P300,000 Pursuant to the wash sales rule, The
100,000/1MX45,000=4,500 XP25,000 P4,166.67 P2,000 capital loss on the sale shall not
Note that the selling price is used to For every be deductible in the computation of the
measure the initial payment ratio, but installment:P50,000/P300,000Xp25,000 annual net capital gains in 2016 since
the contract price is used in determining P 4,166.67 the shares sold were fully replaced
the capital gains tax installment. SPECIAL TAX RULES IN CAPITAL GAIN OR within the 61-day period.
Illustration 2: With mortgage on stocks LOSS MEASUREMNT There is full replacement or cover-up
but not in excess of cost 1. Wash sales of stocks when the quantity of the shares
Assume the stocks were previously 2. Tax-free exchanges acquired in the 61-day period is at least
mortgaged for P600,000 which the a.Exchange of stocks pursuant to equal to the quantity of the shares sold.
buyer assumed. The P400,000 balance a merger or consolidation In this case, the loss is deferred and is
is payable in monthly installments of b.Transfer of stocks resulting in added to the tax basis of the
P100,000 starting November 30, 2016. corporate control replacement shares.
The gain and the capital gains tax shall WASH SALES RULE The adjusted basis of the replacement
be the same as P300,000 and P25,000 Wash sale of securities is deemed to shares acquired on March 1, 2016 shall
respectively. The contract price or total occur when within 30 days before and be:
sum collectible on the sale shall be: 30 days afer the sale (also referred to Purchase price P41,000
Selling price P1,000,000 as the 61-day period), the taxpayer Add: Deferred loss on March 18 wash
100,000/400,000x45,000=11,250 acquired or entered into a contract or sales 2,000
Less:mortgage assumed option to acquire substantially identical Basis on replacement shares P43,000
(*indebtedness) 600,000 securities.Capital losses on wash sales What if the replacement shares are less
Contract price P400,000 by non-dealers in securities are not than the shares sold?
deductible against capital gains. Assume that the shares bought on
The capital gains tax payable every March 1, 2016 were only 8,000 shares
installment shall be P6,250 computed for P32,800.
as P100,000/P400,000XP25,000. 30 days Day of losing sale
Only the portion covered with Basis of 12,000 replacement shares Add:Deferred loss (5,000
replacement shares shall be disallowed. P212,000 shares/8,000XP16k) 10,000
The portion without replacement cover What if replacement shares are less Basis of the 5,000 replacement shares
is a deductible realized loss. Thus, the than the shares sold? on February 15
capital loss shall be split as follows: Assume instead that only 7,000 shares P115,000
Deferred loss( 8,000 shares/10,000 were bought on March 4 for P110,000.
sharesxP2,000) P1,600 In this case, the capital loss shall be split
The adjusted basis of the replacement
Deductible loss(2, 000 shares/10,000 as follows: shares acquired on March 4, 2016 shall
sharesx P2,000) 400 Deferred loss(7,000/10,000XP20,000) be:
Capital loss P2,000 P14,000 Purchase Price P48,000
The adjusted basis of the replacement Deductible loss (3,000/10,000
Add:Deferred loss (3,000 shares/8,000
shares acquired on March 1, 2016 shall XP20,000) 6,000 shares XP16K) 6,000
be: Capital loss P20,000
Basis of 3,000 replacement shares on
Purchase price P32,800 The adjusted basis of the replacement March 4
Add:Deferred loss on March 18 wash shares acquired on March 4, 2016: P54,000
sales 1,600 Purchase Price P110,000Illustration 4:No replacement shares in
Basis of 8,000 replacement shares Add:Deferred loss on wash sales 14,000the 61-day period
P34,400 Basis of 7,000 replacement shares On January 18, 2016, Mr. Malunay
What is the 10,000 shares bought on P124,000 bought 10,000 shares of Gen. Luna
March 1, 2016 were the same shares Illustration 3:Acquisition of Identical
Corporation for P100,000. On February
sold at a loss on March 18, 2016? shares before and after losing sale 6, 2016, he sold the same shares for
Note that wash sales involve the sale of In 2016, Mr. Iriga had the following P95,000. On March 28, 2016, he bought
shares at a loss, but the same shares transactions in the shares of Naga 5, 000 shares for P55,000.
were effectively re-acquired before or Corporation, a domestic corporation: Note that the March 28 acquisition is
after the sale by a covering acquisition. Date Transaction Shares Price/Share
beyond the 61-day period. Since there
In this case, the P2,000 capital loss is Value is no acquisition of replacement shares
not a wash sales since there is no January 4 Purchase 15,000
within the 61-day period, the P5,000 is
acquisition of replacement shares P20.00 P300,000 not a wash sales loss but a deductible
within the 61-day period. Hence, the February 15 Purchase 5,000
realized loss against capital loss against
capital loss is deductible against capital 21.00 105,000 capital gain from the sale of domestic
gains. February 28 Sale* 12,000
directly to a buyer. The basis of the
Illustration 2: Acquisition of identical 18.00 216,000 shares bought on March 28, 2016 shall
shares after a losing sale March 4 Purchase 3,000
be P55,000.
In 2016, Mr. Balangkayan had the 16.00 48,000 Rationale of the wash sales rule
following transactions in the stocks of April 1 Purchase 7,000
The wash sale rule is intended to
Sta. Rita Corporation, a domestic 14.00 98,000 prevent taxpayers from feigning
corporation: The shares sold on February 28 were attemporary losses which could enable
Date Transaction Shares Price Cost a loss shares bought on January 4, them to manipulate their reportable
January 4 Purchase 10,000 P20.00 2016. The capital loss is P24,000 taxable net gain. Hence, the prohibition
P200,000 computed as (P18/share selling price- against the claim of wash sales is not
February 28 Sale 10,000 18.00 P20/share cost)x12,000 shares sold. absolute rule but is a form of deferral of
180,000 There were 12,000 shares sold at a loss
loss.
March 4 Purchase 12,000 16.00 while there were a total of 8,000 The wash sales rule is not applicable to
192,000 replacement shares in the 61-day dealers in securities as it is normal
The capital gains or capital loss shall be period:5,000 shares acquired on business for them to buy and sell stocks
computed as follows: February 15 (i.e., before the sale) and
and realize gains or incur losses within
Selling price P180,000 3,000 shares acquired on March 4 (i.e.,
short term periods of time.
Less: Cost of shares sold 200,000 after the sale). TAX FREE EXCHANGES
Capital loss (P20,000) Since this is a partial replacement, the
Merger or Consolidation
Since there is a full replacement over capital loss shall be split as follows:
Stockholders of a domestic corporation
(i.e., 12,000 shares) within the 61-day Deferred loss (8,000 shares/12,000 may exchange their stocks for the
period (i.e., March 4, 2016) the capital sharesxP24K) P16,000 stocks of another corporation pursuant
loss shall be deferred and included as Deductible loss(4,000/12,000 shares x to a plan of merger of consolidation
part of the cost of the replacement P24K) 8,000 The gains or losses on share-for-share
shares. Capital loss P24,000
swaps pursuant to a plan of merger of
The basis of the replacement shares consolidation will not be recognized for
purchased on March 4 shall be: The adjusted basis of the replacement taxation purposes. In a share-swap
Purchase price P192,000 shares acquired on February 15, 2016 pursuant to a plan of merger or
Add: Deferred loss on wash sale shall be: consolidation, the shareholders of the
20,000 Purchase price P105,000 acquired corporation will be integrated
in the acquiring corporation. The shares
of the acquired corporation will be Less: Cost of Cabanatuan stocks Total consideration received or selling
called in for replacement with the exchanged 2,000,000 price
shares of the acquiring corporation. Indicated loss (P1,050,000+P150,000) P1,200,000
In effect, the transaction merely (P200,000) Less: Cost of stocks exchanged
involves a replacement of shares of The P200,000 indicated loss shall not be 1,000,000
stocks of the shareholders of the recognized. Any indicated gain shall not Indicated gain
absorbed corporation with them being also be recognized. The law views initial P200,000
simply integrated as shareholders of the acquisition of corporate control by not The indicated gain is recognized to the
acquiring corporation. more than persons as an investing extent of the cash and/or other
Illustration transaction. The tax basis of the properties received. The indicated gain
Mr. Santiago was required to surrender Dingalan shares received shall be is considered as follows:
his Carranglan Inc. shares in exchange P2,000,000, the same as the tax basis of Realized gain (up to the value of cash
for Baler shares with total fair value of the Gapan shares exchanged. and other properties received)
P1, 200,000 pursuant to the merger of Exchanged not solely for stocks P150,000
Carranglan Inc., and Baler Inc. The In tax-free exchanges, if stocks are Unrealized return on capital (in excess
Caranglan shares were previously exchanged not solely for stocks but with of the value of cash and other
purchased by Mr. Santiago for other consideration such as cash and properties received) 50,000
P1,000,000. other properties, the gains but not Total indicated gain
Fair value of Baler shares received losses are recognized up to the extent P200,000
(selling price) P1,200,000 of cash and other properties received. 200,000-150,000=50k
Less: Cost of Carranglan shares Illustration 1:Cash and property The substituted tax basis of the Baler
exchanged 1,000,000 received exceed indicated gain shares received shall be:
Indicated gain P200,000-not Assume that pursuant to the plan of Basis of the Carranglan shares
taxable merger between Carranglan Inc. and exchanged P1,000,000
Baler Inc., Mr. Santiago was required to Add: Basis of other properties
The P200,000 indicated gain is not surrender his Carranglan Inc., shares exchanged 0
taxable as the exchange involves stocks costing P1,000,000 in exchange for Less: Return on capital
for stocks. Similarly, an indicated loss Baler shares with total fair value of 0
shall not likewise be recognized. The P900,000 plus P100,000 cash and Basis of the Baler shares received
P1,000,000 tax basis of the Carranglan P200,000 worth of goods. P1,000,000
shares given shall be carried over as the Total consideration received or selling Regulatory Formula on Tax Substituted
substituted basis of the Baler shares price Basis
received. (P900,000+P100,000+P200,000) The regulations prescribe the following
Initial Acquisition of control P1,200,000 formula in computing the tax basis of
No gain or loss shall also be recognized Less:Cost of stocks exchanged properties arising from the tax-free
if property is transferred to a 1,000,000 exchanges:
corporation by a person in exchange for Indicated gain Tax basis of old shares exchanged
the stocks or units or participation in P200,000 Pxxx.xx
such a corporation of which as a result The amount of cash and other Add: Gain recognized on the transfer
of such as exchange, said person, alone properties received is considered xxx.xx
or together with others not exceeding realization of gains to the extent of the Less: Cash or other properties received
four, gains control of said corporation. indicated gain. The excess amount of xxx.xx
“Control” shall mean ownership of cash and other properties received is a Tax basis of new shares received
stocks in a corporation possessing at return of capital. Hence, Pxxx.xxx
least 51% of the total voting power of Realized return on capital If we apply this formula to the two
all classes of stocks entitled to vote. (to the extent of the indicated gain) previous illustrations, we would come
This rule may be relevant only to the P200,000 up with essentially the same tax bases
capital gains tax or the recognition of Return of capital (in excess of the as computed.
capital gains when stocks are exchanged indicated gain) 100,000 Minimum public float requirement of
in the acquisition of corporate control. Total cash and other properties publicly listed corporations
Illustration received P300,000 Listed corporations are mandatorily
Mr. Gapan exchanged his shares in Illustration 2: Indicated gain exceeds required to maintain a minimum public
Cabanatuan Corporation costing cash and other properties received ownership under Philippine Stock
P2,000,000 in exchange for the shares Assume that pursuant to the plan of Exchanges(PSE) regulations.
of Dingalan Corporation with fair value merger between Carranglan Inc. and The minimum public ownership is the
of P1,800,000. The transfer resulted in Baler Inc., Mr. Santiagp was required to higher of:
Mr. Gapan acquiring 51% ownership surrender his Carranglan, Inc. shares 1.The 10% of issued and outstanding
(corporate control) in Dingalan costing P1,000,000 in exchange for shares and
Corporation. Baler shares with total fair value of 2. The minimum public ownership
Total consideration received or selling P1,050,000 plus P150,000 cash required by the Securities and Exchange
price P1,800,000
Commission or the Philippine Stock Hence, it is not subject to capital gains Net dividends
Exchange. tax. receivable(P2x10,000x90%) 18,000
Non-compliance to the minimum public Illustration 5: Sale of stocks ex-dividend Capital gains P28,000
ownership shall result in the de-listing Ms. Pearl bought 10,000 shares of It should be noted that the dividends to
of the stocks of the corporation in the Zuma, a domestic Corporation, at be received by Pearl is net of the 10%
PSE. Under RR16-2012, the sale of listed P10/shares. On February 14, 2016, final withholding tax on dividends of
stocks which fall below their minimum Zuma declared a dividend of P2/share individual taxpayers.
public ownership requirement will be with record date of March 20,2016 and TAX ISSUE: SALE OF STOKCS DIVIDEND-
subject to the 5%-10% capital gains tax payment date of April 20, 2016, Ms. ON TO A CORPORATE BUYER
and not the ½ of 1% stock transaction Pearl sold all the shares for P15 per Dividends may escape taxation when
tax. share directly to a buyer. The selling stocks are sold dividend-on by
Comprehensive Illustrations expenses were P5,000. individual taxpayers to a corporate
Illustration 1: Sale by a security dealer The shareholder’s right to dividend buyer between the date of declaration
Benjie, a security dealer, sold various accrues at the date of declaration. The and the date of record. At the date of
domestic stocks for P1,200,000, net of stocks may pass through different record, the corporate buyer will be
selling expenses. These stocks were hands anytime. However, those who are listed as shareholder in the corporate
acquired at a cost of P800,000. registered as shareholders of the books and will not be subjected to the
The capital gains tax is nil because corporation at record date shall receive 10% dividend tax.
domestic stocks are ordinary assets to a the dividends/ Assuming the same data under
security dealer. The P400,000 net gains Between the date of record and the Illustration 6, the capital gain shall be
is an ordinary gain subject to regular date of payment, stocks are said to be computed as follows:
income tax. selling ex-dividend. The seller receives Total selling price (P15x10,000)
Illustration 2:sale of domestic bonds the dividends/ The price of the stocks P150,000
Carlo, not a security dealer, sold on those dates include only the selling Less: Costs and expenses
domestic bonds directly to a buyer at a price of the stocks. (P10x10,000+P4,000) 104,000
net gain of P200,000. Carlo is not a Thus, the capital gains shall be normally Dividends receivable (P2x10,000x100%)
dealer of domestic bonds. computed as follows: 20,000
The capital gains tax is nil. The gain on Total selling prize (P25x 10,000) Capital gains P26,000
the sale of domestic bonds is a capital P150,000 Note that the individual seller
gain subject to regular income tax. Less: cost of stocks and effectively realizes the entire dividend
Illustration 3: Exchange pf stocks for expenses(P10x10,000+P5,000)105,000 income under the cloak of the dividend
other securities Capital gains P45,000 exemption of the corporate buyer who
Debbie, an NRA-NETB, exchanged her In this case, the dividends to be will be registered as shareholder at the
domestic stocks costing P300,000 for received by Ms. Pearl shall be subject to date of record.
bonds with a fair value of P400,000. the 10% final tax to be withheld by How should the dividend on the stocks
The P100,000 capital gain is subject to Zuma Corporation. sold be taxed?
capital gains since it is not a share-for- Illustration 6:Sale of stocks dividend-on Under the NIRC, all income not
share swap pursuant to a plan of Ms. Pearl owns 10,000 P10 par value expressly exempted or not subjected to
merger or consolidation. The same rule shares of Zuma, a domestic final tax or capital gains tax must be
applies for share-for-share swap not Corporation. On February 14, 2016, included in gross income subject to
pursuant to a plan of merger or Zuma declared a dividend of P2/share regular income tax. Hence, the
consolidation. Non-resident persons not with record date of March 20,2016 and individual seller shall exceptionally
engaged in business in the Philippines payment date of April 20,2016. On report the P20,000 domestic dividend
such as NRA-NETBs and NRFCs are February 18, 2016. Ms. Pearl sold all the in gross income subject to regular
subject to the capital gains and are shares for P15 per share directly to Mr. income tax
required to file a capital gains tax Lover. The selling expenses were Persons not liable to the 5%-10% capital
return. P4,000. gains tax
Illustration 4:Issuance of stocks Between the date of declaration and 1. Dealers in securities
HKG Inc., a domestic corporation, the date of record, stocks are said to be 2. Investors in shares of stocks in a
issued 10,000 P10-par ordinary shares selling dividend-on; that is, the buyer mutual fund company in
in exchange for a vacant lot owned by shall receive the dividends. The selling connection with gains realized
KIT, Inc. The vacant lot has a fair value price of stocks on these dates includes upon redemption of stocks in
of P500,000. Compute the capital gains both the price of the stocks and the the mutual company.
tax. dividends on the stocks. 3. All other persons, whether
The transaction involves issue by HKG Thus, the capital gains of Ms. Pearl on natural or juridical, who are
Inc. of its own shares of stocks. These the disposition of the stocks dividend-on specifically exempt from
stocks do not represent investment in shall be determined as follows: national revenue taxes under
the shares of another corporation. The Total selling price (P15x10,000) 150,000 existing investment incentives
share premium of P300,000,(P500,000- Less:Cost and and other special laws.
(10,000XP10)), is part of HKG’s expenses(P10x10,000+P4,000)104,000 Examples:
corporate capital, not an income.
a.Foreign governments and of P2,500,000. The house had an Within the Philippines All individuals
foreign government-owned and assessed fair value of P2,000,000. Domestic Corporation only
controlled corporations Selling price P5,000,000- Outside the Philippines Not applicable
b.Qualified employee trust Fair value of land(P4M or 2.5 w/e is Not applicable
funds higher)P4,000,000 The 6% capital gains tax is applicable to
SALE, EXCHANGE, AND OTHER Assessed fair value of improvement all individual taxpayers but it applies
DISPOSITION OF REAL PROPERTY 2,000,000 only to domestic corporations.
CLASSIFIED AS CAPITAL ASSET LOCATED Fair market value (HIGHER) P6,000,000 Interestingly, the NIRC did not impose
IN THE PHILIPPINES The capital gains tax shall be computed final capital gains tax on foreign
The sale, exchange, and other at 6% x P6,000,000 or P360,000. corporations. However, in cases where
disposition of real property capital Illustration 3 foreign corporations realize gains from
assets in the Philippines, is subject to a A real property dealer sold a condo unit the sale of real property classified as
tax of 6% of the selling price or the fair costing P1, 200,000 to a client for P1, capital assets, the capital gain shall be
value, whichever is higher. 500,000. The unit has a total assessed subject to the regular income tax.
Under the NIRC, the fair value of real value of P900,000 and zonal value of Under the NIRC, the sale of real
property is whichever is higher of the: land and assessed value on property located abroad is not covered
a.Zonal value, which is the value improvements of P1,000,000 at the by the capital gains tax. Hence, the
prescribed by the Commissioner of date of sale. actual gains on sale, exchange, and
Internal Revenue for real properties for The capital gains tax is nil. The condo other dispositions of properties abroad
purposes of enforcements of internal unit is an ordinary asset to a realty are subject to the regular income tax if
revenue laws, and dealer, lessor or developer. The actual the taxpayer is taxable on global income
b. Assessed value, which is the value gain of P300,000(P1,500,000- such as resident citizens and domestic
prescribed by the City of Municipal P1,200,000) is an ordinary gain subject corporations. For all other taxpayers,
Assessor’s Office for purposes of the to regular income tax. the capital gain realized abroad is
real property tax. BIR TAX CLEARANCE exempt.
Zonal value exists only for land, but No registration of any document EXCEPTIONS TO THE 6% CAPITAL GAINS
assesses value is prescribed separately transferring real property shall be TAX
for land and improvements effected by the Registry of Deeds unless 1. Alternative taxation rule
For lands, the capital gains tax is 6% of the Commissioner or his duly 2. Exemption rules
whichever is the highest of the selling authorized representative has certified a. Exemption under the NIRC
price (bid price in the case of that such transfer has been reported, b. Exemption under the special
foreclosure sales), zonal value, or and the capital gains or creditable laws
assessed value. withholding tax, if any, has been paid. ALTERNATIVE TAXATION
Note that independent appraisal (Sec. 58€, NIRC) An individual seller of property capital
valuation, the fair value commonly used The certificate for purposes of this legal assets has the option to be taxed at
in external financial reporting, is not requirement is referred to as the either:
used in the computation of the capital “Certificate Authorizing Registration a.6% capital gains tax or
gains tax. (CAR)”. b. The regular income tax
Illustration 1 NATURE OF THE 6% CAPITAL GAINS TAX It should be noted that this is
Terry sold a parcel of land for a.Presumption of capital gains permissible only when:
P5,000,000. The land has an appraisal The 6% capital gains tax applies even if 1. The seller is an individual
value of P8,000,000, zonal value of the sale transaction resulted to a loss. taxpayer, and
P6,000,000, assessed value of Gain is always presumed to exist. The 2. The buyer is the government a
P5,000,000 and was previously basis of taxation is the selling price or vacant lot for P800,000. The lot
purchased by Terry for P4,000,000. fair value whichever is higher, not the was purchased for P200,000 in
The highest of the selling price, zonal actual gain. 1980 and had an assessed value
value, and assessed value is the b.Non-consideration to the of P400,000 and zonal value of
P6,000,000 zonal value. Hence, the involuntariness of the sale P500,000 at the date of sale.
capital gains tax would be computed The capital gains tax applies if the sale is Gretchen may opt to be subject to
P6,000,000x6%; hence, P360,000. It involuntary or is forced by tax at 6% OF P800,000 or report the
should be emphasized that the circumstances such as the case of P600,000 (P800,000-P200,000)
independent appraisal value is not expropriation sale, foreclosure sale, actual capital gain in her regular
used. dispositions by judicial order, and other income tax return.
Illustration 2 forms of forced disposition. It also Basis of Alternative Taxation
Anjo sold his residential and lot for applies to conditional sales and pacto The alternative taxation is intended
P5,000,000. Anjo purchased the lot de retro sales. to ease the burden of government
when it was worth P1,000,000 and SCOPE AND APPPLICABILITY OF THE 6% expropriation where taxpayers may
constructed on it the house at a total CAPITAL GAINS TAX incur losses on the forced
cost of P2,500,000. The lot had a zonal Location of the property Taxpayers expropriation sale and are required
value of P4,000,000 and assessed value Individuals to pay tax.
Corporations Illustration
An individual taxpayer bought a It must be emphasized that the sale Partial utilization of proceeds is
house and lot near a highway at a of principal residence must precede partially exempt
cost of P2,000,000. After several the acquisition of the new principal Assume Helen uses only P4,500,000
years, the government invoked its to be exempt. (BIR Ruling No. 038- out of the P5,000,000 proceeds in
power of eminent domain to buy 2015) acquiring the new residence.The
the property for the expansion of Illustration 1 portion representing the unused
the highway. Helen sold her principal residence proceeds shall be subject to tax. The
Assuming the property has a fair with a fair market value of capital gains tax held in escrow
value of P1,800,000 for purposes of P6,000,000 for P5,000,000. Helen account including any accrued
the expropriation, the taxpayer purchased the residence for interest shall be allocated as
would be forced to incur P200,000 P3,000,000 several years ago. The follows:
loss( P1.8,-P2.0M) and still pay the imposable capital gains tax in 6% of To Helen P324,000
6% capital gains tax. This would be P6,000,000 or P360,000. (P4.5M/P5MXP360,000)
too oppressive to the taxpayer. With Helen should indicate her intention To the government 36,000
the alternative regular income tax to apply for exemption in the capital (P0.5M/P5MXP360,000)
option, the taxpayer would be given gains tax return to be filed and Total amount in escrow P360,000
the benefit of deduction of the submit a sworn declaration of Note: Any interest which might have
P200,000 capital loss without being Intent. She will be required to accrued on the escrow fund shall be
imposed the 6% capital gains tax. deposit the P360,000 capital gains released to the taxpayer. The
EXEMPTION TO THE 6% CAPITAL tax in an escrow account in favor of government is entitled to the
GAINS TAX UNDER THE NIRC the government. amount of the unpaid tax only.
The sale, exchange and other Full utilization is exempt Tax basis of the new residence with
disposition of principal residence for Assuming Helen acquires a new less than full utilization
the re-acquisition of a new principal principal residence for P5,200,000 If the proceeds is not fully utilized,
residence by individual taxpayers is within 18 months, the P360,000 the tax basis of the new residence
exempt from the 6% capital gains capital gains tax in escrow will be shall be reduced as accordingly as
tax. released to her. follows:
Principal residence If Helen does not acquire a new Tax basis of old residence x utilized
Principal residence means the principal residence within 18 proceeds/total proceeds
house and lot which is the primary months, the capital gains tax escrow The tax basis of the new principal
domicile of the taxpayer. If the will be taken by the government. residence shall be computed as
taxpayer has multiple residences, Basis of new residence with full P3,000,000 x P4,
his principal residence is deemed utilization 500,000/P5,000,000=P2,700,000.
that one shown in his latest tax If the proceeds is fully utilized, the Illustration 2
declaration. tax basis of the new residence shall Alberto sold his residential lot with
Requisite of exemption: be the basis of the old residence fair value of P1,000,000 for
1.The seller must be a citizen or plus additional cost incurred by the P2,000,000. He purchased a new
resident alien. taxpayer in acquiring the new residence for P1,500,000 within 18
2. The sale involves the principal residence. The additional cost is the months.
residence of the seller-taxpayer. excess of the purchase price of the Alberto will be required to pay
3. The proceeds of the sale is new residence over the selling price P120,000 (P2,000,000 x6%) capital
utilized in acquiring a new principal of the old residence. gains tax whether or not he utilized
residence. Thus, the tax basis of the new the proceeds to acquire a new
4. The BIR is duly notified by the residence shall be: residence. Note that the exemption
taxpayer of his intention to avail of Basis of old residence P3,000,000 rule envisages a sale of a principal
the tax exemption within 30 days of Add: Additional out-of-packet residence for the acquisition of a
the sale through a prescribed return costs(P5.2M-P5M) 200,000 new principal residence.
(BIR Form 1706) and “Sworn Basis of new residence P3,200,000 Illustration 3
Declaration of intent.” Tax basis has no relevance for real Afraid of ghosts that frequently
5. The reacquisition of the new property capital assets because the appear in his mansion residence,
residence must be within 18 months actual gain on the sale is irrelevant Raymund left his mansion and
from the date of sale to capital gains taxation. However, bought a new home for
6. The capital gain is held in escrow when the real property capital P17,000,000 as his principal
in favor of the government. assets subsequently qualifies as residence. Within 3 months,
7. The exemption can only be ordinary assets such as when they Raymund was able to sell his
availed of once in every 10 years. are later employed in business, the mansion for P40,000,000.
8. The historical cost or adjusted tax basis of the property becomes The sale of the mansion will be
basis of the principal residence sold necessary for gain or loss subject to 6% capital gains tax. For
shall be carried over to the new measurement. That’s why the basis purposes of the exemption, the sale
principal residence built or of the new property needs to be of the old residence must precede
acquired. monitored. the purchase of the new.
CAPITAL GAINS TAX EXMEPTION Initial payment (December (P2.5M mortgage-P2.0M
UNDER SPECIAL LAWS installment) P500,000 cost)500,000
1. Sale of land pursuant to the Ratio of initial Contract price
Comprehensive Agrarian Reform payment=P500,000/P4,000,00012.5 P2,000,000
Program 0% The initial payment shall be
2. Sale of socialized housing units The installment sale qualifies under computed as:
by the National Housing the ratio ceiling: hence the capital Constructive downpayment(excess
Authority gains tax can be paid in installment. mortgage)
Sale of land under the The capital gains tax payable every P500,000
Comprehensive Agrarian Reform installment shall be P37,500 December 31 installment
Program computed as 300,000
The sale of agricultural lands by land P500,000/P4,000,000XP300,000. Initial payment
owners pursuant to the Illustration 2:With mortgage not in P800,000
Comprehensive Agrarian Reform excess of cost Ratio of initial
Program of the government shall be Assume that the lot in the previous payment=P800,000/P4,000,000 20%
exempt from capital gains tax. illustration is mortgaged for The installment plan qualifies under
Similarly, interest income on the P1,000,000 which the buyer the ratio ceiling; hence, the capital
selling price that may have agreed assumed and the buyer agreed to gains tax can be paid in installment.
by the land owner and the tenant- pay the P3,000,000 balance in Under the installment method, the
buyer shall be exempt from income P300,000 monthly installments capital gains tax payable shall be:
tax. starting December 31, 2016. For the
Sale of socialized housing units by Capital gains tax=P5,000,000x6% sale:P500,000/P2,000,000Xp300,00
the National Housing Authority P300,000 0 P75,000
The sale of socialized housing units Initial payment (December For every installment:
for the underprivileged and installment) P300,000 P300,000/P2,000,000XP300,000
homeless citizens by the National Ratio of initial payment P45,000
Housing Authority (NHA) pursuant (P300,000/P4,000,000) 7.5 % Illustration 4:Initial payment
to the Urban Development and The contract price shall be exceeds 25% of selling price
Housing Act of 1992 is exempt from computed as follows: Assume that the initial payment of
the capital gains tax. Selling price P4,000,000 the sale of Ms. Batanes exceeds
This exemption is limited to Less: Mortgage assumed by buyer 25% of the selling price.
socialized housing units only. The 1,000,000 The sale would be taxed as if itwere
BIR ruled that the sale of the NHA of Contract price P3,000,000 a cash sale. The capital gains tax
commercial lots which is not part of The capital gains tax payable every shall be paid in lump sum upon
the socialized housing project for installment shall be P50,000 filing of the capital gains tax return.
the poor and homeless is subject to computed as This applies without regard to
capital gains tax or regular tax and P300,000/P3,000,000Xp300,000 whether or not any mortgage on
documentary stamp tax. capital gains tax. the property exceeds the cost of the
To qualify for exemption, the Illustration 3: With mortgage in property disposed.
socialized housing units of the NHA excess of cost Deadline for payment of the capital
must comply with price ceilings set Assume further that the lot is gains tax
by the NIRC and other special laws. mortgage for P2,500,000 which the The 6% capital gains tax will be filed
PAYMENT OF THE 6% CAPITAL buyer assumed and the buyer through BIR From 1706 and is due
GAINS TAX IN INSTALLMENT agreed to pay the P1,500,000 within 30 days from the date of sale
The capital gains tax may be paid in balance in P300,000 monthly or exchange. For foreclosure sales, it
installment if, under the payment installment starting December 31, is due within 30 days from the
terms, the initial payment does not 2016. expiration of the applicable
exceed 25% of the selling price. The It should be recalled that the excess statutory redemption period. When
“initial payment” refers to the of the mortgage over the tax basis the tax on the sale is qualified for
collections in the taxable year the of the property is an indirect installment payment, It is due 30
sale is made. downpayment which must be days upon receipt of every
Illustration 1: Without mortgage included in the initial payment and installment.
On December 1, 2016, Ms. Batanes contract price. Capital gains tax is Statutory redemption period on
sold for P4,000,000 an unused lot P5,000,000 x6% orP300,000. foreclosure sale
with a cost and fair value of The contract price shall be Foreclosed properties are subject to
P2,000,000 and P5,000,000, computed as follows: a right of redemption by individual
respectively. The buyer agreed to Selling price P4,000,000 mortgagor within a year counted
pay P500,000 monthly installments Less: Mortgage assumed 2,500,000 not from the date of sale but from
starting December 31,2016. Cash collectible P1,500,000 the time of registration of the sale
Capital gains tax=P5,000,000x6% Add: Constructive downpayment- in the Office of the Registry of
P300,000 excess mortgage
Deeds. (Santos vs. Register of Deeds same penalties discussed in Chapter engaged in the realty business.
of Manila) 4. True
For juridical persons, redemption ENTITIES EXEMPT FROM CAPITAL 4. Capital assets will not become
must be made before the GAINS TAX ordinary assets when used in
registration of the certificate of The same lists of entities exempt business.False
foreclosure sale with that applicable from final tax in Chapter 5 are 5. The sale of real property capital
Register of Deeds or within 3 likewise exempt from capital gains assets will never be subject to
months from foreclosure, whichever tax. regular income tax.-False
is earlier. CHAPTER 6: SELF-TEST EXERCISES 6. The sale of real property capital
DOCUMENTARY STAMP TAX ON THE Discussion Questions assets will never be subject to
SALE OF CAPITAL ASSETS 1. What are ordinary assets and regular income tax.False
Documentary stamp tax on the sale, capital assets? Discuss 7. Donated assets become
exchange, and other dispositions of 2. Enumerate the two types of ordinary assets even if the done
domestic stocks directly to a buyer capital assets subject to capital do not employ the same in
The sale of domestic stocks is gains tax. business-True
subject to a documentary stamp tax 3. What are the transactions 8. An ordinary asset continues to
of P0. 75 for every P200 of the par considered as “other be an ordinary asset even if
value of the stocks sold.(RA. 9243) disposition” of domestic stocks? idled for more then two years if
Illustration What transactions are not the taxpayer is engaged in realty
A taxpayer sold domestic stocks considered as “other business. True
with total par value of P800,000 for dispositions”? 9. The real properties used by
P1,200,000. The stocks have a fair 4. Discuss the rules on tax basis of exempt corporations in their
value of P1,250,000 and were stocks acquired by purchase, exempt operations are capital
acquired for P1,000,000. inheritance, donation, for an assets- True
The documentary stamp tax shall be inadequate consideration, and 10. Dealers in realties are subject to
P3,000 computed as P0.75/P200 x under a tax-free exchange. the regular tax on their sale of
P800,000. 5. Enumerate the methods in properties. True
Documentary Stamp tax on the sale costing stocks in order of 11. Capital gains from assets other
of real properties priority than domestic stocks and real
The sale of real property capital is 6. Discuss the compliance properties are subject to regular
subject to a documentary stamp tax requirements of the two-tiered income tax. True
on the gross selling price or fair capital gains tax. 12. Dealers in securities are not
value whichever is higher. 7. Explain the concept of a wash subject to the stock transaction
The documentary stamp tax is P15 sale. tax but are subject to the
for every P1,000 and fractional parts 8. Enumerate and discuss the tax- regular income tax on gains
of the tax basis thereof. However, if free exchanges realized upon the sale of stocks
the government is a party to the 9. What are the criteria of through the Philippine stock
sale, the basis shall be the alternative taxation to the 6% exchange True
consideration paid. capital gains tax? 13. Unit of participations in golf,
Illustration 10. Enumerate the exemption polo, and similar clubs are
A taxpayer disposed a real property requirements to the 6% capital considered domestic stocks.True
capital asset acquired for gains tax 14. The excess premium on the re-
P2,000,000 10 years ago for 11. Discuss the nature of the 6% issuance of treasury stocks is
P4,000,000. The property has a capital gains tax. subject to capital gains tax-false
zonal value of P5,000,000 and 12. Discuss the nature of the 6% 15. The issuance of shares of stock
declared real property per real capital gains tax. for property is subject to capital
property tax declaration of 13. Compare the taxpayers covered gains tax. False
P3,000,000. by the 5%-10% capital gains tax 16. The sale of foreign stocks
The documentary stamp tax shall be and the 6% capital gains tax. directly to a buyer is subject to
computed from the fair value since True or false capital gains tax.-False
it is higher than the selling price. 1. A vacant and unused lot is an 17. The two-tiered final tax cannot
Hence, the documentary stamp tax ordinary asset to a real estate apply unless and until there is a
shall be P75,000 computed as dealer. True gain on the sale, exchange, and
P15/P1,000 x P5,000,000. 2. For taxpayers not engaged other disposition of stocks
PENALTIES FOR LATE/NON-FILING business, assets shall cease to directly to a buyer. True
OR NON-PAYMENT OF CAPITAL be ordinary assets when they 18. The stock transaction ax on the
GAINS TAX are discontinued from active use sale of stocks through the PSE
The late filing and payment of for more than two years.False cannot apply unless there is a
capital gains tax at the time or times 3. Real and other properties gain on the transaction. False
required by law is subject to the acquired are ordinary assets to
banks even if they are not
19. The 6% capital gains tax cannot 11. Control means more than 50% a. Vacant lot held for future
apply unless there is a gain on ownership in the voting power development
the sale of real property. False of a corporation. True b. Construction equipment
20. The sale of real properties 12. The sale of delisted stocks is c. Domestic stocks-
located abroad is subject to the subject to stock transaction tax d. Head office building of the
6% capital gains tax.False and not to capital gains tax. developer
True or false 2 False 4. Which is an ordinary asset?
1. The annual capital gains tax 13. Fain and loss in a share-for-share a. Home appliances
return is simultaneously due swap pursuant to a plan of b. Personal car
with the annual regular income merger consolidation shall be c. Personal cellphone
tax return. True recognized up to the extent of d. Office supplies-
2. The basis of properties received the cash and other properties 5. Which of the following assets, if
by way of inheritance is the received. True not used in business, is subject
basis in the hands of the last 14. The sale by the National Housing to regular tax?
owner who did not acquire the Authority of commercial lots is a.Real property
same by donation. False subject to capital gains tax. True b. Domestic stock rights
3. When specific identification is 15. If the assessed value is lower c. Domestic stock option
impossible, the cost of the than the selling price, then the d. Taxpayer’s personal car-
stocks sold is determined by the fair value of the property is the
6. Which is a capital asset for a security
weighted average method. False zonal value. False dealer?
4. The basis of the stocks received 16. Title to a property shall not be
a. Domestic stocks
in tax-free exchanges is the basis registered by the Registry ofb. Domestic bonds
of the share from the date of deeds unled the Commissioner c. Real property held for speculation-
sale. True or his representatives has D.Office equipment
5. The gain on the sale of stocks for certified that the tax on the7.Which is subject to the 5%-10%
stocks pursuant to a plan of transfer has been paid. True capital gains tax?
merger and consolidation is 17. Domestic corporations are
a.Sale of domestic stocks directly to a
exempt if it resulted in the exempt from capital gains tax on
buyer within or outside the
transferor acquiring corporation the sale, exchange, and otherPhilippines.=
over the absorbed corporation. disposition of real properties.
b. Sale of domestic bonds directly to a
True False buyer within the Philippines.
6. The gain on the sale of stocks for 18. The sale of land pursuant to the
c.Sale of domestic stocks through the
stocks pursuant to a plan of Agrarian Reform Program is Philippine stock exchange
merger consolidation is exempt exempt from capital gains tax.
d. All of the above
if it resulted in the transferor True 8. Who is not subject to capital gains tax
acquiring corporation over the 19. Foreign corporations are
on the sale of domestic stocks directly
absorbed corporation. True required to pay capital gains tax
to a buyer?
7. Installment payment of capital on the sale of domestic stocks
a. Dealer of cars
gains tax is allows if the ratio of and on the sale of real b. Real property developer
downpayment over the selling properties capital assets. False
c.Dealer of securities-
price of the sales does not 20. The alternative taxation on an
d. Realty dealer
exceed 25%. expropriations sale is not 9.Which of the following, when sold is
False applicable to corporate
not subject to capital gains tax?
8. The selling price is used to taxpayers. True a. Boarding house
determine the propriety of using Multiple Choice-Theory part 1 b. Warehouse
the installment indirect method 1. Which is an ordinary asset? c. House and lot
but the contract price is used to a. Personal car d.A and B-
determine the capital gains tax b. Principal residence of the10. Which is not subject to the 6%
payable in installment. True taxpayer capital gains tax?
9. The excess of mortgage over the c. Delivery truck- a. Donation of property-
basis assumed by the buyer d. Wedding ring of the taxpayer
b.Foreclosure of a mortgaged property
constitutes an indirect receipt 2. Which is not an ordinary asset?
c. Expropriation of one’s property in
which is part of the initial a. Personal laptop of the favor of the government.
payment and the selling taxpayer- d. Sale of property for an insufficient
price.False b. Machineries and equipment consideration.
10. Wash sales occur when there is c. Real property held for sale
11.Statement 1: Capital gains may arise
a repurchase of shares within 30 d. Leasehold improvements from sale, exchange, and other
days before and 30 days after disposition of movable properties used
the date of disposal of securities in business.
at a loss. True 3. Which is a capital asset to a Statement 2: Ordinary gains may arise
realty developer? from sale, exchange, and other
disposition of real properties not used 2. Which of the following 9. Statement 1:The sale or
in business. assets may be subject to exchange must result to an
Which is true? capital gains tax upon -actual gain before the 5%
a.Statement 1 is correct disposal? -10% capital gains tax is
b. Statement 2 is correct a. Parking lot imposed.
c. Both statements are false- b. Dormitory Statement 2:The sale or
d. Both statements are correct. c. Farm lot- exchange must result to an
12. Statement 1:The gains on sale of d. Office supplies actual gain before the 6%
domestic stocks directly to a buyer is 3. The sale of an office building capital gains tax is imposed.
presumed. will be subject to a. Both statements are
Statement 2:The gain on sale of real a. ½ of 1% percentage tax. correct
properties is presumed. b. 6% capital gains tax b. Both statements are
Which of the following correct? c. 5%-10% capital gains tax incorrect
a. Both the statements are true. d. Regular tax- c. Only statement 1 is
b. Both statements are false 4. The term “asset disposition” correct-
c. Only statement 1 is true covers d. Only statement 2 is
d. Only statement 2 is true- a. Foreclosure sales correct
b. Auction sales 10. When the annualized capital gains
13.Which of the following c. Expropriation by the tax exceeds the transactional capital
properties when sold may be government gains tax the excess is a
subject to capital gains tax? d. Any of these- a. Tax credit
a. Domestic stocks- 5. Which of the following sales b. Tax payable-
b. Foreign stocks of domestic stocks is subject c. Tax refundable
c. Patent to capital gains tax? d. A or B
d. Office buildings a. Sale of domestic stocks 11.1st statement:Properties acquired by
14. Statement 1:Only through the PSE real estate dealers are ordinary assets
depreciable properties of b. Issue of domestic stocks 2nd statement:Properties of real estate
business can be classified as to subscribers dealers continue to be classified as
ordinary assets. c. Sale of domestic stocks ordinary asset even if they change the
Statement 2:Land used in directly to a buyer- nature of their business.
business is a capital asset since d. Exchange of stocks for a. First statement is correct
it is not subject to depreciation. stocks in a corporate b. Second statement is correct
Which of the following correct? merger. c. Neither statement is correct
a. Statement 1 is false 6.The sale of listed shares d. Both statements are correct.-
b. Statement 2 is false may be subjected to 12. 1st statement:When realty
c. Both statements are false- a. 6% of capital gains tax businesses discontinue used of assets
d. A, B and C only. for more than 2 years, the same shall be
15.Statement 1:Ordinary gains b. ½ of 1% percentage tax reclassified as capital assets/
arise from sale, exchange, and only 2nd statement: When realty businesses
other dispositions of real c. 5%-10% capital gains tax discontinue used of assets for more
properties used in business. only- than 2 years, the same shall be
Statement 2:Capital gain may d. Any of these reclassified as capital assets.
arise from sale, exchange, and 7. The sale of non-listed a. First statement is correct
other dispositions of real shares may be subjected to b. Second statement is correct
properties not used in business. a. 6% capital gains tax only c. Neither statement is correct-
Which is false? b. ½ of 1% percentage tax d. Both statement are correct
a. Statement 1 is correct only 13.Which is an incorrect statement?
b. Statement 2 is correct c.5%-10% capital gains tax a. The capital gains tax on the
c. Both statements are false only.- disposition of capital stock presumes
d. Both statements are d. Any of these the existence of gain on the sales
correct.- 8. Which of the following transaction
Multiple Choice-Theory:Part 2 when sold may be exempted b. The buyer of real property capital
1. Which of the following from the 6% capital gains asset shall withhold the tax at source
properties, when sold, may tax? and remit the same to the government-
be covered by regular a.Unused land to the c. Capital gains tax is identified under
income tax? government the NIRC as a form of final tax.
a. Share options b. Residential lot d. The capital gains tax on the
b. Preferred stocks c. Developed residential disposition of real property presumes
c. Share warrants properties for sale the existence of gain on the sales
d. Promissory notes- d. Principal residence- transaction.
14. Which of these shall pay the two- d. on or before the 15 th day of the 9. Partial taxation under the 6% capital
tiered capital gains tax? fourth following the close of the quarter gains tax will result when
a. A real property developer- when the sale was made a. The proceeds from the sale of the old
b. A dealer in stocks 3.Capital gains that is not payable on property exceeds both its cost and the
c. A merchandiser or trader of goods installment basis is due acquisition price of the new property.
d. A or B a.within 30 days from the date of sale b. The proceeds of the sale exceeds its
15. The sale of real properties which or exchange- zonal value and assessed value
would otherwise be subject to the 6% b.within 30 days from the end of month c. The proceeds of the old property
capital gains tax may nevertheless be of sale exceeds the acquisition price of the new
subject to regular income tax if all of c. on or before the 15th day of the property regardless of the tax basis,
the following conditions are met, except fourth month following the close of the zonal value, and assessed value of the
one. Which is the exception? quarter when the sale was made. old property.-
a. the seller must be an individual d. on or before the 15 th day of the d. The zonal value is greater than the
taxpayer fourth month following the taxpayer’s sale proceeds of the old property.
b. the sale involves the principal year-end 10. The transactional capital gains tax
residence of the taxpayer- 4. Installment payments of the 6% on domestic stocks is
c. the buyer is the government capital gains tax is due a. not a final tax.
d. the taxpayer opted to be subjected to a.Within 10 days from the date of sale b. included to the regular income tax.
regular tax. of each installment payment. c.creditable to the regular income tax.
16. Which of these pay the 6% capital b.Within 30 days from the date of each d. creditable to the annual capital gains
gains tax? installment payment- tax due-
a. Security dealer- c. Within 15 days from the date of each 11. The 5%-10% capital gains tax does
b. Real property dealer installment payment not apply to
c. Real property developer d. Within 20 days from the date of each a. Resident citizen dealers of stocks-a
d. None of these installment payment. b. Non-resident citizen dealers of cars
17. The sale of a principal residence is 5. The installment payment of capital c. Resident alien dealers of computer
exempt from the capital gains tax if all gains tax is applicable to the parts
of the following conditions are met, a.5% and 10% capital gains tax only d. Domestic corporations dealing in real
except b. 6% capital gains tax only properties
a. The proceeds is fully utilized in c. Both A and B- 12. The documentary stamp tax on the
acquiring a new principal residence d. Neither A nor B sale of domestic stocks directly to a
b. The reacquisition must be by 6. The installment payment of capital buyer is based on
purchase- gains tax is applicable to a. Selling price
c. The reacquisition must have been a. Individual taxpayers only b. Par value-
made within 18 months from the date b. Corporate taxpayers only c. Fair value
of sale. c. Dealers in properties d. Cost
d. The capital gains tax must be d. A or B- 13. The documentary stamp tax on the
deposited in escrow. 7. Which of these capital gains is sale of real property is based on
Multiple choice-Theory: Part 3 subject to capital gains tax? a. Selling price
1. The transactional 5%-10% a. Gain on the sale of stock rights- b. Fair value
capital gains tax is to be paid b. Gain on the sale of derivative c. Cost
a.Within 30 days from the date financial instruments linked to d. A or B, whichever is higher-
of sale or exchange- commodity prices 14. The 6% capital gains tax does not
b. Within 30 days from the end c. Gain on sale of interest in a apply to
of month of sale professional partnership a. Domestic corporations
c. On the 15th day the fourth d. Gain on sale of bonds b. Resident aliens
month following the close of the 8. Paulo indicated in his return his c. Non-resident citizens
quarter when the sale was made intent to avail of the exemption from d. Foreign corporations-
d. On the 15th day of the fourth the 6% capital gains tax. Under what 15. Who shall file the capital gains tax
month following the taxpayer’s condition will he be exempted? return for the sale, exchange, and other
year-end. a.When the proceeds of the sale disposition of real property?
2.The annual 5%-10% capital gains tax exceeds the cost basis of the property a.Seller
return is due sold b. Buyer
a. within 30 days from the end of the b. When the proceeds of the sale c. Transfer agent
month of sale exceeds the acquisition price of the new d. The registry of deeds
b. within 30 days from the date or residence Multiple choices-Problems:Part 1
exchange c. When the cost basis of the property 1. Mr. Dionisio sold domestic
c.on or before the 15th day of the fourth sold exceeds its selling price stocks directly to a buyer at a
month following the taxpayer’s year- d. When the acquisition price of the mark-up on cost of P200,000. He
end- new property exceeds the proceeds of paid P5,000 broker’s
the old property sold- commission and P8,000
documentary stamp tax on the loan from the latter. Digong a.P20,000
sale. Compute the capital gains acquired the shares for b.P16,000
tax. P300,000. Ignoring c.P12,800
a. P15,000 documentary stamp tax, d.P0-
b. P14,500 compute the capital gains 12.Compute the taxable gain on the
c. P14,200 tax on the transaction. December 7 sale
d. P13,700- a. P0 a. P64, 118
2. Mr. Abdul, a non-resident alien, b. P7,500 b. P60,000-
sold domestic stocks directly to c. P5,000 c.P51, 467
a buyer at a net gain of P70,000. d. P2,500- d.P44,000
Compute the capital gains tax. 7. On January 5, 2017, Mercy, a e.P44,000
a. P8,000 stock dealer, disposed the 13. Mr. Trinidad has the following
b. P6,000 following shares directly to a transactions during the year on the
c. P4,000 buyer: common stocks of Philippines Pines, a
d. P3,500- Shares Selling price Cost domestic non-listed company:
3. Mr. Panay, a non-resident Stock rights P200,000 Date Transaction Gain(loss)
citizen, sold domestic stock P170,000 5/8/2017 Sale P120,000
rights directly to a buyer at a net Common stocks100,000 8/5/2017 Sale (10,000)
gain of P320,000. Compute the 110,000 9/8/2017 Sale 250,000
capital gains tax Ignoring the documentary Compute the final capital gains tax on
a.P30,000 stamp tax, the capital gains April 15, 2018
b.P25,000 tax payable on the sale is a.P29,000
c.P27,000- a. P0- b.P22,000
d.P15,000 b. P1,000 c.P3,000-
4.Mr. Digos sold shares of a resident c. P1,500 d.P0
foreign corporation directly to a buyer. d. P3,000 14. Mr. Kalibo shows the following
The shares were purchased for 8.Kidapawan, Inc. a domestic service transactions in the shares of Aklan
P100,000 and were sold at a net selling company, has the following transactions Corporation, a closely held corporation:
price of P210,000. Compute the capital on the sale of another domestic Date Transaction Quantity Price
gains tax. corporation: 2/8/2017 Buy 10,000
a.P11,000 Transaction Quantity Net Price P120,000
b.P6,000 Purchase 20,000 P40,000 4/5/2017 Sell 10,000
c.P5,500 Purchase 30,000 63,000 100,000
d.P0 Sale 40,000 92,000 5/1/2017 Buy 20,000
5. Grace sold domestic shares directly Assuming the first-in, first-out method, 240,000
to buyer. The following relates to the compute the capital gains tax on the What is the tax basis of the shares
sale: sale acquired on May 1, 2017?
Fair market value of shares P400,000 a.P500- a. P300,000
Selling price 300,000 b.P480 b. P260,000-
Cost 150,000 c.P400 c. P240,000
Compute the capital gains tax. d.P0 d. P220,000
a.P7,500 9. Assuming the moving-average 14. Mrs. Aurora, a resident citizen,
b.P9,925 method, compute the capital gains tax purchased 100,000 shares of
c.P10,000- on the sale PhilHotdogs, a domestic listed
d.P15,000 a.P500 company. The shares were
6. Texas Inc. exchanged its investments b.P480- acquired at P200,000. She
representing domestic shares for a c.P400 disposed the shares through the
piece of land owned by E Inc. d.P0 Philippine stock Exchange at a
Fair market value of shares P400,000 10. Koron Company, a trading company, fair value of P250,000.
Fair market value of land 500,000 made the following transactions during Compute the capital gains tax.
Par value of shares 300,000 the year involving the stocks of Xurpas, a.P0-
Cost of shares 350,000 a domestic corporation: b.P5,000
Compute the capital gains tax Date Transaction Shares Net price c.P2,500
a. P5,000 6/15/2017 Purchase 10,000 P30 d.P10,000
b. P7,500 9/30/2017 Sale 8,000 28 15.Mr. Bosun disposed various stocks at
c. P9,988- 10/3/2017 Purchase 15,000 25 a total consideration of P400,000 and
d. P15,000 12/7/2017 Sale 10,000 32 paid thereon stock transaction tax of
6. Digong Inc. exchanged its Koron uses the FIFO method in using P2,000. Aggregate gains realized totaled
share investment from Bee the Xurpas stocks. P98,000 after the stock transaction tax.
Inc., as payment of its Compute the deductible loss on the What is the capital gains tax?
P350,000 long outstanding September 30 sale. a.P0-
b.P4,900 The deductible capital loss against property. What is the classification of
c.P5,000 capital gain on the wash sale is the property?
d.P9,800 a.P0 a. Ordinary asset-
Multiple Choice-Problems:Part 2 b.P13,333 b.Capital asset
1. A certain taxpayer shows the c.P10,000- c.Either A or B at the discretion of the
following over-the-counter d.P20,000 BIR
transactions shares of a 6. What is the cost of the 15,000 shares d.Either A or B depending on the intent
domestic corporation: acquired in the preceding problem? of the buyer
Date Transaction Quantity Net a. P150,000 2. Assuming the same date in the
price b.P160,000 preceding number except that the
2/8/2015 Purchase 10,000 P112,000 c.P180,000 property was not disposed of but the
4/5/2015 Sale 10,000 100,000 d.P190,000- same was used as a sales outlet which it
5/1/2015 Purchase 8,000 80,000 7. Isidro sold 1,500 shares of stocks of became vacant for more than two
6/7/2015 sale 5,000 60,000 Achievers Corporation directly to a years. What is the classification of the
Compute the capital gains on June 7, buyer. The share’s par value per shares property?
2015 that is subject to capital gains tax. was P85. Isidro purchased the shares a.Ordinary asset, regardless of the
a.P4,000 for P90 each. On the date of sale, the taxpayer
b.P5,000 shares had a selling price of P120 per b.Capital asset, regardless of the
c.P10,000 share. taxpayer
d.P12,000 What is the capital gains tax on the c.Ordinary asset, if taxpayer is not
2.A non-security dealer sold domestic sale? engage in real estate business
stocks directly to a buyer on October 1, a.P2,65 d.Capital asset, if the taxpayer is not
2017 under the following terms: b.P2,250- engaged in real estate business-
Selling price P500,000 c.P14,000 3.Anderson disposes a vacant lot for
Cost 200,000 d.P11,375 P3,000,000. The lot has an assessed
Downpayment 10% 8. Mr. Palangdan purchased domestic value of P2,800,000, a zonal value of
Installments in 2015 50,000 stocks which were priced at 150% P3,200,000, and an appraisal value of
Compute the total capital gains tax in above their par values. After two years, P3,500,000. What is the capital gains
2017. he sold the stocks when the fair value tax?
a.P30,000 doubled. He paid P3, 750.00 a.P0
b.P25,000 documentary stamp and P10,000 in b.P180,000
c.P6,250 commission expenses on the sale. c.P192,000-
d.P5,000 Compute the selling price of the stocks. d.P210,000
3.ABC realized the following gains or a.P3,000,000- 4.Puerto Prinsesa Company sold its
losses in selling various securities: b.P2,500,000 parking lot for P2,000,000. The lot has a
Gain on sale of domestic stocks C.P1,500,000 zonal value of P2,500,000 and appraisal
P300,000 d.P1,000,000 value of P1,800,000. The capital gains
Par value of domestic stocks sold 9.Compute the capital gains tax. tax on the sale of lot is
200,000 a.P143, 625- a.P0
Gain on the sale of interest in a b.P145,000 b.P108,000
partnership 200,000 c.P152,750 c.P120,000
Gain on the sale of stocks of foreign d.P153,725 d.P150,000
corporations 150,000 10. On June 20, 2016, Mr. Lito filed the 5. Mr. Antonio disposed his principal
Compute the capital gains tax capital gains tax return involving the residence for P2,000,000 and
a.P45,000 sale of domestic stocks on February 20, immediately acquired a new one for
b.P40,000 2016. The net gain was P140,000. P1,800,000. The old residence cost Mr.
c.P35,000 Compute the total amount due Antonio P1,000,000 and had a fair
d.P25,000- including penalties except compromise market value of P2, 500,000 on the date
4. Compute the documentary stamp tax penalty. of sale.
in the preceding problem a.P11, 250 Compute the capital gains tax to be
a.P0 b.P11,500 deposited in escrow.
b.P562.50 c.P11,700- a.P0
c.P1,125 d.P12,250 b.P60,000
d.P750- Multiple Chocie-Problems:Part 3 c.P120,000-
5. A wash sale of domestic shares 1.A taxpayer purchased a building to be d.P150,000
wherein 20,000 shares were disposed used as a future plant site. The building 6. What would be the tax basis of Mr.
at a loss of P40,000 were subsequently remained unused for 3 years due to a Antonio’s new residence?
covered up within the 30-day period by significant decline in customer’s a. P1,800,000
purchase of 15, 000 shares for demand in product of the taxpayer. The b.P1,000,000
P12/share taxpayer eventually disposed the c.P900,000
d.P800,000-
7. How much of the capital gains tax will Multiple choice-Problems:Part 4 Fair value of other properties received
be released to the taxpayer? 1.Mr. Quirino exchanged his stock in 250,000
a.P150,000 Carmen Corporation for the shares of Compute the capital gains tax.
b.P135,000- stock of Dingalan Corporation. The a.P0
c.P120,000 stocks acquired by Mr. Quirino b.P5,000
d.P15,000 represent 60% of the stocks of Dingalan c.P10,000-
8. On August 15, 2011, Ms. Mones sold Corporation. d.P20,000
a 500-square meter residential house Basis of the stocks given P3,000,000 7. What is the tax basis of the B
and lot for P3,000,000. The house was Fair market value of the stocks given company received by Raymund?
acquired in 2005 at P2,000,000. The 5,000,000 a.P0
assessed fair market values of the Fair market value of stocks received 4, b.P1,100,000-
house and lot, respectively, were 500,000 c.P1,200,000
P1,500,000 and P1,000,000. The zonal What is the capital gains tax? d.P1,350,000
value of the lot was P5,000 per square a.P0- 8. What is the basis of the “boot” or
meter. b.P45,000 other properties received by Raymund?
What is the capital gains tax? c.P145,000 a.P0
a. P180,000 d.P195,000 b.P150,000
b. P120,000 2.In the immediately preceding c.P250,000-
c. P150,000 problem, what is the basis of the stocks d.P400,000
d. P240,000- received by Mr. Quirino? 9. What is the basis of the A company
9.Manny, a resident Filipino citizen sold a.P0 shares received by B company?
his principal residence (house and lot) b.P3,000,000- a.P0
at its original purchase price of c.P4,500,000 b.P1,100,000
P11,000,000. The property had a d.P5,000,000 c.P1,200,000
P13,000,000 fair value at that time. 3. Mr. Eller exchanged his DEF shares d.P1,350,000
If the proceeds of the sale were not for shares of EFG pursuant to a plan of 10. Mrs. Joson sold a residential lot on
invested in the new principal residence merger. Mr. Eller bought his shares for July 1, 2014 for P2,000,000. The
but, instead, new funds of P15,000,000 P1,000,000. The shares had a fair value property had a zonal value of
were used to construct it, the capital of P1,500,000 on the date of exchange, P2,500,000 and an assessed value of
gains tax is Mr. Eller received EFG shares with fair P1,000,000.
a.P0 value of P1,300,000 plus cash of On July 1, 2014, Mrs. Joson was
b.P660,000 P200,000. compelled to pay the capital gains upon
c.P750,000 Compute the capital gains tax the request of the buyer. The
d.P780,000- a.P0 compromise penalty was determined to
Numbers 8 through 10 are based on the b.P10,000 be P200,000.
following information: c.P15,000- Compute the total tax due.
Mr. Pepito sold his residential land in d.P45,000 a. P150,000
Manila with fair market value of 4.What is the basis of the shares b. P180,000
P12,000,000 for P10,000,000. received by Mr. Eller? c. P217, 500
10. If Mr. Pepito utilized all of the a.P0 d. P237,500-
P10,000,000 in buying a house and lot b.P1,000,000- 11. Basic Company paid P9,000
to be used as his new principal c.P1,200,000 documentary stamp tax on the sale of a
residence, the final tax due from him is d.P1,300,000 real property capital asset. Compute
a. P720,000- 5. What is the basis of the DEF shares the capital gains tax on the sale.
b.P600,000 received by EFG Company? a.P9,000
c.P120,000 a.P0 b.P16,000
d.P0 b.P1,000,000 c.P36,000-
11. If Mr. Pepito utilized only c.P1,200,000- d.P42,000
P7,000,000 from the proceeds of the d.P1,300,000 12. Mr. Bassit Unay sold a residential
sale in acquiring a new residence, the 6. Raymund exchanged his A company land for P4,000,000. The land had a fair
final tax due from him is shares pursuant to a plan of value of P3,500,000 and an assessed
a.P720,000- consolidation where A company will be value of P2,000,000. What is the total
b.P216,000 integrated with B company. The income and documentary stamp tax
c.P180,000 following relates to the exchange: due?
d.P0 Basis of A company shares given a. P0
12.The documentary stamp tax due on P1,200,000 b.P300,000-
the sale is Fair value of A company shares given c.P400,000
a.P179, 895 1,300,000 d.P450,000
b.P180,000- Fair value of B company shares received CHAPTER 7
c.P149,985 1,100,000 INTRODUCTION TO REGULAR INCOME
d.P150,000 TAX
This chapter provides an overview of The regular tax is an imposition on Excluded income vs. exempt income
the regular income tax intended to residual profits or gains after Excluded income is also exempt income.
acquaint readers with the nature and deductions for expenses and Both are not included in gross income,
tax structures of the regular income tax. personal exemptions allowable by but differ only as to source. Excluded
It also discusses regular tax reporting law. income traces their origin from the
and income tax determination. Annual tax NIRC, but exempt income trace their
Subsequent chapters deals with specific The regular income tax applies on origin from either the NIRC or special
aspects of the regular income tax. yearly profits or gains. The gross laws.
After this chapter, readers are expected income and expenses of the ALLOWABLE DEDUCTIONS
to demonstrate knowledge on the taxpayer are measured using the Allowable deductions, or simply
following: accounting methods adopted by the “deductions” are expenses in the
1. The scope of regular income and taxpayer and are reported to the conduct of business or exercise of
its tax model government over the accounting profession. Deductions can be claimed
2. The features of the regular period selected by the taxpayer. itemized wherein the taxpayer supports
income tax Creditable withholding taxes every item of deduction or standardized
3. The concept of inclusion and Most items of regular income are through the Optional Standard
exclusions from gross income subject to creditable withholding Deductions wherein the deduction is
4. The types of gross income tax(CWT). The CWT is withheld at simply presumed as a percentage of
subject to regular tax source by customers or clients but is gross sales, gross receipts, or gross
5. The concept of deduction and not a final tax. It is an advance tax income.
personal exemption deductible against the annual
6. The concept of deductions income tax due of the taxpayer. The book sub-divided the vast topic of
compared to personal Final Withholding tax (FWT) vs. deductions as follows:
exemptions Creditable Withholding Tax (CWT) 1.Principles of deductions-Chapter 13
7. Measurement of gross income Similarities of FWT and CWT 2.Regular allowable Itemized
from employment and business 1.In both cases, the income payor deductions-Chapter 13-A
and the treatment of other withholds a fraction of the income 2.Special allowable Itemized Deductions
income and remits the same to the and net operating loss carry over
8. The concept of operating government. Chapter 13-B
income or revenue and the 2.By collecting at the moment cash 4.The standard optional
difference in tax presentation of is available, both serve to minimize deductions(OSD)-Chapter 13-C
individuals and corporate cash flow problems to the taxpayer PERSONAL EXEMPTIONS
taxpayers. and collection problems to the Ideally, income taxation should not
9. The procedural computation of government, apply to the basic subsistence and
taxable income of corporations Differences between FWT and CWT support of individual taxpayers because
and different individual imposing tax on these would be
taxpayers Final Withholding Tax tantamount to killing the goose that
10. The computation of the regular Creditable Withholding tax lays the golden eggs. The amount
tax for individuals and Income tax withheld Full Only a exempted by law in lieu of the personal,
corporations portion living, and family expenses of an
11. The deadline of the regular tax Coverage Certain passive income individual taxpayer is referred to as
returns Certain passive and active income personal exemption. Personal
12. Applicability of the quarterly Who remits the actual tax Income exemption will be discussed in detail in
filing and its deadlines payor Income payor for the CWT Chapter 14.
THE REGULAR INCOME TAX MODEL and the taxpayer for the balance Allowable Deduction vs. Personal
Gross Income Pxxx,xxx Necessity of an annual consolidated exemption
Less: Allowable deductions xxx,xxx return Not required 1. Allowable deductions pertain to
Personal exemptions xxx,xxx GROSS INCOME costs or expenses of earning
Taxable income Pxxx,xxx For purposes of the regular income tax, items of gross income while
CHARACTERISTICS OF THE REGULAR gross income constitutes all items of personal exemptions are
INCOME TAX income that are neither excluded in deductions in lieu of the
1. General in coverage gross income nor subjected to final tax personal, family and cost of
2. A net income tax or capital gains tax. The items of gross living of individual taxpayers.
3. An annual tax income subject to regular income tax 2. Personal exemption is applicable
4. Creditable withholding tax will be extensively discussed in Chapter only to individual taxpayers.
General coverage 9. Corporations cannot claim
The regular income tax applies to all Exclusion from Gross Income personal exemptions. However,
items of income except those that These pertain to items of income that both corporations and
are subject to final tax, capital gains are excluded; hence, exempt from individuals can claim
tax, and special tax regimes. income taxation. These will be deductions.
Net Income taxation discussed in detail in Chapter 8.
3. Allowable deductions are Regular pay Compensation income Freight-in xxx
generally deductible by Compensation income Total goods available for sale Pxxx
corporations. For individual Fringe Benefits Compensation income Less: Ending inventory xxx
taxpayers, only self-employed Fringe Benefit Cost of goods sold Pxxx
individuals or those engaged in Compensation income is subject to Under the perpetual system, the cost of
business or exercise of a regular income tax. The fringe benefits goods sold is determined through bar
profession can claim deductions. of rank and file employees are included codes of the goods sold by stock cards
No deduction is allowed to as part of compensation income and indicating the costs of the goods sold.
individuals who are purely are subject to regular income tax while Under the periodic system, the cost of
employed while exemption is the fringe benefits of managerial or goods sold is established by counting
allowed to individuals whether supervisory employees are excluded in the inventories. The cost of missing
purely employed or self- compensation income are subject to a items at every reporting date is
employed. special final tax, called fringe benefit considered sold. For purposes of
4. Allowable deductions are tax. This special taxation will be costing, the freight costs of the goods
deductible only from gross discussed in Chapter 11. purchased are allocated to all units
income from business or TAX REPORTING OF COMPENSATION purchased.
exercise of a profession. INCOME Cost of sales of a manufacturing
Personal exemption is Compensation income is presented in business
deductible against all types of the income tax return as follows: The cost of goods sold of a
gross income subject to regular Gross compensation income Pxxx manufacturing business is computed in
income tax. Less: Non-taxable compensation almost the same way with those of a
5. Allowable deductions are xxx(SSS, PHIC, Pag-ibig, Union Dues) trading business. The detailed costing
considered in the determination Gross taxable compensation income Pxx procedures for inventories of a
of net income from business or Less:personal exemption xxx manufacturing business will be
profession whereas personal Net taxable income discussed in Chapter 13.
exemption is considered in the Pxxx Illustration
computation of taxable income. Non-taxable compensation includes A taxpayer had the following data
TYPES OF GROSS INCOME SUBJECT items of compensation income that are during the year:
TO REGULAR INCOME TAX exempted by law, contracts, or treaty Gross sales P4,000,000
1. Compensation income from income taxation. The detailed tax Sales discounts 100,000
2. Business or professional income rules on compensation income will be Sales return 200,000
3. Other income-non-business discussed in Chapter 10. Beginning inventory 600,000
income and non-compensation BUSINESS INCOME Purchases 2, 500,000
such as: Business Income arises from habitual Purchases returns and allowances
a. gains from dealings in engagement in any commercial activity 150,000
properties involving regular sales of goods or Freight-in 200,000
b. other active or passive services by an individual or a Ending inventory 800,000
income not subject to final corporation. The income from business, The cost of sales shall be computed as
tax legal, or illegal, registered or follows:
THE REGULAR TAX MODEL IN unregistered, is taxable. Beginning inventory P600,000
EXPANDED FORM The business gross income from the Net purchases(P2.5 M-P150K) 2,
Employment Business or sale of goods is computed as: 350,000
profession Others Total Sales Pxxx Freight-in
Gross income xxx xxx xxx Less: Cost of goods sold(cost of sales) 200,000
Less: xxx Total goods available for sale P3,
Deductions (xxx) Gross income Pxxx 150,000
Personal exemptions(xxx) Cost of sales Less: Ending inventory
Taxable income xxx+xxx+xxx=xxx Cost of sales pertains to the acquisitions 800,000
COMPENSATION INCOME cost of the goods sold or the Cost of sales
In taxation, the term “compensation manufacturing cost of the goods sold. P2,350,000
income” generally comprises all Cost of sale of a trading business The business gross income shall be
remunerations under an employer- The cost of goods sold may be computed as follows:
employee relationship, such as the determined by the specific Sales(P4M-P100K-P200K) P3,700,000
regular pay of employees every payroll identification using perpetual inventory Less:cost of sales 2, 350,000
period and other benefits or incentives system with the aid of point-of- Gross income P1,350,000
other than the basic pay which sale(POS) machines or by periodic PROFESSIONAL INCOME
commonly known as fringe benefits. inventory system using the following The gross income from exercise of a
Classification of employee formula: profession or business gross income
remunerations Beginning inventory Pxxx from the sales of services is measured
Rank and file employees Purchases, net of returns and as follows:
Managerial or supervisory employees allowances xxx Revenues or gross receipts Pxxx
Less: Cost of services xxx Less: Cost of services 2,112,000 3. A private hospital has patient service
Gross income Pxxx Gross income P2,388,000 fees as its primary revenue, but may
Service providers using the accrual basis TAX REPORTING OF BUSINESS OR have room rental and sale of medicines
shall report their revenues while those PROFESSIONAL INCOME at its other operating revenues.
using the cash basis shall report their Tax Reporting by Individual Taxpayers 4. A dormitory has boarding fees as its
gross receipts or collections. Net Sales/Revenues/Receipts/Fees Pxxx primary revenue, but may have laundry
Cost of services Add:Other taxable income from fees and canteen income as other
Cost of services pertains to all direct operation not subject to final tax operating revenues.
costs of rendering the services such as xxx 5. A retail store has its sales of
cost of labor, materials, and overhead Total sales/revenues/receipts/fees merchandise as its primary revenue,
costs. The cost of services should be Pxxx,xxx but may earn consignment commission
distinguished from the costs of Less:Cost of sales or services xxx,xxx income as other operating revenues.
administration and marketing of the Gross Income from 6. A bus transport company has the
business. These two are separately business/professionPxxx,xxx receipts from passengers and baggage
presented under the deduction Add:Non-operating income xxx,xxx as primary revenue, but may earn
category “Regular allowable itemized Total Gross income Pxxx,xxx income from bus stop restaurants and
deductions.” Less: Allowable deductions xxx,xxx washrooms as other operating
Illustration Net Income Pxxx,xxx revenues.
A practicing auditor had the following Sales, revenues, receipts, and fees and Non-operating income
income and expenses during the year: fees distinguished Non-operating income includes all other
Billing for services rendered and out-of- Revenue is a general term which items of gross income such as:
pocket costs P4,500,000 pertains to the gross inflow of benefits 1.Gains from dealings in properties
Salaries of audit staff 1, 400,000 (total return) arising from the primary Gains are net of cost of the property
Salaries of administrative employees operations of the business. Sales sold. They are gross income items
200,000 pertains to revenue from the sale of rather than revenue. Hence, they are
Transportation expenses to and from goods while “fees” pertains to revenue excluded under
clients 12,000 from the sale of service. Receipts “sale/Revenues/Receipts/Fees” of
Supplies used in various engagements pertains to cash collections from the individual taxpayers and are included
250,000 sale of goods or services. under “Non-operating income.”
Supplies and general utilities 120,000 The terms sales or fees simply revenue Dealings in properties pertain to the
Depreciation of office equipment are commonly used to denote the sale, exchange, and other disposition of
80,000 income of taxpayers using the accrual properties by the taxpayer. The rules on
Depreciation of laptops issued to audit basis while the term receipts is used to gains in dealing in properties not
staff 50,000 denote the income of taxpayers using covered by the capital gains tax will be
Insurance expense on office properties the cash basis. discussed in Chapter 12 under regular
20,000 Revenue vs. Gross income income taxation.
Rent expenses allocable to workspaces Revenue is a gross concept pertaining 2.Income distribution from a general
400,000 to the total return in a transaction professional partnership, taxable trusts
Rent expenses allocable to which includes the return on capital or estate, or from an exempt joint
administrative offices 50,000 and the return on capital. Gross income venture
Bad debts expense on non-paying is a net concept pertaining to the return Income distributions from these entities
clients 100,000 on capital in a transaction. Gross are not revenue, but items of gross
The cost of services shall include only income is net of the cost of sales or cost income, hence, included as part of the
those directly incurred or related gross of services. non-operating income of individuals
revenue from the rendition of services Other taxable income from operations 3.Casual active income
such as: Other taxable income from operations This includes active income from
Salaries of audit staff includes revenues or receipts from isolated or one time transaction such as
P1,400,000 incidental or secondary operations casual capentry income of a person not
Transportation expenses to and from aside from the primary operations. engaged in carpentry business. Any
clients 12,000 Examples: expense on casual transactions is set off
Rental expenses on staff workspaces 1.A school has tuition fees as primary with the casual income. The net gain or
400,000 revenue, but its income from its income is a non-operating income.
Supplies used in various engagements bookstore. canteen or student 4.Passive income not subject to final
250,000 dormitories constitutes other operating tax
Depreciation of laptops revenues. This includes passive income not
50,000 2. A manufacturing firm has its gross connected with the business of the
Total cost of services P2, income from sale of finished goods as taxpayer and is not subjected to final
112,000 its primary revenue, but its income tax such as interest on advances to
The gross income shall be computed as from scrap sales constitutes other employees and dividends from foreign
follows: operating revenues. corporations. Similar to casual
Revenue P4,500,000 income,these do not arise from the
regular business operations, hence, Illustration if the method employed does not
classified as non-operating income. Assuming the same data in the previous clearly reflect the income, the
Illustration illustration except that the taxpayer is a computation shall be made in
An individual taxpayer who is using the corporation, the business income shall accordance with such method that in
accrual basis in his manufacturing be reported as follows: the opinion of the Commissioner,
business reported the following results Net clearly reflects the income.
of operations in the preceding year. sales/Revenues/Receipts/Fees(P4M+P0. TAXABLE INCOME OF CORPORATE
Sales, net of returns, and discounts 2M) P4,200,000 TAXPAYERS
P4,000,000 Less:Cost of sales The taxable income of corporate
Cost of sales 1,800,000 1,800,000 taxpayers is simply the net income from
Dividends income, net of final tax Gross Income from operations business because corporations cannot
36,000 P2,400,000 claim personal exemptions.
Business expenses Add: Other taxable income not subject TAXABLE INCOME OF INDIVIDUAL
1,600,000 to final tax TAXPAYERS
Gain on sale of old equipment Gain on sale of equipment The computation of the taxable income
100,000 P100,000 of an individual taxpayer depends on
Sale of scrap metals Interest income on employee whether he or she is a:
200,000 advances 45,000 1. Pure compensation income
Interest income on employee advances 145,000 earner
45,000 Total gross income P2,545,000 2. Pure business income earner or
Gain on sale of domestic stocks directly Less:Allowable deductions(Business pure professional income earner
to buyer 10,000 expenses) 1, 600,000 3. Mixed income earner
The business income of the individual Net Income P945,000 Pure compensation income earner
will be presented in the income tax The difference in presentation between The taxable income of a pure-
return as follows individuals and corporations is compensation income earner is
Net Sales/Revenues/Receipts/Fees necessitated by the Optional Standard computed as follows:
P4,000,000 Deduction(OSD). The basis of the OSD Gross taxable compensation income
Add:Other taxable Income from for individual taxpayers is the total Pxxx
operations-Scrap sales revenues or receipts from operations Less:Personal exemptions xxx,xxx
200,000 while the basis of the OSD for Taxable compensation income Pxxx,xxx
Total sales/ Revenues/ Receipts/ Fees corporations is on total gross income Individuals deriving pure compensation
P4,200,000 subject to regular income tax whether income from one employer and without
Less:Cost of sales or services or not they arise from the regular other income outside their employment
1,800,000 business operations. may avail of the substituted filing
Gross Income from Business/Profession Separate bookkeeping for business and system where the employer withholds
P2,400,000 professional practice the total income tax on compensation
Add:Non-operating income Individual taxpayers engaged in of the employee income. The employee
Gain on sale of equipment business or exercise of a profession need not file an income tax return
P100,000 must maintain a separate record of provided that the employer withheld
Interest income on employee their transactions from business or the correct amount of tax.
advances 45,000 professional transactions. The personal Treatment of other income of a pure
Total Gross Income P2, transactions of the individual taxpayer compensation earner
545,000 must not be mixed with the The other income is simply added to
Less:Allowable dedcutions(Business transactions of the business or the taxable compensation income as
Expenses) 1,600,000 professional practice. follows:
Net Income This is important in the tax treatment of Gross taxable compensation income
P945,000 expenses. The personal expense of the Pxxx
Note:For corporate taxpayers, revenues taxpayer cannot be deducted against Taxable compensation income Pxxx
or receipts from secondary or incidental the gross income of the business. The Add:other income xxx
operations will be included under the allowable personal exemption fixed by Taxable income Pxxx
classification ,”Sales/Revenues/Receipts law for individual taxpayers is in lieu of Note: Employed individuals with other
/Fees.” all the actual personal, family, and cost income outside employment are
Other taxable income not subject to of living expenses of the taxpayer. required to file the annual income tax
final tax DETERMINATION OF TAXABLE INCOME return for pure compensation earner.
This category includes other items of The taxable income shall be computed (BIR Form 1700).
gross income whether or not arising upon the basis of the taxpayer’s annual Pure Business or professional income
from the operations of the corporation accounting period in accordance with earner
such as gains in dealings in properties, the method of accounting regularly The taxable income of a pure business
income distribution from an exempt employed in keeping the books of such or professional earner is the net income
joint venture, and other passive income taxpayer; however, if no such method from business or profession less the
not subject to final tax. of accounting has been so employed, or personal exemptions:
Net income from business or profession Case 1: A compensation earner with Over But Basi Pl Of
Pxxx other gross income not c tax us excess
Less: personal exemption xxx Compensation income P400,000 over over
Taxable business income Pxxx Less:Personal exemption 125,000 P0.00 P10,00 P0 5 P 0.00
Treatment of other income of a pure Taxable compensation income P275,000 0.00 %
business or professional income earner Add:other gross income 20,000 10,000 30,000 500 10 10,000
The other income of individuals is Taxable income P295,000 .00 .00 .00
integrated in the business or Case 2: A business income earner with %
professional net income through the other gross income 30,000 70,000 2,50 15 30,000
category “non-operating income.” Gross business income P400,000 .00 .00 0 .00
%
Mixed Income Earner Add:other gross income 20,000 70,000 140,00 8,50 20 70,000
The taxable income of individual Total gross income P420,000 .00 0.00 0 % .00
taxpayers deriving income from both Less:Allowable deductions 280,000 140,00 250,00 22,5 25 140,00
employment and self-employment in Net income P140,000 0.00 0.00 00 % 0.00
business or exercise of a profession Less: Personal exemption 125,000 250,00 500,00 50,0 30 250,00
shall be determined as follows: Taxable income P15,000 0.00 0.00 00 % 0.00
Taxable compensation income Pxxx,xxx Case 3: A mixed income earner- 500,00 - 150, 32 500,00
Net income from business or profession negative taxable compensation income 0.00 000 % 0.00
xxx, xxx Gross taxable compensation income
Total taxable income Pxxx,xxx P100,000 Note:Examinees are not required to
Note: self-employed individuals, estates Less: personal exemption 125,000 memorize this tax table for Board exam
and trusts, and mixed income earners Taxable compensation income purposes
shall file BIR From 1701. (P25,000) Scope of progressive tax
A negative taxable compensation Gross business income P400,000 The progressive tax covers all
income is deductible from net business Other gross income 20,000 individuals including taxable estates and
or professional income in consonance Total gross income P420,000 trusts except those subject to final
with the rule that personal exemption is Less: deductions 280,000 income tax:
deductible against all types of income 140,000 a.NRA-NETB-subject to 25% final tax on
subject to regular tax. Taxable income gross income
However, a net operating loss on P115,000 b. Special aliens or special employees-
business or professional practice is not Note: A negative taxable compensation subject to 15% final tax on gross income
deductible against compensation income is deductible against business or from employment
income. A net operating loss is allowed professional net income. Illustration 1:Income tax computation
to be carried over as an item of Case 4:Mixed income earner-with A resident citizen with a personal
deduction (Net Operating Loss Carry- business/professional net loss exemption of P50,000 has a
over or NOLCO) against any net income Gross taxable compensation income compensation income of P150,000
in the succeeding three years. This will P400,000 within the Philippines and P100,000
be discussed under chapter 13-B. Less: personal exemption 125,000 from abroad.
Illustration:Individual income taxpayer Taxable compensation income The taxable income of the resident
Case Case Case Case P275,000 citizen shall be computed as follows:
1 2 3 4 Gross business income P200,000 Gross compensation income
Compe P400 P100 P400 Add:other gross income 20,000 (P150K+P100K) P250,000
nsation ,000 ,000 ,000 Total gross income P220,000 Less: Personal exemption 50,000
income Less:deductions 280,000 Taxable compensation income P200,000
Gross P400 400, 200, Net Loss(NOLCO) (P60,000) The income tax due shall be computed
busines ,000 000 000 Taxable income as follows:
s P275,000 Tax due
income Note: A net operating loss from Taxable compensation income P200,000
Deducti 280, 280, 280, business or profession is not deductible Less: Lower limit of the income bracket
ons 000 000 000 against taxable compensation income. where the taxable income
Other 20,0 20,0 20,0 20,0 Types of Regular Income tax qualifies 140,000
gross 000 00 00 00 1.Individual income tax P22,500
income 2. Corporate income tax Excess P60,000
Persona 125, 125, 125, 125, INIDIVIDUAL INCOME TAX Multiply by: 25% 15,000
l 000 000 000 000 The individual income tax or Total income tax due P37,500
exemp progressive tax is determined in Over But Basic Pl Of
tion reference to a tax table of progressive not tax us excess
Taxable income shall be determined in tax rates. over % over
each of the above case as follows: The Income tax table for Individual P70,0 P140,0 P8,50 2 P70,0
Taxpayers 00.00 00.00 0 0 00.00
% income. The RCIT applies to any monitoring compliance with the
140,0 250,00 22,5 2 140,0 corporation other then those: creditable withholding tax by exempt
00.00 0.00 00.0 5 00.00 a. Subject to final tax such as non- corporations.
0 % resident foreign corporation and Exempt corporations with gross income
250,0 500,00 50,00 3 250,0 FCDU interest income not subject to the regular corporate income
00.00 0.00 0.00 0 00.00 subjected to final tax tax shall file BIR From 1702-RT for such
% b. Special corporations or those income.
subject to special tax regimes RULES IN ROUNDING OF CENTAVOS IN
Note:Recall that a resident citizen is such as PEZA and TIEZA- THE INCOME TAX RETURN
taxable on global income. registered enterprises. The requirement for entering centavos
Illustration 2:Income tax Computation c. Exempt corporations on their in the latest version of the income tax
A resident alien with a P100,000 exempt income. return (June 2013 version) has been
personal exemption has a business net Illustration eliminated. If the amount of centavos is
income of P750,000 in the Philippines A corporation has a net income of 49 or less, the centavos are dropped
and P250,000 from abroad. P1,200,000 in the Philippines and down. If the amount is 50 centavos or
The taxable income of a resident alien P800,000 from abroad. more, it is rounded up to the next peso.
shall be computed as follows: Assuming the corporation is a domestic Hence, an amount for P100.49 shall be
Business net income corporation, the income tax due shall entered in the income tax return as
P750,000 be computed as follows: P100. An amount of P100.50 shall be
Less:Personal exemption 100,000 Taxable income (world) rounded to P101.00.
Taxable net income P2,000,000 DEADLINE OF FILING THE INCOME TAX
P650,000 Multiply by:Tax rate RETURN
30% The annual income tax return is due for
The income tax due shall be computed Income tax due P600,000 filing on the 15th day of the fourth
as follows: month following the taxable year of the
Tax due Assuming the corporation is a resident taxpayer. The income tax due shall be
Taxable income P650,000 foreign corporation, the income tax due paid upon filing.
Less: Lower limit of the income bracket shall be computed as follows: REQUIRED ATTACHMENT IN THE
where the taxable income Taxable Income(Philippines)P1,200,000 ANNUAL INCOME TAX RETURN
qualifies500,000 Multiply by:tax rate 30% 1. Certificate of Independent CPA-if
P125,000 Income tax due P360,000 gross quarterly sales, earnings,
Excess P150,000 Note: receipts, or output exceed
Multiply by: bracket marginal rate 32% 1. Recall that a domestic P150,000)
48,000 corporation is taxable on global 2. Supplemental form for taxpayers
Total income tax due income while resident foreign with multiple activities per tax
P173,000 corporation is taxable on regime
Philippine income. 3. Account information form and
Over But Basi Pl Of 2. Under RR2-2014, corporations financial statements(FS)
not c tax us excess file their annual income tax showing:
over % over return using: a.Sales/receipts/fees
140,00 250,00 22,5 25 140,00 a.BIR Form 1702-RT for b.Cost of sales/services
0.00 0.00 00 % 0.00 corporations subject only to c.Non-operating and other
250,00 500,00 50,0 30 250,00 regular income tax taxable income
0.00 0.00 00 % 0.00 b. BIR Form 1702-EX for exempt d. Itemized deductions (if
500,00 - 125, 32 500,00 corporations under the NIRC taxpayer did not avail of OSD)
0.00 000 % 0.00 and special laws with no other e. Taxes and licenses
taxable income f. Other information prescribed
c.BIR Form 1702-MX for to be disclosed in the FS
Note:Recall that a resident alien is corporations with income 4. Statement of management
taxable only on Philippine income. subject to multiple income tax responsibility (SMR)
Special Alien Employees rates or with income subject or 5. Certificate of income payments not
These pertain to employees who are preferential rate. subjected to Withholding Tax (BIR Form
subject to a 15% final tax on gross Exempt Corporations 2304)
compensation income. These will be It should be noted that exempt 6. Certificate of creditable withheld at
discussed in detail in Chapter 10. corporations are required to report source (BIR Form 2307)
CORPORATE INCOME TAX their results of operations through BIR 7. Duly approved Tax debit memo, if
The corporate income tax, commonly From 1702-EX even if they do not have applicable
referred to as the regular corporate taxable income. They are mandated to 8. Proof of prior year’s excess credits, if
income tax (RCIT), is a proportional or use the Itemized deductions in their applicable
flat tax at a rate of 30% on taxable income tax return. The rule is 9. Proof of foreign tax credits, if
apparently intended to assist the BIR in applicable
10. For amended return, proof of tax 8. What composes the compensation income and is subject to
payment and the return previously filed compensation income of a rank regular income tax-True.
11. Certificate of tax treaty and file employee and a True or false 2
relief/Entitlement issued by the managerial and supervisory 1.The gross income from compensation
concerned investment promotion employee? is measured as gross compensation less
agency (IPA) 9. How does the tax presentation the non-taxable compensation
QUARTERLY FILING OF INCOME TAX of the composition of gross income.True
RETURN income in the tax return differ 2.The deadline of filing the corporate
Corporations and individuals engaged in between individuals and quarterly return is the same with the
business and those engaged in the corporate taxpayers? deadline of the quarterly income tax
practice of a profession are required to 10. Distinguish “other taxable return of individuals.False
file three quarterly returns aside from income from operations” from 3.Business expenses can be deducted
the annual consolidated income tax non-operating income. against all types of gross income subject
return. 11. Distinguish revenue from sales, to regular tax-False
Individual tax payers engaged in fees, and receipt 4, Deductions are considered in the
business or practice of profession shall 12. Discuss how the taxable income determination of net income wile
file their income tax returns using BIR of the following is determined: personal exemption is considered in the
Form 1701 Q. Corporations shall file a.Corporate taxpayer determination of taxable income-True
their quarterly income tax returns using b.Purely compensation earner 5.Personal exemption relates to
BIR Form 1702Q. c.Purely business or professional individual taxpayers while deductions
Taxpayers make estimated quarterly tax income earner relate to corporations.False
payments. These quarterly tax d.Mixed income earner 6.The gross income from business is
payments are claimed as tax credit 13. What is the treatment of negative measured as sales or gross receipts less
(deductions) to the annual consolidated taxable compensation income and cost of sales or cost of services. True
income tax due of the taxpayer. negative business income? 7. The tax due of individuals is
Deadline of Quarterly Income tax 14. What are the deadlines of the determines by means of a tax table.
returns quarterly regular income tax for True
Income Taxpayers Corporations individuals and corporations? 8. The tax due of corporations is
tax Individual True or false determined by multiplying their taxable
returns s 1.There are two types of regular income income by 30%. True
st
1 April 15, 60 days end tax: proportional income tax for 9. The deadline of the annual income
quarter same year of 1st Qtr corporations and progressive income tax return using the calendar year is
ITR tax for individuals.True similar to the deadline for individual
2nd August 15, 60 days end 2.NRA-NETBs and NRFCs are also taxpayers. True
Quarter same year of 2nd Qtr subject to regular income tax.-False 10. Personal exemptions are deductible
ITR 3.All taxpayers are subject to final tax against all types of gross income subject
3rd November 60 days end False to regular tax. False
Quarter 15, same of 3rd QTR 4. Taxable income is synonymous to net Multiple choice-Theory:Part 1
ITR year income.False 1. The general rule in income
CHAPTER 7:SELF-TEST EXERCISES 5.For all taxpayers, taxable income taxation is
Discussion Questions means the pertinent items of gross a. Final income taxation
1. Discuss the scope of the regular income not subject to capital gains tax b. Capital gains taxation
income tax especially on passive and final tax less allowable deductions c. Regular income taxation-
income and capital gains. personal exemption.True d. Fringe benefit taxation
2. Enumerate the characteristics of 6.All taxpayers are subject to regular 2. Active income is subject to
the regular income tax income tax.-False a.regular tax-
3. What are exclusion in gross 7. Employed taxpayers can claim b.Capital gains tax
income? expenses from their employment as c.final tax
4. Distinguish allowable deductions deductions against their compensation d.any of these
from personal exemption income. False 3.Question 3 and 4 are based on the
5. Demonstrate the computation 8. Items of gross income subject to final following:
of the gross income from tax and capital gains tax are excluded in A.Regular tax B. Final tax C. Capital
employment and the gross gross income subject to regular income gains tax
income from business or tax. True Which of the foregoing are passive
exercise of a profession 9.The personal exemption allowable to incomes subject to?
6. How is cost of goods sold individual taxpayers is in lieu of the a.A only
determined? personal and business expenses.False b.B only
7. What are included in cost of 10. The basic pay of both rank and file c.Both A and B
services? employees and managerial or d.Either A or B-
supervisory employees is considered 4. Which of the foregoing are capital
gains subject to?
a. A only b. Compensation earners only d.Divididends from domestic
b. Conly c.Individuals engaged in business corporations
c. Either A or C- d. both individuals and corporations Multiple Choice-Theory:Part 2
d.Both A and C 15.Which of the following individual 1. The following may be relevant in
5. The net amount of regular income taxpayers is not subject to tax on the determination of taxable
subject to regular tax is called taxable income? income”
a.Taxable income- a.Non-resident citizen A.Gross income subject
b.compensation income b.Resident alien toregular tax
c. net income c.Non-resident alien not engaged in B.Gross income subject to final
d.gross income trade or business- tax
6.Which is not generally subject to d.Non-resident alien engaged in C.Deductions from gross income
regular income tax? business D.Personal exemption
a. Compensation income 16.Which of the following employers is Which is not considered in the
b. Business income least likely to have special alien determination of corporate
c.Professional income employees? taxable income?
d.Passive income- a. Regional operating headquarters of a.A and D
7.What are allowable deductions multinational companies b. C and D
against gross income? b. offshore banking units c. D only
a.Business expenses- c. Petroleum service contractors under d. B and D only-
b.Family support service contracts with the government 2. In the immediately preceding
c.Personal expenses by the taxpayer d.Foreign currency deposit unit- problem, which is not considered in the
d.Expenses of employment 17. Which of the following corporate determination of individual taxable
8.Deductions are allowed to taxpayers is not subject to tax on income?
a. employed taxpayers. taxable income? a.A and D
b.inidividual taxpayers only a.Domestic corporation b. B and D
c.corporate taxpayers only b.Business partnership c. D only
d.taxpayers engaged in business- c.Non-resident foreign corporation- d.B only-
9. Personal exemptions are allowed to d.Resident foreign corporation 3. Which is correct statement regarding
a.employed taxpayers 18.Which is a source of income subject exclusion in gross income?
b.inidividual taxpayers only- to regular income tax? a. They are included in gross income
c.corporate taxpayers only a.Employment subject to regular income tax.
d.taxpayers engaged in business b.Trade or business or exercise of b.They are ignored in the determination
10.Which is not a feature of the regular profession of gross income.
income tax? c.Casual sales transactions c. They are presented in gross income
a.Net income tax d.All of these- but are presented as deductions.-
b.Final withholding tax- 19. Which interest income will not be d.They are subject to final tax.
c.Annual tax included in the income tax return? 4.Which of these of employees may be
d.Creditable withholding tax a.Interest income from bank deposits- subject to final fringe benefit tax?
11.Which is true with the final b.Interest income from lending a.Managerial employees
withholding tax? c.Interest income from notes b.Supervisory employees
a.The taxpayer still needs to file an d.Interest income from employees c.Rank and file employees
annual consolidated return 20. What type of employee benefit may d.A and B-
b. It applies to all items of gross income be subjected to final tax? 5.Which is not considered an operating
c. It constitutes a partial payment of a.Fringe benefit- income?
income tax b.Compensation income a.Consignment commission income by a
d. It applies to certain passive income- c.De minimus benefits retail store-
12.Which is not true with the creditable d.B and C b.Fees from the rendering of services
withholding tax? 21.Which is not subject to final tax? c.Interest income from advances to
a.Advances to the annual tax due a.Prizes amounting to P11,000 employees
b.No need to pay further taxes- b.Interest income from bank deposits d.Sale of scrap
c.Need to file annual income tax return c.Winnings from the Philippines 6.Which is a non-operating income?
d.Applicable to items of regular income d.Share on the net income of general a.Gain on sale of office building-
13.Progressive income tax is applicable professional partnership- b.Sale of goods by a retail store
to 22.Which of the following is a passive c.Gate receipts of cockpits
a.corporate taxpayers income but is nevertheless subject to d.Gate receipts of cinemas
b.individual taxpayers- regular tax by virtue of exclusion under 7.Which is an incorrect statement?
c.compensation earners only final income taxation? a.Business expenses are deductible by
d.individuals in business only a.Prizes amounting to P10,000- individuals and corporations
14.Proportional regular income tax is b.Service income b.Personal exemptions are deductible
applicable to c.Merchandising income by individuals and corporations-
a.Corporation only-
c.Personal exemptions are deductible 16. Avida Corporation is filing its income non-operating income and is included in
by individuals, estates and trusts. tax return for the quarter ending net income.
d.Deductions are considered in the February 28, 2014. The return must be c, The other income of a mixed income
determination of net income. field on or before earner is also treated as part of non-
8.Which of the following will least likely a.April 15, 2015 operating income and is included in net
to be considered an operating income b.August 15, 2014 income.
of a security dealer? c.April 20,2014- d.The other income is simply ignored in
a.Dividend income from domestic d. March 30, 2014 the computation of taxable income.-
corporation- 17.Which of these taxpayers is required 23.Statement 1:Individuals with higher
b. Gain on sale of stocks to file an income tax return? income are subject to higher tax rates.
c.Gain on sale of bonds a.An employee covered by the Statement 2:Corporations with higher
d. Interest income from bonds substituted filing system? income are subject to higher tax rates.
9.The distinction between operating b. A taxpayer deriving purely passive Which is correct regarding the regular
and non-operating income is not income subject to final tax. income tax?
required in the income tax return of c. A special alien with respect to his a.Statement 1 only-
a.Self-employed individuals in business compensation income. b.Statement 2 only
b.Mixed income earners d.A resident citizen who derives his c.Both statements 1 and 2
c.self-employed professionals entire income from sources outside d.Neither statement 1 nor 2
d.Purely employed individuals- Philippines.- 24.Which is incorrect in the
10.The reporting classification of gross 18.The taxable income of corporate determination of the taxable income of
income into operating and non- taxpayers is the individual taxpayers?
operating income is unnecessary for a.Net income from business- a.A negative compensation income is
a.Corporate taxpayers- b.Net income from business less deducted against net income from
b.Individual taxpayers personal exemption business.
c.Both A and B c.Taxable compensation income b.A net operating loss is deductible
d.Neither a nor b d.Taxable compensation income plus against taxable compensation income-
11.Which is not part of compensation net income from business c.The taxable compensation income is
income? 19.The taxable income of a pure added to the net income from business
a.Basic pay of rank and file employees compensation income earner is the d.Personal exemption is deducted
b.Fringe benefits of managerial or a. net income from business less against compensation income.
supervisory employees- personal exemption 25.Statement 1:Corporations with the
c.Basic pay of managerial or supervisory b. taxable compensation income plus same net income may not have the
employees net income from business same tax due.
d.Fringe benefits of rank and file c.taxable compensation income- Statement 2:Individuals with the same
employees d. net income from business net income may not have the same tax
12.Who cannot claim deductions? 20.The taxable income of amazed due
a.Employed taxpayers- income earner is the Which statement is incorrect regarding
b.Self-employed taxpayers in business a. net income from business less the regular income tax?
c.self-employed professional taxpayers personal exemption a.Statement 1-
d.B and C b. net income from business b. Statement 2
13.Who are required to file quarterly c. taxable compensation income c. Both statements 1 and 2
declaration of income? d.taxable compensation income plus d. None
a.Individuals engaged in business net income from business- Multiple choice-Problems 1
b.Corporations and individuals engaged 21.The taxable income from a pure 1. Mrs. Sanchez Mira had a gross
in business- professional income earner is the taxable compensation income of
c.Corporations a. net income from business P400,000 and her money in time
d.All individuals and corporations b. taxable compensation income deposits plus P3,000 interest
14.Mr. Jones wishes to file his 2014 c. taxable compensation income plus income from lending money to a
income tax return. To avoid penalty, he net income from business friend.
must file his return on or before d. net income from profession less Compute her taxable income.
a. April 15, 2014 personal exemption- a.P303,000-
b.April 15, 2015- 22. Which of the following statements is b.P302,000
c.August 15,2015 incorrect with respect to the c.P300,000
d.November 15, 2015 determination of the taxable income of d.P305,000
15. An individual taxpayer must file his individual taxpayers with other income? 2.Ms. Claveria had a business net
income tax return for the third quarter a.The other income of pure income of P300,000. She also
of 2014 on or before compensation earners is simply earned P5,000 commission from
a.April 15, 2015 included taxable compensation income. selling cellular cards and P12,000
b.August 15, 2014 b. The other income of a professional dividends from a domestic
c.November 15, 2015 income earner is included as part of corporations. Ms. Claveria has
d.November 15, 2014- P500,000 personal exemption.
Compute her taxable income. d.P270,000 Compute the taxable income if Lesde,
a.P250,000 8. In 2015, Ms. Balayan earned Inc., was a domestic corporation.
b.P262,000 P450,000 gross compensation income a.P3.5M
c.P255,000- but incurred P120,000 net loss in her b.P3.7M-
d.P267,000 business. What is her taxable income C.P1.380M
3. Mr. Pamplona earned total gross assuming she had a personal exemption d.P1.3M
receipts of P800,000 and paid of P100,000? 2.Compute the income tax due in the
P300,000 in expenses in his a.P350,000- immediately preceding problem.
accounting practice. During the b.P450,000 a.P1, 149,000
same year. he also earned a total of c.P230,000 b.P390,000
P60,000 net gain from the sale of d.P0 c.P1,110,000-
domestic stocks directly to a buyer. 9.In the immediately preceding d.P381,000
He also disposed a vacant lot at a problem, compute the income tax due 3. Compute the taxable income
net gain of P140,000. of Ms. Balayan. assuming Lesde Corporation was a
What is that taxable income of Mr. a.P110,000 resident foreign corporation.
Pamplona assuming he has b.P105,000 a.P3,700,000
P100,000 personal exemption? c.P80,000- b.P3,500,000
a.P400,000- d.P0 c.P1,380,000
b.P460,000 10.Mr. Gudani, with a P75,000 personal d.P1,300,000-
c.P540,000 exemption, had the following data in 4.Coompute the income tax due in the
d.P600,000 2015: immediately preceding problem.
4.Mr. Monreal earned a gross Philippines Abroad a.P390,000-
compensation income of P200,000 Gross income from sales P4M P600K b.P1,110,000
and gross business income of Interest income from deposits 40K 80K c.P1,149,000
P500,000 before expenses of Less:Deductions 2M d.P381,000
P200,000. He also earned book 3.6M 5.Compute the tax due if Lesde, Inc.
royalties of P10,000 and P8,000 Compute the taxable income if Mr. was a non-resident foreign corporation.
interest income from client’s Gudani was a resident citizen. Assume that the tax sparing is not
promissory notes. Mr. Monreal has a.P4,405,000- applicable to Cavite.
a personal exemption of P150,000 b.P4,445,000 a.P3,015,000
and personal expenses of P170,000 c.P1,965,000 b.P3,039,000-
during the year. d.P1,925,000 c.P3,000,000
What is Mr. Monreal’s taxable 11.In the immediately preceding d.P429,000
compensation income? problem. compute the taxable income if 6. Mr. Allan derived the following
a.P0 Mr. Gudani was a non-resident citizen. income in 2015:
b.P68,000 a.P4,445,000 Business income P200,000
c.P58,000 b.P4,405,000 Compensation income 300,000
d.P50,000- c.P1,925,000- Interest income from 5/6 lending to
5. What is Mr. Monreal’s net income d.P1,965,000 clients 100,000
from business? 12. What is the tax due in the Interest Income from bank deposits
a.P308,000- immediately preceding problem? 20,000
b.P300,000 a.P581,000- Royalty Income 100,000
c.P310,000 b.P589,500 Capital gain on sale of personal car
d.P518,000 c.P593,800 20,000
6.What is Mr. Monreal’s taxable d.P577,500 Capital gain on the sale of stocks
income? 13. What is the total tax due from the directly to a buyer 40,000
a.P518,000 taxpayer if Mr. Gudani was a non- Ordinary gain on sale of old office
b.P350,000 resident alien not engaged in trade or furniture 10,000
c.P358,000- business? Dividend income 50,000
d.P368,000 a.P510,000 Compute the total passive income
7. Mr. Bangul earned a compensation b.P1,000,000 subject to final tax.
income of P120,000 and net income c.P991,250 a.P170,000-
from business of P300,000. He also d.P1,010,000- b.P1,370,000
earned P8,000 prizes from a dancing Multiple choice-Problems 2 c.P270,000
competition and P45,000 royalties from 1.Lesde, Inc., had the following income d.P1,470,000
his musical composition. Mr. Bangul has in 2015: 7. Compute the total income subject to
P150,000 personal exemption. Philippines Abroad regular tax.
Compute the taxable income. Rent income P10M P12M a.P1,730,000
a.P323,000 Dividend-domestic 50K 200K b.P630,000-
b.P278,000- Royalties 80K 200K c.P1,470,000
c.P308,000 Business expenses 8.7M 9.8M d.P530,000
8. Compute the capital gain subject to 7.PERA investment income and PERA the extended payment. The
capital gains tax distributions 1,000,000 proceeds will not be
a.P70,000 8. 13th month pay and other benefits taxed upon collection, but the
b.P40,000- not exceeding P90,000. P50,000 excess representing
c.P60,000 9. Gains from sale of bonds, interest is a taxable of gross income.
d.P0 debentures, or certificates of Illustration 1:Life Insurance
CHAPTER 8 indebtedness with maturity of more contracts
REGULAR INCOME TAX:EXCLUSIONS than 5 years. Alberto is insured is a P1,000,000
FROM GROSS INCOME 10. Gains from redemption of shares in life insurance policy with annual
Chapter Overview and Objectives mutual fund premium payments of P20,000 for
This chapter discusses the items of EXCLUSION FROM GROSS INCOME 10 years. If Alberto outlives the
income that are excluded from gross A. Proceeds of a Life Insurance policy after the 10th year, he will be
income, hence not subject to income policy-The proceeds of life paid a P500,000 maturity value.
tax under the NIRC. It also discussions insurance policies paid to the Scenario 1
of other exempt income under special heirs or beneficiaries upon the Alberto died on the 8th year of
laws, treaties, or contracts. death of the insured, whether in coverage and his heirs collected the
After this chapter, readers must be able a single sum or otherwise; P1,000,000 proceeds. The entire
to demonstrate: however, if such amounts are insurance proceeds of P1,000,000 is
1.Mastery of the list of exclusions from held by the insurer under an not taxable.
gross income agreement to pay interest Scenario 2
2.Comprehension of exclusion thereon, the interest payments Upon the death of Alberto, the
conditions or limitations of certain shall be included in gross insurance company negotiated for
items of income income. Life is regarded as a an extension of the payment of the
3.Knowledge of the list of entities capital item with infinite value. proceeds wherein the insurance
exempt under the NIRC and special Hence, the proceeds of life company shall pay P1,050,000 on
laws. insurance is a return of capital. the extended payment. The
EXCLUSIONS FROM GROSS INCOME B. Amount received by the insured P1,000,000 proceeds will not be
Exclusions from gross income are as a return of premium-The taxed upon the collection, but the
income which will not be subject to amount received by the insured P50,000 excess representing interest
income tax. They are not included in as a return of premiums paid by is a taxable item of gross income.
gross income subject to regular tax, him under life insurance, Scenario 3
capital gains tax, or final tax. endowment, or annuity Alberto outlived the policy and
Under Sec. 32(B) of the NIRC, the contracts, either during the term collected the maturity value of
following items shall not be included in or at the maturity of the term P500,000.
gross income and shall be exempt from mentioned in the contract or The total proceeds shall be analyzed
taxation: upon surrender of the contract. as:
A.Proceeds of life insurance policy The amount received by the insured Total proceeds
B.Amount received by the insured as a as a return of premium on any Return of premium (P20,000x10
return of premium insurance contract is a return of years) P500,000
C. Gift, Bequest, or descent capital;hence, it is included from Return of premium (P20,000x10
D.Compensation for injuries or sickness gross income. years) (200,000)
E.Income exempt under treaty Illustration 1:Life insurance Return on capital (item of gross
F.Retirement benefits, pensions, contracts income) P300,000
gratuities, etc. Alberto is insured in a P1,000,000 Scenario 4
G.Miscellaneous items life insurance policy with annual After 6 years of payment, Alberto
1. Income in the Philippines of foreign premium payments of P20,000 for assigned the policy to Glino who
government or foreign government- 10 years. If Alberto outlives the paid him P130,000. Glino continued
owned and controlled corporations policy after the 10th year, he will be the premium payments for two
2. Income of the government and its paid a P500,000 maturity value. more years after which Alberto
political subdivisions Scenario 1 died. Glino collected the P1,000,000
3. Prizes, and awards in recognition of Alberto died on the 8th year and his insurance proceeds.
religious, charitable, scientific, heirs collected the P1,000,000 The assignment or sale of proceeds
educational, artistic, literary, or civic proceeds.The entire insurance by Glino resulted in P170,000 return
achievements. proceeds of P1,000,000 is not of capital(P130,000+
4. Prizes and awards in athletic sports taxable. (P20,000x2))and P830,000 taxable
competitions Scenario 2 return on capital. There is loss of life
5. Contributions to GSIS, SSS, Upon the death of Alberto, the in this scenario but it does not
Philihealth, Pag0Ibig, and union Dues insurance company negotiated for pertain to the purchaser of the life
6. Contributions to Personal Equity an extension of the payment of the insurance policy. Hence, the excess
Retirement Account (PERA) proceeds wherein the insurance must be taxable to the heirs.
company shall pay P1,050,000 on
Illustration 2:Life Insurance of donated income shall be included in a recovery of lost profit; hence,
company officers gross income but in the income tax an item of gross income.
Alberto is insured by his employer return of the donor.The P150,000 Illustration 2
corporation for P1,000,000 with the income of the donated property Mr. Pogi was driving his brand
employer corporation as the afer the perfection of the donation new P1,200,000 car when a
beneficiary. Alberto subsequently is included as item of gross income truck bumped it resulting to the
died, and the corporation collected in the tax return of Mark, the total wreckage of his car. Luckily,
the P1,000,000 life insurance donee. he managed to escape the
proceeds. Gift distinguished from exchange incident unharmed. He received
It is interesting to note that the The transferor’s intention or motive P1,300,000 indemnity from the
entire proceeds under this insurance must be evaluated in determining accident.
arrangement are held within the whether a transfer is a gift or an The P100,000 excess indemnity
purview of the NIRC exemption; exchange. Gifts are characterized by is an item of gross income. Note
hence, it is not taxable. pure liberality or disinterested that the law pertains to
Property insurance contracts generosity and are given without personal physical injury rather
The proceeds of property insurance any consideration. An exchange than injury to rights or property.
contracts in excess of the tax basis always involves a consideration. E.Income exempt under treaty
of the property lost is destroyed is a Employment Gratuities Income items that are excluded
taxable return on capital. Gratuities given under an employer- by international agreement to
Illustration:Property insurance employee relationship are normally which the Philippine
Aztec Company secured a fire treated in exchange for services government is a signatory are
insurance covering the entire rendered by employees. Hence, excluded from income tax. It
P2,000,000 fair value of its they are subject to income tax. The must be recalled that treaty
depreciated building. The building transfer of properties by the agreements override provisions
was completely destroyed by fire employer to managerial or of our revenue tax laws in case
when the depreciated cost(tax supervisory employees is generally of conflict under the exemption
basis) of the building was subject to fringe benefit tax. under the exemption doctrine of
P1,800,000. Aztec recovered the Christmas or major anniversary gifts international comity
P2,000,000 insurance proceeds. granted by the employer to F.RETIREMENT BENEFITS,
The total proceeds shall be analyzed employees are de minimis benefit PENSIONS, GRATUITIES AND
as follows: subject to income tax. OTHER BENEFITS
Total proceeds P2,000,000 D. Compensation for injuries and 1.Retirement benefit under R.A
Less:Basis of property destroyed sickness-amounts received 7641 and those received by
(return of capital) 1,800,000 through accident or health officials and employees of
Return on capital (item of gross insurance or under Workmen’s private firms in accordance with
income) P200,000 compensation acts as a reasonable private benefit
C. Gifts, Bequests and Devises or compensation for personal plan maintained by the
Descent-The value of property injuries or sickness, plus the employer.
acquired by gift, bequest, amounts of any damages Requisites of exemption:
devise, or descent. Provided, received, whether by suit or (Pneumonica:1-10-50-RPBP)
however, that income from such agreement, on account of such a. This is the first time
property as well as gift, bequest, injuries or sickness. availment of retirement
devise. or descent of income Illustration 1 benefit exemption
from any property, in cases of Andrew was hit by a jeepney. He b. The retiring official or
transfers of dividend interest, spent 3 months in the hospital employee has been in the
shall be included in gross and paid P100,000 for services of the same
income. hospitalization expenses. He employer for at least ten
Illustration sued the jeepney driver and was (10) years.
Mark received a restaurant business awarded by the court an anguish c. The retiring employee is at
as a gift on April 1, 2010. On that and sufferings;P40,000 for his least fifty(50) years of age at
date, the restaurant had total lost salaries, and P100,000 as the time of retirement.
properties amounting to P400,000 reimbursements for his hospital d. The employer maintains a
including P50,000 cash income bills. reasonable benefit plan
earned since January 1, 2010. The The P200,000 indemnity and the A reasonable private benefit
restaurant posted an additional P100,000 reimbursement for plan means a pension, gratuity,
P150,000 cash income from April 1 hospitalization expenses are stock bonus or profit-sharing
to December 31, 2010. non-taxable returns of capital. plan maintained by an employer
The transfer of business properties Note that health is a capital for the benefit of some or all of
worth P400,000 to Mark is a item with infinite value. his officials or employees,
gratuity subject to transfer tax, not However, the P40,000 wherein contributions are made
income tax. However, the P50,000 reimbursement for lost salary is by such employer for the
officials or employees, or both, her first employer and worked accumulated unused leave
for the purpose of distributing therein for 20 years after which credits (BIR Ruling No. 199-
to such officials and employees she retired at 50. She 2011)
the earnings, or both, for the immediately joined another To avail of the tax
purpose of distributing to such employer and retired after 10 exemption, the employee or
officials and employees the years of service when she was his heirs shall request for a
earnings and principal of the 60 years old. ruling or certificate of
fund this accumulated, and The first retirement benefit from exemption(CTE) from the
wherein it is provided in said the first employer is exempt BIR. The request for the CTE
plan that at no time shall any since Angel is 50 years old and and other required
part of the corpus or income of has rendered at least 10 years documents shall be filed at
the fund be used for, or be of service (i.e, 20 years). The the RDO where the
diverted to, any purpose other second retirement benefit from employer is registered.
than for the exclusive benefit of the second employer is taxable Illustration 1
the said officials and employees. even if she met the residency Yvonne is an employee of
To be exempt, the retirement and age requirements since Goldfish Company which
benefit plan must be “trusteed” these retirement benefit closed its business during
plan where the fund is held exemption can be availed of the year.
under the management of a only once in a lifetime. Yvonne’s last paycheck shows the
trustee free from both employer 2.Separation or Termination following details:
and employee control. Requisite of exemption: Unpaid salary in the last two months
The 10-year service period 1.The separation or termination P30,000
requirement pertains to must be due to job-threatening Current month salary 15,000
cumulative years of employment sicknesss, deaths, or other Separation pay 100,000
with the same employer. It does physical disability; and Total pay P145,000
not need to be continuous years 2. The same must be due to any The current month salary and the
of employment. A requirement cause beyond the control of the P30,000 backwages are subject to
for continuous employment employee or official such as: income tax. The P100,000 separation
would be prejudicial to working a.Redundancy pay is an exclusion from gross income;
women. b. Rentrenchment hence, not taxable.
Illustration 1 c.Closure of employer’s business Illustration 2
Angel was employed in 1990 d.Employee lay-off Henson’s employer was downsizing its
when she was 25 years old. In e.Downsizing of employer’s business operations. Henson was
2010, she availed of the early business identified among others to be laid off.
retirement program of her f.Sickness or death of the To avoid implications of inefficiencies on
employer. employee his part, Henson filed a resignation
Angel satisfied the 10-year The phrase “beyond the control letter to the company and received a
cumulative employment of the employee” connotes separation pay of P120,000.
requirement but she is only 45 involuntariness on the part of The separation pay is taxable as
years old(i.e.,25+(2010-1990))at the employee. In other words, compensation income since the
the time of her retirement. The the separation must not be of underlying reason of the severance of
retirement benefit is taxable. It his own making. the employment(i.e., resignation) is
is an inclusion in gross income Abandonment of office such as within the control of the employee. If
as compensation income. the registration and subsequent Henson got terminated without
Illustration 2 appointment to another office is resigning, the separation pay would be
Assume that Angel joined considered as a voluntary exempt.
another employer and worked separation and does not fall Illustration 3
therein for 7 more years after within the purview of the phrase Mr. Swabe was diagnosed to have
which she retired from her “for any cause beyond the sexually transmitted disease (STD). Due
employment. control of such official or to this, his employer decided to
Although Angel is 50 years old employee”.(BIR Ruling 054-2001) terminate his services but granted him
by then, she is only 7 years The exemption of termination or P1,000,000 separation pay.
under the employ of her second separation benefits does not The P1,000,000 separation pay is
employer. The second extend to: taxable as STD does not normally
retirement benefit is also 1. Backwages or illegal render the employee incapable of
taxable as compensation income deductions repaid by the working.
since she failed the residency employer upon 3.Social Security Benefits, Retirement
requirement. termination(BIR Ruling 003- Gratuities, and Other similar benefits
Illustration 3 2004) from foreign government and other
Assume instead that Angel was 2. Terminal leave pay or the institutions, private or public,
30 years old when she joined commutation of
received by resident or non-resident b. Exercise of essential government These pertains to the employee share in
citizens or aliens who come to settle function the premium contributions to GSIS, SSS,
permanently in the Philippines. Government agencies and Philhealth, Pag-Ibig and union dues. The
Illustration instrumentalities portion of the salary thus contributed is
John was an OFW employed by The general rule with government exempt from income tax.
Microsoft Corporation in the USA. John agencies and instrumentalities is Under RMC No. 21-2011, the exclusion
retired and returned to permanently exemption because of their public pertains only to the mandatory or
settle in the Philippines. He is paid a service nature. However, taxation compulsory monthly contributions.
$2,000 monthly pension from aoolies when they engage in income- Voluntary contributions to Pag-Ibig 2,
Microsoft’s pension fund and another producing activities which are GSIS, or SSS in excess of the mandatory
$800 monthly benefit from the US proprietary or commercial in nature. monthly contribution are taxable. Note
social security benefit. This exemption does not extend to that Pag-Ibig is now called the Home
Both the pension and the social security government-owned and controlled development Mutual Fund or HDMF.
benefits are exempt. Note that these corporations(GOCCs), GOCCS are Illustration
benefits were earned abroad when the generally taxable as regular corporation An employee has a gross compensation
taxpayer was a non-resident. Under because their operations are income of P400,000 in 2016. His
situs rule, the foreign income of non- proprietary in nature. employer deducted P5,000 SSS, P4,000
residents is not taxable in the 3. Prizes and Awards made Philhealth, P3,000 HDMF, P2,000 union
Philippines. This holds true even if the primarily in recognition of dues and P80,000 creditable
taxpayer subsequently receives the religious, charitable, withholding tax.
income as a resident of the Philippines. scientific, educational, Illustration
4.United States Veterans artistic, literary, or civic An employee has a gross compensation
Administration(USVA) –administered achievements but only if: income of P400,000 in 2016. His
benefits a. The recipient was employer deducted P5,000 SSS, P4,000
under the laws of the United States selected without any Philhealth, P3,000 HDMF, P2,000 union
received by any person residing in the action on his part to dues and P80,000 creditable
Philippines enter the contest or withholding tax.
Illustration proceeding; and Thus, the gross income subject to
Mr. Jackson is a retires US serviceman b. The recipient is not regular tax shall be computed as
from the Iraqui war. He married a required to render follows:
beautiful Filipina and settled in the substantial future Gross compensation income
Philippines. He is receiving a $1,000 services as a condition to P400,000
monthly benefit from the USVA receiving the prize or Less: Excluded compensation income or
The USVA benefit is excluded in gross award. contributions
income. The same rule applies to USVA Prizes of this kind partake the nature of Contributions to SSS P5,000
benefits for beneficiaries of Filipino a unilateral transfer and hence, exempt Contributions to Philhealth 4,000
veterans who fought under the from income tax. These transfers are Contributions to HDMF 3,000
American flag in World War 11. also exempt by law from transfer tax. If Union dues 2,000
5.Social security system (sss) benefits the recipient exerted effort for the grant 14,000
under RA 8282 of the prize such as joining a contest or Gross taxable compensation income
6.GSIS benefits under RA 8291 is required to render service for its P386,000
including retirement gratuity received grant, the prize would be construed as 6. Contributions to Personal
by government officials and employees. received in an exchange; hence, taxable Equity Retirement
G. Miscellaneous items as income. Account(PERA)
1.Income derived on investments in Examples of exempt prizes: PERA is a contributor’s voluntary
the Philippines in loans, stocks, bonds a.Nobel Prize award retirement account established from
or other domestic securities, or from b.Gawad ng Sining Award qualified contributions of the
interest on deposits in banks in the c. CNN hero of the year contributor and or his employer for the
Philippines by: d. Most outstanding citizen sole purpose of being invested in
a. Foreign governments 4. Prizes and Awards in Sports qualified PERA investment products.
b.Financing institutions owned, Competitions granted to athletes: Each OFW is allowed to contribute up to
controlled, or enjoying refinancing from a.In local or international competitions P200,000 per year to a PERA account.
foreign government. and tournaments; Non-OFWs are allowed P100,000
c. International or regional financial b.Whether held in the Philippines or contributions per year. Husband and
institutions established by foreign abroad; and wife can each contribute up to the
governments c.Sanctioned by their national sports maximum allowable contribution.
These are exempt under the exemption associations. Contributions to PERA accounts are
doctrine of international comity. 5.Contributons for GSIS, SSS, exclusions in gross income. This is an
2.Income derived by the government Philhealth, Pag-Ibig, and Union dues of additional exclusion and is separate
and its political subdivisions from: individuals. with the exclusion for contributions to
a. Any public utility or SSS or GSIS. Moreover, PERA
contributors are allowed to claim 5% of The term mutual fund company shall including holiday pay, overtime pay,
their PERA contributions as tax credit mean an open-end and close-end night shif differential pay and hazard
against any internal revenue taxes. investment company as defined under pay.
7.PERA investment income and PERA the Investment Company Act. Barangay Micro-Business
distributions Mutual funds pool the money invested Enterprise(BMBE)
PERA investment income are exempt by different investors and invest the A BMBE is a business entity or
from taxes (i.e., final tax, capital gains money to earn investment income enterprise engaged in the production,
tax and regular income tax). The PERA which shall add up to the net assets of processing, or manufacturing of
account assets will be distributed back the fund. A participating investor must products or commodities, including
to the contributor either in lump sum, purchase participation shares from the those arising from loans but exclusive of
life pension or in installment upon fund at their Net Asset Value(NAV). the land on which the particular
reaching the age of 55 or to his heirs or Upon redemption of his participation business entity’s office, plant, and
beneficiaries upon his or her death. shares, the investor gains or losses by equipment are situated, do not exceed
PERA distributions are likewise in gross his proportionate share in the increase P3,000,000.
income of the contributor or his heirs or or decrease in the Net Asset Value of The term service excludes those
beneficiaries as the case may be. the fund. rendered by licensed professionals and
8.13th Month pay and other benefits Illustration partnership and corporations engaged
received by officials and employees of A taxpayer bought 10,000 shares from in consultancy, advisory and similar
public or private entities not exceeding Golden Dragon Mutual Fund at P120 services and corporations are
P90,000 NAV per share. The taxpayer redeemed essentially carried out through licensed
13th month pay and other benefits will his shares when the NAV per share was professionals.
be discussed in detail in Chapter 10. P180. A BMBE shall include any individual
9.Gains from sale of bonds, The P600,000 gain, computed as((P180- owning such business entity or
debentures, or other indebtedness P120)X10,000), on redemption is enterprise, partnership, cooperative,
with a maturity of more than 5 years. excluded from gross income; hence corporation, association, or other entity
This exemption is grounded upon the exempt from taxation. incorporated and/ or organized and
same assumption that long-term The exemption is apparently intended existing under Philippine laws and
indebtedness is diverted to the to mitigate double taxation. Most of the registered with the office of the
financing of long-term projects which is items of income of mutual funds are treasurer of a city or municipality.
viewed as beneficial to the subject to final tax at source. The To qualify as a BMBE, an enterprise
development of the country. subsequent distribution of these to the must not be a branch or a subsidiary of
The term “gain” however, does not investors at redemption should no a large scale enterprise such as in the
include “interest”.(Nippon Life longer be subject to income tax. On the case of franchises.
Insurance Company of the Philippines other hand, the exemption may have To avail of the benefits and privileges of
vs. CIR, CTA Case No. 6142) been intended to promote the growth a BMBE, an applicant must secure a
Illustration of mutual funds which are widely certificate of authority to operate as a
On September 1, 2014, an individual regarded as key participants in BMBE from the office of the Treasurer
taxpayer sold 6-year term bond providing liquidity in most financial of the city or municipality that has
investment for P1,100,000. These markets. jurisdiction.
bonds bear 8% interest payable every OTHER EXEMPT INCOME UNDER THE Tax exemptions on Income from
December 31 and were previously NIRC AND SPECIAL LAWS Operations.
acquired at P1,000,000 face value on 1. Minimum wage and certain Aside from other incentives afforded by
January 1, 2014. benefits of Minimum wage the law, the income of BMBE from their
The gain on sale will be computed as earners operation is exempt;, hence, excluded
follows: 2. Income of Barangay Micro- from the gross income subject to
Selling price P1,100,000 Business Enterprises Act(RA regular income tax. BMBEs file an
Less: Cost of bonds sold 1,000,000 9178) annual information return in lieu of the
Interest 3. Income of cooperatives (RA income tax return. However, their non-
accrued(P1MX8%X9mos./12mos.)60,00 9520) operating, passive, and capital gains are
0 4. Income of non-stock, non-profit subject to the appropriate type of
Gain on sale P40,000 entities income tax.
The gain on sale of the long-term bonds 5. Income of qualified employee Illustration
is exempt because the bonds have a trust funds William has a bakery with total assets of
maturity period of more than 5 years. Minimum Wage Earners P4,000,000 inclusive of a lot with a
However, the accrued interest income is A minimum wage earner is an individual book value of P1,200,000.
an item of gross income subject to recipient of a minimum wage as fixed Gross income from sales of bread
regular income tax. by the Regional Tripartite Productivity P300,000
10. Gains realized from redemption of Wage and Productivity Board of the Interest on promissory notes of retail
shares in a mutual fund company by Department of Labor and Employment. store clients 12,000
the investor A minimum wage earner is exempt Royalties on sale of recipe books 36,000
from income tax on the minimum wage
Dividend income from domestic stocks Cooperatives that transact business Exclusions from gross income are not
10,000 purely with members are exempt from included in the amount of reportable
Note that William’s total asset is all taxes and fees. Cooperatives that gross income in the income tax return.
P2,800,000, excluding the lot. Hence, transact business with non-members The amount of deductions is initially
William’s business qualifies as a BMBE, are likewise exempt from all taxes and included in the amount of gross income
If William obtained a certificate of fees if their accumulated reserve and but is separately presented as
authority to operate as a BMBE. If undivided savings do not exceed P10M. deduction against gross income in the
William obtained a certificate of Otherwise, the amount of surplus income tax return.
authority to operate as a BMBE, the allocated for interest on capital is
following items of operating income are subject to regular tax.
exempt from income tax: However, the income of any Note to readers:
Gross income from sales of bread cooperative from non-related sources is
P300,000 fully taxable to regular tax. Exclusions in gross income
Other operating income: Non-Stock and Non-profit Entities represents one of the
Interest income from client Non-stock entities that are not exceptions to the general
promissory notes 12,000 organized for profit are exempt from scope of the regular income
Total exempt income income tax on their income from tax. Readers are advised to
P312,000 operations. However, their income from master or, at least, familiarize
Assuming William’s bakery is not unrelated sources is taxable. themselves with the list and
registered as a BMBE, the P312,000 Qualified Employees’ Trust Fund their respective exclusion
total operational income will be subject An employees’ trust fund which forms criteria before proceeding to
to the regular income tax. part of a pension, stock bonus or profit the next chapters of the book.
Either way, the royalty income and sharing plan of an employer for the This is important in assisting
dividend income are exclusions in the benefit of some or all of his employees readers in mastering the
gross income subject to regular tax but is exempt from any income tax under regular income tax.
are inclusions in the gross income the NIRC.
subject to final tax. Conditions for exemptions of
Another illustration employee trust funds CHAPTER 8:SELF-TEST EXERCISES
Chris Santa has an accounting and a.Contributions are made to the trust Discussion Questions
auditing firm with total assets of by such employer, or employees, or 1.Enumerate the exclusions from gross
P2,500,000. He derived a total both for the purpose of distributing to income
operating income of P1,000,000 in such employees the earnings and 2. What capital items are considered
2014. principal of the fund accumulated by with infinite value?
The entire P`1,000,00 is taxable since the trust in accordance with such plan. 3.Enumerate the exclusion conditions of
Mr. Santana is a professional service b. The asset of the fund shall not be retirement benefits.
provider not qualified to be a BMBE. diverted for other purposes other than 4.Discuss the exclusion condition of
Revocation of BMBE Tax Exemptions the exclusive benefit of the employees. termination benefits.
The income tax exemption of a BMBE QUALIFICATION OF EXEMPTION OF 5.Discuss the rules on taxation of prizes
may be revoked for any of the following EXEMOT ENTITIES 6.Enumerate the exclusions from gross
reasons: Tax incentive or exemption is highly compensation income in the
1.Transfer of place of business disfavored by law. It is not automatic. determination of gross taxable
2.Value of assets exceeds P3,000,000 Taxpayers with exemptions or tax compensation income.
3. Voluntary surrender of the Certificate incentives under any existing laws or 7.Who is a minimum wage earner?
of Authority contracts must establish their 8. Enumerate the benefits of minimum
4.Death of the registered individual entitlement by filing required wage earners exempt from income tax
owner; violation or non-compliance documents with the BIR. BMBEs need 9.What is a BMBE?
with the provisions of RA 9178 to secure a Certificate of Authority. 10.What are the exemption conditions
4.Merger or consolidation with an Cooperatives need to secure a of an employee trust fund?
entity which is not eligible to be a Certificate of Tax Exemption/Ruling True or false
BMBE. (CTE). Once exemption is established, it 1.The proceeds of life insurance
6.Sale or transfer of the BMBE if a sole only operates prospectively. received by the heirs of the insured
proprietorship without prejudice to the INCOME SUBJECT TO FINAL TAX OR upon his death is excluded in gross
transferee applying for registration. CAPITAL GAINS TAX income-True
7.Submission of fake or falsified Items of income that are subject to final 2.The amount received in excess of the
documents tax or capital gains tax are not items of premium paid in an insurance contract
8. Retirement from business, or gross income subject to regular income constitutes an item of gross income.
cessation/suspension of operations for tax. Also income items that are True
one year exempted in the coverage of final tax or 3.Donated income is included in the
9. Making false or omitting required capital gains tax are not taxable to the gross income of the donee.False
declarations or statements regular income tax.
Cooperatives EXCLUSIONS VS. DEDUCTIONS
4.Compensation for injuries and 6.The employer’s share to SSS, 7.Progressive income tax does not apply
sickness constitutes profit; hence, an PhilHealth, and Pag-Ibig contributions to a special alien
inclusion in gross income.False are an exclusion in gross income.False 8.Proportional income tax does not
5. It is sufficient that the employee 7.Compared to exclusion, deduction is apply to a General professional
rendered more than 10 years of service included in the amount of gross income partnership
for his retirement benefit to be but both exclusion and deductions are 9.Which is not included under the term
exempt.False not reflected in the amount of taxable “corporation”?
6.An employee can secure retirement income.True. Co-ownership
benefit exemption only once in a 9. Cooperatives that transact business /Business partnership, non-profit
lifetime.True only with members will, in no case, be charitable institution, joint venture
7.It is a must that the employer subject to income tax-False 10.The highest marginal tax rate for
maintains a reasonable pension benefit 10. Cooperatives, regardless of their individual income tax payers is
plan for the retirement benefit to be classification, are taxable on income a.25%
exempt.True from their unrelated activities.True b.30%
8.An employee must have rendered 11.The gain on the sale of long-term c.35%
more than 10 years of service before bonds with a maturity of five years is an d.32%-
claiming exemption for his termination exclusion in gross income.False 11.Which corporate taxpayer is not
benefits.False 12.A non-stock, non-profit entity is subject to regular income tax?
9.The income of the Philippine subject to tax on income from a.Non-resident foreign corporation-
government from essential public unrelated activities.True b.Domestic corporation
functions is exempt from any income 13.A general professional partnership c.Resident foreign corporation
tax.True can be registered as a BMBE. d.Business partnership
10. Prizes paid to corporations are an 14.A general professional partnership 12.Which individual income taxpayer is
inclusion in gross income subject to can be registered as a BMBE.False not subject to regular tax?
final tax.False 15. A BMBE must have a net asset not a.Non-resident alien not engaged in
11.Only the mandatory portion of GSIS, exceeding P3,000,000 to be trade or business
SSS, PhilHealth, and union dues can be exempt.False //Resident alien not engaged in trade or
excluded in gross compensation Multiple Choice-Theory 1 business, Non-resident citizen not
income. True. 1.Statement 1:Items of passive income engaged in trade or business, Non-
12.Social security benefits, retirement from abroad are subject to regular resident alien engaged in trade or
gratuities, and other benefits from income tax business
foreign governments are excluded in Statement 2:Items of passive income 13.Which is correct with respect to
gross income.True. from the Philippines are generally exclusions from gross income?
13.Social security benefits, retirement subject to final income tax. a.They are included as part of gross
gratuities, and other benefits from Which statement is generally correct? income but are subsequently deducted.
foreign private entities are included in a.Both statements b.They are not included in gross income
gross income.False 2.Which is true with the regular income but are added to the taxable income
14.The gain from redemption of shares tax? c.They are not considered in the
in mutual fund is an exclusion in gross a.Tax is payable at regular intervals computation of taxable income-
income subject to regular tax because it 3. Which is not a feature of regular d.They are synonymous with deductions
is an inclusion in gross income subject income tax? 14.Which is correct with respect to
to capital gains tax-False c.Gross income tax deductions from gross income?
15.13th month pay and other benefits //It generally applies to all items of a.They pertain to expenses of
are taxable only up to P82,000.False gross income not subject to final tax, generating items of business or
True or False 2 creditable withholding tax, accounting professional gross income.-
1.GSIS and SSS benefits are included in period. b. They are excluded from the
gross income to the extent they exceed 4.Statement 1:Capital gains are determination of taxable income
P82,000.False generally subject to capital gains tax. c.They include all expenses incurred in
2.Prizes awarded upon the condition Statement 2:Items of passive income in the generation of any income
that the recipient shall render specified the Philippines are generally subject to d.They include personal exemptions
future services is an item of gross regular income tax. 15.Which constitutes a taxable item of
income.True False gross income?
3. Prizes from contests are included in 5.Deductions from gross income are- a.Compensation for personal injuries
gross income subject to regular income Business expenses b.Gain from sale of shares in mutual
tax.False 6. Which of the following statements funds
4.The income of government-owned best distinguishes deductions from c.Gain from sale of government bonds-
and controlled corporations is an item exclusions from gross income?- d.Income exempt under treaty
of gross income.True Deductions are outflows from gross Multiple Choice-Theory 2
5.The employer’s share to SSS, incomes while exclusions are not 1.The proceeds of an insurance policy
PhilHealth, and Pag-Ibig contributions outflows from gross income. received by the corporation as
are an exclusion in gross income.True
beneficiary on the life insurance of its 8.Which is not a requisite of exemption Multiple Choice-Problem:Part 1
officers is of a retirement benefit plan? 1.Mr. Bisligo collected the P1,000,000
a.a gift a.10 years of employment insurance proceeds of Mr. Pantukan
b.a taxable income b.The employer maintains a reasonable which he bought from the latter for
c.an inheritance pension benefit plan P400,000. Before the death of Mr.
d.exempt from income tax- c.The retiree must be a senior citizen.- Pantukkan, Mr. Bisligo paid total
2.Mr. Buguey was insured in a life d.First time availment of retirement premiums of P200,000
insurance with his daughter, Ybon, as exemption Determine respectively the exclusion in
the irrevocable beneficiary. Ybon was 9.Termination benefits are exempt from gross income and the inclusion in gross
paid the entire proceeds when Mr. income tax provided that the reason for income.
Buguey died. The proceeds constitute termination is a.P1,000,000;P0
a.A taxable inheritance a.beyond the employer’s control- b.P0;P1,000,000
b.a taxable gift b.within the employer’s control c.P400,000;P600,000
c.a taxable income c.within the employee’s control d.P600,000;P400,000-
d.an exclusion from gross income- d.beyond the employer’s control 2.Mr. Kabacan surrendered his life
3.A policy holder who outlived the 10.Which is not an item of exclusion insurance policy and received a cash
policy and received a cash surrender from gross income? surrender value of P800,000 after
value in excess of premiums paid is a.SSS benefits contributing P700,000 in annual
exempt upon b.Income of the government and its premiums. Determine respectively the
a.The amount representing a return of political subdivisions total exclusion in gross income and the
premiums- c.Income from government-owned and inclusion in gross income.
b.The entire amount received controlled corporations- a.P800,000;P0
c.the excess of the amount received d.Income of foreign governments b.P0;P800,000
over the premiums paid 11. Which of the following government- c.P100,000;P700,000
d.None of these owned and controlled corporations is d.P700,000;P100,000-
4.The assignment of an insurance policy subject to income tax? 3.Mr. Tarragoza died. His heirs collected
at an amount in excess of the premium a.Social security system the P2,000,000 proceeds of his
paid on the policy is subject to b.National Development corporation- insurance policy. Mr. Tarragoza
a.donor’s tax c.Philippine Charity Sweepstakes office previously paid a total payment of
b.income tax- d.Philippine Health Insurance P500,000 in premiums. Determine
c.estate tax Corporation respectively the exclusion in gross
d.any of these 12.Which is not an acceptable ground income and the inclusion in gross
5.A widow who collected the life for exemption of termination pay? income.
insurance proceeds of her deceased a.Mass employee lay-off a.P2,000,000;P0
husband is b.Closure of employer’s business b.P500,000;P1,500,000
a.exempt to the entire amount of the c.Grave misconduct and neglect of c.P2,000,000;P0-
proceeds- duty- d.P0;P2,000,000
b.taxable to the excess of the proceeds d.Retrenchment of employer’s business 4. Mr. Malalag collected the P5,000,000
over the premiums paid by the husband 13.Which is not an item of gross income fire insurance proceeds of his building
c.taxable to the excess of the proceeds for taxation purposes? which was destroyed by fire. The
over the premiums paid by the widow a.Income earned between related building had a tax basis of P4,500,000 at
d.exempt with respect to the portion of parties the occurrence of the fire. Determine
the proceeds representing returns of b.Unrealized income- respectively the total exclusion in gross
premium c.Advanced income income and the inclusion in gross
6.The policyholder of a life insurance d.All of these income.
contract outlived his insurance policy, 14.Which is subject to income tax? a.P5,000,000;P0
He was paid P300,000 upon maturity of a.Gain on sale of 6-year bonds b.P0;P5,000,000
the policy. He paid P250,000 total b.Gain on sale of shares in mutual fund c.P4,500,000;P500,000-
premium. What is the inclusion in gross c.Interest income on long-term bonds d.P500,000;P4,500,000
income? with maturity period exceeding five 5.Mr. Cateel insured his crops for a
a.P300,000 years- P1,000,000 insurance cover against
b.P250,000 d. Interest income on long-term calamities. He paid and expensed
c.P50,000- deposits by individual taxpayers P100,000 insurance premium. How
d.P0 15.Which of the following is not an much will be included in gross income?
7.Which of the following is subject to exclusion from gross income? a.P900,000
tax? a.Income from government properties- b.P1,000,000-
a.Proceeds of crop insurance b.Income taxes collected by the Bureau c.P100,000
b.Proceeds of livestock insurance of Internal Revenue d.P0
c.Indemnity under patent infringement c.Dividend Income payable to a foreign 6.Ms. Sindangan received a
suit government condominium including its accrued
d.All of these- d.Social Security benefits income as inheritance from her
deceased grandfather on April 1, 2014. a.P0 Compute the total exemptions and
The following data relates to the b.P5,000,000 exclusions from gross income
property: c.P100,000,000- a.P46,000-
Fair value of property P125,000,000 d.P105,000,000 b.P56,000
Rent income earned before death of 11. Mr. Bangus, a rank and file c.P38,000
decedent 4,000,000 employee, received the following 13 th d.P36,000
Rent income earned after death of month pay and other benefits during 15.Mang Antonio collected P2,000,000
decedent 6,000,000 the year: from the fire insurance company that
Interest on deposits of rentals(40% 13th month pay P48,000 insured his building which had a tax
accruing after death) 100,000 Productivity pay 22,000 basis of P1,800,000 when the fire
How much of the above income will be Excess de minimis above legal limits occurred. During the same period, he
included in the gross income of Mr. 24,000 also collected P300,000 crop insurance
Sindangan and in the gross income of Compute the total exemptions and proceeds pertaining to his crops
the decedent? exclusions from gross income: destroyed by frost. The total item of
a.P6,040,000;P4,060,000- a.P82,000- gross income is
b.P6,000,000;P4,000,000 b.P94,000 a.P2,300,000
c.P4,060,000;P6,040,000 c.P70,000 b.P200,000
d.P4,000,000;P6,000,000 d.P46,000 c.P300,000
7.Mr. Dimaling was hurt in a bus 12. Mr. Alvarez had the following d.P500,000-
accident. He received a total indemnity income during the year: Multiple Choice-Problems:Part 2
of P800,000 from the insurer of the bus. Gross compensation income 1.Wary of his deteriorating health
Mr. Dimaling paid P250,000 in hospital including P25,000 13th month pay conditions, Mr. Benigno resigned from
bills due to the accident. Compute the P325,000 his job at age 40 after working as a
total amount to be excluded in gross Less:tardiness or absences (10,000) supervisor for 12 years. He was paid
income. Net Compensation income P315,000 P2,000,000 as separation pay. Is the
a.P0 SSS deductions 12,000 P2,000,000 separation pay subject to
b.P250,000 PhilHealth deductions 9,000 income tax?
c.P550,000 Pag-Ibig deductions 10,000 a.No, because the reason for Mr.
d.P800,000- Union dues 5,000 Benigno’s termination was beyond his
8.At the age of 54, Mrs. Sindangan was Withholding tax 40,000 control
awarded a retirement gratuity of Net pay P239,000 b.Yes, because Mr. Benigno resigned-
P2,000,000 for her 30 years of service in Compute the total exclusions from c.No, because Mr. Benigno worked for
the Sirawai Company. The employer’s gross income. the company for more than 10 years
retirement benefit plan was with an a.P36,000 d. Yes, because Mr. Benigno is not yet
employee participation feature where b.P61,000- 50 years old.
Mrs. Sindangan contributed a total c.P66,000 2. On December 25, 2014, Mr. Reynon
premium of P800,000 in the fund. Mrs. d.P71,000 was terminated by his employer at age
Sindangan also received P500,000 13. Mr. Henares received the following 60 due to his failing eyesight. He joined
benefit from the SSS. during the year: the company in February 2006 and has
Compute the total exclusion in gross Donated properties P200,000 since then worked as treasurer of the
income Income of donated property before company. Is Mr. Reynon’s retirement
a.P1,700,000- donation 50,000 pay exempt from income tax?
b.P1,300,000 Income of donated property after a.Yes, because his termination was
c.P800,000 donation 30,000 beyond his control-
d.P2,500,000 Inherited properties 100,000 b.No, because he was employed for less
9. Mrs. Candoni retired from the How much is taxable to Mr. Henares? than 10 years
government after 30 years of service at a.P380,000 c. Yes, because he is over 50 years old
the age of 55. He received a total b.P80,000 d.No, because the employee is already
retirement pay of P1,800,000 plus c.P30,000- a senior citizen.
P400,000 GSIS benefits. How much will d.P50,000 3. Mr. Henson retired from her job after
be excluded in gross income? 14. Mr. Tacurong has the following data 25 years of service. She joined the
a.P0 during the year: company at the age of 23 and was
b.P400,000 Basic salary P400,000 promoted from an accounting clerk to
c.P2,200,000 Income tax withheld 50,000 VP Finance. She was paid P2,000,000
d.P1,800,000- 13th month pay 40,000 total retirement pay from the
10.The professional regulations SSS 2,000 employer’s contributory pension plan
commission collected a total sum of Philhealth 1,800 which was duly registered with the BIR.
P100,000,000 from professional license Pag-Ibig 1,700 out of the total proceeds, Ms. Henson
fees. It also collected P5,000,000 from Union dues 500 contributed P600,000. This was Ms.
rentals of government properties. What Capital build up contribution in a social Henson’s first retirement from
is the total exclusion in gross income? fund 2,000 employment.
How much is excluded from gross 7.The following relates to the b.P415,800
income? compensation income of Ms. Lamitan in c.P15,800
a.P600,000- 2107: d.P0
b.P1,400,000 Compensation P2,400,000 11.In the immediately preceding
c.P2,000,000 Contributions to SSS, Philhealth, and problem, compute the total inclusions
d.P0 HDMF: in gross income subject to final tax.
4.Assuming Ms. Henson transferred to -Mandatory contributions 125,000 a.P0
another company and was retired after -Voluntary contributions 150,000 b.P15,800-
15 years of service. The second Contribution to PERA 120,000 c.P6,800
employer paid P1,500,000 out of its Creditable withholding taxes 190,000 d.P9,000
non-contributory pension fund as Employer’s share in SSS, Philhealth and 9,000+6800=15800
retirement pay to Ms. Henson. The HDMF 105,000 13. KKB, a multi-purpose credit
pension fund was also duly registered What is the total exclusion in gross cooperative, had the following
with the BIR. Is the second retirement income? income in 2015:
pay exempt from income tax? a.P225,000-120+105=225 Income from related activities P400,000
a.No, because this is the second time b.P230,000 Income from unrelated activities:
Ms. Henzon retired from employment. c.P395,000 -Dividend from stocks 20,000
b.Yes, because Ms. Henson is already a d.P465,000 -Income from time deposits 18,000
senior citizen 8. In 2014, Ms. Kabacan invested -Rent income 60,000
c.Yes, because this is the first time Ms. P8,000,000 in the 10-year bonds of Compute the total exclusion from gross
Henson qualifies for retirement pay Compostela Mining Corporation. She income subject to regular tax of the
exemption- disposed the investment in 2016, for a cooperative
d.No, because an employee must work total consideration of P8,500,000 a.P0
under one employer until retirement to inclusive of the P400,000 accrued b.P38,000
qualify for retirement exemption. interest. What are respectively the c.P438,000-
5. Mr. Baracho won P500,000 in a local inclusion in gross income and the d.P400,000
chess competition that was sanctioned exclusion in gross income? P400+20+18=P438,000
by a sports organization that is currently a.P500,000;P0 14.A non-stock, non-profit charitable
applying for accreditation from the b.P100,000;P400,000 entity received the following during
national sports association. Is the c.P400,000;P100,000- 2015:
P500,000 prize considered an item of d.P0;P500,000 Contributions from the public
gross income? 9. Ms. Sibuco invested in the mutual P1,400,000
a.Yes, because all prizes are subject to fund and savings deposit of BCBC Bank. Income from the sale of merchandise
income tax She acquired a 100,000 participation 500,000
b.No, because the chess competition is shares when the net asset value per Gain on the sale of properties
not an international competition unit of the fund was P98.00. She pulled 300,000
c.No, because the sport competition is a out her investment when the net asset What is the total exclusions from gross
local competition value per unit was P101.00. Ms. Sibuco income subject to regular tax?
d.Yes, because the organizer is not an also had P100,000 accrued interest in a.P2,200,000
accredited sports organization- her savings deposit. b.P1,900,000
6. Mr. Sibuco discovered teleport Which statement is incorrect? c.P1,400,000-
technology where people can be a.The P300,000 gain is an exclusion in d.P1,300,000
transported over thousand miles in gross income. 15. Mr. Santiago purchased a life
seconds. Due to this, he was awarded b.The P100,000 interest income is an annuity for P100,000 which will pay him
by the scientific community the most exclusion in gross income subject to P10,000 a year. The life expectancy of
coveted Nobel Prize award in 2015. The regular tax. Mr. Santiago is 12 years. Which of the
total award was $150,000,000. C.The P100,000 interest income is an following can Mr. Santiago exclude from
What is the most correct statement inclusion in gross income subject to his gross income?
regarding the taxation of the award? final tax. a.P10,000
a.The award is an inclusion in gross d.The P300,000 is an inclusion in gross b.P20,000
income subject to regular income tax income subject to regular tax, but the c.P120,000
since final taxes do not apply abroad. P100,000 is an exclusion in gross d.P100,000-
b.The award is an inclusion in gross income subject to final tax.- CHAPTER 9
income subject to final tax since it is 10. The following income relates to a REGULAR INCOME TAX:
more than P10,000. proprietorship registered as a BMBE. INCLUSION IN GROSS INCOME
c.The award is an exclusion in gross Gross Income from sales P400,000 Chapter Overview and Objectives
income subject to final tax since it is an Dividend income-domestic 9,000 This chapter discusses inclusion in gross
inclusion in items of gross income Interest on deposits 6,800 income subject to the regular income
subject to final tax. Compute the total exclusion in gross tax
d.The award is an exclusion in gross income subject to regular tax. After this chapter, readers are expected
income.- a.P400,000- to demonstrate:
1.Mastery of the NIRC of items of gross pertains to the types of employee The gains or losses in dealing in
income subject to regular tax and their benefits that are subject to regular tax. ordinary assets are subject to regular
measurement rules The fringe benefits of managerial or income tax.Dealings in capital assets
2.Knowledge of the boundary between supervisory employees are not other than domestic stocks and real
income subject to final tax or capital considered compensation income and properties are also subject to regular
gains tax and those to regular income are subject to final tax. income tax.
tax To minimize the complexity of this Ordinary gains are included as items of
3.Knowledge of the link between items section, compensation income is gross income. Ordinary losses are items
of exempt income and income subject separately discussed in Chapter 10. of deductions against gross income. The
to regular income tax Gross income from the conduct of net capital gain from other capital
4.Comprehension of the effect of trade, business or exercise of a assets after deducting capital losses is
accounting methods and situs rules on profession also included as an item of gross
the reportable amount of gross income This includes income from any trade or income. A net capital loss is not an item
5.Knowledge of the treatment of business, legal or illegal, and whether of deduction against gross income.
creditable withholding tax registered or unregistered. To avoid complicating this section, the
6. Understanding the treatment of Gross income from business or tax rules on measurement and
income from pass-through entities profession is determined as follows: recognition of gains from dealings in
7.Mastery of the rules on recoveries of Sales/Revenues/Receipts/Fees Pxxx,xx properties are discussed in detail in
past deductions Less:Cost of sales or services xxx,xx Chapter 12.
8. Appreciation of the essence and Gross income from operations Pxxx,xx Interest income
purposes of transfer pricing regulation The following business income shall not This particularly refers to interest
ITEMS OF GROSS INCOME be included in gross income subject to income other than passive interest
The term items of gross income or regular tax: income subject to final tax. A taxable
inclusions in gross income is a broad 1.Business income exempt from interest income must have been
category pertaining to all items of income tax such as: actually paid out of an agreement to
income subject to taxation, namely: a.Gross income from a Barangay Micro- pay interest. It cannot be imputed. (CIR
1.Gross income subject to final tax Business Enterprise(BMBE) under RA vs. Filinvest Development Corporation,
2.Gross income subject to capital gains 9178 GR 163653 and 167689).
tax b. Gross income from enterprises Examples of interest income subject to
3.Gross income subject to regular tax enjoying tax holiday incentives under regular income tax:
Items of gross income subject to final EO 226 which have not yet graduated to 1.Interest income from lending
tax and items of gross income subject their income tax holiday incentives activities to individuals and
to capital gains tax are respectively 2.Business income subject to special corporations by banks.
discussed in Chapter 5 and Chapter 6. tax regime such as: 1.Interest income from lending
This chapter focuses on the items of a.Philippine Economic Zone activities to individuals and
gross income subject to regular tax. Authority(PEZA) –registered enterprises corporations by banks, finance
ITEMS OF GROSS INCOME SUBJECT TO subject to 5% gross income tax companies and other lenders
REGULAR TAX b.Tourism infrastructure and Enterprise 2.Interest income from bonds and
Gross income includes, but is not Zone Authority(TIEZA)-registered promissory notes
limited to, the following items: enterprises subject to 5% gross income 3. Interest income from bank deposits
1.Compensation for services in tax abroad
whatever form paid 3.Business income subject to final tax Exempt interest income
2.Gross income from the conduct of when not subjected to final tax by the The following are exempt from regular
trade, business, or exercise of a payor income taxation:
profession a.Subcontractors of petroleum service 1.Interest income earned by
3.Gains derived from dealings in contractos subject to 8% final tax landowners in disposing their lands to
properties b.Business income of foreign currency their tenants pursuant to the
4.Interest deposit units(FCDUs) and offshore Comprehensive Agrarian Reform Law
5.Rents banking units(OBUs) from Philippine 2.Imputed Interest income
6.Royalties residents subject to 10% final tax Imputed interest income (the
7.Dividends Taxpayers with multiple type of opportunity cost of money) does not
8.Prizes and winnings business income shall report their gross constitute an actual income; hence; it is
9. Annuities income subject to regular tax under the exempt from income tax.
10.Pensions column “Regular” in the computation of The power of the Commissioner to
11.Partner’s distributive share from the tax per regime in the annual income tax allocate income and deduction does not
net income of general professional return. Gross income subject to special include the power to impute
partnership rate and those exempt are presented “theoretical interest”.(Ibid)
Compensation for services in whatever respectively under the columns”Total Illustration
form paid Speical”and “Total Exempt”. Sapphire Bank has the following income
Under current tax rules, the term Gains from Dealings in properties in 2014:
“compensation income” technically
Interest income from loans final income tax except when they are Liquidating dividend is not income. The
P3,000,000 active by nature. Active royalty income liquidating dividend are considered an
Interest income from deposits with and royalties earned from sources amount in exchange for the investment
other banks 400,000 outside the Philippines are subject to of the investor and are subject to the
Interest income from notes regular income tax. rules of dealings in properties in
rediscounting 100,000 Illustration 1 Chapter 12.
Interest income from Treasury notes Forressoftware is a distributor of a Illustration
50,000 computer program and earns royalties Caloocan Corporation, a domestic
Only the interest income from loans and from its licensed users. Computer corporation, received cash dividends
notes rediscounting are items of gross programs are specifically tailored to from the following corporations:
income subject to regular income tax. each client and regular continuing Domestic corporations P400,000
The interest on deposits and treasury maintenance services are provided. Resident foreign corporations 200,000
notes are items of gross income subject During the year, client-users remitted a Non-resident foreign corporations
to final tax. total of P500,000 royalty payments. 300,000
Rents The entire P500,000 is subject to The P400,000 inter-corporate dividends
Rent income arises from leasing regular income tax since the royalty is declared by a domestic corporation is
properties of any kind. It is a passive an active income to Forresofware. exempted from final tax. Therefore, it is
income but is not subject to final tax Illustration 2 not an item of gross income subject to
under the NIRC; hence, it is subject to Mang Damian has the following regular income tax.
regular income tax. royalties: The P500,000 total dividends from the
Special consideration on rent Royalties from mining properties in the resident and non-resident foreign
1.Obligations of the lessor that are Philippines P550,000 corporations are items of regular
assumed by the lessee are additional Royalties from books published in the income subject to regular income tax
rental income to the lessor. Philippines 200,000 and shall be reported as follows:
2.Advance rentals are Royalties from books published abroad Scenario 1:
a.Items of gross income upon receipt if: 300,000 Assuming Caloocan Corporation is a
i.Unrestrected or Royalties from franchise exercised domestic corporation, the P500,000
ii.Restricted to be applied in future abroad 400,000 total dividend from foreign corporations
years upon the termination of the lease The royalties from mining properties shall be included in gross income
b.Not an item of gross income is: and from books in the Philippines are because domestic corporations are
i.It constitutes a loan subject to final tax. The royalties from taxable on world income.
ii.It is a security deposit to guarantee sources abroad aggregating P700,000 Scenario 2:iAssuming Caloocan
payment or rent subject to contingency are items of gross income subject to Corporation is a resident foreign
which may or may not happen. regular income tax. Remember that the corporation, only a portion of the
3.Leasehold improvements made by the final withholding tax does not apply to P200,000 dividends from the resident
lessee on the leased property are foreign income. corporation determined as earned
recognized by the lessor as income dividends within by the pre-dominance test
using the spread-out method or This pertains to dividends declared by discussed in chapter 3 shall be included
outright method discussed in Chapter 4. foreign corporations. It should be in gross income. Note that the situs of
Illustration recalled that dividends declared by dividends from the non-resident foreign
Under the Iba Leasing Corporation’s domestic corporations are generally corporation is abroad.
standard lease contract, leases shall run subject to 10% final tax if recipient is an Annuities
for a non-preterminable 12-month individual taxpayer and exempt if the The excess of annuity payments
period for a monthly rental of P25,000. recipient is a domestic or a resident received by the recipient over premium
The lessee shall pay three months foreign corporation. Cash, property, and paid is taxable income in the year of
rental in advance plus one month script dividends from foreign receipt.
security deposit. The rent for the last corporations are items of gross income Illustration
two months of the lease shall be taken subject to regular income tax Andrew purchased an annuity contract
from the advance while the security stock dividends for P100,000 which shall pay him
deposit will be returned if there are no It should be recalled that stock P10,000 annually until he does.
damages sustained by the property dividends are exempt from income tax, The receipt of the first 10 annual
during the lease term. but when the declaration confers to the annuity payments is a return of capital.
The entire P75,000 rental payments for recipient a different interest of right Any further receipt from year 11
the current month and the advanced after the stock dividend declaration or onwards is an item of gross income
rental for the last two months is an when stocks dividend are subsequently subject to regular income tax.
item of income subject to regular tax. redeemed such that it amounts to Prizes and winnings
The P25,000 security deposit is not an payment of cash dividend, the fair Prizes and winnings that are exempted
income. market value of the stock dividends from final tax are not items of gross
Royalties received is taxable. income subject to regular income tax.
Royalties earned from sources within Liquidating dividends Exempt prizes and winnings:
the Philippines are generally subject to
1.Prizes received without effort to join a share in the net income of the general NIRC, the regular income tax has a
contest professional partnership. catch-all provision for all income
2.Prizes in athletic competitions For this purpose, the net income of the derived from whatever sources that are:
sanctioned by their respective national general professional partnership shall 1.not subject to final tax, capital gains
sports association include items of income which are tax, and special tax regime, and
3.Winnings from PCSO or lotto exempted from final tax or capital gains 2.not excluded or exempted by law,
The following table summarizes the tax to the general professional treaty, or contract from taxation.
rules of taxable prizes and winnings to partnership. OTHER SOURCES OF GROSS INCOME
individual taxpayers: For this purpose, the net income of the
Earned from general professional partnership shall 1.Income distributions from taxable
sources include items of gross income which are estates or trusts
Within Abroad exempted from final tax or capital gains 2.Share from the net income of other
Prizes: tax to the general professional pass-through entities:
P10,000 and below Regular tax partnership. a.Exempt joint venture
Regular tax Illustration b.Exempt co-ownership
More than P10,000 Final tax Regular Zef and Siegfried practice their 3.Farming income
tax profession in a general professional 4.Recovery of past deductions
Winnings other than PCSO and lotto partnership and share profits 60:40. 5.Reimbursement of expenses
Final tax Regular tax Their firm reported the following: 6.Cancellation of indebtedness for a
Note: PCSO and lotto winnings are Gross receipts P2,000,000 consideration
exempt from any income tax. However, Less:Professional expenses 1,200,000 Income Distributions from taxable
lotto winnings from abroad are items of Net income from operations P800,000 estates or trusts
gross income subject to regular tax for Interest income from banked deposits Any income distributions received by an
taxpayers taxable on global income. 20,000 heir or beneficiary from a taxable
The final taxation of prizes and winnings Distributive net income P820,000 estates or trust shall be included in his
for corporations is not contemplated in The share of the partners in the net gross income subject to regular tax,
the NIRC. Hence, the taxable prizes and income of the partnership shall be provided that such income must not
winnings of corporations are subject to computed as: have been subjected to final tax or
regular income tax. Total distribution of Zef(60%Xo820,000) capital gains tax.
Illustration P492,000 Illustration:Estates
The city of Baguio held its Panagbenga Total distribution to Roman is one of several heirs to the
flower festival. During the festivities, Siegfried(40%P820,000)328,000 business estate of his father which is
Mr. Erorita, the proprietor of Mr. Sexy Distributive net income under judicial settlement. The
Body Gym, won the P500,000 second P820,000 administrator distributed the following
prize in the flower float competition. The partners shall include their income for the support of Roman:
John Hay Management Corporation respective shares in their gross income Domestic dividends P22,500
won the P600,000 first prize. subject to regular income tax. Business income 70,000
The city of Baguio shall withhold 20% Note that this rule applies to other Roman shall include in his gross income
final tax on the winnings of Mr. Erorita. pass-through entities such as: subject to regular income tax the
The prize of John Hay Management 1.Exempt joint ventures P70,000 distribution from business
Corporation shall be subjected to a 20% 2.Exempt c-ownership income. The estate shall present the
final tax but to creditable withholding Business partnership and taxable joint same amount as a deduction against its
tax. John Hay shall include the prize in venture gross income. The P22,500 dividend
its gross income subject to regular These entities are subject to corporate income shall not be reported by Roman
income tax. income tax. The distributive share of since this was already subjected to final
Pensions the partner, venture, or co-owner from tax at source.
These pertains to pensions and the net income of these entities, if Illustration:Trusts
retirement benefits that fail to meet the organized within the Philippines, is Horace received the following income
exclusion criteria and hence subject to subject to 10% final withholding tax. distributions in his capacity as
regular tax. However, if these entities are organized beneficiary to an irrevocable trust
Partner’s distributable share from the or constituted abroad, the share from designated by his grandmother:
net income of the professional their profit is subject to regular income Net capital gains on sale of domestic
partnership tax for taxpayers taxable on global stocks P9,500
It should be recalled that general income. It should be recalled again that Rental income 12,000
professional partnerships are not the final income taxation is territorial Horace shall report only the P12,000
subject to income tax(i.e., final tax, and does not apply to foreign income. rental income in his gross income
capital gains tax or regular income tax) GENERAL CRITERIA FOR ITEMS OF subject to regular income tax. The net
because they are merely viewed as GROSS INCOME gain on the sale of stocks is subject to
pass-through entities. The partners are Items of gross income subject to regular capital gains tax to the trust.
the ones subject to regular tax on their income tax are not limited to the Share from the net income of exempt
aformentioned NIRC list. Under the joint ventures and co-ownerships
The same tax treatment on recognition the year of recovery so that the Less:NOLCO application
of share in the net income of a general government will recover the tax (20,000)
professional partnerships applies to the lost from the deduction. Net income for ’16-P80K
share from the net income of exempt Tax benefit The entire P90K deduction is a
joint ventures and co-ownerships. There are two ways a taxpayer tax benefit. The taxpayer
Farming income may benefit from a deduction: benefited by the P70K reduction
Farming operations can be classified as: a.Direclty,through reduction of in 2015 taxable income plus the
1. Raise and sell operation taxable income in the year P20,000 carry-over of NOLCO.
The proceeds on the sales of deduction is made The P60,000 recovery from the
livestock or farm products is b. Indirectly, through reduction deduction in 2017 is a tax
included in gross income subject of future taxable income benefit subject to tax. The
to regular income tax. Animal through carry-over of net reportable net income in 2017
raising expenses are presented operating loss. shall be P180,000.
as items of deductions against Note: Illustration 3:With expired
gross income. 1.Under our tax laws, the excess NOLCO before recovery
2.Purchase and sell operation of deductions over gross income Suppose a tax payer deducted a
The gross profit from the sale in a taxable year is carries over P500 interest expense in 2012
(sales less cost of purchase) is as or NOLCO, Because of this, but failed to pay the same due
included in gross income. almost all prior year deductions to financial difficulty. The lender
Taxpayers may follows accrual or have tax benefit; hence, their condoned the interest in 2016.
cash basis in accounting for recovery is taxable. ’12 ‘13’14’15’16
inventories of livestock or farm 2.NOLCO will be discussed in Income before interest P100
products. The crop year basis Chapter 13-B, It is partially (P130) P80 (P160) (P70)
discussed in Chapter 4 may be discussed here because of its Less:Interest expense(500) – 0 –
used for recognizing for long- relevant to the topic at hand. 500
term crops. Illustration 1:With Net Income Net income(NOLCO)(P400)
It should be recalled that the in the year of deduction (P130) P80 (P160) P???
proceeds of crop or livestock A taxpayer incurred P60,000 bad NOLCO application (P80)
insurance constitute a taxable debts expense in 2015 out of P???
item of gross income because which P35,000 was recovered in Net income P0
they are recovery of lost profits. 2017: The interest expense saved the
RECOVERIES OF PAST 2015 2016 2017 2012 P100 pre-tax income and
DEDUCTIONS Net income P100,000 P80k the 2014 P80 net income from
When past year deductions from P120k taxation. Note than NOLCO can
gross income are subsequently (Bad debts expense)/Recoveries be deducted only against net
recovered by the taxpayer or (60K) 0 35K income in the next three years.
when accrued expense Net income after bad debts The P320 remaining NOLCO
previously deducted are expense P40K 80K ??? expired in 2015 without tax
subsequently paid as an amount The entire P60,000 deduction in benefit. The P500 interest
less than the deduction claimed, 2015 is a tax benefit to the deduction only benefited the
they should be analyzed taxpayer. Hence, the P35,000 taxpayer P180. Hence, only
whether or not they resulted in recovery from this deduction is a P180 of the P500 recovery in
tax benefit to the taxpayer. tax benefit which must be 2016 be reverted back to the
Examples of recoveries of past reverted back to gross income in 2016 gross income.
deductions: 2017. The taxable net income in Illustration 4:With operating
1.Recovery of previously 2017 shall be P155,000. loss in the year of recovery
claimed bad debts expense Illustration 2:With operating A taxpayer incurred a P90,000
2.Refund of local tax expense loss and NOLCO carry-over bad debt expense in 2015 out of
3. Refund of foreign tax before recovery which P60,000 was recovered in
previously claimed as deduction A taxpayer incurred a P90,000 2016.
4.Re-commissioning of bad debts expense in 2015 out ’15 ‘16
abandoned petroleum service of which P60,000 was Net income/(loss) before bad
contracts or mining properties recovered in 2017 debts/recovery P70K (P15K)
5.Release of reserve funds of 2015 2016 2017 (Bad debts expense)/Recovery
insurance companies Net income before bad debts (90K) 45K
6.Interest expense which were expense P70k P100K 120K Net income after bad debts
subsequently condoned by the (Bad debts expense)/Recoveries expense (P20K) P???
lender. (90K) 0 60K An increase in NOLCO which has
Past deductions that created Net income after bad debts not expired before the beginning
benefit to the taxpayer must be expense (P20,000) of the taxable year in which the
reverted back to gross income in P100,000 ?? recovery takes place shall be
treated as tax benefit. Thus, the Less:Net income as reported in 2015 2.When a client reimburses the out-of-
entire P90,000 is a tax benefit to (recovery is unknown) 0 pocket expenses of a professional
the taxpayer. Hence, the Tax benefit of the bad debts expense practioner, the reimbursements are
P45,0000 recovered out of it is a P40,000 income to the practitioner.
tax benefit which must be P40,000 out of the P60,000 recovery in CANCELLATION OF INDEBTEDNESS
reverted back to gross income in 2017 constitutes tax benefits which The cancellation of indebtedness may
2016. must be included in the 2017 gross amount to gratuity or payment of
Net Loss before recovery income. The 2017 net income shall be income.
(P15,000) P160,000. The cancellation of debt:
Add: Recovery 45,000 Illustration 6:Taxpayer is exempt in the a.In consideration of service or goods-t
Net income P30,000 year of deduction treated as income
Less:NOLCO application-2015 Kalinga Corporation is an exempt BMBE b.As an act of gratuity-treated as gift;
20,000 taxpayer in 2015 but became a taxable not as income
Taxable net income P10,000 regular income taxpayer in 2016. It c.As capital transaction such as
Illustration 5:Without benefit of deducted P120,000 bad debts expense forfeiting the right to receive dividends
NOLCO carry-over in 2015. In 2016, it recovered P40,000 in exchange of the debt-treated as
A corporate taxpayer had a out of bad debts. dividend income.
change in 80% of its 2015 2016 SPECIAL CONSIDERATIONS IN
shareholders in 2016. Thus, any Net Income before bad debt expense REPORTING OF GROSS INCOME
net operating loss incurred P70,000 P100,000 1.Accounting methods
before 2015 is not allowed to be (Bad debts expense)/Recovery 2.Situs rules
carried over. A P90,000 bad debt (120,000) 40,000 3.Effect of value added tax
write-off was made in 2015 out Net Income after bad debts expense 4.Creditable withholding tax
of which P60,000 was recovered (P50,000) P??? 5.Power of the CIR to redistribute
in 2017. Deductions have no tax benefits to a income and expenses
’15 ’16 ‘17 taxpayer who is exempt from tax.Future ACCOUNTING PERIOD
Net Income before bad debts recoveries from deductions made in the The accounting period method adopted
expense P70K P100K P120K year of exemption are non-taxable. The by the taxpayer has a direct effect on
(Bad debts P40,000 recovery is not income. The the reportable amount of gross income
expense)/Recoveries(90K) - 2016 net income shall be P100,000. subject to regular income tax.
60K Refund of non-deductible expense For instance, cash-basis taxpayers will
Net Income after bad debts Expenses or payments which are non- report their revenue consisting of
expense (P20K) 60K deductible against gross income in the collected and uncollected income as
??? computation of taxable net income will gross income.
The tax benefit of the P90,000 never create benefit to the taxpayer. As It must be recalled also that regardless
bad debts expense to the such, their recovery should not be of the accounting methods of the
corporation in this case shall be included in gross income. taxpayer, advanced income must be
determined using the As-IF Hence, the refund of the following non- included in gross income in the period
Approach deductible items is not taxable: received.
Re-compute the net income in 1.Philippine income tax SITUS RULES
the year of deduction by 2.Estate or donor’s tax The situs rules of taxation also affects
adjusting the deduction as if the 3.Income tax paid or incurred to a the extent of income included as items
subsequent deduction recovery foreign country if the taxpayer claimed of gross income of the taxpayer. It must
is known. The computed net a credit for such tax in the year it was be recalled that all taxpayers are
income is compared to what paid or incurred. taxable only on Philippine income
was previously reported to 4.Stock transaction tax in disposing except resident citizens and domestic
determine the income that is stocks through the Philippine Stocks corporations which are taxable on
saved from taxation. Exchange global income.
Assuming the future recovery is known, 5. Special assessments For taxpayers taxable only on Philippine
the 2015 net income should have been: REIMBURSEMENTS OF EXPENSES income,only their items of gross income
Net income before bad debts expense Expenses of the taxpayer that are subject to regular tax from sources
P70,000 reimbursed or paid by the customer or within the Philippines are included in
Less: Bade debts expense if recovery is client constitute additional income to gross income.
known(P90,000-P60,000) 30,000 the taxpayer. For taxpayers taxable on global income,
Net income if recovery in known Examples: their items of gross income subject to
P40,000 1.When the lessee pays the ownership regular tax from sources within and
The tax benefit is the income that costs of the lessor such as real property without the Philippines are included in
escaped taxation in 2015 computed as: tax and insurance on the property, the gross income.
Net Income if subsequent recovery is payment constitutes income to the Integrative Illustration
known P40,000 lessor. Nomisma, a finance corporation, lends
to various clients:
Interest income from loans to Philippine Illustrative Illustration 2 2.The 5%-10% capital gains tax is the
residents P400,000 A certain taxpayer had the following most universal rule in taxation that
Interest income from loans to non- details of income during the year. applies to all taxpayers regardless of
residents clients 500,000 Service fees from Philippine clients classification.
Interest income from bank deposits in P400,000 3.Inter-corporate dividends are exempt
the Philippines 20,000 Service fees from foreign clients from final tax, except when the
Interest income from bank deposits 500,000 recipient is a non-resident foreign
abroad 10,000 Gain on sale of domestic stocks directly corporation.
Required: to a buyer 150,000 EFFECTS OF VALUE ADDED TAX
Determine the total amount of gross Dividends from domestic corporations Under the NIRC, taxpayers who are
income subject to final tax and the 5,000 engaged in business or exercise of a
reportable amount of gross income Interest income on bank deposits profession are required to register as
subject to regular income tax assuming abroad 30,000 business taxpayers.
the taxpayer is a: Required: Business taxpayers are either:
1.Non-resident foreign corporation Determine the amount of gross income a.VAT taxpayers-these are usually large
2.Resident foreign corporation subject to regular income tax, final tax taxpayers with sales or receipts
3.Domestic corporation and capital gains tax assuming that the exceeding P1,915, 500 in any 12-month
Solution: taxpayer is: period.
An analysis of the situs of the above 1.an individual b.Non-vat taxpayers-these are usually
income is shown below: a. Non-resident alien not engaged in smaller taxpayer below the VAT
Within Without trade or business (NRA-NETB) threshold and those designated by the
Loan interest income P400,000 b.Non-resident alien engaged in trade law to pay percentage taxes.
P500,000 or business(NRA-ETB), a resident alien Every VAT taxpayers is mandatorily
Bank interest income 20,000 (RA) or a non-resident citizen(NRC) required to charge 12% output tax on
10,000 c. Resident citizen(RC) their sales or receipts. The regulations
Total P420,000 2.a corporation presume that the amount charged to
P510,000 a. Non-resident foreign corporation customers is inclusive of the 12% VAT.
The following are the amounts subject b.Resident foreign corporation The output VAT will be paid to the
to final tax and the amounts to be c.Domestic corporation government net of VAT paid by the
reported in gross income subject to Solution: taxpayer (input VAT) on his purchases.
regular income tax: An analysis of the situs of the foregoing As such, the amount of reportable gross
Subject to income is as follows: income shall not include the output
Final tax Regular Within Without VAT.
tax Service fees P400,000 P500,000 Illustration 1:VAT taxpayers
1.Non-resident foreign corporation Gain on sale of domestic stocks 150,000 A VAT-registered taxpayer charged
P420,000 P0 Domestic dividends 5,000 P78,400 to a client for rental.
2.Resident foreign corporation 20,000 Interest income from foreign bank – The VAT-taxpayer shall split the billing
400,000 30,000 as follows:
3.Domestic corporation 20,000 Total P555,000 Rental income (P78400/112%) P70,000
910,000(420,000+510,000) P530,000 Plus:Output VAT (P78,400X12/112)
Note: 8,400
1.Non-resident foreign corporations are The following are the amounts to be Invoice price P78,400
subject to final income tax on gross included in gross income subject to final Only the rental income is subject to
income within Philippine residents shall income tax(FIT), capital gains tax(CGT), income tax. The output VAT shall be
withhold 30% final tax on their gross and regular income tax(RIT): recorded as a liability. The collection of
income. Taxpayers FIT CGT RIT the rental is recorded in accounting as
2. The interest income from banks in Individuals follows:
the Philippines is an item of gross 1.NRA-NETB P405,000 P150,000 P- Cash P78,400
income subject to 20% final tax. The 2.NRA-ETB, RA, or NRF 5,000 150,000 Rent Income 70,000
gross income subject to regular income 400,000 Output vat 8,400
tax to resident foreign corporations 3.RC 5,000 150,000 930,00 Illustration 2:Non-VAT taxpayers
includes only those earned from Corporations A non-VAT taxpayer charged P78,400 to
sources within 4.NRFC P405,000 P150,000 P- a client for rental.
3.Income from sources abroad, passive, 5.RFC - 150,000 Non-VAT taxpayers are not subject to
or active, are subject to regular income 400,000 VAT. The entire amount they charge for
tax for taxpayers subject to tax on 6.DC - 150,000 their sales of goods or services is gross
global income. The gross income of 930,000 income subject to income tax.
domestic corporations includes items of Note: CREDITABLE WITHHOLDING TAX
gross income subject to regular tax 1.NRFCs and NRA-NETBs are subject to Creditable withholding taxes(CWT)
from sources within and outside the final tax on Philippine income. deducted by income taxpayors against
Philippines. Thus, P400k+P500k+P10K. the gross income of the taxpayer are
not exclusions in gross income. These withheld tax by the client evidenced by Philippines while the entire gross
should be added back to the reportable a BIR From 2307. income will be recognized abroad
amount of gross income. CWTs are tax The VAT taxpayer shall compute his where no tax is imposed.
credits that are deductible against the service income subject to regular tax as 2.A foreign corporation subject to 10%
annual income tax due of the taxpayer. follows: corporate tax in its home country has a
Illustration Cash received P55,000 branch in the Philippines which is
Denzo, Inc., a non-VAT domestic Plus:Withholding tax(BIR From 2307) subject to the 30% corporate income
corporation, reported the following: 1,000 tax herein. The foreign corporation
Rent income, net of 5% or P25,000 CWT Invoice price (inclusive of VAT) transfers goods at a pricing method that
P475,000 P56,000 will allow very minimal profit for the
Professional fees, net of 10% of P40,000 Less:Output VAT (P56,000X12/112) Philippine branch to minimize exposure
CWT 360,000 6,000 to higher income tax.
Interest income, net of 20% final tax Service fees(Gross income) 3. Mr. Wais has a business enjoying a
40,000 P50,000 tax holiday under an investment
Dividends from a domestic corporation Illustration 2 promotion law. Mr. Wais also has a
10,000 A VAT taxpayer collected P85,600 rental business that is subject to regular
Business expenses 500,000 from a lessee who withheld 5% income tax. Mr. Wais orders his taxable
Required: creditable withholding tax. business to sell goods and supplies at
Determine the total reportable gross The VAT taxpayer shall compute his rent cost to his exempt business thereby
income and the income tax due and still income subject to regular tax as follows: shifting profits to the exempt business
due under the regular income tax. Cash received(inclusive of 12% but net to save from income tax.
The total reportable gross income shall of 5% CWT) P85,600 The transfer pricing guideline
be: Divide by: (100%+12%-5%) 107% Those enumerated scenarios are just a
Rent Rent income(Gross income) few of the problems in taxation brought
income[P475K/95%or(P475K+P25K)] P80,000 about by unfair pricing practices. To
P500,000 The VAT and withholding tax are as limit these unfair practices and to
Professional follows: properly reflect the income of
fees[P360K/90%or(P360K+P40K) Rent income P80,000 associated enterprises, the BIR and the
400,000 Plus:Output VAT (P80,000X12%) 9,600 Department of Finance promulgated
Total Gross income P900,000 Less:Withholding tax(P80,000x5%) Revenue Regulations No. 2 series of
Note:The interest income and the 4,000 2013 (RRS-2013) on transfer pricing.
dividend income are subject to final tax. Cash payments to the lessor P85,600 What are associated enterprises?
They should not be included in the item POWER OF THE CIR TO REDISTRIBUTE Under RR2-2013, two or more
of gross income subject to regular INCOME AND DEDCUTIONS enterprises are associated if one
income tax. In the case of two or more participates directly or indirectly in the
The income tax due and still due shall organizations, trades or management, control, or capital of the
be determined as: businesses(whether or not incorporated other;or if the same persons participate
Total gross income P900,000 and whether or not organized in the directly or indirectly in the
Less: Allowable deductions(business Philippines) owned or controlled management, control, or capital of the
expenses) 500,000 directly or indirectly by the same enterprises. Associated enterprises are
Taxable net income P400,000 interests, the Commissioner is also called”related parties”.
Multiply:Corporate tax rate 30% authorized to distribute, apportion, or Examples of associated enterprises:
Income tax due P120,000 allocate gross income or deductions 1.Parent corporation and its subsidiary
Less:Creditable withholding tax between or among such organizations, corporation
-Rent P25,000 trade or business, if he determined that 2.Sister companies or businesses
-Professional fess 40,000 65,000 such distributions, apportionment or owned by the same parent corporation
Income tax still due allocation is necessary in order to 3.All corporations controlled under the
P55,000 prevent evasion of taxes or clearly to same holding company
Note: reflect the income of any such 4.Businessess owned by the same
1.The CWTs are actually advances to the organization, trade or business. (Sec. person
annual income tax due of corporations 50,NIRC) The arm’s length principle
and individuals and are deductible(i.e., Examples: Under RR2-2013, transfer pricing
creditable)thereto. Same procedures 1.A domestic corporation which is between associated enterprises shall be
are applied with individuals except that subject to 30% corporate tax in the made under comparable conditions and
their income tax dues are determined Philippines has a subsidiary that circumstances as those entered into
by progressive tax rates. operates in a tax haven country where between independent parties where
2. The final taxes should not be credited no income tax is imposed. The domestic market forces drive the terms and
against the annual income tax due. corporation transfers goods to its conditions of the transaction rather
Creditable withholding tax and VAT foreign subsidiary at a transfer pricing than being controlled solely by reason
VAT taxpayers shall revert back to gross based on production cost so that no of special relationship between the
income amounts of withholding tax but gross income will be recognized in the associated enterprises.
In other words, an uncontrolled pricing expected to realize from engaging in the Applying the APA, the gross income
method determined by free market transaction or transactions. earned from within and outside the
forces also called arms’s length pricing, a.Residual profit split approach-Profit Philippines shall be computed as:
is preferred. The failure to comply may is first allocated to provide a basic Within Outside Total
expose the taxpayer to a transfer return appropriate for the type of Sales through the branch(intra-
pricing adjustment where the BIR re- transaction the participant is engaged company sales):
computes the proper income of the in. The residual profit after such Sales *P10,500,000 P12,000,000
associated enterprises. allocation is further allocated among P12,000,000
The arm’s length principle shall be the parties based on an analysis of how Less:Cost of sales 7,000,000 10,500,000
applied to: the residual would have been divided 7,000,000
1.Cross-border transactions between between independent parties. Gross income P3,500,000 P1,500,000
associated enterprises b.Contribution profit split approach-The P5,000,000
2.Domestic transactions between combined profits from controlled
associated enterprises transactions are divided between Sales to unassociated domestic
When operations are conducted cross- associated enterprises in a single stage enterprises:
boarder, the taxpayer may enter into an based upon the parties’ relative Sales P3,000,000 P0
“advanced pricing agreement” with contribution to the profit or the relative P3,000,000
the BIR where a pricing rate is pre- value of the functions performed by Less:cost of sales 1,200,000 0
agreed to apply for a period of time. each of the associated enterprises 1,200,000
Although this is not a mandatory participating in the controlled Gross income P1,800,000 P0
requirement, this may serve as a safety transactions. P1,800,000
net for the taxpayer to avoid the risk of 5.Transactional net margin Total gross income P5,300,000
transfer pricing examination and method(TNMM)-This is similar to the P1,500,000 P6,800,000
adjustment and the inconvenience it cost plus and the resale price methods Note:
may possible cause. in the sense that it uses the margin 1.The total income on the P12,000,000
Transfer pricing methods approach by reference to the operating sales is split between the Philippines
When the pricing methods between profit earned in comparable gross income and foreign gross income
associated enterprises do not reflect uncontrolled transactions. using the APA rate. The Philippine sales
arm’s length pricing, the BIR will adjust When no comparatives can be derived shall be measured as:P7,000,000 cost of
the controlled transactions to their within the industry of the subject goods transferred and
arm’s length values using the most taxpayer, the BIR may consider; soldx150%=P10,500,000.The amount
appropriate of the following method a.Extension of the transfer pricing determined as Philippines sales shall be
considering the circumstance of the methods using comparatives derived the deemed cost of sales of the foreign
taxpayer: from another industry segment branch*
1.Comparable uncontrolled price(CUP) b.Use a combination of the transfer 2.Transfer pricing rule does not apply to
method-The transaction is valued in pricing methods or other methods. transactions with unrelated parties.
reference to the amount charged in a Illustration 1:With an advanced pricing Scenario 1:Crosby is a domestic
comparable uncontrolled transaction agreement corporation
under comparable circumstances. The Crosby Manufacturing Corporation Crosby shall report P6,800,000 gross
This method works best for standard usually bills its foreign branch at income since Crosby is taxable on global
tangible goods sold in an open market. cost;however, it entered into an income.Note that the transfer pricing
It does not apply to products containing advanced pricing agreement (APA) with rule is only important in the
unique characteristics such as those the BIR which fixed its cross-border measurement of the proper income
patented products or those containing pricing to its foreign branch at 150% of from foreign sources for purposes of
trade secrets. cost. the computation of the foreign tax
2.Resale price method(RPM) –The Crosby compiled the following costs credit.
transaction is valued based on the and sales during the year: Scenario 2:Crosby is a domestic
functions performed by the reselling Philippines Branch corporation, and the foreign operation
party to the product. This is used when Sales through the branch (intra- is not a branch but a foreign subsidiary
products purchased from a related company sales): incorporated abroad.
party are resold to an independent Sales P7,000,000 P12,000,000 Crosby shall report P5,300,000 gross
party. Less:cost of sales 7,000,000 7,000,000 income. The income of the foreign
3.Cost plus method(CPM)-The Gross income P0 P5,000,000 subsidiary is not taxable in the
transaction is measured by valuing the Sales to unassociated domestic Philippines as it is a non-resident
function performed by the supplier of enterprises: foreign corporation. Note the
the property or services. Sales P3,000,000 importance of the transfer pricing rule
4.Profit split method(PSM)-The profit Less:Cost of sales 1,200,000 in measuring the fair and proper
or loss on the transaction is split based Gross Income P1,800,000 amount of the reportable gross income
on the division of profits(or losses) that Total gross income P5,300,000 from within. The profits on the sales of
independent enterprises would have Crosby to the foreign affiliates could
have escaped taxation without this rule.
The same procedures in this scenario The amount of all items of gross income 8.Items of passive royalty income are
will be applied if Crosby is a resident shall be included in the gross income subject to regular income tax while
foreign corporation. for the taxable year in which received items of active royalty income are
Illustration 2:Without an advanced by the taxpayer, unless, under methods subject to regular income tax.True
pricing agreement of accounting permitted, any such 9.Compensation income is an inclusion
Hot Corporation, a foreign corporation, amounts are to be properly accounted in gross income subject to regular tax
sells cosmetics products in the for as of a different period. except compensation income of special
Philippines through its branch. The CHAPTER 9:SELF-TEST EXERCISES aliens.True
Philippine branch filed an income tax Discussion Questions 10.The reportable gross income from
return reflecting the following gross 1.Enumerate the NIRC list of items of business or the exercise of a profession
income: gross income. is net of the cost of goods sold or cost
Sales P4,000,000 2.What are the broad categories of of services. True
Less:Cost of goods sold 3,500,000 gross income 11.Items of income which are included
Gross income P500,000 3.Discuss in detail the taxation of in gross income subject to final tax are
Scenario 1:The BIR determined that interest income. Which is subject to excluded in gross income subject to
Hot Corporation is billing its branch at final tax?Which is subject to regular regular income tax.True
200% of cost. Meanwhile, other income tax? 12.Imputed interest income is an item
competitors of Hot Corporation which 4.Discuss the treatment of gains from of gross income subject to regular
offer similar product lines are billing dealings in properties, Which gains are income tax. False
their Philippine independent subject to capital gains tax?Which gains 13.Advanced rentals are income in the
distributors at an average of 150% of are subject to regular income taxation year received.True
cost. 5.Discuss the taxation of dividends 14.Real property tax and insurance on
The gross income of the Hot 6.Discuss the taxation of royalties the property if assumed by the lessee
Corporation for purposes of Philippine 7.Discuss the taxation of prizes and constitute income to the lessor.True
income tax shall be restated by re- winnings 15.Corporate winnings are exclusions in
measuring the cost of goods sold as 8.Compare actual distribution and the gross income;hence, they are exempt
follows: share in the net income of the from income tax.False
Cost of sales at 200% billing partnership.Which one is included in 16. Stock dividend are never subject
rate(i.e.200% xCost) P3,500,000 the gross income of the partner? income tax.False
Cost to the selling party 9. Discuss the taxability of recoveries of 17.Pensions or retirement benefits are
(P3,500,000/200%) 1,750,000 past deductions inclusions in gross income subject to
Cost at arms’ length price 10. Enumerate examples of pass- regular income tax if the employee is
(150%XP1,750,000) 2,625,000 through entities. Are they taxable to terminated due to any cause within his
Thus, the gross income of Hot final tax, capital gains tax, or regular control.True.
Corporation shall be treated as follows: income tax? 18.Prizes in athletic competitions
Sales P4,000,000 11. Enumerate and discuss the transfer sanctioned by Philippine government
Less:Cost of goods sold(arm’s length pricing methods? are exclusions in gross income subject
cost) 2,625,000 True or False 1 to final tax, but are inclusions in gross
Gross income P1,375,000 1.Items of gross income subject to income subject to regular income tax.
Note:Hot corporation will be given an regular income tax and capital gains tax False
assessment by the BIR for the are reportable to the government-True 19.Corporate prizes are exclusions in
incremental tax plus penalties that may 2.Rent is a passive income, but is not gross income subject to final tax but are
be due from this restatement. subject to final tax.-True inclusions in gross income subject to
Scenario 2:Hot Corporation bills its 3.The interest income from bonds regular income tax.True
branch at established market prices issued by banks is subject to final tax.- 20. Stock splits are never subject to
Since the transfer price of Hot False income tax.True
Corporation reflect’s arms’ length 4.Gains from dealings in capital assets True or false 2
pricing, no transfer pricing adjustment are generally subject to the regular 1.The distributable net income of a
shall be made. income tax-True general professional partnership is
Selection of Transfer Pricing method 5.The gross income from operations subject to creditable withholding
To minimize the risks of transfer pricing enjoying tax holiday are included in tax.True
adjustments, taxpayers may also gross income subject to regular tax, but 2.Exempt joint ventures and co-
consider using the transfer pricing are presented as deductions in the ownerships are treated as pass-through
methods used by the BIR in pricing their income tax return.False entities and are subject to income
transactions with associated 6.The share in a business partnership is tax.False
enterprises. The taxpayer must support subject to final tax, but the share is a 3.The distribution by the GPP of items
the propriety of the method adopted general professional partnership is of passive income is an inclusion in
through proper documentation. subject to regular income tax.True gross income of the partner subject to
PERIOD IN WHICH OF GROSS INCOME 7.Gains from dealings in ordinary assets regular income tax.True
ARE INCLUDED are subject to regular income tax.True
4.General professional partnerships are 22. The taxpayer must enter into an b.Interest income from long-term bank
exempt from tax and hence, exempt advanced pricing agreement with the deposits
from withholding.True BIR for its cross-border transfer pricing c.Ordinary gain on sale of properties
5.The share from the net income of a with associated enterprises.False d.Interest on notes receivables
joint venture organized abroad is 23. Transfer pricing between associated 6.Which is an income exempt from
subject to 10% final withholding enterprises must be made at arm’s income tax?
tax.False length.True a.Income of a general professional
6.Income distribution from taxable 24. The transfer pricing regulations partnership
estates and trusts is an inclusion in apply only to cross-border transfers of b.Foreign dividends
gross income subject to regular tax by goods and services between associated c.Taxes collected by the government
the heir or beneficiary.True enterprises.False d.Income of government-owned and
7.The recovery of past deduction must 25. Corporations under the direct and controlled corporations
be reverted back to gross income of indirect control of the same controlling 7.All of these are subject to regular
taxpayers using the accrual basis.True individual or corporation are associated income tax. except
8. The recovery of bad debts need not enterprises.False a.Professional fees
be reverted to gross income of 26. Under the accrual basis of b.Wages and commissions
taxpayers using the cash basis.False accounting, items of gross income are c.Business income
9. The recovery of deduction from an reported in the period they are d.Capital gain from the sale of real
exempt year is subject to tax.False received.False property located in the Philippines
10. General professional partnerships 27. Basically, transfer pricing 8. Which is exempt from regular tax?
are not exempt from regular tax but are adjustment is needed when the income a.Income from construction
subject to final tax and capital gains reported for Philippine taxation is b.Income of qualified pension plans
tax.False underrated.True. c.Income from merchandising or trading
11. An indebtedness cancelled by the Multiple Choice-Theory:Part 1 d.Income from financing or leasing
creditor out of mercy is an income to 1.What is not subject to income tax? 9. Which item of gross income is not
the debtor.False a.Gain from sale of shares in mutual subject to regular tax?
12. When there is a net loss in the funds by the investor a.Interest income from foreign bank
period the deduction is taken, the b.Prizes in recognition of civic, religious, deposits
subsequent recovery of the deduction and artistic achievements b.Capital gains on the sale of binds
will not have any tax benefit.False c.PCSO and lotto winnings with more than 5 years maturity
13.The refund or recovery of non- d.All of these c.Gain on sale of domestic stocks by a
deductible taxes shall not be reverted 2.Which is an item of gross income security dealer
back to gross income.False subject to regular tax? d.Capital gain on sale of patent
14. The loss of the partnership can be a.Gain on sale of lot by a realty dealer 10.Which is not part of compensation
claimed by the partners as deduction in b.Interest income from bank deposits income subject to regular tax?
their income tax returns.False c.Passive royalty income a.Director’s fees
15. The accounting period of the d.Capital gain on the sale of domestic b.Bonuses and fixed allowances
taxpayer has a direct impact upon the stocks c.Portion of salary contributed to SSS
amount of gross income to be 3.Which is not an item of gross income d.Portion of salary used to pay salary
reported.False subject to final tax? loans
16. The power of the CIR to redistribute a.Dividends from a domestic 11.Which is included in the gross
income and expense includes the corporation income subject to regular tax of a
power to impute income between b.Prizes in excess of P10,000 by an resident alien?
affiliated enterprises.False individual a.Gross income from the sale of goods
17. The situs of taxation has an impact c.Share in the income of a general abroad
on the extent of the reportable gross professional partnership b.Interest income from promissory
income.True d.Winnings notes of resident clients
18. Creditable withholding taxes are 4.Which of these employee benefits is c.Interest income from relatives abroad
added back to the amount of subject to final tax? d.Gain from sale of domestic stocks
reportable gross income.True a.Fringe benefits to rank and file directly to a buyer
19. The output VAT must be included as employees 12. Which of the following deductions
part of gross income of VAT b.Regular pay of supervisory and from gross compensation income is
taxpayers.False managerial employees included as part of gross income subject
20. The requirement to revert back to c,Fringe benefits to supervisory and to regular tax?
gross income the amount of withheld managerial employees a.Pag-Ibig Contributions
taxes applies only to VAT d.Regular pay of rank and file b.Contributions to union
taxpayers.False employees c.PhilHealth Contributions
21. Generally, all items of income of 5.All of these are items of gross income d.dues Withholding tax
NRA-NETB and NRFCs from the subject to regular tax except one. Select 13. Which interest income is subject to
Philippines are inclusions in gross the exception. regular tax?
income subject to final tax.True a.Compensation income a.Interest income from notes
b.Interest income from bonds Multiple Choice-Theory:Part 2 a.fiscal year
c.Interst income from lending 1.Which of the following will not be b.a calendar year
d.All of these reported in gross income? c.either a fiscal or calendar year
14. Dividends subject to regular tax a.Receipt of inheritance 9. Corporaions are allowed to report
includes b.Share in the net income of a foreign their income on
a.Foreign dividends partnership a.a fiscal year
b.Domestic dividends c.Royalties from foreign sources b.a calendar year
c.Both a and b d.Income distributions from a taxable c.either a fiscal year or calendar year
d.Neither a nor b estate 10. Which is subject to progressive tax
15.Statement 1:All prizes earned abroad 2.Which is not a reportable type of to an individual taxpayer?
are subject to regular tax. gross income? a.Proceeds of life insurance policy
Statement 2:All prizes in the Philippines a.Passive royalties b.Gift
are subject to final tax. b.Dividends from a foreign corporations c.Amounts received by the insured in
Which statement is generally correct? c.Rent income excess of premiums paid
a.Statement 1 d.Capital gains from the sale of d.Compensation for personal injuries
b.Statement 2 domestic stocks through the PSE 11.Which of these is subject to
c.Both statements 1 and 2 3.Which of the following is included in Philippine regular income tax to a
d. Neither statements is true gross income subject to regular tax? foreigner?
16.Which is an item of gross income a.Sub-contract income from petroleum a.Rent income on properties located
subject to regular tax? service contractors abroad
a.Lottery winnings abroad b.Interest income from government b.Dividend income from a domestic
b. Imputed interest income securities corporation
c.Advanced rent representing security c.Share in the net income of a taxable c.Interest income on a deposit abroad
deposit for contingency which may or co-ownership d.Interest income from domestic
not may not happen d.Farming income bonds
d.Leasehold improvement with useful 4.All income earned abroad that would 12. If not covered by the substantiated
life not extending beyond the lease otherwise be subject to final taxes if filing system, employed individual
term earned within the Philippines shall be taxpayers shall report their regular
17. Which of the following is not subject to progressive tax of a income
subject to regular tax of a domestic a.domestic corporation a.monthly
corporation or resident citizen? b.resident citizen b.annually
a.Deposit interest income from abroad c.resident alien c.quarterly
b.Prize not exceeding P10,000 from the d.all taxpayers .quarterly and annually
Philippines 5.All items if passive income earned 13.Corporations and individuals
c.Income from abroad exempt under abroad are subject to regular tax to engaged in business or in the exercise
treaty a.resident citizen only of a profession are required to report
d.Royalties from abroad b.a domestic corporation only their regular income
18.Which is subject to regular tax to a c.a resident citizens and domestic a.monthly
non-resident foreign corporation or corporations b.quarterly
non-resident alien not engaged in trade d.all taxpayers c.annually
or business? 6. All itmes of passive income earned d.quarterly and annually
a.Business income from the Philippines abroad are subject to regular tax to 14.Which is incorrect concerning
b.Capital gain from the sale of stocks a.a resident citizen only transactions between associated
directly to a buyer in the Philippines b.a domestic corporation only enterprises?
c.Dividends from domestic corporations c.a resident citizens and domestic a.Transactions between related parties
d.None of these corporations should not be controlled
19.Which is subject to regular tax to a d.all taxpayers b.Pricing should be determined by free
resident foreign corporation? 7. Which individual taxpayer is not market forces
a.Service fees abroad subject to progressive tax? c.Pricing should be motivated by the
b.Gain from sale of real property a.NRA-ETB need to save from total income tax
capital assets in the Philippines b.Special aliens d.Non-arms’ length between related
c.Dividends from a domestic c.Resident alien parties may be restated by the BIR to
corporation d.Resident citizen reflect the arms’ length value of
d.Gain from dealings in properties 8.Which corporate taxpayer is not transactions
abroad subject to regular tax? 15.Which is not an associated
20. The proceeds of life insurance a.Domestic corporation enterprise to the controlling individual
received by the wife of the insured is b.resident corporation of a holding company?
a.exempt from income tax c.Business partnership a.An associate of a subsidiary in the
b.subject to final tax d.Non-resident foreign corporation group
c.part of taxable income 8.Individual taxpayers shal report b. The parent company
d.partly exempt and partly taxable income on c.A direct subsidiary company
d.A subsidiary of a subsidiary in the Interest income, net of final tax 10,000 Advances for future services to be
group Dividend income 50,000 rendered P30,000
Multiple choice-Problems:Part 1 Total Income Collections for past services
1.Mr Lolong, a supervisory employee, P1,780,000 rendered 70,000
received the following income in 2015: Less:Admin and Selling expenses Reimbursements for client expenses
Gross compensation income, before 1,000,000 40,000
contributions to SSS, PhilHealth, and Net Income P780,000 Reimbursements for out-of-pocket
HDMF totaling P124,000 Compute the total gross income subject expenses 10,000
P800,000 to regular tax. How much will be included in
Fringe benefits 200,000 a.P1,720,000-until interest income Croki’s gross income for regular
Gain from redemption of shares in a from customers income tax purposes?
mutual fund P100,000 b.P1,770,000 a.P150,000
Commission income 150,000 c.P1,780,000 b.P140,000
Gain on sale through the PSE 400,000 d.P840,000 c.P110,000-
Determine the total income to be 5.Precy, Inc., a domestic corporation, d.P100,000
reported by Mr. Lolong in gross income. reported the following income in 2014: all are subject to gross income for
a.P1,526,000 Philippines Abroad regular income tax purposes
b.P1,426,000 Service fees P400,000 P300,000 9.The Big bird Security Agency
c.P1,026,000 Interest income-bank 40,000 70,000 (BBSA) received P3,000,000 from its
d.P826,000 Royalties-franchise 80,000 30,000 clients. P2,400,000 of this was
2.Calixto, employed, derived the Compute the total gross income subject designated for salaries of guards
following income during the year: to regular income tax. assigned to various client
Gross salaries P400,000 a.P920,000 establishments.
th
13 month pay and other benefits b.P860,000 How much will be included in the
40,000 c.P800,000- gross income of BBSA?
SSS, PhilHealth, and Pag-Ibig d.P700,000 a.P600,000
contributions 20,000 Service fees from ph of P400,000 add b.P2,400,000
Deductions for loans repayments interest income from abroad and c.P3,000,000
50,000 royalties from abroad regular tax. d.P0
Deductions for withholding tax 60,000 6. If Precy, Inc., is a resident 10.Farmers, Inc., purchased an
Compute the compensation income to foreign corporation, compute agricultural lot for P1,000,000. It
be reported in the annual income tax the gross income subject to was later discovered that the lot
return. regular tax. had gold deposits. Thus fair value
a.P440,000 a.P520,000 increased to P4,000,000. This
b.P330,000 b.P480,000 increase in fair value is
C.P380,000 c.P400,000- a.exempt from income tax
d.P390,000 add the withholding tax d.P440,000 b.subject to income tax
3.Corazon company resigned in 2015 only in Philippines is subject to c.partially exempt and partially
after 12 years of service. She had the regular tax when earn in Ph.of taxable
following income during the year: service fees d.Any of these
Salary, net of P80,000 withholding tax, 7. Andres leases a building to a 11.Northern Cattle Company
P20,000 SSS, P18,000 Philhealth and client. During the year, he produces beef meat. In 2015, it
P40,000 13th month pay P480,000 received the following reported the following:
Separation pay 1,000,000 remittance from the lessee: Sale of live cattle P600,000
Compute the gross income subject to Rental, net of 5% creditable Sale of young feeders 200,000
progressive (regular) tax. withholding tax P1,900,000 Increase in value of cattle inventory
a.P1,480,000 Real property tax for leased building 300,000
b.P1,560,000 50,000 Compute the income subject to
c.P560,000 Reimbursement for utilities used by regular tax
d.P480,000 the lessee paid by Andres 200,000 a.P1,100,000
add the salary and separation pay plus How much will be included in gross b.P800,000
wt for subject to tax income subject to regular tax? c.P900,000
4.Iriga Corporation is engaged in the a.P2,050,000 all are subject to d.P600,000
sales of goods. It reported the following regular tax.- 12.Don Juanito has the following
summarized financial statements during b.P2,250,000 income in 2015:
the year: c.P2,000,000 Sales from vegetables P400,000
Sales P3,500,000 d.P1,950,000 Sales from fruits
Less:Cost of sales 2,000,000 8.Mr. Croki, a professional 200,000
Gross profit P1,500,000 practitioner, received the following Sales from carabao(acquired for
Commission on consignment 200,000 from his clients: P30,000 in 2015) 35,000
Interest income from customers 20,000
Interest income from tenants on the operating loss of NCC to P50,000. b.P4,000
sale of agricultural land pursuant to NCC was very profitable from 2013 c.P10,000
the Agrarian Reform program to the present. How much of the d.P0
12,000 P40,000 recovery is subject to tax? 22. An accrual basis taxpayer recovered
The gross income subject to a.P60,000 a P20,000 local tax expense which was
progressive tax is or regular b.P10,000 refunded by the local government in
a.P647,000 c.P40,000 2015. The local tax expense was paid in
b.P617,000 d.P0 2013 when the taxpayer sustained a
c.P600,000 18. West oil abandoned an oil P5,000 net operating loss. How much
d.P605,000- facility in 2012 and expensed the shall be reverted to income?
P400,000+200,000+5k=605k P300,000 unrecovered investment a.P0
13.Mr. Conner purchased a life in the facility as abandonment loss. b.P5,000
annuity for P1,000,000 which will The 2012 taxable income before c.P20,000
pay him P100,000 a year. What will provision for the loss was P100,000. d.P15,000 20k-5k=15k
Mr. Conner include in his gross West posted continuous losses until 23.A taxpayer under the cash basis
income on the 11th year of the 2015. With increasing prices of wrote-off P50,000 receivables in 2010.
policy? crude oil in 2016, West re- In 2015, P30,000 of the receivables was
a.P1,000,000 commissioned the facility for use. recovered.Determine the amount to be
b.P100,000 a year How much will be included in its included in gross income in 2015
c.P200,000 gross income in 2016? assuming that the taxpayer incurred a
d.P1,200,000 a.P300,000 net operating loss of P40,000 in 2010.
14. Edwin purchased the life b.P200,000 a.P0
insurance policy of Paulo for c.P100,000 B.P10,000
P50,000. He continued the policy by d.P0 c.P30,000
paying P20,000 premium after 19. Sarah Baby International d.P40,000
which Paulo died. Edwin collected graduated from its income tax 24.Multiple Choice-Problems:Part 2
the P500,000 proceeds of the policy. holiday incentive and is effectively 1. Mr. Cordillera owns 20% interest in a
How much will Edwin exclude from subject to tax beginning 2015. In joint venture engaged in construction
hi gross income? 2016, it collected a P4M, from a projects. In 2015, the joint venture
a.P500,000 P4M receivable which was written reported profits of P500,000, inclusive
b.P430,000 off as bad debt expense in 2013. of P20,000 from time deposits
c.P70,000-P50,000+P20,000 Before the write off, Sarah Compute the total income to be
d.0 International had P1M profit. Sarah reported in gross income of Mr.
15.Mr. Benson insured his life with posted profits in 2014 and 2015 in Cordillera?
his children as beneficiaries. He died excess of its operating loss in 2013. a.P0
after paying P200,000 premiums, Compute the amount of recovery b.P4,000
His children collected the subject to regular income tax c.P96,000
P1,000,000 life insurance proceeds. a.P6,000,000 d.P100,000
How much will be excluded from b.P1,000,000 P500k times 20%=96k
Mr. Benson’s gross income? c.P4,000,000 2. Mr. Buwaya had the following
a.P800,000 d.P0 data to be reported for his
b.P1,000,000 20. In 2015, an accrual basis income tax return in 2015:
c.P200,000 taxpayer received a cash refund for Sales P490,000
d.P0 an income tax assessment which he Gross compensation income 120,000
16. Pedro’s crop was destroyed by paid in 2011 consisting of the Personal exemption 150,000
incessant rains. He received following: Cost of sales 200,000
P200,000 from an insurer that Basic tax P40,000 Administrative and selling expenses
insured his crop. The proceeds of Surcharge 10,000 120,000
the insurance is an Interest 4,000 Personal expenses 70,000
a.Item of gross income subject to Total taxes paid P54,000 What is the net income?
regular tax. How much must be reverted back to a.P170,000
b.item of gross income subject to gross income in 2015? b.P290,000
final tax a.P0 c.P100,000
c.exclusion from gross income b.P4,000 d.P20,000do not include personal
d.exempt from gross income c.P10,000 expenses
17.In 2016, Northern Crest d.P44,000 3. Compute the taxable compensation
Corporation(NCC) reported a 21.In the immediately preceding income.
P40,000 recovery from bad debts problem, compute the tax benefit if the a.(P150,000)
that was claimed as deduction refund was of local tax instead of b.(P30,000)
against gross income in 2012. In income tax. C.P0
2012, the write-off increased the a.P44,000 D.120,000
Personal exemption 150,000 in gross income assuming the employee What is the amount to be reported in
Less:admin and se 120,000 is a Filipino rank and file employee? gross income if the taxpayer is a
30k a.P0 resident citizen?
170k-30k=140k b.P1,147,500 a.P900,000
or ni=170- c.P1,500,000 b.P1,100,000
140-170=-30k d.P1,350,000 c.P1,500,000
4.Compute the taxable income. gross income d.P2,000,000
a.P70,000 8. A corporation had the following 14.In the immediately preceding
b.P100,000 gains from dealings in problem, what is the reportable gross
c.P140,000 properties: income assuming the taxpayer is a
d.P0 Sale of delivery truck P150,000 resident alien?
5. Raymund and Zeus practice their Sale of domestic stocks 50,000 a.P0
accounting profession through general Sale of 3-year corporate bonds 12,500- b.P900,000
professional partnership. They short term c.P600,000
contributed equal capital and agreed to Sale of 6-year corporate bonds 7,500 d.P400,000
share in profits equally. The following What is the total amount of gain to be 15. In 2015, the taxpayer received the
relates to their gross receipts and included in gross income? following prizes and winnings:
expenses. a.P150,000 Passive income Philippines Abroad
Gross receipts P4,000,000 b.P162,500 Prizes P10,000 P400,000
Less:Cost of services 1,800,000 c.P170,000 Winnings 400,000 100,000
Gross income from operations d.P212,500 What is the reportable item of gross
P2,200,000 10. Shown below is a compilation of the income if the taxpayer respectively is a
Add:other non-operating income gain on the sale of real properties: resident citizen and a non-resident
Gain on sale of equipment P100,000 Real properties classified as Philippines citizen?
Interest on time deposit 40,000 Abroad a.P500,000;P0
140,000 Ordinary assets P300,000 P800,000 b.P510,000;P10,000
Total gross income P2,340,000 Capital assets 400,000 200,000 c.P0;P500,000
Less:Allowable deductions 1,200,000 What is the amount of income to be d.P910,000;P410,000
Net profits P1,140,000 included in gross income subject to 16. What is the reportable item of gross
What is the reportable income in the regular income tax assuming the income of the taxpayer is a domestic
tax return of Raymund? taxpayer is a domestic corporation? corporation and a resident foreign
a.P1,170,000 a.P1,300,000 corporation, respectively?
b.P500,000 b.P1,100,000 a.P500,000;P0
C.P550,000 c.P1,700,000 b.P510,000;P10,000
D.P570,000 d.P300,000 c.P910,000;P410,000
6.In the immediately preceding 11.What is the gain to be included in D.P500,000;P10,000
problem, determine the amount of gross income subject to regular income 17.Mang Sipalay registered his business
income to be reported by Zeus tax if the taxpayer is a resident foreign as a BMBE. He made a total sales of
assuming that their partnership is a corporation? P500,000 and incurred cost of sales of
beauty parlor. a.P300,000 P400,000. He also earned P10,000
a.P0 b.P400,000 interest income from time deposits.
b.P570,000 c.P700,000 What is the total reportable gross
c.P550,000 d.P1,100,000 income?
d.P500,000 12.What is the gain to be included in a,P0
7.An alien employee in an RHQ had the gross income if the taxpayer is a b.P190,000
following in 2015: resident alien? c.P210,000
Gross compensation income a.P700,000 d.P400,000
P1,500,000 b.P400,000 18. Mr. Siayan is a 5-6 lender. During the
Contributions to SSS, PhilHealth and c.P300,000 year, he granted loans totaling
HDMF 150,000 d.P1,100,000 P2,000,000 and collected P400,000 in
Creditable withholding tax 202,500 13.A taxpayer collected the following interest. He also earned P8,000 in
What is the total amount to include in passive income during the year: temporary investments in domestic
gross income subject to regular income Passive income Philippines Abroad bonds plus additional P6,000 from bank
tax? Interest income from banks P300,000 deposits substitutes. Direct cost of
a.P0 P800,000 lending was P100,000.
b.P1,147,500 Royalties from books 200,000 What is the total amount to be reported
c.P1,350,000 100,000 in gross income subject to regular tax?
d.P1,500,000 Rent of properties 400,000 a,P0
8. In the immediately preceding 200,000 b.P300,000
problem, what is the amount to include c.P414,000
d.P308,000
19. Boracay company is registered as a d.P470,000 What is the amount of gross income
TIEZA locator subject to 5% gross 3. A non-VAT taxpayer collected subject to Philippine tax?
income tax.During the year, it made a P45,000 net of P5,000 withholding tax. a.P2,000,000
total P400,00 gross receipts from Compute the gross income subject to b.P4,800,000
various tourist assistance services. It regular tax. c.P2,400,000
also incurred P210,000 in direct a.P5,000 d.P6,000,000
services. b.P40,000 10. Ms. Nene Gosio registered a
What is the amount to be included in c.P45,000 manufacturing business as a BMBE
gross income subject to regular income d.P50,000 exempt from tax. She also owns
tax? 4.A VAT taxpayer collected P66,600, another taxable business which is
a.P0 inclusive of P7,200 VAT and net of P600 engaged in the reading of goods. Ms.
b.P190,000 withholding tax. Compute the gross Gosio ordered her BMBE business to
c.P210,000 income subject regular income tax. sell its production to her trading
D.P400,000 a. P59,400 business at ultimate sales prices.
20. Mr. Asuncion received the following b.P67,200 You were tasked by your audit
royalties from the following sources: c.P66,000 supervisor to conduct a transfer pricing
Mining claims P150,000 d.P60,000- evaluation of Ms. Gosio’s businesses.
Novel,”Alicia in Wonderland” 250,000 P66,000+P600-P7,200=P60,000 Based on your study, you determined
Basic Accounting textbook 80,000 5.A non-VAT taxpayer collected that the retail profit rate(on sales) of
Musical composition “Dayang-Dayang” P79,200, net of 1% withholding tax. trading businesses with similar
P40,000 What is the amount subject to regular operations involving similar goods is
What is the total amount to be reported income tax? 40%. During the year, the trading
in gross income? a.P71,351 business made a total purchases of
a.P0 b.P71,429 P400,000 from the BMBE and sold 75%
b.P150,000 c.P80,000 of these for P500,000.
c.P400,000 d.P79,200 What is the gross income of the trading
d.P520,000 6.A VAT taxpayer received P45,900 business to be subjected to regular
Multiple choice-Problems:Part 3 inclusive of VAT and net of 10% income tax following the arm’s length
1. Mrs. Kapalong has several creditable withholding tax. Compute principle?
interests in various businesses the gross income subject to regular tax. a.P0
and partnerships. He received a.P45,000 b.P100,000
the following income during the b.P45,900 C.P200,000
year: c.P45,536 d.P120,000
Dividends from a domestic corporation d.P51,000 400,000x75%=300,000/500,000=0.60
P120,000 7.A non-VAT taxpayer received P8,000 200,000/500,000=40%
Dividends from a resident foreign interest income, net of 20% final Cost of selling
corporation 80,000 withholding tax. Compute the amount party(400,000x0.60=240,000)
Share in net income of a business subject to regular income tax. 500,000x40%=200,000
partnership 200,000 a.P0 CHAPTER 10
Share in the net income of a b.P6,400 COMPENSATION INCOME
professional partnership 100,000 c.P8,000 Chapter Overview and Objectives
What is the total income to be reported d.P10,000 This Chapter discusses the employee
in gross income? 8. A VAT-registered taxpayer received benefits considered as compensation
a.P0 P18,000 dividend, net of 10% final income. It discusses the types of
b.P380,000 withholding tax. Compute the amount employees, and exempt and taxable
c.P300,000 subject to regular income tax. benefits. It also delineates the gap
d.P180,000 a.P0 between the compensation income
2. Ms. Panabo received a total P200,000 b.P17,647 subject to regular income tax and the
from her father for her support. During c.P17,857 fringe benefits subject to fringe benefits
the year, she also received a P150,000 d.P20,000 tax.
total distribution from the trust 9.A resident foreign corporate taxpayer After this chapter, readers are expected
irrevocably designated by her entered into an advanced pricing to demonstrate:
grandfather in her favor. She also agreement(APA) with the BIR with 1.Understanding of the concept of an
received P120,000 income distribution respect to the pricing of its export sales employer-employee relationship
from the estate of her grandmother to a foreign country. A markup ratio of 2.Knowledge of the classifications of
undergoing judicial settlement. 50% of the cost is set in the APA. During employees and the tax treatment of
What is the total amount to be included the year, the corporation manufactured their compensation income and fringe
in her gross income? goods costing P12,000,000 and benefits
a.P200,000 exported 80% of the production to its 3.Mastery of the list of employee
b.P270,000 foreign affiliate at a price of benefits exempted under the NIRC and
c.P350,000 P12,000,000. special laws and the de minimis list
4. Knowledge of the condition for The following are not considered of a minimum wage but with other
exemption of employee benefits under employees: taxable income is a regular employee.
treaty or international agreements 1.Consultants Special alien
5. Understanding of the concept of 2.Directors without management Special aliens refer to aliens holding
“employer convenience” rule and the function managerial or technical position in a:
“necessity of the employer” rule. 3.Talents and artists on TV shows or a.Regional or area headquarters(RHQ)
6. Understanding of the conditions of radio broadcasts(Sonza vs. ABS-CBN or Regional operating headquarters
exemption of a minimum wage earner Broadcasting Corporation, G.R No. (ROHQ) of multinational companies.
7. Knowledge of the classification rules 138051) b.Offshore banking units
for items of gross taxable compensation The income or fees of these individuals c.Petroleum service
income are not compensation income but are cotractors/subcontractors
8. Mastery of the composition of “13th business or professional income. An RHQ or an ROHQ is branch of a
month pay and other benefits” for rank TYPES OF EMPLOYEES AS TO FUNCTION multinational company or foreign form
and file employees and managerial or 1.Managerial employees-Those who are or entity engaged in international trade,
supervisory employees given powers or prerogatives to lay with affiliates, subsidiaries, or branch
9. Comprehension of the fringe benefits down and execute managerial policies offices in the foreign markets, and is
subject to regular tax and fringe and/or to hire, transfer, suspend, lay-off, governed by Book 3 of Executive Order
benefits subject to fringe benefits tax recall,discharge, assign, or discipline No. 226 as amended by RA 8756
10.Understanding of the conditions for employees An offshore banking unit is a division of
citizens to be considered special aliens 2.Supervisory employees-Those who a foreign bank which is authorized to
EMPLOYER-EMPLOYEE RELATIONSHIP effectively recommend such managerial transact business transactions in foreign
Employer-refers to any person for actions if the exercise of such authority currencies in the Philippines
whom an individual performs any is not merely routinary or clerical in A petroleum service contractor is an
service of whatever nature as employee nature but requires the use of awardee of a petroleum service
of such person. independent judgement. contract under the Oil Exploration and
An employer is the person who has 3. Rank and file employees-Those who Development Act of 1972 either alone
control over the payment of the hold neither managerial nor supervisory or in consortium with others.
employee remuneration. However, if functions. Taxation of special aliens
such person is a non-resident not TYPES OF EMPLOYEES AS TO Special aliens are subject to 15% tax on
engaged in trade or business in the TAXABILITY gross income received such as salaries,
Philippines, the employer is deemed 1.Minimum wage earners-Employees wages, annuities, compensation,
the person paying remuneration in their who are recipients of minimum wage remuneration, and other emoluments
behalf. are exempt from income taxation. such as honoraria and allowance except
Employee-refers to any individual who those subject to fringe benefit tax.
is a recipient of wages and includes 2.Special aliens-Alien employees who Optional Final income tax for Filipino
officer, employee or elected official of are subject to the 15% final income tax employees
the Government of the Philippines or on compensation income Filipinos who are occupying the same
an political subdivisions, agency or positions as those held by special aliens
instrumentality thereof. The term also 3.Regular employees –Employees who may opt to be taxed at 15% final income
includes an officer of a corporation. are subject to the regular progressive tax on gross income or the regular
Elements of employer and employee income tax income tax. Hence, the term “special
relationship under case law: Minimum Wage Earner alien” includes qualifying Filipino
1.Selection and engagement of A minimum wage earner refers to a employees who opted to be taxed at
employees-There is a screening process worker in the private sector who is paid 15% of gross income.
for employees to hire the minimum wage or to an employee Requirement to Filipinos to be
2.Payment of wages-The employer in the public sector with compensation classified as Special Alien
usually fixes and controls the payment income of not more than the statutory To qualify as special alien employees,
of wages minimum wage(i.e., those with salary Filipino employed by RHQs or ROHQs,
3.Power of dismissal-Employer has grade 1 to 3) in the non-agricultural OBUs or petroleum service contractors
power to retrench or terminate sector where he or she is assigned. or subcontractors must meet all of the
employees when incurring heavy losses The statutory minimum wage refers to following:
or other reasonable basis. rate fixed by the Regional Tripartite 1.Position and function test-The Filipino
4. Power to control-The employer has Wage and Productivity Board of the employee must be occupying and
power to control the employee on the Department of Labor and Employment actually exercising a managerial or
means and methods by which the work or P5,000/month or P60,000/year, technical position.
is accomplished whoever is higher. 2.Compensation threshold test-The
An arrangement which do not manifest To be considered as a minimum wage Filipino employee must have a gross
all the elements is not an employer- earner, the employee must not have annual taxable income of at least
employee relationship but an other income aside from their minimum P975,000.
independent contract for the provision wage and exempt benefits. A recipient 3.Exclusivity test-The Filipino employee
of services. is not a consultant or contractual
personnel, and is solely employed by under an employer-employee As originally conceived, other petty
the RHQ or ROHQ. relationship. fringe benefits which fall within the
Special alien is not a general NON-TAXABLE OR EXEMPT purview of de minimis even if not part
classification COMPENSATION of the deminimis list are normally
It must be noted that the “Special alien” A.Benefits excluded and/or exempted treated as de minimis and are also
classification is not a general under the NIRC and special laws exempt from income tax.
classification but a special category that B.Benefits exempt under treaty or However, the BIR and the Department
exists only in tax law for purposes of international agreements of Finance changed the rule under RR5-
taxing compensation income.For C.Benefits necessary to the trade, 2011 wherein the term “de minimis
purposes of taxation of other income, business, or conduct of profession of benefits” was restricted to means only
special aliens are classified according to the employer the following:
the appropriate general classification. D.Benefits for the convenience or 1.Monetized unused vacation leave
Illustration advantage of the employer credits of private employees not
Mr. Kuonoman is a non-resident A.EXEMPT BENEFITS UNDER THE NIRC, exceeding 10 days during the year
Japanese expert hired as a technical AS AMENDED, AND SPECIAL LAWS 2.Monetized unused vacation and sick
employee of FilDrill, a petroelem 1.Remunerations received as incidents leave credits paid to government
service contractor developing the on employment officials and employees
Malampaya Service Contract No.32. a.Exempt retirement benefits under 3.Medical cash allowance to
Mr. Kuonoman shall be considered as RA 7641 including exempt retirement dependents of employees not
special alen with respect to his gratuities to government officials and exceeding per employee per semester
compensation income from FillDrill. He employees per month of P1500-250
shall be considered a non-resident alien b.Exempt termination bebefits 4.Rice subsidy not exceeding 1 sack of
on all other items of income from the c.Benefits from the United States 50-kg rice per month amounting to not
Philippines. Veterans Administration more than 2,000
Illustration 2 d.Social security, retirement gratuities, 5.Actual medical
Mr. Camacho, a resident Filipino, is pensions, and similar benefits from assistance,e.g.,medical allowance to
hired as operations manager of the foreign government agencies and other cover medical and healthcare needs,
offshore banking unit of Eurobank in institutions, private or public annual medical/executive check-up,
the Philippines. He opted to be taxed to e.Benefits from SSS, under the SSS Act maternity assistance, and routine
the 15% final income tax. of 1954, as amended consultations not exceeding P10,000
Mr. Camacho shall be considered a f.Benefits from GSIS under the GSIS Act per annum.
special alien with respect to his of 1937, as amended. 6.Uniform and clothing allowance-not
compensation income from the OBU. 2.Employee mandatory contributions exceeding P6,000 per annum (RR8-
He shall be classified as a resident to GSIS, SSS, PhilHealth, HDMF, and 2012)
citizen for all other items of income he union dues 7.Laundry allowance-not exceeding
may derive outside the OBU. 3.Certain benefits of minimum wage P1,200
Illustration 3 earners 8.Emloyee achievement award-e.g., for
Mr.Escala, a resident Filipino, occupies a 4.13th month pay and other benefits length of service or safety achievement,
managerial position in the ROHQ of not exceeding P90,000 which must be in the form of tangible
Cosmetics International. He derived an Exempt benefits of minimum wage property other than cash or gift
annual income of P1,000,000 inclusive earners certificates, with an annual monetary
of P300,000 annual part-time income 1.Basic minimum wage value not exceeding P10,000 received
from outside employment. 2.Holiday pay by the employee under an established
Mr. Escala shall be treated as a regular 3.Overtime pay written plan which does not
employee subject to the regular tax 4.Night shift differential pay discriminate in favor of highly paid
since he failed both the compensation 5.Hazard pay employees.
threshold test and the exclusivity test. To be exempt from regular income tax, 9.Gifts given during Christmas and
Note that he has concurrent a minimum wage earner must not have major anniversary celebrations not
employment and that he only derives other items of taxable income aside exceeding P5,000 per employee per
an annual income of from these employee benefits annum (i.e., Christmas gifts and
P800,000(P1,100,000-P300,000) from De minimis benefits anniversary gifts)
the ROHQ. De minimis benefits are facilities or 10. Daily meal allowance for overtime
THE TAX MODEL ON COMPENSATION privileges such as entertainment, work and night or graveyard shift not
INCOME medical services, or courtesy discounts exceeding 25% of the basic minimum
Gross compensation income Pxxx,xx on purchases that are of relatively small wage on a per region basis (i.e.,
Less:Non-taxable compensation xxx,xx value and are furnished by the overtime meal)
Gross taxable compensation income employer merely as a means of 11.Benefits received by an employee
Pxxx,xx promoting the health, goodwill, by virtue of a collective bargaining
GROSS COMPENSATION INCOME contentment, or efficiency of hid agreement (CBA) and productivity
Gross compensation income generally employees. De minimis benefits are incentives schemes provided that the
includes all remunerations received petty fringe exempt from income tax. total annual monetary value received
from both CBA and productivity (BIR Ruling No. 227-2013, June 20, 2. Only meals for overtime or
incentives schemes combined do not 2013) graveyard shifts are considered
exceed P10,000 per employee per 3. The rice subsidy and allowance were de minimis. Other meal benefits
taxable year.(RA 10653, effective likewise annualized by multiplying their are no longer considered de
January 1, 2015 under RR3-2015) monthly limit by 12. The de minimis minimis
Taxable de minimis benefits benefits within the limits are exempt Note that in all three illustrations, if the
1. Excess de minimis over their from income tax. employee is a managerial or
limits Illustration 2 supervisory employee, the entire excess
2. Other benefits relatively small Giovanni, a government rank and file de minimis shall be considered as other
value that are not included in employee, received the following fringe benefits subject to fringe benefits
the list of de minimis benefits benefits: tax.
Treatments of taxable de minimis Monetized unused vacation leave Commutation of accumulated leave
benefits credits (10 days) P6,000 credits
a.For rank and file employees-taxable Monetized unused sick leave credits (15 The terminal leave pay or the
de minimis is treated as other days) 9,000 commutation of unused leave credits
compensation income under the Uniform allowance 5,000 due to involuntary separation from
category “13th month pay and other Laundry allowance 4,800 employment of the employee is now
benefits” Required:Determine the amount to be treated as de minimis benefits subject
b. For managerial and supervisory included in other benefits. to the 10-day leave credit limit and is no
employees-taxable de minimis is Solution: longer exempt as part of exempt
treated as fringe benefits subject to Actual Limit termination benefits.
final fringe benefit tax.(RR5-2011 and Excess 13th month pay and other benefits not
RMC20-2011) Monetized unused VL P6,000 exempt in excess of P82,000
DETERMINATION OF EXCESS DE P0 The composition of the “13th month pay
MINIMIS BENEFITS Monetized unused SL 9,000 exempt 0 and other benefits”will be discussed
Illustration 1:De minimis limits Uniform allowance 5,000 5,000 0 later under taxable benefits
Alexander, a private employee who is Laundry allowance 4,800 3,600 B.BENEFITS EXEMPT UNDER TREATY
paid a P600 daily rate, receives the 1,200 OR INTERNATIONAL AGREEMENTS
following benefits during the year 2015: Taxable de minimis as “other benefits” Employee benefits of non-Filipino
Monetized unused vacation leave 1,200 P1,200 nationals and/or non-permanent
credits 9 days Note: It is clear under RR5-2011 that residents of the Philippines from
Monetized unused sick leave credits 9 the vacation leave and sick leave of foreign governments, embassies, or
days government employees are not subject diplomatic missions, and international
Medical assistance P7,000 to the 10-day limit rule organizations are exempt from income
Rice subsidy(P2,000 per month) 24,000 Illustration 3 tax.
Clothing allowance 8,000 Professor Radvic was one of the Hall of Exemption from withholding tax does
Laundry allowance 6,000 Fame awardees of Youbee University. not mean income tax exemption
Required:Determine the taxable He was granted P25,000 cash as loyalty Foreign government embassies,
amount of de-minimis benefits award for his 30 years of service. He diplomatic missions and international
Solution: was also given P10,000 Christmas gift organizations are immune from income
Actual Limit and an additional P10,000 gift during tax including the obligation to withhold
Excess the institution’s Founding day income tax by virtue of international
Monetized unused VL P5,400 P6,000 P0 anniversary. Besides, he was also given comity as embodied in several
Monetized unused SL 5,400 0 5,400 free lunch meals with a total value of international agreements to which the
Medical assistance 7,000 10,000 0 P15,000 during the same year. Philippines is a signatory.
Rice subsidy 24,000 18,000 Required:Compute the total taxable de- However, this exemption from the
6,000 minimis benefits as other benefits obligation to withhold tax does not
Clothing allowance 8,000 5,000 Solution: mean income tax exemption of their
3,000 Actual Limit Excess Filipino employees. In fact, most of the
Laundry allowance 6,000 3,600 Loyalty or service award P25,000 P0 international agreements to which the
2,400 P25,000 Philippines is a signatory limit
Taxable de minimis as “other benefits” Christmas and anniversary gift 20,000 exemption only to non-Filipino
P16,800 5,000 15,000 nationals and/or non-residents of the
Note:Private employees Meals 15,000 0 Philippines.
1.The actual value of the monetized 15,000 Filipino employees of foreign
unused VL was computed as P600 x 9 Total taxable de minimis as “other governments, international missions
while limit was P600 x10. benefits” P55,000 and organizations are taxable as a rule
2.The 10-day rule applies only to Note: except only to employees of the
vacation leaves. Monetization of sick 1. The limit on loyalty or sevice following organizations:
leaves of private employees is taxable awards applies only if it is given 1.United Nations(UN)
in kind
2.Specialized Agencies of the United C.BENEFITS REQUIRED BY THE NATURE available always for assignment and
Nations OF, OR NECESSARY TO, THE TRADE, consultation (BIR Ruling DA-233-07)
3.Australian Agency for International BUSINESS OR CONDUCT OF 2.outstation allowance for employees
Development (AUSAID) PROFESSION OF THE EMPLOYER who will be out from office site at least
4.Food and Agriculture Benefits or allowances furnished by the 8 hours to visit lotto franchise holders
Organization(FAO) employer to the employees to enable for repairs and/or inspection of
5.World Health Organization (WHO) them to appropriately and effectively equipment leased by the employer (BIR
6.United Nations Development execute their duties as required by their Ruling No. 013-02)
Programme(UNDP) employment are exempt from income 3.Grant of housing privilege to
7.International Organization for tax. This is referred to as “necessity of employees working at distant or remote
Migration(IOM) the employer rule”. facilities even if the dwelling is
8.International Seabed Authority(ISA) Examples: distanced from the facility in
These organizations have exemption 1.Necessary travelling, representation, compliance to labor safety standardsd.
provisions that extend even to their or entertainment expenses that are (BIR Ruling No. 055-99)
Filipino employees. Other aid agencies subject to an accounting or liquidation 4. Car incentives to medical doctors on-
or international organizations may have in accordance with specific call who are required to report on duty
tax free provisions in their articles of requirements of substitution of anytime.
agreement for Filipino employees. expense. 5.Scholarship grants to employees
Confirmation of tax exemptions 2.Allowances which essentially under contract to remain in service for
The exemption of Filipino employees is constitute reimbursement to a specified period upon completion of
not automatic. Filipinos claiming government personnel for expenses the study
exemptions under the terms of they incurred in the performance of 6.Housing privilege of military officials
international agreements or under their official duties, such as: of the AFP located inside or near the
provisions of special laws granting a.Representation and transportation military camps.
privileges to international organizations allowance(RATA) of public officers and These types of employer spending are
shall file an application for confirmation employees under the General regarded as business expenses and are
of tax exemption with the BIR’s Appropriation Act not considered as employee reward
International Tax Affairs Division(ITAD). b.Personnel Economic Relief Allowance because they are not intended for the
The confirmation shall serve as proof of (PERA) (RR10-2008) free personal consumption or disposal
exemption. Without the confirmation 3.Reasonable amounts of of the employees but as implements of
certificate, the employee is taxable. reimbursements or advances to the employer’s business to ensure the
Employees of Philippine embassies or employees for travelling and employer’s convenience.
consulate offices representation which are pre-numbered However, if the expenses is
It should be recalled that employees only a daily basis and which are paid to unreasonably excessive making it
working in Philippine embassies or any employee while on assignment or depart from the nature of a reasonable
Philippine consulate offices are not duty. business expense such as when it is
considered non-resident citizens and These amounts given to the employee deliberately granted to include a benefit
are therefore subject to Philippine are not income but are expenses of the for the employee, the portion of the
income tax. trade, business or profession of the expense representing provision or
Summary of rules employer that are incurred or paid privilege to the employee is considered
Foreign Philippin through the employee.These are not a taxable fringe benefit. These types of
embassy, e benefits since they are mere advances expense are regarded as “hybrid
missions, or embassy or replenishments of what are supposed expenses” because they are partially
organizatio or to be direct cash outflows from the business expense and partially
n consulate employer; hence, they are not employee benefits.
office considered as compensation income. GROSS TAXABLE COMPENSATION
In the Taxable* n/a D.BENEFITS FOR THE CONVENIENCE OR INCOME
Philippine Exempt N/A ADVANTAGE OF THE EMPLOYER Classification of gross taxable
s Benefits or allowance which are compensation income
-Filipino intended for the furtherance of the 1. Regular compensation-This
citizens interest of the employer’s business or pertains to the fixed
-Aliens to ensure its smooth operations are remunerations received by the
Abroad Exempt Taxable likewise exempt from income tax. This is employee every payroll period.
-Filipino Exempt Exempt referred to as the “convenience of the
citizens employer rule.” 2.Supplemental compensation-
-Aliens Examples: This pertains to other
*Taxpayer must prove if there is an 1.Work-related mobile phone allowance performance-based pays to
exemption grant under contract or and transportation allowance employees with or without
special law. particularly to employees of call centers regard to the payroll period.
which are operated on a 24-hour basis 3.13th month pay and other
where employees are required to be benefits-This is a residual
category which generally The paid absences of an employee The gross taxable compensation shall
includes incentive pays and all applied against his vacation or sick be computed as follows:
other taxable employee leave credits which are normally Gross compensation
benefits. received as part of the regular salary is income(P200K+P20K+P100K) P320K
The amount of 13th month pay and part of the regular compensation. Less:Non-taxable compensation
other benefits not exceeding P82,000 is Non-compensation items 100,000
an exclusion from gross income. The 1.Fees Gross taxable compensation income
amount in excess of P82,000 is Retainer fees of consultants, talents, P220,000
considered as supplemental and directors who have no Note:
compensation. management function in the business 1.Reimbursement for transportation
Illustration are professional income, not expense is not an income to the
An employee received P400,000 regular compensation income of the recipient. employee
compensation, P100,000 supplemental 2.Commissions to non-employees such 2.The termination pay is included in
compensation , and P90,000 13th month as independent sales agents are gross compensation income, but is also
pay and other benefits. business income to the sales agents deducted as non-taxable compensation
The gross taxable compensation income 3.Tips and gratuities because the reason of termination is
shall be computed as follows: Tips and gratuities paid directly to an beyond the employee’s control.
Regular compensation P400,000 employee by customers of the SUPPLEMENTARY COMPENSATION
Supplemental compensation 100,000 employer which are not accounted for Supplementary or additional
13th month pay and other by the employee to the employer are compensation includes performance-
benefits(P90,000-P82,000) 8,000 not considered as compensation based remunerations to an employee in
Gross taxable compensation income income, but are to be reported as addition to the regular compensation
P508,000 “other income” in the income tax return with or without regard to the payroll
REGULAR COMPENSATION INCOME of the employee. period.
The regular compensation includes Valuation of compensation paid in kind The following are the additional
fixed remunerations due to be received Compensation in kind is taxable at the compensation under current tax rules:
by an employee every period such as: fair value of the consideration received. 1.Overtime pay
1.Basic salary If received in shares, the fair value of 2.Hazard pay
2.Fixed allowances such as cost-of-living the shares at the date of services were 3.Night shift differential pay
allowance, fixed housing allowance, provided is used. 4.Holiday pay
representation, transportation, and Illustration 1 5.Commisions
other allowances paid to an employee The following pertains to an employee 6.Fees, including director’s fees(if
every payroll period. in 2015: director is an employee)
Fixed allowances Gross salaries P400,000 7.Emoluments and honoraria
Allowances which are fixed in amounts Cost-of-living allowance 36,000 8.Taxable retirement and separation pay
and regularly received as part of the Fixed monthly transportation allowance 9.Value of living quarters and separation
basic, monthly, bi-weekly, weekly or (P2KX12) 24,000 pay
daily salaries or wages are part of Deduction for withholding tax on 10.Gains on exercise of stock
regular compensation. This applies even compensation 10,000 options(BIR ruling 119-2012)
if a portion of the allowances are Deduction for employee share in SSS, 11.Profit sharing and taxable bonuses
actually used in the employer’s PHIC, and HDMF 32,000 Overtime, holiday, and night shift
business. The gross taxable compensation income differential pay
Exception rule on the taxability of shall be computed as follows: These constitute additional to
allowances: Gross salaries P400,000 compensation, except when derived by
a.If the allowance is an ordinary and Cost-of-living allowance 36,000 a minimum wage earner.
necessary travelling, representation,or Fixed monthly transportation allowance Commissions, emoluments and
entertainment expense of employee in 24,000 honoraria
the pursuit of his trade, business or Gross compensation income Commissions are incentives intended to
profession. P460,000 stimulate sales. These may be given as a
b.The expense is subject to accounting Less:Non-taxable compensation 32,000 profit sharing or performance bonus.
or liquidation Gross taxable compensation income Emoluments pertain to any pay in
c.Any excess advances are returned to P428,000 general while honoraria are additional
the employer Illustration 2 payments for attending to special tasks
Hence, variable and liquidated An employee who was terminated in or assigments.
allowances are not subject to tax. 2015 due to business closure of the Living quarters or meals
However, amounts of allowances that employer received the following: If an employee receives free living
are retained by the employee for Unpaid 2015 salaries P200,000 quarters or meals in addition to salary
himself shall be considered Unpaid 2014 salaries 20,000 for services rendered, the value to the
compensation. Reimbursement for transportation employee of such living quarters or
Paid vacation and sick leave allowances expenses 10,000 meals is included in compensation
Termination pay 100,000 income. However, when the same was
furnished to an employee for the employees such as salesmen, division Christmas bonus of private employees
convenience of the employer or out of heads, key officers, or by all employees which is a non-performance-based
necessity of the employer’s business, collectively. incentive pay is part of other benefits.
the value thereof is not compensation Bonuses are supplemental or additional Christmas bonus in the nature of profit
income, but a business expense. compensation. However, if they are sharing should be treated as additional
Stock option plans linked solely to productivity incentive compensation income, not as “other
Under various stock option plans, plan of the employer pursuant to RA benefits” The nature of the Christmas
employees are given the privilege to 6971, they should be considered as de bonus of private employees shall
buy shares at an agreed exercise price minimis benefits. determine its tax classification.
after meeting stipulated vesting Productivity incentive bonus The Christmas gift of government
conditions. The option will have value The productivity Act of 1990(RA 6971) employees is specifically designated as
when the stock of the employer encourages private employers to set-up part of “13TH month pay and other
increases in value above the exercise productivity incentive programs. benefit” under Sec. 32 (B)(7)( e) (i) of
price at exercise date. The value of the A productivity incentive is linked to the NIRC. RR5-2011 includes Christmas
option is the discount at exercise date. improvements in productivity usually in gift in the list of de minimis benefits.
In the past, stock options were not terms of cost savings through waste But since revenue regulations cannot
taxed at exercise date under the view reduction, efficient labor utilization, or amend the law they implement. RR5-
that the discount becomes realized only increase in volume of production. 2011 should be interpreted to apply
when the stocks are disposed. Under Under the NIRC, productivity incentive only to Christmas gifts of private
current tax rules, the discount (i.e. bonus is considered as part of “other employees.
market price-exercise price) at exercise benefits” under “13th month pay and Hence,
date is viewed as compensation in kind. other benefits”Under the revision of RA Governmen Private
The gains from the exercise of stock 10653, productivity incentive is now a t employees employee
options constitute a taxable de minimis benefit. s
th
compensation income, unless they Productivity incentive distinguished Christma 13 month 13th
qualify as fringe benefits subject to final from profit sharing bonus s bonus pay and month pay
tax.(RMC88-2012) Productivity incentive is anchored on other and other
Illustration improvements in the factors of benefits benefits
Mr. Anthony met the vesting condition production and is usually enjoyed Christma 13 moth th
De
of his employer’s stock option plan collectively by employees due to the s gift pay and minimis
where he is entitled to buy 10,000 of his inherent difficulty of tracing other
employer’s share at a strike price of productivity to individual performance. benefits
P100. Mr. Anthony exercised the option It is based upon cost savings, hence, it is Bonus vs. Gift
when the share of his employer was payable even if the business poses a Bonus is performance-based and is non-
selling P150/share. loss. Profit sharing is payable only when discretionary to the employer while a
The discount of P500,000 computed as the business post a profit. gift is a gratuity and is discretionary
[(P150-P100)X10,000), shall be reported 13th month pay and other BENEFITS upon the employer.
and treatment as additional “13TH Month pay and Other Benefits” Other fringe benefits
compensation income of Mr. Anthony. includes: Other fringe benefits include all other
Treatment of the subsequent sale of 1.13th month pay taxable fringe benefits not specifically
the shares 2.Other benefits included in compensation income as
If the employer is a: a.Christmas bonus of private regular, supplementary or 13th month
1.Domestic corporation, and the sale of employees pay, and other benefits under current
the stocks is made: b.Cash gifts other than Christmas or tax rules such as:
a.through the PSE, the sale is subject anniversary gifs of private 1.Employee personal expenses
to a transaction tax ½ of 1% of the gross employees(RR2-98 as amended by RR5- shouldered by the employer
selling price. 2011) 2.Taxable de minimis benefits such as:
b.directly to buyer, the net gain on the C.Additional compensation a.Excess de minimis
sale is subject to the 5%-10% capital allowance(ACA) of government b.Benefits not included in the de
gains tax. personnel (RR8-2000) minimis list
2.Foreign corporation, the net gain on D.14th month pay, 15th month pay, etc. Employee personal expenses
the sale is a capital gain subject to the e.Other fringe benefits of rank and file Employee personal expenses such as,
regular income tax employees but not limited to, rental of residence,
Profit sharing or taxable bonus Christmas bonus and Christmas gift grocery, association or club
Profit sharing is a reward for churning The Christmas bonus of government membership dues or vacation expense
the business to post a profit. It is a employees is their 13th month pay. In or tuition fees, when assumed or paid
compensation for controlling all the private companies, the term “Christmas by the employer, constitute fringe
factors that influence profit such as bonus “ may pertain to the 13th month benefits to the employee. This fact
marketing and sales, productivity, and pay, a separate incentive pay, or to a holds true even if expense is receipted
administrative factors. It is a reward profit sharing. in the name of the employer.
which can be enjoyed by individual
Taxable de minimis benefits P90,000 is subject to the withholding Danger exposure allowance(hazard
All other benefits of relatively small tax on compensation pay) 6,000
value which are not included in the list RR3-98, the revenue regulation Housing privilege 38,000
of de minimis benefits shall not be implementing the fringe benefit tax, Uniform and clothing allowance 7,000
considered as de minimis but as also provides that it does not cover Laundry allowance 6,000
ordinary fringe benefits. Corollary to benefits forming part of compensation The housing privilege to the annual
this rule, excess de minimis benefits income subject to the withholding tax value of the employee’s living quarters
should be considered as taxable on compensation. furnished by the employer for staying
ordinary fringe benefits. Hence, the excess of 13th month pay on-site.
Tax Treatment of other fringe benefits and other benefits”over P82,000 should Required:Compute the excess 13th
a.For rank and file employees-treated be treated as compensation income month pay and other benefits
as compensation income as part of subject to regular income tax. Solution:
“other benefits” under “13th month pay Illustration 1 De minimis Limit Other
and other benefits” A government rank and file employee benefits
b. For managerial or supervisory received the following benefits aside
employee-treated as fringe benefits from the basic pay in 2015: Christmas gift P22,000 P5,000 P17,000
subject to fringe benefits tax. Christmas bonus P50,000 Uniform and clothing allowance 7,000
It must be emphasized than the “other Cash gift 5,000 5,000 2,000
fringe benefits” of managerial or Additional compensation allowance Laundry allowance 6,000 3,600 2,400
supervisory employees are excluded 36,000 Excess de minimis benefits(other fringe
from their “13th month pay and other Personnel Economic Relief Allowance benefits) P21,400
th
benefits.”. (P2,000/month) 24,000 13 month pay 67,000
Illustration 1 Monetized value of vacation leave and Total 13th month pay and other benefits
The employer pays for the tuition fee of sick leave(18 days) 9,000 P89,400
the employee in addition to his regular Uniform and clothing allowance 5,000 Less:Exclusion threshold
compensation. Required: Determine the taxable “13th 82,000
The tuition fee paid is a fringe benefit month pay and other benefits” Taxable 13th month pay and other
which will be treated as follows: Solution: benefits P7,400
1.As a compensation income as part of Christmas bonus(13th month pay of Note:
“other benefits” under “13th month pay gov’t employees) P50,000 1.Performance bonus is a supplemental
and other benefits”if the employee is a Cash gift 5,000 or additional compensation.
rank and file employee. Additional compensation allowance 2.The laundry allowance limit is
2.As a fringe benefit subject to fringe 36,000 computed as P300x12=P3,600
benefit tax if the employee is a Total 13th month pay and other benefits 3.Christmas gift is a de minimis benefit
managerial or supervisory employee P91,000 for private employees under RR5-2011.
3.As an exempt fringe benefit, Less:Exclusion Threshold 82,000 4.The housing privilege is exempt under
regardless of the type of employee, is Taxable 13th month pay and other the convenience of the employer rule.
the same was given by the employer for benefits P9,000 5.The “13th month pay and other
his convenience or business necessity Note of government employees: benefits” of rank and file employees
such as when the employee is required 1.Personnel Economic Relief Allowance includes “other fringe benefits.”
to study to acquire expertise for the is not subject to income tax and Illustration 3
future use of the employer’s business. withholding tax (Under RR8-2000,as A managerial employee received the
Illustration 2 affirmed by RR10-2008) following benefits in 2015:
An employee receives a monthly rice 2. The P5,000 Christmas gift of 13th month pay P75,000
allowance of P2,000 a month which is government employees is designated by Rental expense on condominium unit
P500 in excess of the P1,500 a month the NIRC to be part of “13th month pay 18,000
de minimis limit for rice allowance. and other benefits”; hence, it is not a Salary of personal body guard 12,000
The P500 monthly excess constitutes a de minimis benefit. Profit sharing 12,000
taxable de minimis benefit taxable as 3.Under RR5-2011, the monetization of Rice allowance 21,600
compensation as part of “other vacation leave and sick leave credits of Clothing allowance 6,000
benefits” for a rank and file employee. government officials is an exempt de Night shift differential pay 11,000
It is a fringe benefit subject to final minimis benefit without regard to the Productivity incentive bonus 8,000
fringe benefit tax for a managerial or number of days Required:Determine the taxable “13th
supervisory employee. Illustration 2 month pay and other benefits”,
TAX TREATMENT OF 13TH MONTH PAY A private rank and file employee additional compensation, and the fringe
AND OTHER BENEFITS working in a remote tower station of benefit subject to fringe benefit tax.
RR2-98 provides that 13th month pay No-signal Telecom received the Solution:
and other benefits are exempt from following benefit during 2014: De minimis Limit
withholding on compensation provided 13th month pay P67,000 Other benefits
they do not exceed P82,000. It follows, Performance bonus 15,000 Rice allowance P21,600 P18,000
therefore, that the excess above Christmas gift 22,000 P3,600
Clothing allowance 6,000 5,000 1,000 Uniform allowance P5,000 P5,000 – Daily transportation allowance
Productivity incentive bonus 8,000 Total P5,000 P79,000 16,000
10,000 0 P32,000 Total P40,000 P0
Excess de minimis P4,600 Exclusion threshold (up to P82,000) P309,000
Other fringe benefits: 32,000 -32,000 13th month pay and other benefits
Rent of residence paid by employer Taxable excess 13th month pay and 13th month pay P25,000
18,000 other benefits P0 0 Other benefits:
Salary of personal body guard 12,000 Gross taxable compensation income Vacation expense paid by employer
Total fringe benefit subject to fringe P307,000 Excess de minimis benefits:
benefit tax P34,600 Note: Rice subsidy P25,200 18,000 7,200
Profit sharing 1.The taxable regular compensation Monetized VL 10,000 10,000 -
P12,000 income is computed as(P324,000- Monetized SL 8,000 0 8,000
Night shift differential pay 11,000 P32,000) Productivity incentive 26,000
Total supplemental compensation 2.The excess of the P82,000 threshold 10,000 16,000
P23,000 over the actual 13th month pay and Uniform allowance 9,000 5,000
Total 13th month pay and other benefits other benefits is non-deductible to 4,000
P75,000 other items of compensation income. Total P78,200 P83,000 P84,200
Less:Exclusion threshold 82,000 (RR3-2015) P309,000
th
Taxable 13 month pay and other Presentation in the Income tax return Exclusion threshold 82,000 -82,000
benefits P0 of the employee: Total non-taxable benefits P165,000
INTEGRATIVE Gross compensation income(i.e., P2,200
ILLUSTRATIONS:COMPENSATION P352K+p66K) P418,000 Taxable 13th month pay and other
INCOME Less:Non-taxable income 111,000 benefits -2,200
Integrative Illustration 1 Gross taxable compensation income 2,200
A government rank and file employee P307,000 Gross taxable compensation income
had the following summary of his Integrative Illustration 2 P311,200
compensation and benefits in 2015: A private employee derived the Required:Determine the taxable “13th
Gross compensation income following remunerations and benefits in month pay and other benefits.”
P324,000 2015: Solution:
Less:Employee payroll deductions Basic compensation, net of P24,000 Christmas bonus(13th month pay of
Employee contributions to GSIS, SSS, PHIC, PhilHealth,HDMF, union gov’t employees) P50,000
PHIC;HDMF P32,000 dues, and P30,000 withholding tax Cash gift 5,000
Employee deduction for withholding P246,000 Additional compensation allowance
tax 34,000 66,000 Overtime pay 21,000 36,000
Net regular payroll P258,000 Productivity incentive 26,000 Total 13th month pay and other benefits
Representation and Transportation Vacation expenses of the employee P91,000
allowance 18,000 paid by employer 24,000 Less:Exclusion Threshold 82,000
th
Christmas bonus 27,000 Cost-of-living allowance(COLA) 12,000 Taxable 13 month pay and other
Personnel Economic Relief Allowance Pre-computed daily transportation benefits P9,000
24,000 allowance 16,000 Note on government employees:
Uniform allowance 5,000 Rice subsidy(12 cavans worth P2,100 1.Personnel Economic Relief Allowance
Christmas gift 5,000 each) 25,200 is not subject to income tax and
Honoraria 15,000 13th month pay 25,000 withholding tax(Under RR8-2000,as
Total compensation P352,000 Monetized unused leave credit(10VL affimed by RR10-2008)
The gross taxable compensation income and 8SL) 18,000 2.The P5,000 Christmas gift of
shall be computed as follows: Uniform allowance 9,000 government employees is designated by
De minimis Non-taxable Total compensation income P422,200 the NIRC to be part of “13th month pay
Other benefits Taxable Benefits The non-taxable compensation income and other benefits:;hence, not a
Regular and supplementary and the gross taxable compensation deminimis benefit.
compensation: income shall be computed as follows: 3.Under RR5-2011,the monetization of
Regular compensation P32,000 For a rank and file employee vacation leave and sick leave credits of
P292,000 De minimis Non-taxable Other government officials is an exempt de
Supplemental compensation benefits Taxable benefits minimis benefit wihout regard to the
Honoraria 15,000 Regular and supplementary number of days.
RATA 18,000 compensation: Illustration 2
PERA 24,000 Regular compensation P24,000 A private rank and file employee
Total P74,000 P0 P307,000 P276,000 working in a remote tower station of
13th month pay and other benefits: Supplementary compensation No-Signal Telecom received the
Christmas bonus P27,000 Overtime pay 21,000 following benefit during 2014:
Christmas gift 5,000 COLA 12,000 13th month pay P67,000
Excess De minimis: Performance bonus 15,000
Christmas gift 22,000
Danger exposure allowance(hazard
pay) 6,000
Housing privilege
38,000
Uniform and clothing allowance
7,000
Laundry allowance 6,000
The housing privilege pertains to the
annual value of the employees’ living
quarters furnished by the employer for
staying on-site.
Required:Compute the excess 13th
month pay and other benefits
Solution:
De minimis Limit Other
benefits
Christmas gift P22,000 P5,000
P17,000
Uniform and clothing allowance 7,000
5,000 2,000
Laundry allowance 6,000 3,600
2,400
Excess de minimis benefit(other fringe
benefits) P21,400
13th month pay and other benefits
P89,400
Less:Exclusion threshold 82,000
th
Taxable 13 month pay and other
benefits P7,400
Note:Private employees
1.Performance bonus is a supplemental
or additional compensation.
2.The laundry allowance limit is
computed as P300x12[P3,600
3.Christmas gift is a de minimis benefit
for private employees under the RR5-
2011.
4.The housing privilege is exempt under
the convenience of the employer rule.
5.The “13th month pay and other
benefits” of rank and file employees
includes “other fringe benefits.”
Illustration 3

You might also like